Sei sulla pagina 1di 293

A.

Vaginal instillation of conjugated estrogen


B. Urinary catheterization
PNLE I for Foundation of Nursing C.
D.
Nasogastric tube insertion
Colostomy irrigation
1. Which element in the circular chain of infection can be
eliminated by preserving skin integrity? Answer: B. The urinary system is normally free of
microorganisms except at the urinary meatus. Any procedure
A. Host that involves entering this system must use surgically aseptic
B. Reservoir measures to maintain a bacteria-free state
C. Mode of transmission
D. Portal of entry 7. Sterile technique is used whenever:
Answer: D. In the circular chain of infection, pathogens must A. Strict isolation is required
be able to leave their reservoir and be transmitted to a B. Terminal disinfection is performed
susceptible host through a portal of entry, such as broken skin. C. Invasive procedures are performed
D. Protective isolation is necessary
2. Which of the following will probably result in a break in
sterile technique for respiratory isolation? Answer: C. All invasive procedures, including surgery,
catheter insertion, and administration of parenteral therapy,
A. Opening the patient’s window to the outside require sterile technique to maintain a sterile environment.
environment All equipment must be sterile, and the nurse and
B. Turning on the patient’s room ventilator the physician must wear sterile gloves and maintain surgical
C. Opening the door of the patient’s room leading into asepsis. In the operating room, the nurse and physician are
the hospital corridor required to wear sterile gowns, gloves, masks, hair covers,
D. Failing to wear gloves when administering a bed bath and shoe covers for all invasive procedures. Strict isolation
requires the use of clean gloves, masks, gowns and
Answer: C. Respiratory isolation, like strict isolation, equipment to prevent the transmission of highly
requires that the door to the door patient’s room remain communicable diseases by contact or by airborne routes.
closed. However, the patient’s room should be well Terminal disinfection is the disinfection of all contaminated
ventilated, so opening the window or turning on the supplies and equipment after a patient has been discharged to
ventricular is desirable. The nurse does not need to wear prepare them for reuse by another patient. The purpose of
gloves for respiratory isolation, but good hand washing is protective (reverse) isolation is to prevent a person
important for all types of isolation with seriously impaired resistance from coming into contact
who potentially pathogenic organisms.
3. Which of the following patients is at greater risk for
contracting an infection? 8. Which of the following constitutes a break in sterile
technique while preparing a sterile field for a dressing change?
A. A patient with leukopenia
B. A patient receiving broad-spectrum antibiotics A. Using sterile forceps, rather than sterile gloves, to handle
C. A postoperative patient who has undergone orthopedic a sterile item
surgery B. Touching the outside wrapper of sterilized material
D. A newly diagnosed diabetic patient without sterile gloves
C. Placing a sterile object on the edge of the sterile field
Answer: A. Leukopenia is a decreased number of leukocytes D. Pouring out a small amount of solution (15 to 30 ml)
(white blood cells), which are important in resisting infection. before pouring the solution into a sterile container
None of the other situations would put the patient at risk for
contracting an infection; taking broadspectrum antibiotics Answer: C. The edges of a sterile field are considered
might actually reduce the infection risk. contaminated. When sterile items are allowed to come in
contact with the edges of the field, the sterile items also
4. Effective hand washing requires the use of: become contaminated
A. Soap or detergent to promote emulsification
9. A natural body defense that plays an active role in
B. Hot water to destroy bacteria
C. A disinfectant to increase surface tension preventing infection is:
D. All of the above A. Yawning
B. Body hair
Answer: A. Soaps and detergents are used to help remove C. Hiccupping
bacteria because of their ability to lower the surface tension D. Rapid eye movements
of water and act as emulsifying agents. Hot water may lead to
skin irritation or burns Answer: B. Hair on or within body areas, such as the nose,
5. After routine patient contact, hand washing should last at traps and holds particles that contain microorganisms.
least: Yawning and hiccupping do not prevent microorganisms
from entering or leaving the body. Rapid eye movement
A. 30 seconds
marks the stage of sleep during which dreaming occurs.
B. 1 minute
C. 2 minute
10. All of the following statement are true about donning
D. 3 minutes
sterile gloves except:
Answer: A. Depending on the degree of exposure to A. The first glove should be picked up by grasping the
pathogens, hand washing may last from 10 seconds to 4 inside of the cuff.
minutes. After routine patient contact, hand washing for 30 B. The second glove should be picked up by inserting the
seconds effectively minimizes the risk of gloved fingers under the cuff outside the glove.
pathogen transmission. C. The gloves should be adjusted by sliding the gloved
fingers under the sterile cuff and pulling the glove over
6. Which of the following procedures always requires surgical the wrist
asepsis?
D. The inside of the glove is considered sterile
Answer: D. The inside of the glove is always considered to Answer: A. Platelets are disk-shaped cells that are essential for
be clean, but not sterile. blood coagulation. A platelet count determines the number of
thrombocytes in blood available for promoting hemostasis and
11.When removing a contaminated gown, the nurse should be assisting with blood coagulation after injury. It also is used to
careful that the first thing she touches is the: evaluate the patient’s potential for bleeding; however, this is
not its primary purpose. The normal count ranges from
A. Waist tie and neck tie at the back of the gown 150,000 to 350,000/mm3. A count of 100,000/mm3 or less
B. Waist tie in front of the gown indicates a potential for bleeding; count of less than
C. Cuffs of the gown 20,000/mm3 is associated with spontaneous bleeding.
D. Inside of the gown
16.Which of the following white blood cell (WBC) counts
Answer: A. The back of the gown is considered clean, the clearly indicates leukocytosis?
front is contaminated. So, after removing gloves and washing
hands, the nurse should untie the back of the gown; slowly A. 4,500/mm³
move backward away from the gown, holding the inside of B. 7,000/mm³
the gown and keeping the edges off the floor; turn and fold C. 10,000/mm³
the gown inside out; discard it in a contaminated linen D. 25,000/mm³
container; then wash her hands again.
Answer: D. Leukocytosis is any transient increase in the
12.Which of the following nursing interventions is considered number of white blood cells (leukocytes) in the blood. Normal
the most effective form or universal precautions? WBC counts range from 5,000 to 100,000/mm3. Thus, a count
of 25,000/mm3 indicates leukocytosis
A. Cap all used needles before removing them from their
syringes 17. After 5 days of diuretic therapy with 20mg of furosemide
B. Discard all used uncapped needles and syringes in (Lasix) daily, a patient begins to exhibit fatigue, muscle
an impenetrable protective container cramping and muscle weakness. These symptoms probably
C. Wear gloves when administering IM injections indicate that the patient is experiencing:
D. Follow enteric precautions
A. Hypokalemia
Answer: B. According to the Centers for Disease Control B. Hyperkalemia
(CDC), blood-to-blood contact occurs most commonly when C. Anorexia
a health care worker attempts to cap a used needle. Therefore, D. Dysphagia
used needles should never be recapped; instead they should
be inserted in a specially designed puncture resistant, labeled Answer: A. Fatigue, muscle cramping, and muscle weaknesses
container. Wearing gloves is not always necessary are symptoms of hypokalemia (an inadequate potassium level),
when administering an I.M. injection. Enteric precautions which is a potential side effect of diuretic therapy. The
prevent the transfer of pathogens via feces. physician usually orders supplemental potassium to prevent
hypokalemia in patients receiving diuretics. Anorexia is
13.All of the following measures are recommended to prevent another symptom of hypokalemia. Dysphagia means
pressure ulcers except: difficulty swallowing.
A. Massaging the reddened are with lotion 18.Which of the following statements about chest X-ray is
B. Using a water or air mattress false?
C. Adhering to a schedule for positioning and turning
D. Providing meticulous skin care A. No contradictions exist for this test
B. Before the procedure, the patient should remove all
Answer: A. Nurses and other health care professionals jewelry, metallic objects, and buttons above the waist
previously believed that massaging a reddened area with C. A signed consent is not required
lotion would promote venous return and reduce edema to the D. Eating, drinking, and medications are allowed before this
area. However, research has shown that massage only test
increases the likelihood of cellular ischemia and necrosis to
the area Answer: A. Pregnancy or suspected pregnancy is the only
14.Which of the following blood tests should be performed contraindication for a chest X-ray. However, if a chest X-ray
before a blood transfusion? is necessary, the patient can wear a lead apron to protect the
pelvic region from radiation. Jewelry, metallic objects, and
A. Prothrombin and coagulation time buttons would interfere with the X-ray and thus should not
B. Blood typing and cross-matching be worn above the waist. A signed consent is not required
C. Bleeding and clotting time because a chest X-ray is not an invasive examination. Eating,
D. Complete blood count (CBC) and electrolyte levels. drinking and medications are allowed because the X-ray is of
Answer: B. Before a blood transfusion is performed, the the chest, not the abdominal region.
blood of the donor and recipient must be checked for
compatibility. This is done by blood typing (a test that 19.The most appropriate time for the nurse to obtain a sputum
determines a person’s blood type) and cross-matching specimen for culture is:
(a procedure that determines the compatibility of the donor’s
and recipient’s blood after the blood types has been matched). A. Early in the morning
If the blood specimens are incompatible, hemolysis and B. After the patient eats a light breakfast
antigen-antibody reactions will occur C. After aerosol therapy
D. After chest physiotherapy
15.The primary purpose of a platelet count is to evaluate the:
Answer: A. Obtaining a sputum specimen early in this
A. Potential for clot formation morning ensures an adequate supply of bacteria for culturing
B. Potential for bleeding and decreases the risk of contamination from food or
C. Presence of an antigen-antibody response medication.
D. Presence of cardiac enzymes
20.A patient with no known allergies is to receive penicillin A. 18G, 1 ½” long
every 6 hours. When administering the medication, the nurse B. 22G, 1” long
observes a fine rash on the C. 22G, 1 ½” long
D. 25G, 5/8” long
patient’s skin. The most appropriate nursing action would be
to: Answer: D. A 25G, 5/8” needle is the recommended size for
insulin injection because insulin is administered by the
A. Withhold the moderation and notify the physician subcutaneous route. An 18G, 1 ½” needle is usually used for
B. Administer the medication and notify the physician I.M. injections in children, typically in the vastus lateralis. A
C. Administer the medication with an antihistamine 22G, 1 ½” needle is usually used for adult I.M. injections,
D. Apply corn starch soaks to the rash which are typically administered in the vastus lateralis or
ventrogluteal site.
Answer: A. Initial sensitivity to penicillin is commonly
manifested by a skin rash, even in individuals who have not 25.The appropriate needle gauge for intradermal injection is:
been allergic to it previously. Because of the danger of
anaphylactic shock, he nurse should withhold the drug A. 20G
and notify the physician, who may choose to substitute B. 22G
another drug. Administering an antihistamine is a dependent C. 25G
nursing intervention that requires a written physician’s order. D. 26G
Although applying corn starch to the rash may relieve
discomfort, it is not the nurse’s top priority in such Answer: D. Because an intradermal injection does not
a potentially life-threatening situation. penetrate deeply into the skin, a small-bore 25G needle is
recommended. This type of injection is used primarily to
21.All of the following nursing interventions are correct when administer antigens to evaluate reactions for allergy
using the Ztrack method of drug injection except: or sensitivity studies. A 20G needle is usually used for I.M.
injections of oilbased medications; a 22G needle for I.M.
A. Prepare the injection site with alcohol injections; and a 25G needle, for I.M. injections; and a 25G
B. Use a needle that’s a least 1” long needle, for subcutaneous insulin injections.
C. Aspirate for blood before injection
D. Rub the site vigorously after the injection to promote 26.Parenteral penicillin can be administered as an:
absorption
A. IM injection or an IV solution
Answer: D. The Z-track method is an I.M. injection B. IV or an intradermal injection
technique in which the patient’s skin is pulled in such a way C. Intradermal or subcutaneous injection
that the needle track is sealed off after the injection. This D. IM or a subcutaneous injection
procedure seals medication deep into the muscle,
thereby minimizing skin staining and irritation. Rubbing the Answer: A. Parenteral penicillin can be administered I.M. or
injection site is contraindicated because it may cause the added to a solution and given I.V. It cannot be administered
medication to extravasate into the skin subcutaneously or intradermally.

22.The correct method for determining the vastus lateralis site 27.The physician orders gr 10 of aspirin for a patient. The
for I.M. injection is to: equivalent dose in milligrams is:

A. Locate the upper aspect of the upper outer quadrant of A. 0.6 mg


the buttock about 5 to 8 cm below the iliac crest B. 10 mg
B. Palpate the lower edge of the acromion process and the C. 60 mg
midpoint lateral aspect of the arm D. 600 mg
C. Palpate a 1” circular area anterior to the umbilicus
D. Divide the area between the greater femoral trochanter Answer: D. gr 10 x 60mg/gr 1 = 600 mg
and the lateral femoral condyle into thirds, and select the
middle third on the anterior of the thigh 28.The physician orders an IV solution of dextrose 5% in
water at 100ml/hour. What would the flow rate be if the drop
Answer: D. The vastus lateralis, a long, thick muscle that factor is 15 gtt = 1 ml?
extends the full length of the thigh, is viewed by many
clinicians as the site of choice for I.M. injections because it A. 5 gtt/minute
has relatively few major nerves and blood vessels. The middle B. 13 gtt/minute
third of the muscle is recommended as the injection site. C. 25 gtt/minute
The patient can be in a supine or sitting position for an D. 50 gtt/minute
injection into this site.
Answer: C. 100ml/60 min X 15 gtt/ 1 ml = 25 gtt/minute

23.The mid-deltoid injection site is seldom used for I.M. 29.Which of the following is a sign or symptom of a
injections because it: hemolytic reaction to blood transfusion?

A. Can accommodate only 1 ml or less of medication A. Hemoglobinuria


B. Bruises too easily B. Chest pain
C. Can be used only when the patient is lying down C. Urticaria
D. Does not readily parenteral medication D. Distended neck veins

Answer: A. The mid-deltoid injection site can accommodate Answer: A. Hemoglobinuria, the abnormal presence of
only 1 ml or less of medication because of its size and hemoglobin in the urine, indicates a hemolytic reaction
location (on the deltoid muscle of the arm, close to the (incompatibility of the donor’s and recipient’s blood). In this
brachial artery and radial nerve reaction, antibodies in the recipient’s plasma combine rapidly
with donor RBC’s; the cells are hemolyzed in
24.The appropriate needle size for insulin injection is: either circulatory or reticuloendothelial system. Hemolysis
occurs more rapidly in ABO incompatibilities than in Rh
incompatibilities. Chest pain and urticaria may be symptoms Answer: D. A drug-allergy is an adverse reaction resulting
of impending anaphylaxis. Distended neck veins are from an immunologic response following a previous
an indication of hypervolemia. sensitizing exposure to the drug. The reaction can range from
a rash or hives to anaphylactic shock. Tolerance to a drug
30.Which of the following conditions may require fluid means that the patient experiences a decreasing
restriction? physiologic response to repeated administration of the drug in
the same dosage. Idiosyncrasy is an individual’s unique
A. Fever hypersensitivity to a drug, food, or other substance; it appears
B. Chronic Obstructive Pulmonary Disease to be genetically determined. Synergism, is a drug interaction
C. Renal Failure in which the sum of the drug’s combined effects is
D. Dehydration greater than that of their separate effects.

Answer: C. In real failure, the kidney loses their ability to 35.A patient has returned to his room after femoral
effectively eliminate wastes and fluids. Because of this, arteriography. All of the following are appropriate nursing
limiting the patient’s intake of oral and I.V. fluids may be interventions except:
necessary. Fever, chronic obstructive pulmonary disease, and
dehydration are conditions for which fluids should A. Assess femoral, popliteal, and pedal pulses every 15
be encouraged. minutes for 2 hours
B. Check the pressure dressing for sanguineous drainage
31.All of the following are common signs and symptoms of C. Assess a vital signs every 15 minutes for 2 hours
phlebitis except: D. Order a hemoglobin and hematocrit count 1 hour
after the arteriography
A. Pain or discomfort at the IV insertion site
B. Edema and warmth at the IV insertion site Answer: D. A hemoglobin and hematocrit count would be
C. A red streak exiting the IV insertion site ordered by the physician if bleeding were suspected. The other
D. Frank bleeding at the insertion site answers are appropriate nursing interventions for a patient
who has undergone femoral arteriography.
Answer: D. Phlebitis, the inflammation of a vein, can be
caused by chemical irritants (I.V. solutions or medications),
mechanical irritants (the needle or catheter used during 36.The nurse explains to a patient that a cough:
venipuncture or cannulation), or a localized allergic reaction to
the needle or catheter. Signs and symptoms of A. Is a protective response to clear the respiratory tract
phlebitis include pain or discomfort, edema and heat at the I.V. of irritants
insertion site, and a red streak going up the arm or leg from B. Is primarily a voluntary action
the I.V. insertion site. C. Is induced by the administration of an antitussive drug
D. Can be inhibited by “splinting” the abdomen
32.The best way of determining whether a patient has learned Answer: A. Coughing, a protective response that clears the
to instill ear medication properly is for the nurse to: respiratory tract of irritants, usually is involuntary; however
it can be voluntary, as when a patient is taught to perform
A. Ask the patient if he/she has used ear drops before coughing exercises. An antitussive drug inhibits coughing.
B. Have the patient repeat the nurse’s instructions using her Splinting the abdomen supports the abdominal muscles when
own words a patient coughs
C. Demonstrate the procedure to the patient and encourage
to ask questions
D. Ask the patient to demonstrate the procedure 37.An infected patient has chills and begins shivering. The
best nursing intervention is to:
Answer: D. Return demonstration provides the most certain
evidence for evaluating the effectiveness of patient teaching. A. Apply iced alcohol sponges
B. Provide increased cool liquids
33.Which of the following types of medications can be C. Provide additional bedclothes
administered via gastrostomy tube? D. Provide increased ventilation

A. Any oral medications Answer: C. In an infected patient, shivering results from the
B. Capsules whole contents are dissolve in water body’s attempt to increase heat production and the production
C. Enteric-coated tablets that are thoroughly dissolved in of neutrophils and phagocytotic action through increased
water skeletal muscle tension and contractions. Initial
D. Most tablets designed for oral use, except for vasoconstriction may cause skin to feel cold to the touch.
extended-duration compounds Applying additional bed clothes helps to equalize the
body temperature and stop the chills. Attempts to cool the
Answer: D. Capsules, enteric-coated tablets, and most body result in further shivering, increased metabloism, and
extended duration or sustained release products should not be thus increased heat production.
dissolved for use in a gastrostomy tube. They are
pharmaceutically manufactured in these forms for valid 38.A clinical nurse specialist is a nurse who has:
reasons, and altering them destroys their purpose. The
nurse should seek an alternate physician’s order when an A. Been certified by the National League for Nursing
ordered medication is inappropriate for delivery by tube. B. Received credentials from the Philippine Nurses’
Association
34.A patient who develops hives after receiving an antibiotic C. Graduated from an associate degree program and is a
is exhibiting drug: registered professional nurse
D. Completed a master’s degree in the prescribed
A. Tolerance clinical area and is a registered professional nurse.
B. Idiosyncrasy
C. Synergism Answer: D. A clinical nurse specialist must have completed a
D. Allergy master’s degree in a clinical specialty and be a registered
professional nurse. The National League of Nursing accredits D. Maintain the drainage tubing and collection bag
educational programs in nursing and provides a testing service below bladder level to facilitate drainage by gravity
to evaluate student nursing competence but it does not certify
nurses. The American Nurses Association identifies Answer: D. Maintaing the drainage tubing and collection bag
requirements for certification and offers examinations for level with the patient’s bladder could result in reflux of urine
certification in many areas of nursing., such as medical into the kidney. Irrigating the bladder with Neosporin and
surgical nursing. These certification (credentialing) clamping the catheter for 1 hour every 4 hours must be
demonstrates that the nurse has the knowledge and the ability prescribed by a physician
to provide high quality nursing care in the area of her
certification. A graduate of an associate degree program is not 44.The ELISA test is used to:
a clinical nurse specialist: however, she is prepared to provide
bed side nursing with a high degree of knowledge and skill. A. Screen blood donors for antibodies to human
She must successfully complete the licensing examination to immunodeficiency virus (HIV)
become a registered professional nurse. B. Test blood to be used for transfusion for HIV antibodies
C. Aid in diagnosing a patient with AIDS
39.The purpose of increasing urine acidity through dietary D. All of the above
means is to:
Answer: D. The ELISA test of venous blood is used to assess
A. Decrease burning sensations blood and potential blood donors to human immunodeficiency
B. Change the urine’s color virus (HIV). A positive ELISA test combined with various
C. Change the urine’s concentration signs and symptoms helps to diagnose acquired
D. Inhibit the growth of microorganisms immunodeficiency syndrome (AIDS)

Answer: D. Microorganisms usually do not grow in an acidic 45.The two blood vessels most commonly used for TPN
environment. infusion are the:

40.Clay colored stools indicate: A. Subclavian and jugular veins


B. Brachial and subclavian veins
A. Upper GI bleeding C. Femoral and subclavian veins
B. Impending constipation D. Brachial and femoral veins
C. An effect of medication
D. Bile obstruction Answer: D. Tachypnea (an abnormally rapid rate of breathing)
would indicate that the patient was still hypoxic (deficient in
Answer: D. Bile colors the stool brown. Any inflammation or oxygen).The partial pressures of arterial oxygen and carbon
obstruction that impairs bile flow will affect the stool pigment, dioxide listed are within the normal range. Eupnea refers to
yielding light, clay-colored stool. Upper GI bleeding results in normal respiration.
black or tarry stool. Constipation is characterized by small,
hard masses. Many medications and foods will discolor stool –
for example, drugs containing iron turn stool black.; beets turn 46.Effective skin disinfection before a surgical procedure
stool red. includes which of the following methods?

41.In which step of the nursing process would the nurse ask a A. Shaving the site on the day before surgery
patient if the medication she administered relieved his pain? B. Applying a topical antiseptic to the skin on the evening
before surgery
A. Assessment C. Having the patient take a tub bath on the morning of
B. Analysis surgery
C. Planning D. Having the patient shower with an antiseptic soap on
D. Evaluation the evening v=before and the morning of surgery

Answer: D. In the evaluation step of the nursing process, the Answer: D. Studies have shown that showering with an
nurse must decide whether the patient has achieved the antiseptic soap before surgery is the most effective method of
expected outcome that was identified in the planning phase. removing microorganisms from the skin. Shaving the site of
the intended surgery might cause breaks in the skin, thereby
42.All of the following are good sources of vitamin A except: increasing the risk of infection; however, if indicated, shaving,
should be done immediately before surgery, not the day
A. White potatoes before. A topical antiseptic would not remove microorganisms
B. Carrots and would be beneficial only after proper cleaning and rinsing.
C. Apricots Tub bathing might transfer organisms to another body site
D. Egg yolks rather than rinse them away.

Answer: A. The main sources of vitamin A are yellow and 47.When transferring a patient from a bed to a chair, the nurse
green vegetables (such as carrots, sweet potatoes, squash, should use which muscles to avoid back injury?
spinach, collard greens, broccoli, and cabbage) and yellow
fruits (such as apricots, and cantaloupe). Animal sources A. Abdominal muscles
include liver, kidneys, cream, butter, and egg yolks. B. Back muscles
C. Leg muscles
43.Which of the following is a primary nursing intervention D. Upper arm muscles
necessary for all patients with a Foley Catheter in place?
Answer: C. The leg muscles are the strongest muscles in the
A. Maintain the drainage tubing and collection bag level body and should bear the greatest stress when lifting. Muscles
with the patient’s bladder of the abdomen, back, and upper arms may be easily injured.
B. Irrigate the patient with 1% Neosporin solution three
times a daily 48.Thrombophlebitis typically develops in patients with which
C. Clamp the catheter for 1 hour every 4 hours to maintain of the following conditions?
the bladder’s elasticity
A. Increases partial thromboplastin time
B. Acute pulsus paradoxus
C. An impaired or traumatized blood vessel wall
D. Chronic Obstructive Pulmonary Disease (COPD)

Answer: C. The factors, known as Virchow’s triad,


collectively predispose a patient to thromboplebitis; impaired
venous return to the heart, blood hypercoagulability, and
injury to a blood vessel wall. Increased partial thromboplastin
time indicates a prolonged bleeding time during fibrin
clot formation, commonly the result of anticoagulant (heparin)
therapy. Arterial blood disorders (such as pulsus paradoxus)
and lung diseases (such as COPD) do not necessarily impede
venous return of injure vessel walls.

49.In a recumbent, immobilized patient, lung ventilation can


become altered, leading to such respiratory complications as:

A. Respiratory acidosis, ateclectasis, and hypostatic PNLE II for Community Health


pneumonia
B. Appneustic breathing, atypical pneumonia and Nursing and Care of the
respiratory alkalosis
C. Cheyne-Strokes respirations and spontaneous Mother and Child
pneumothorax 1. May arrives at the health care clinic and tells the nurse that
D. Kussmail’s respirations and hypoventilation her last menstrual period was 9 weeks ago. She also tells the
nurse that a home pregnancy test was positive but she began
Answer: A. Because of restricted respiratory movement, a to have mild cramps and is now having moderate vaginal
recumbent, immobilize patient is at particular risk for bleeding. During the physical examination of the client, the
respiratory acidosis from poor gas exchange; atelectasis from
nurse notes that May has a dilated cervix. The nurse
reduced surfactant and accumulated mucus in the bronchioles,
determines that May is experiencing which type of abortion?
and hypostatic pneumonia from bacterial growth caused by
A. Inevitable
stasis of mucus secretions
B. Incomplete
50.Immobility impairs bladder elimination, resulting in such C. Threatened
disorders as D. Septic

A. Increased urine acidity and relaxation of the perineal Answer: Answer: (A) Inevitable. An inevitable abortion is
muscles, causing incontinence termination of pregnancy that cannot be prevented.
B. Urine retention, bladder distention, and infection Moderate to severe bleeding with mild cramping
C. Diuresis, natriuresis, and decreased urine specific gravity and cervical dilation would be noted in this type of abortion
D. Decreased calcium and phosphate levels in the urine
2. Nurse Reese is reviewing the record of a pregnant client for
Answer: B. The immobilized patient commonly suffers from her first prenatal visit. Which of the following data, if noted
urine retention caused by decreased muscle tone in the
on the client’s record, would alert the nurse that the client is
perineum. This leads to bladder distention and urine
at risk for a spontaneous abortion?
stagnation, which provide an excellent medium for bacterial
growth leading to infection. Immobility also results in A. Age 36 years
more alkaline urine with excessive amounts of calcium, B. History of syphilis
sodium and phosphate, a gradual decrease in urine C. History of genital herpes
production, and an increased specific gravity. D. History of diabetes mellitus

Answer: Answer: (B) History of syphilis. Maternal infections


such as syphilis, toxoplasmosis, and rubella are causes of
spontaneous abortion.

3. Nurse Hazel is preparing to care for a client who is newly


admitted to the hospital with a possible diagnosis of ectopic
pregnancy. Nurse Hazel develops a plan of care for the client
and determines that which of the following nursing actions is
the priority?
A. Monitoring weight
B. Assessing for edema
C. Monitoring apical pulse
D. Monitoring temperature

Answer: Answer: (C) Monitoring apical pulse. Nursing care


for the client with a possible ectopic pregnancy is focused on
preventing or identifying hypovolemic shock and
controlling pain. An elevated pulse rate is an indicator of
shock

4. Nurse Oliver is teaching a diabetic pregnant client about


nutrition and insulin needs during pregnancy. The nurse
determines that the client understands dietary and insulin
needs if the client states that the second half of pregnancy could result in injury to the mother and the fetus if Pitocin is
require: not discontinued.
A. Decreased caloric intake
B. Increased caloric intake 9. Calcium gluconate is being administered to a client with
C. Decreased Insulin pregnancy induced hypertension (PIH). A nursing action that
D. Increase Insulin must be initiated as the plan of care throughout injection of
the drug is:
Answer: (B) Increased caloric intake. Glucose crosses the A. Ventilator assistance
placenta, but insulin does not. High fetal demands for glucose, B. CVP readings
combined with the insulin resistance caused by hormonal C. EKG tracings
changes in the last half of pregnancy can result in elevation D. Continuous CPR
of maternal blood glucose levels. This increases the mother’s
demand for insulin and is referred to as the diabetogenic Answer: Answer: (C) EKG tracings. A potential side effect of
effect of pregnancy. calcium gluconate administration is cardiac arrest.
Continuous monitoring of cardiac activity (EKG) throught
5. Nurse Michelle is assessing a 24 year old client with a administration of calcium gluconate is an essential part of
diagnosis of hydatidiform mole. She is aware that one of the care.
following is unassociated with this condition?
A. Excessive fetal activity. 10. A trial for vaginal delivery after an earlier caesareans,
B. Larger than normal uterus for gestational age. would likely to be given to a gravida, who had:
C. Vaginal bleeding A. First low transverse cesarean was for active herpes type
D. Elevated levels of human chorionic gonadotropin. 2 infections; vaginal culture at 39 weeks pregnancy was
positive.
Answer: Answer: (A) Excessive fetal activity. The most B. First and second caesareans were for cephalopelvic
common signs and symptoms of hydatidiform mole includes disproportion.
elevated levels of human chorionic gonadotropin, C. First caesarean through a classic incision as a result of
vaginal bleeding, larger than normal uterus for gestational severe fetal distress.
age, failure to detect fetal heart activity even with sensitive D. First low transverse caesarean was for breech position.
instruments, excessive nausea and vomiting, and early Fetus in this pregnancy is in a vertex presentation.
development of pregnancy-induced hypertension. Fetal
activity would not be noted. Answer: Answer: (D) First low transverse caesarean was for
breech position. Fetus in this pregnancy is in a vertex
6. A pregnant client is receiving magnesium sulfate for severe presentation. This type of client has no obstetrical indication
pregnancy induced hypertension (PIH). The clinical findings for a caesarean section as she did with her first caesarean
that would warrant use of the antidote , calcium gluconate is: delivery.
A. Urinary output 90 cc in 2 hours.
B. Absent patellar reflexes. 11.Nurse Ryan is aware that the best initial approach when
C. Rapid respiratory rate above 40/min. trying to take a crying toddler’s temperature is:
D. Rapid rise in blood pressure. A. Talk to the mother first and then to the toddler.
B. Bring extra help so it can be done quickly.
Answer: Answer: (B) Absent patellar reflexes. Absence of C. Encourage the mother to hold the child.
patellar reflexes is an indicator of hypermagnesemia, which D. Ignore the crying and screaming.
requires administration of calcium gluconate
Answer: Answer: (A) Talk to the mother first and then to the
7. During vaginal examination of Janah who is in labor, the toddler. When dealing with a crying toddler, the best
presenting part is at station plus two. Nurse, correctly approach is to talk to the mother and ignore the toddler first.
interprets it as: This approach helps the toddler get used to the nurse before
A. Presenting part is 2 cm above the plane of the ischial she attempts any procedures. It also gives the toddler an
spines. opportunity to see that the mother trusts the nurse.
B. Biparietal diameter is at the level of the ischial spines.
C. Presenting part in 2 cm below the plane of the ischial 12.Baby Tina a 3 month old infant just had a cleft lip and
spines. palate repair. What should the nurse do to prevent trauma to
D. Biparietal diameter is 2 cm above the ischial spines. operative site?
A. Avoid touching the suture line, even when cleaning.
Answer: Answer: (C) Presenting part in 2 cm below the plane B. Place the baby in prone position.
of the ischial spines. Fetus at station plus two indicates that C. Give the baby a pacifier.
the presenting part is 2 cm below the plane of the ischial D. Place the infant’s arms in soft elbow restraints.
spines.
Answer: Answer: (D) Place the infant’s arms in soft elbow
8. A pregnant client is receiving oxytocin (Pitocin) for restraints. Soft restraints from the upper arm to the wrist
induction of labor. A condition that warrant the nurse in- prevent the infant from touching her lip but allow him to hold
charge to discontinue I.V. infusion of Pitocin is: a favorite item such as a blanket. Because they could damage
A. Contractions every 1 ½ minutes lasting 70-80 seconds. the operative site, such as objects as pacifiers, suction
B. Maternal temperature 101.2 catheters, and small spoons shouldn’t be placed in a baby’s
C. Early decelerations in the fetal heart rate. mouth after cleft repair. A baby in a prone position may rub
D. Fetal heart rate baseline 140-160 bpm. her face on the sheets and traumatize the operative site. The
suture line should be cleaned gently to prevent infection,
Answer: Answer: (A) Contractions every 1 ½ minutes lasting which could interfere with healing and damage the cosmetic
70-80 seconds. Contractions every 1 ½ minutes lasting 70-80 appearance of the repair.
seconds, is indicative of hyperstimulation of the uterus, which
13. Which action should nurse Marian include in the care plan B. Provincial Health Office
for a 2 month old with heart failure? C. Regional Health Office
A. Feed the infant when he cries. D. Rural Health Unit
B. Allow the infant to rest before feeding.
C. Bathe the infant and administer medications before Answer: Answer: (D) Rural Health Unit. R.A. 7160 devolved
feeding. basic health services to local government units (LGU’s ). The
D. Weigh and bathe the infant before feeding. public health nurse is an employee of the LGU.

Answer: Answer: (B) Allow the infant to rest before 19.Tony is aware the Chairman of the Municipal Health Board
feeding. Because feeding requires so much energy, an infant is:
with heart failure should rest before feeding A. Mayor
B. Municipal Health Officer
14.Nurse Hazel is teaching a mother who plans to discontinue C. Public Health Nurse
breast feeding after 5 months. The nurse should advise her to D. Any qualified physician
include which foods in her infant’s diet?
A. Skim milk and baby food. Answer: Answer: (A) Mayor. The local executive serves as the
B. Whole milk and baby food. chairman of the Municipal Health Board
C. Iron-rich formula only.
D. Iron-rich formula and baby food. 20.Myra is the public health nurse in a municipality with a
total population of about 20,000. There are 3 rural health
Answer: Answer: (C) Iron-rich formula only. The infants at midwives among the RHU personnel. How many more
age 5 months should receive iron-rich formula and that they midwife items will the RHU need?
shouldn’t receive solid food, even baby food until age 6 A. 1
months. B. 2
C. 3
15.Mommy Linda is playing with her infant, who is sitting D. The RHU does not need any more midwife item.
securely alone on the floor of the clinic. The mother hides a
toy behind her back and the infant looks for it. The nurse is Answer: Answer: (A) 1. Each rural health midwife is given a
aware that estimated age of the infant would be: population assignment of about 5,000.
A. 6 months
B. 4 months 21.According to Freeman and Heinrich, community health
C. 8 months nursing is a developmental service. Which of the following
D. 10 months best illustrates this statement?
A. The community health nurse continuously develops
Answer: Answer: (D) 10 months. A 10 month old infant can himself personally and professionally.
sit alone and understands object permanence, so he would B. Health education and community organizing are
look for the hidden toy. At age 4 to 6 months, infants can’t sit necessary in providing community health services.
securely alone. At age 8 months, infants can sit securely alone C. Community health nursing is intended primarily for
but cannot understand the permanence of objects. health promotion and prevention and treatment of
disease.
16.Which of the following is the most prominent feature of D. The goal of community health nursing is to provide
public health nursing? nursing services to people in their own places of
A. It involves providing home care to sick people who are residence.
not confined in the hospital.
B. Services are provided free of charge to people within Answer: Answer: (B) Health education and community
the catchments area. organizing are necessary in providing community health
C. The public health nurse functions as part of a team services. The community health nurse develops the health
providing a public health nursing services. capability of people through health education and
D. Public health nursing focuses on preventive, not community organizing activities
curative, services.
22.Nurse Tina is aware that the disease declared through
Answer: Answer: (D) Public health nursing focuses on Presidential Proclamation No. 4 as a target for eradication in
preventive, not curative, services. The catchments area in the Philippines is?
PHN consists of a residential community, many of whom are A. Poliomyelitis
well individuals who have greater need for preventive rather B. Measles
than curative services. C. Rabies
D. Neonatal tetanus
17.When the nurse determines whether resources were
maximized in implementing Ligtas Tigdas, she is evaluating Answer: Answer: (B) Measles. Presidential Proclamation No.
A. Effectiveness 4 is on the Ligtas Tigdas Program.
B. Efficiency
C. Adequacy 23.May knows that the step in community organizing that
D. Appropriateness involves training of potential leaders in the community is:
A. Integration
Answer: Answer: (B) Efficiency. Efficiency is determining B. Community organization
whether the goals were attained at the least possible cost. C. Community study
D. Core group formation
18.Vangie is a new B.S.N. graduate. She wants to become a
Public Health Nurse. Where should she apply? Answer: Answer: (D) Core group formation. In core group
A. Department of Health formation, the nurse is able to transfer the technology of
community organizing to the potential or informal Answer: Answer: (A) Change the diaper more
community leaders through a training program. often. Decreasing the amount of time the skin comes contact
with wet soiled diapers will help heal the irritation.
24.Beth a public health nurse takes an active role in
community participation. What is the primary goal of 29.Nurse Carla knows that the common cardiac anomalies in
community organizing? children with Down Syndrome (tri-somy 21) is:
A. To educate the people regarding community health A. Atrial septal defect
problems B. Pulmonic stenosis
B. To mobilize the people to resolve community health C. Ventricular septal defect
problems D. Endocardial cushion defect
C. To maximize the community’s resources in dealing
with health problems. Answer: (D) Endocardial cushion defect. Endocardial cushion
D. To maximize the community’s resources in dealing defects are seen most in children with Down syndrome,
with health problems. asplenia, or polysplenia
Answer: (D) To maximize the community’s resources in
dealing with health problems. Community organizing is a 30.Malou was diagnosed with severe preeclampsia is now
developmental service, with the goal of developing the receiving I.V. magnesium sulfate. The adverse effects
people’s self-reliance in dealing with community health associated with magnesium sulfate is:
problems. A, B and C are objectives of contributory A. Anemia
objectives to this goal. B. Decreased urine output
C. Hyperreflexia
D. Increased respiratory rate
25.Tertiary prevention is needed in which stage of the natural
history of disease? Answer: Answer: (B) Decreased urine output. Decreased
A. Pre-pathogenesis urine output may occur in clients receiving I.V. magnesium
B. Pathogenesis and should be monitored closely to keep urine output
C. Prodromal at greater than 30 ml/hour, because magnesium is excreted
D. Terminal through the kidneys and can easily accumulate to toxic levels.

Answeer: Answer: Answer: (D) Terminal. Tertiary prevention 31.A 23 year old client is having her menstrual period every 2
involves rehabilitation, prevention of permanent disability weeks that last for 1 week. This type of menstrual pattern is
and disability limitation appropriate for convalescents, the bets defined by:
disabled, complicated cases and the terminally ill (those in A. Menorrhagia
the terminal stage of a disease B. Metrorrhagia
C. Dyspareunia
26.The nurse is caring for a primigravid client in the labor and D. Amenorrhea
delivery area. Which condition would place the client at risk
for disseminated intravascular coagulation (DIC)? Answer: Answer: (A) Menorrhagia. Menorrhagia is an
A. Intrauterine fetal death. excessive menstrual period.
B. Placenta accreta.
C. Dysfunctional labor. 32. Jannah is admitted to the labor and delivery unit. The
D. Premature rupture of the membranes. critical laboratory result for this client would be:
A. Oxygen saturation
Answer: Answer: (A) Intrauterine fetal death. Intrauterine B. Iron binding capacity
fetal death, abruptio placentae, septic shock, and amniotic C. Blood typing
fluid embolism may trigger normal clotting mechanisms; if D. Serum Calcium
clotting factors are depleted, DIC may occur. Placenta accreta,
dysfunctional labor, and premature rupture of the Answer: Answer: (C) Blood typing. Blood type would be a
membranes aren’t associated with DIC. critical value to have because the risk of blood loss is always
a potential complication during the labor and delivery
27.A fullterm client is in labor. Nurse Betty is aware that the process. Approximately 40% of a woman’s cardiac output is
fetal heart rate would be: delivered to the uterus, therefore, blood loss can occur
A. 80 to 100 beats/minute quite rapidly in the event of uncontrolled bleeding
B. 100 to 120 beats/minute
C. 120 to 160 beats/minute 33.Nurse Gina is aware that the most common condition
D. 160 to 180 beats/minute found during the second-trimester of pregnancy is:
A. Metabolic alkalosis
Answer: Answer: (C) 120 to 160 beats/minute. A rate of 120 B. Respiratory acidosis
to 160 beats/minute in the fetal heart appropriate for filling C. Mastitis
the heart with blood and pumping it out to the system. D. Physiologic anemia

28.The skin in the diaper area of a 7 month old infant is Answer: Answer: (D) Physiologic anemia. Hemoglobin values
excoriated and red. Nurse Hazel should instruct the mother and hematocrit decrease during pregnancy as the increase in
to: plasma volume exceeds the increase in red blood cell
A. Change the diaper more often. production.
B. Apply talc powder with diaper changes.
C. Wash the area vigorously with each diaper change. 34.Nurse Lynette is working in the triage area of an
D. Decrease the infant’s fluid intake to decrease saturating emergency department. She sees that several pediatric
diapers. clients arrive simultaneously. The client who needs to be
treated first is:
A. A crying 5 year old child with a laceration on his scalp. must understand that, although the “fertile” period is
B. A 4 year old child with a barking coughs and flushed approximately mid-cycle, hormonal variations do occur and
appearance. can result in early or late ovulation. To be effective, the
C. A 3 year old child with Down syndrome who is pale and diaphragm should be inserted before every intercourse.
asleep in his mother’s arms.
D. A 2 year old infant with stridorous breath sounds, 39.Hypoxia is a common complication of
sitting up in his mother’s arms and drooling. laryngotracheobronchitis. Nurse Oliver should frequently
assess a child with laryngotracheobronchitis for:
Answer: Answer: (D) A 2 year old infant with stridorous A. Drooling
breath sounds, sitting up in his mother’s arms and B. Muffled voice
drooling. The infant with the airway emergency should be C. Restlessness
treated first, because of the risk of epiglottitis. D. Low-grade fever
Answer: (C) Restlessness. In a child, restlessness is the earliest
35.Maureen in her third trimester arrives at the emergency sign of hypoxia. Late signs of hypoxia in a child are associated
room with painless vaginal bleeding. Which of the following with a change in color, such as pallor or cyanosis.
conditions is suspected?
A. Placenta previa 40.How should Nurse Michelle guide a child who is blind to
B. Abruptio placentae walk to the playroom?
C. Premature labor A. Without touching the child, talk continuously as the
D. Sexually transmitted disease child walks down the hall.
B. Walk one step ahead, with the child’s hand on the
Answer: Answer: (A) Placenta previa. Placenta previa with nurse’s elbow.
painless vaginal bleeding. C. Walk slightly behind, gently guiding the child forward.
D. Walk next to the child, holding the child’s hand.
36.A young child named Richard is suspected of having
pinworms. The community nurse collects a stool specimen to Answer: (B) Walk one step ahead, with the child’s hand on
confirm the diagnosis. The nurse should schedule the the nurse’s elbow. This procedure is generally recommended
collection of this specimen for: to follow in guiding a person who is blind.
A. Just before bedtime
B. After the child has been bathe 41.When assessing a newborn diagnosed with ductus
C. Any time during the day arteriosus, Nurse Olivia should expect that the child most
D. Early in the morning likely would have an:
A. Loud, machinery-like murmur.
Answer: Answer: (D) Early in the morning. Based on the B. Bluish color to the lips.
nurse’s knowledge of microbiology, the specimen should be C. Decreased BP reading in the upper extremities
collected early in the morning. The rationale for this timing D. Increased BP reading in the upper extremities.
is that, because the female worm lays eggs at night around
the perineal area, the first bowel movement of the day will Answer: (A) Loud, machinery-like murmur. A loud,
yield the best results. The specific type of stool specimen machinery-like murmur is a characteristic finding associated
used in the diagnosis of pinworms is called the tape test with patent ductus arteriosus

37.In doing a child’s admission assessment, Nurse Betty 42.The reason nurse May keeps the neonate in a neutral
should be alert to note which signs or symptoms of chronic thermal environment is that when a newborn becomes too
lead poisoning? cool, the neonate requires:
A. Irritability and seizures A. Less oxygen, and the newborn’s metabolic rate
B. Dehydration and diarrhea increases.
C. Bradycardia and hypotension B. More oxygen, and the newborn’s metabolic rate
D. Petechiae and hematuria decreases.
C. More oxygen, and the newborn’s metabolic rate
:Answer: (A) Irritability and seizures. Lead poisoning primarily increases.
affects the CNS, causing increased intracranial pressure. This D. Less oxygen, and the newborn’s metabolic rate
condition results in irritability and changes in level of decreases.
consciousness, as well as seizure disorders, hyperactivity, and
learning disabilities. Answer: (C) More oxygen, and the newborn’s metabolic
rate increases. When cold, the infant requires more oxygen
38.To evaluate a woman’s understanding about the use of and there is an increase in metabolic rate. Non-shievering
diaphragm for family planning, Nurse Trish asks her to explain thermogenesis is a complex process that increases the
how she will use the appliance. Which response indicates a metabolic rate and rate of oxygen consumption, therefore,
need for further health teaching? the newborn increase heat production.
A. “I should check the diaphragm carefully for holes every
time I use it” 43.Before adding potassium to an infant’s I.V. line, Nurse Ron
B. “I may need a different size of diaphragm if I gain or must be sure to assess whether this infant has:
lose weight more than 20 pounds” A. Stable blood pressure
C. “The diaphragm must be left in place for atleast 6 hours B. Patant fontanelles
after intercourse” C. Moro’s reflex
D. “I really need to use the diaphragm and jelly most D. Voided
during the middle of my menstrual cycle”.
Answer: (D) Voided. Before administering potassium I.V. to
Answer: (D) “I really need to use the diaphragm and jelly any client, the nurse must first check that the client’s kidneys
most during the middle of my menstrual cycle”. The woman are functioning and that the client is voiding. If the client is
not voiding, the nurse should withhold the potassium and person-to-person transmitted infections, such as sexually
notify the physician. transmitted diseases

44.Nurse Carla should know that the most common causative 49.A 33-year old female client came for consultation at the
factor of dermatitis in infants and younger children is: health center with the chief complaint of fever for a week.
A. Baby oil Accompanying symptoms were muscle pains and body
B. Baby lotion malaise. A week after the start of fever, the client noted
C. Laundry detergent yellowish discoloration of his sclera. History showed that he
D. Powder with cornstarch waded in flood waters about 2 weeks before the onset of
symptoms. Based on her history, which disease condition will
Answer: Answer: (C) Laundry detergent. Eczema or you suspect?
dermatitis is an allergic skin reaction caused by an offending A. Hepatitis A
allergen. The topical allergen that is the most common B. Hepatitis B
causative factor is laundry detergent C. Tetanus
45.During tube feeding, how far above an infant’s stomach D. Leptospirosis
should the nurse hold the syringe with formula?
A. 6 inches Answer: (D) Leptospirosis. Leptospirosis is transmitted
B. 12 inches through contact with the skin or mucous membrane with
C. 18 inches water or moist soil contaminated with urine of infected
D. 24 inches animals, like rats

Answer: (A) 6 inches. This distance allows for easy flow of 50.Mickey a 3-year old client was brought to the health
the formula by gravity, but the flow will be slow enough not center with the chief complaint of severe diarrhea and the
to overload the stomach too rapidly passage of “rice water” stools. The client is most probably
suffering from which condition?
46. In a mothers’ class, Nurse Lhynnete discussed childhood A. Giardiasis
diseases such as chicken pox. Which of the following B. Cholera
statements about chicken pox is correct? C. Amebiasis
A. The older one gets, the more susceptible he becomes D. Dysentery
to the complications of chicken pox.
B. A single attack of chicken pox will prevent future Answer: (B) Cholera. Passage of profuse watery stools is the
episodes, including conditions such as shingles. major symptom of cholera. Both amebic and bacillary
C. To prevent an outbreak in the community, quarantine dysentery are characterized by the presence of blood and/or
may be imposed by health authorities. mucus in the stools. Giardiasis is characterized by fat
D. Chicken pox vaccine is best given when there is an malabsorption and, therefore, steatorrhea.
impending outbreak in the community.
51.The most prevalent form of meningitis among children
Answer: (A) The older one gets, the more susceptible he aged 2 months to 3 years is caused by which microorganism?
becomes to the complications of chicken pox. Chicken pox is A. Hemophilus influenzae
usually more severe in adults than in children. Complications, B. Morbillivirus
such as pneumonia, are higher in incidence in adults. C. Steptococcus pneumoniae
D. Neisseria meningitides
47.Barangay Pinoy had an outbreak of German measles. To
prevent congenital rubella, what is the BEST advice that you Answer: (A) Hemophilus influenzae. Hemophilus meningitis is
can give to women in the first trimester of pregnancy in the unusual over the age of 5 years. In developing countries, the
barangay Pinoy? peak incidence is in children less than 6 months of age.
A. Advice them on the signs of German measles. Morbillivirus is the etiology of measles. Streptococcus
B. Avoid crowded places, such as markets and movie pneumoniae and Neisseria meningitidis may cause meningitis,
houses. but age distribution is not specific in young children.
C. Consult at the health center where rubella vaccine may
be given. 52.The student nurse is aware that the pathognomonic sign
D. Consult a physician who may give them rubella of measles is Koplik’s spot and you may see Koplik’s spot by
immunoglobulin. inspecting the:
A. Nasal mucosa
Answer: (D) Consult a physician who may give them rubella B. Buccal mucosa
immunoglobulin. Rubella vaccine is made up of attenuated C. Skin on the abdomen
German measles viruses. This is contraindicated in pregnancy. D. Skin on neck
Immune globulin, a specific prophylactic against German
measles, may be given to pregnant women. Answer: (B) Buccal mucosa. Koplik’s spot may be seen on the
mucosa of the mouth or the throat.
48.Myrna a public health nurse knows that to determine
possible sources of sexually transmitted infections, the BEST 53.Angel was diagnosed as having Dengue fever. You will say
method that may be undertaken is: that there is slow capillary refill when the color of the nailbed
A. Contact tracing that you pressed does not return within how many seconds?
B. Community survey A. 3 seconds
C. Mass screening tests B. 6 seconds
D. Interview of suspects C. 9 seconds
D. 10 seconds
Answer: (A) Contact tracing. Contact tracing is the most
practical and reliable method of finding possible sources of
Answer: (A) 3 seconds. Adequate blood supply to the area
allows the return of the color of the nailbed within 3 59.Nurses are aware that diagnosis of leprosy is highly
seconds dependent on recognition of symptoms. Which of the
following is an early sign of leprosy?
54.In Integrated Management of Childhood Illness, the nurse A. Macular lesions
is aware that the severe conditions generally require urgent B. Inability to close eyelids
referral to a hospital. Which of the following severe C. Thickened painful nerves
conditions DOES NOT always require urgent referral to a D. Sinking of the nosebridge
hospital?
A. Mastoiditis Answer: Answer: (C) Thickened painful nerves. The lesion of
B. Severe dehydration leprosy is not macular. It is characterized by a change in skin
C. Severe pneumonia color (either reddish or whitish) and loss of sensation,
D. Severe febrile disease sweating and hair growth over the lesion. Inability to close
Answer: (B) Severe dehydration. The order of priority in the the eyelids (lagophthalmos) and sinking of the nosebridge are
management of severe dehydration is as follows: late symptoms.
intravenous fluid therapy, referral to a facility where IV
fluids can be initiated within 30 minutes, Oresol or 60.Marie brought her 10 month old infant for consultation
nasogastric tube. When the foregoing measures are not because of fever, started 4 days prior to consultation. In
possible or effective, then urgent referral to the hospital is determining malaria risk, what will you do?
done. A. Perform a tourniquet test.
B. Ask where the family resides.
55.Myrna a public health nurse will conduct outreach C. Get a specimen for blood smear.
immunization in a barangay Masay with a population of D. Ask if the fever is present everyday.
about 1500. The estimated number of infants in the barangay
would be: Answer: (B) Ask where the family resides. Because malaria is
A. 45 infants endemic, the first question to determine malaria risk is where
B. 50 infants the client’s family resides. If the area of residence is not a
C. 55 infants known endemic area, ask if the child had traveled within the
D. 65 infants past 6 months, where she was brought and whether she
stayed overnight in that area.
Answer: (A) 45 infants. To estimate the number of infants,
multiply total population by 3%. 61.Susie brought her 4 years old daughter to the RHU
because of cough and colds. Following the IMCI assessment
56.The community nurse is aware that the biological used in guide, which of the following is a danger sign that indicates
Expanded Program on Immunization (EPI) should NOT be the need for urgent referral to a hospital?
stored in the freezer? A. Inability to drink
A. DPT B. High grade fever
B. Oral polio vaccine C. Signs of severe dehydration
C. Measles vaccine D. Cough for more than 30 days
D. MMR
Answer: (A) Inability to drink. A sick child aged 2 months to 5
Answer: (A) DPT. DPT is sensitive to freezing. The appropriate years must be referred urgently to a hospital if he/she has
storage temperature of DPT is 2 to 8° C only. OPV and one or more of the following signs: not able to feed or drink,
measles vaccine are highly sensitive to heat and require vomits everything, convulsions, abnormally sleepy or
freezing. MMR is not an immunization in the Expanded difficult to awaken
Program on Immunization.
62.Jimmy a 2-year old child revealed “baggy pants”. As a
57.It is the most effective way of controlling schistosomiasis nurse, using the IMCI guidelines, how will you manage Jimmy?
in an endemic area? A. Refer the child urgently to a hospital for confinement.
A. Use of molluscicides B. Coordinate with the social worker to enroll the child in a
B. Building of foot bridges feeding program.
C. Proper use of sanitary toilets C. Make a teaching plan for the mother, focusing on menu
D. Use of protective footwear, such as rubber boots planning for her child.
D. Assess and treat the child for health problems like
Answer: (C) Proper use of sanitary toilets. The ova of the infections and intestinal parasitism.
parasite get out of the human body together with feces.
Cutting the cycle at this stage is the most effective way of Answer: (A) Refer the child urgently to a hospital for
preventing the spread of the disease to susceptible hosts. confinement. “Baggy pants” is a sign of severe marasmus. The
best management is urgent referral to a hospital.
58.Several clients is newly admitted and diagnosed with
leprosy. Which of the following clients should be classified as 63.Gina is using Oresol in the management of diarrhea of her
a case of multibacillary leprosy? 3-year old child. She asked you what to do if her child vomits.
A. 3 skin lesions, negative slit skin smear As a nurse you will tell her to:
B. 3 skin lesions, positive slit skin smear A. Bring the child to the nearest hospital for further
C. 5 skin lesions, negative slit skin smear assessment.
D. 5 skin lesions, positive slit skin smear B. Bring the child to the health center for intravenous fluid
therapy.
Answer: (D) 5 skin lesions, positive slit skin smear. A C. Bring the child to the health center for assessment by
multibacillary leprosy case is one who has a positive slit skin the physician.
smear and at least 5 skin lesions.
D. Let the child rest for 10 minutes then continue giving C. 1 year
Oresol more slowly. D. 2 years

Answer: (D) Let the child rest for 10 minutes then continue Answer: (B) 6 months. After 6 months, the baby’s nutrient
giving Oresol more slowly. If the child vomits persistently, needs, especially the baby’s iron requirement, can no longer
that is, he vomits everything that he takes in, he has to be be provided by mother’s milk alone
referred urgently to a hospital. Otherwise, vomiting is
managed by letting the child rest for 10 minutes and then 69.Nurse Ron is aware that the gestational age of a conceptus
continuing with Oresol administration. Teach the mother to that is considered viable (able to live outside the womb) is:
give Oresol more slowly. A. 8 weeks
B. 12 weeks
64.Nikki a 5-month old infant was brought by his mother to C. 24 weeks
the health center because of diarrhea for 4 to 5 times a day. D. 32 weeks
Her skin goes back slowly after a skin pinch and her eyes are Answer: (C) 24 weeks. At approximately 23 to 24 weeks’
sunken. Using the IMCI guidelines, you will classify this infant gestation, the lungs are developed enough to sometimes
in which category? maintain extrauterine life. The lungs are the most immature
A. No signs of dehydration system during the gestation period. Medical care for
B. Some dehydration premature labor begins much earlier (aggressively at 21
C. Severe dehydration weeks’ gestation
D. The data is insufficient.
70.When teaching parents of a neonate the proper position
Answer: (B) Some dehydration. Using the assessment for the neonate’s sleep, the nurse Patricia stresses the
guidelines of IMCI, a child (2 months to 5 years old) with importance of placing the neonate on his back to reduce the
diarrhea is classified as having SOME DEHYDRATION if he risk of which of the following?
shows 2 or more of the following signs: restless or irritable, A. Aspiration
sunken eyes, the skin goes back slow after a skin pinch. B. Sudden infant death syndrome (SIDS)
C. Suffocation
65.Chris a 4-month old infant was brought by her mother to D. Gastroesophageal reflux (GER)
the health center because of cough. His respiratory rate is
42/minute. Using the Integrated Management of Child Illness Answer: (B) Sudden infant death syndrome (SIDS). Supine
(IMCI) guidelines of assessment, his breathing is considered positioning is recommended to reduce the risk of SIDS in
as: infancy. The risk of aspiration is slightly increased with the
A. Fast supine position. Suffocation would be less likely with an
B. Slow infant supine than prone and the position for GER requires
C. Normal the head of the bed to be elevated.
D. Insignificant
71.Which finding might be seen in baby James a neonate
Answer: (C) Normal. In IMCI, a respiratory rate of 50/minute suspected of having an infection?
or more is fast breathing for an infant aged 2 to 12 months. A. Flushed cheeks
B. Increased temperature
66.Maylene had just received her 4th dose of tetanus toxoid. C. Decreased temperature
She is aware that her baby will have protection against D. Increased activity level
tetanus for
A. 1 year Answer: (C) Decreased temperature. Temperature instability,
B. 3 years especially when it results in a low temperature in the neonate,
C. 5 years may be a sign of infection. The neonate’s color often changes
D. Lifetime with an infection process but generally becomes ashen or
mottled. The neonate with an infection will usually show
Answer: (A) 1 year. The baby will have passive natural a decrease in activity level or lethargy.
immunity by placental transfer of antibodies. The mother will
have active artificial immunity lasting for about 10 years. 5 72.Baby Jenny who is small-for-gestation is at increased risk
doses will give the mother lifetime protection. during the transitional period for which complication?
A. Anemia probably due to chronic fetal hyposia
67.Nurse Ron is aware that unused BCG should be discarded B. Hyperthermia due to decreased glycogen stores
after how many hours of reconstitution? C. Hyperglycemia due to decreased glycogen stores
A. 2 hours D. Polycythemia probably due to chronic fetal hypoxia
B. 4 hours
C. 8 hours Answer: (D) Polycythemia probably due to chronic fetal
D. At the end of the day hypoxia. The small-for-gestation neonate is at risk for
developing polycythemia during the transitional period in an
Answer: (B) 4 hours. While the unused portion of other attempt to decreasehypoxia. The neonates are also at
biologicals in EPI may be given until the end of the day, only increased risk for developing hypoglycemia and hypothermia
BCG is discarded 4 hours after reconstitution. This is why BCG due to decreased glycogen stores.
immunization is scheduled only in the morning.
73.Marjorie has just given birth at 42 weeks’ gestation. When
the nurse assessing the neonate, which physical finding is
68.The nurse explains to a breastfeeding mother that breast expected?
milk is sufficient for all of the baby’s nutrient needs only up to: A. A sleepy, lethargic baby
A. 5 months B. Lanugo covering the body
B. 6 months C. Desquamation of the epidermis
D. Vernix caseosa covering the body D. To prepare for an imminent delivery.

Answer: (C) Desquamation of the epidermis. Postdate fetuses Answer: (B) To assess for prolapsed cord. After a client has an
lose the vernix caseosa, and the epidermis may become amniotomy, the nurse should assure that the cord isn’t
desquamated. These neonates are usually very alert. Lanugo prolapsed and that the baby tolerated the procedure well.
is missing in the postdate neonate. The most effective way to do this is to check the fetal heart
rate. Fetal well-being is assessed via a nonstress test. Fetal
74.After reviewing the Myrna’s maternal history of position is determined by vaginal examination. Artificial
magnesium sulfate during labor, which condition would nurse rupture of membranes doesn’t indicate an imminent delivery
Richard anticipate as a potential problem in the neonate?
A. Hypoglycemia 79.Which of the following would be least likely to indicate
B. Jitteriness anticipated bonding behaviors by new parents?
C. Respiratory depression A. The parents’ willingness to touch and hold the new born.
D. Tachycardia B. The parent’s expression of interest about the size of the
Answer: (C) Respiratory depression. Magnesium sulfate new born.
crosses the placenta and adverse neonatal effects are C. The parents’ indication that they want to see the
respiratory depression, hypotonia, and bradycardia. The newborn.
serum blood sugar isn’t affected by magnesium sulfate. The D. The parents’ interactions with each other.
neonate would be floppy, not jittery.
Answer: (D) The parents’ interactions with each
75.Which symptom would indicate the Baby Alexandra was other. Parental interaction will provide the nurse with a good
adapting appropriately to extra-uterine life without difficulty? assessment of the stability of the family’s home life but it has
A. Nasal flaring no indication for parental bonding. Willingness to touch and
B. Light audible grunting hold the newborn, expressing interest about the newborn’s
C. Respiratory rate 40 to 60 breaths/minute size, and indicating a desire to see the newborn are behaviors
D. Respiratory rate 60 to 80 breaths/minute indicating parental bonding.

Answer: (C) Respiratory rate 40 to 60 breaths/minute. A 80.Following a precipitous delivery, examination of the
respiratory rate 40 to 60 breaths/minute is normal for a client’s vagina reveals
neonate during the transitional period. Nasal flaring, a fourth-degree laceration. Which of the following would be
respiratory rate more than 60 breaths/minute, and audible contraindicated when caring for this client?
grunting are signs of respiratory distress. A. Applying cold to limit edema during the first 12 to 24
hours.
76. When teaching umbilical cord care for Jennifer a new B. Instructing the client to use two or more peripads to
mother, the nurse Jenny would include which information? cushion the area.
A. Apply peroxide to the cord with each diaper change C. Instructing the client on the use of sitz baths if ordered.
B. Cover the cord with petroleum jelly after bathing D. Instructing the client about the importance of perineal
C. Keep the cord dry and open to air (kegel) exercises.
D. Wash the cord with soap and water each day during a
tub bath. Answer: (B) Instructing the client to use two or more peripads
to cushion the area. Using two or more peripads would do
Answer: (C) Keep the cord dry and open to air. Keeping the little to reduce the pain or promote perineal healing. Cold
cord dry and open to air helps reduce infection and hastens applications, sitz baths, and Kegel exercises are important
drying. Infants aren’t given tub bath but are sponged off until measures when the client has a fourth-degree laceration.
the cord falls off. Petroleum jelly prevents the cord from
drying and encourages infection. Peroxide could be painful 81. A pregnant woman accompanied by her husband, seeks
and isn’t recommended. admission to the labor and delivery area. She states that she’s
in labor and says she attended the facility clinic for prenatal
77.Nurse John is performing an assessment on a neonate. care. Which question should the nurse Oliver ask her first?
Which of the following findings is considered common in the A. “Do you have any chronic illnesses?”
healthy neonate? B. “Do you have any allergies?”
A. Simian crease C. “What is your expected due date?”
B. Conjunctival hemorrhage D. “Who will be with you during labor?”
C. Cystic hygroma
D. Bulging fontanelle Answer: (C) “What is your expected due date?” When
obtaining the history of a client who may be in labor, the
Answer: (B) Conjunctival hemorrhage. Conjunctival nurse’s highest priority is to determine her current status,
hemorrhages are commonly seen in neonates secondary to particularly her due date, gravidity, and parity. Gravidity and
the cranial pressure applied during the birth process. Bulging parity affect the duration of labor and the potential for labor
fontanelles are a sign of intracranial pressure. Simian creases complications. Later, the nurse should ask about chronic
are present in 40% of the neonates with trisomy 21. Cystic illnesses, allergies, and support persons.
hygroma is a neck mass that can affect the airway.
82.A neonate begins to gag and turns a dusky color. What
78.Dr. Esteves decides to artificially rupture the membranes should the nurse do first?
of a mother who is on labor. Following this procedure, the A. Calm the neonate.
nurse Hazel checks the fetal heart tones for which the B. Notify the physician.
following reasons? C. Provide oxygen via face mask as ordered
A. To determine fetal well-being. D. Aspirate the neonate’s nose and mouth with a bulb
B. To assess for prolapsed cord syringe.
C. To assess fetal position
Answer: (D) Aspirate the neonate’s nose and mouth with a D. 38 to 40 weeks
bulb syringe. The nurse’s first action should be to clear the
neonate’s airway with a bulb syringe. After the airway is clear Answer: (C) 30 to 32 weeks. Individual twins usually grow at
and the neonate’s color improves, the nurse should comfort the same rate as singletons until 30 to 32 weeks’ gestation,
and calm the neonate. If the problem recurs or the neonate’s then twins don’t’ gain weight as rapidly as singletons of the
color doesn’t improve readily, the nurse should notify the same gestational age. The placenta can no longer keep pace
physician. Administering oxygen when the airway isn’t clear with the nutritional requirements of both fetuses after 32
would be ineffective. weeks, so there’s some growth retardation in twins if they
remain in utero at 38 to 40 weeks.

83. When a client states that her “water broke,” which of the 87. Which of the following classifications applies to
following actions would be inappropriate for the nurse to do? monozygotic twins for whom the cleavage of the fertilized
A. Observing the pooling of straw-colored fluid. ovum occurs more than 13 days after fertilization?
B. Checking vaginal discharge with nitrazine paper. A. conjoined twins
C. Conducting a bedside ultrasound for an amniotic fluid B. diamniotic dichorionic twins
index. C. diamniotic monochorionic twin
D. Observing for flakes of vernix in the vaginal discharge. D. monoamniotic monochorionic twins

Answer: (C) Conducting a bedside ultrasound for an amniotic Answer: (A) conjoined twins. The type of placenta that
fluid index. It isn’t within a nurse’s scope of practice to develops in monozygotic twins depends on the time at which
perform and interpret a bedside ultrasound under these cleavage of the ovum occurs. Cleavage in conjoined twins
conditions and without specialized training. Observing for occurs more than 13 days after fertilization. Cleavage that
pooling of straw-colored fluid, checking vaginal discharge occurs less than 3 day after fertilization results in diamniotic
with nitrazine paper, and observing for flakes of vernix are dicchorionic twins. Cleavage that occurs between days 3 and
appropriate assessments for determining whether a client 8 results in diamniotic monochorionic twins. Cleavage that
has ruptured membranes. occurs between days 8 to 13 result in monoamniotic
monochorionic twins.

84. A baby girl is born 8 weeks premature. At birth, she has 88. Tyra experienced painless vaginal bleeding has just been
no spontaneous respirations but is successfully resuscitated. diagnosed as having a placenta previa. Which of the following
Within several hours she develops respiratory grunting, procedures is usually performed to diagnose placenta previa?
cyanosis, tachypnea, nasal flaring, and retractions. She’s A. Amniocentesis
diagnosed with respiratory distress syndrome, intubated, and B. Digital or speculum examination
placed on a ventilator. Which nursing action should be C. External fetal monitoring
included in the baby’s plan of care to prevent retinopathy of D. Ultrasound
prematurity?
A. Cover his eyes while receiving oxygen. Answer: (D) Ultrasound. Once the mother and the fetus are
B. Keep her body temperature low. stabilized, ultrasound evaluation of the placenta should be
C. Monitor partial pressure of oxygen (Pao2) levels. done to determine the cause of the bleeding. Amniocentesis
D. Humidify the oxygen. is contraindicated in placenta previa. A digital or speculum
examination shouldn’t be done as this may lead to severe
Answer: (C) Monitor partial pressure of oxygen (Pao2) bleeding or hemorrhage. External fetal monitoring won’t
levels. Monitoring PaO2 levels and reducing the oxygen detect a placenta previa, although it will detect fetal distress,
concentration to keep PaO2 within normal limits reduces the which may result from blood loss or placenta separation.
risk of retinopathy of prematurity in a premature infant
receiving oxygen. Covering the infant’s eyes and humidifying 89. Nurse Arnold knows that the following changes in
the oxygen don’t reduce the risk of retinopathy of respiratory functioning during pregnancy is considered
prematurity. Because cooling increases the risk of acidosis, normal:
the infant should be kept warm so that his respiratory A. Increased tidal volume
distress isn’t aggravated. B. Increased expiratory volume
C. Decreased inspiratory capacity
85. Which of the following is normal newborn calorie intake? D. Decreased oxygen consumption
A. 110 to 130 calories per kg.
B. 30 to 40 calories per lb of body weight. Answer: (A) Increased tidal volume. A pregnant client
C. At least 2 ml per feeding breathes deeper, which increases the tidal volume of gas
D. 90 to 100 calories per kg moved in and out of the respiratory tract with each breath.
The expiratory volume and residual volume decrease as the
Answer: (A) 110 to 130 calories per kg. Calories per kg is the pregnancy progresses. The inspiratory capacity increases
accepted way of determined appropriate nutritional intake during pregnancy. The increased oxygen consumption in the
for a newborn. The recommended calorie requirement is 110 pregnant client is 15% to 20% greater than in the
to 130 calories per kg of newborn body weight. This level will nonpregnant state.
maintain a consistent blood glucose level and provide enough
calories for continued growth and development. 90. Emily has gestational diabetes and it is usually managed
by which of the following therapy?
86. Nurse John is knowledgeable that usually individual twins A. Diet
will grow appropriately and at the same rate as singletons B. Long-acting insulin
until how many weeks? C. Oral hypoglycemic
A. 16 to 18 weeks D. Oral hypoglycemic drug and insulin
B. 18 to 22 weeks
C. 30 to 32 weeks
Answer: (A) Diet. Clients with gestational diabetes are Answer: (B) An indurated wheal over 10 mm in diameter
usually managed by diet alone to control their glucose appears in 48 to 72 hours. A positive PPD result would be an
intolerance. Oral hypoglycemic drugs are contraindicated in indurated wheal over 10 mm in diameter that appears in 48
pregnancy. Long-acting insulin usually isn’t needed for blood to 72 hours. The area must be a raised wheal, not a flat
glucose control in the client with gestational diabetes circumcised area to be considered positive.

91. Magnesium sulfate is given to Jemma with preeclampsia 95. Dianne, 24 year-old is 27 weeks’ pregnant arrives at her
to prevent which of the following condition? physician’s office with complaints of fever, nausea, vomiting,
A. Hemorrhage malaise, unilateral flank pain, and costovertebral angle
B. Hypertension tenderness. Which of the following diagnoses is most likely?
C. Hypomagnesemia A. Asymptomatic bacteriuria
D. Seizure B. Bacterial vaginosis
C. Pyelonephritis
1. Answer: (D) Seizure. The anticonvulsant mechanism of D. Urinary tract infection (UTI)
magnesium is believes to depress seizure foci in the Answer: (C) Pyelonephritis. The symptoms indicate acute
brain and peripheral neuromuscular blockade. pyelonephritis, a serious condition in a pregnant client. UTI
Hypomagnesemia isn’t a complication of preeclampsia. symptoms include dysuria, urgency, frequency, and
Antihypertensive drug other than magnesium are suprapubic tenderness. Asymptomatic bacteriuria doesn’t
preferred for sustained hypertension. Magnesium cause symptoms. Bacterial vaginosis causes milky white
doesn’t help prevent hemorrhage in preeclamptic vaginal discharge but no systemic symptoms
clients.
96. Rh isoimmunization in a pregnant client develops during
which of the following conditions?
92. Cammile with sickle cell anemia has an increased risk for A. Rh-positive maternal blood crosses into fetal blood,
having a sickle cell crisis during pregnancy. Aggressive stimulating fetal antibodies.
management of a sickle cell crisis includes which of the B. Rh-positive fetal blood crosses into maternal blood,
following measures? stimulating maternal antibodies.
A. Antihypertensive agents C. Rh-negative fetal blood crosses into maternal blood,
B. Diuretic agents stimulating maternal antibodies.
C. I.V. fluids D. Rh-negative maternal blood crosses into fetal blood,
D. Acetaminophen (Tylenol) for pain stimulating fetal antibodies.

Answer: (C) I.V. fluids. A sickle cell crisis during pregnancy is Answer: (B) Rh-positive fetal blood crosses into maternal
usually managed by exchange transfusion oxygen, and L.V. blood, stimulating maternal antibodies. Rh isoimmunization
Fluids. The client usually needs a stronger analgesic than occurs when Rh-positive fetal blood cells cross into the
acetaminophen to control the pain of a crisis. maternal circulation and stimulate maternal
Antihypertensive drugs usually aren’t necessary. Diuretic antibody production. In subsequent pregnancies with Rh-
wouldn’t be used unless fluid overload resulted. positive fetuses, maternal antibodies may cross back into
the fetal circulation and destroy the fetal blood cells
93. Which of the following drugs is the antidote for
magnesium toxicity? 97. To promote comfort during labor, the nurse John advises
A. Calcium gluconate (Kalcinate) a client to assume certain positions and avoid others. Which
B. Hydralazine (Apresoline) position may cause maternal hypotension and fetal hypoxia?
C. Naloxone (Narcan) A. Lateral position
D. Rho (D) immune globulin (RhoGAM) B. Squatting position
C. Supine position
Answer: (A) Calcium gluconate (Kalcinate). Calcium gluconate D. Standing position
is the antidote for magnesium toxicity. Ten milliliters of 10%
calcium gluconate is given L.V. push over 3 to 5 minutes. Answer: (C) Supine position. The supine position causes
Hydralazine is given for sustained elevated blood pressure in compression of the client’s aorta and inferior vena cava by
preeclamptic clients. Rho (D) immune globulin is given to the fetus. This, in turn, inhibits maternal circulation, leading
women with Rh-negative blood to prevent antibody to maternal hypotension and, ultimately, fetal hypoxia. The
formation from RH-positive conceptions. Naloxone is used to other positions promote comfort and aid labor progress. For
correct narcotic toxicity. instance, the lateral, or side-lying, position improves maternal
and fetal circulation, enhances comfort, increases maternal
94. Marlyn is screened for tuberculosis during her first relaxation, reduces muscle tension, and eliminates pressure
prenatal visit. An intradermal injection of purified protein points. The squatting position promotes comfort by taking
derivative (PPD) of the tuberculin bacilli is given. She is advantage of gravity. The standing position also takes
considered to have a positive test for which of the following advantage of gravity and aligns the fetus with the pelvic angle.
results?
A. An indurated wheal under 10 mm in diameter appears 98. Celeste who used heroin during her pregnancy delivers a
in 6 to 12 hours. neonate. When assessing the neonate, the nurse Lhynnette
B. An indurated wheal over 10 mm in diameter appears expects to find:
in 48 to 72 hours. A. Lethargy 2 days after birth.
C. A flat circumcised area under 10 mm in diameter B. Irritability and poor sucking.
appears in 6 to 12 hours. C. A flattened nose, small eyes, and thin lips.
D. A flat circumcised area over 10 mm in diameter appears D. Congenital defects such as limb anomalies.
in 48 to 72 hours.
Answer: (B) Irritability and poor sucking. Neonates of heroin-
addicted mothers are physically dependent on the drug and
experience withdrawal when the drug is no longer supplied.
Signs of heroin withdrawal include irritability, poor sucking, 3. A male client is admitted to the emergency department
and restlessness. Lethargy isn’t associated with neonatal following an accident. What are the first nursing actions of
heroin addiction. A flattened nose, small eyes, and thin lips the nurse?
are seen in infants with fetal alcohol syndrome. Heroin use
during pregnancy hasn’t been linked to specific congenital A. Check respiration, circulation, neurological response.
anomalies. B. Align the spine, check pupils, and check for hemorrhage.
C. Check respirations, stabilize spine, and check
99. The uterus returns to the pelvic cavity in which of the circulation.
following time frames? D. Assess level of consciousness and circulation.
A. 7th to 9th day postpartum.
B. 2 weeks postpartum. Answer: (C) Check respirations, stabilize spine, and check
C. End of 6th week postpartum. circulation. Checking the airway would be priority, and a
D. When the lochia changes to alba. neck injury should be suspected
Answer: (A) 7th to 9th day postpartum. The normal
involutional process returns the uterus to the pelvic cavity in 4. In evaluating the effect of nitroglycerin, Nurse Arthur
7 to 9 days. A significant involutional complication is the should know that it reduces preload and relieves angina by:
failure of the uterus to return to the pelvic cavity within the
prescribed time period. This is known as subinvolution A. Increasing contractility and slowing heart rate.
B. Increasing AV conduction and heart rate.
100. Maureen, a primigravida client, age 20, has just C. Decreasing contractility and oxygen consumption.
completed a difficult, forceps-assisted delivery of twins. Her D. Decreasing venous return through vasodilation.
labor was unusually long and required oxytocin (Pitocin)
augmentation. The nurse who’s caring for her should stay Answer: (D) Decreasing venous return through
alert for: vasodilation. The significant effect of nitroglycerin is
A. Uterine inversion vasodilation and decreased venous return, so the heart does
B. Uterine atony not have to work hard.
C. Uterine involution
D. Uterine discomfort 5. Nurse Patricia finds a female client who is post-myocardial
infarction (MI) slumped on the side rails of the bed and
Answer: (B) Uterine atony. Multiple fetuses, extended labor unresponsive to shaking or shouting. Which is the nurse next
stimulation with oxytocin, and traumatic delivery commonly action?
are associated with uterine atony, which may lead to
postpartum hemorrhage. Uterine inversion may precede or A. Call for help and note the time.
follow delivery and commonly results from apparent B. Clear the airway
excessive traction on the umbilical cord and attempts to C. Give two sharp thumps to the precordium, and check
deliver the placenta manually. Uterine involution and some the pulse.
uterine discomfort are normal after delivery. D. Administer two quick blows.

Answer: (A) Call for help and note the time. Having
PNLE III for Care of Clients with established, by stimulating the client, that the client
Physiologic and Psychosocial is unconscious rather than sleep, the nurse should
immediately call for help. This may be done by dialing the
Alterations (Part 1) operator from the client’s phone and giving the hospital
code for cardiac arrest and the client’s room number to the
operator, of if the phone is not available, by pulling the
1. Nurse Michelle should know that the drainage is normal 4 emergency call button. Noting the time is important
days after a sigmoid colostomy when the stool is: baseline information for cardiac arrest procedure

A. Green liquid
B. Solid formed 6. Nurse Monett is caring for a client recovering from gastro-
C. Loose, bloody intestinal bleeding. The nurse should:
D. Semiformed
A. Plan care so the client can receive 8 hours of
Answer: (C) Loose, bloody. Normal bowel function and soft- uninterrupted sleep each night.
formed stool usually do not occur until around the seventh B. Monitor vital signs every 2 hours.
day following surgery. The stool consistency is related to C. Make sure that the client takes food and medications
how much water is being absorbed at prescribed intervals.
D. Provide milk every 2 to 3 hours.
2. Where would nurse Kristine place the call light for a male
client with a right-sided brain attack and left homonymous Answer: (C) Make sure that the client takes food and
hemianopsia? medications at prescribed intervals. Food and drug therapy
will prevent the accumulation of hydrochloric acid, or will
A. On the client’s right side neutralize and buffer the acid that does accumulate.
B. On the client’s left side
C. Directly in front of the client 7. A male client was on warfarin (Coumadin) before
D. Where the client like admission, and has been receiving heparin I.V. for 2 days. The
partial thromboplastin time (PTT) is 68 seconds. What should
Answer: (A) On the client’s right side. The client has left
Nurse Carla do?
visual field blindness. The client will see only from the right
side A. Stop the I.V. infusion of heparin and notify the physician.
B. Continue treatment as ordered. 12. A male client has active tuberculosis (TB). Which of the
C. Expect the warfarin to increase the PTT. following symptoms will be exhibit?
D. Increase the dosage, because the level is lower than
normal. A. Chest and lower back pain
B. Chills, fever, night sweats, and hemoptysis
Answer: (B) Continue treatment as ordered. The effects of C. Fever of more than 104°F (40°C) and nausea
heparin are monitored by the PTT is normally 30 to 45 D. Headache and photophobia
seconds; the therapeutic level is 1.5 to 2 times the normal
level. Answer: (B) Chills, fever, night sweats, and
hemoptysis. Typical signs and symptoms are chills, fever,
8. A client undergone ileostomy, when should the drainage night sweats, and hemoptysis. Chest pain may be present
appliance be applied to the stoma? from coughing, but isn’t usual. Clients with TB typically have
low-grade fevers, not higher than 102°F (38.9°C). Nausea,
A. 24 hours later, when edema has subsided. headache, and photophobia aren’t usual TB symptoms.
B. In the operating room.
C. After the ileostomy begin to function. 13. Mark, a 7-year-old client is brought to the emergency
D. When the client is able to begin self-care procedures. department. He’s tachypneic and afebrile and has a
respiratory rate of 36 breaths/minute and has a
Answer: (B) In the operating room. The stoma drainage bag is nonproductive cough. He recently had a cold. Form this
applied in the operating room. Drainage from the ileostomy history; the client may have which of the following conditions?
contains secretions that are rich in digestive enzymes and
highly irritating to the skin. Protection of the skin from A. Acute asthma
the effects of these enzymes is begun at once. Skin exposed B. Bronchial pneumonia
to these enzymes even for a short time becomes reddened, C. Chronic obstructive pulmonary disease (COPD)
painful, and excoriated. D. Emphysema

9. A client undergone spinal anesthetic, it will be important Answer:(A) Acute asthma. Based on the client’s history and
that the nurse immediately position the client in: symptoms, acute asthma is the most likely diagnosis. He’s
unlikely to have bronchial pneumonia without a productive
A. On the side, to prevent obstruction of airway by tongue. cough and fever and he’s too young to have developed
B. Flat on back. (COPD) and emphysema.
C. On the back, with knees flexed 15 degrees.
D. Flat on the stomach, with the head turned to the side. 14. Marichu was given morphine sulfate for pain. She is
sleeping and her respiratory rate is 4 breaths/minute. If
Answer: (B) Flat on back. To avoid the complication of a action isn’t taken quickly, she might have which of the
painful spinal headache that can last for several days, the
following reactions?
client is kept in flat in a supine position for approximately 4 to
12 hours postoperatively. Headaches are believed to be A. Asthma attack
causes by the seepage of cerebral spinal fluid from the B. Respiratory arrest
puncture site. By keeping the client flat, cerebral spinal fluid C. Seizure
pressures are equalized, which avoids trauma to the neurons. D. Wake up on his own

10.While monitoring a male client several hours after a motor Answer: (B) Respiratory arrest. Narcotics can cause
vehicle accident, which assessment data suggest increasing respiratory arrest if given in large quantities. It’s unlikely the
intracranial pressure? client will have asthma attack or a seizure or wake up on his
own.
A. Blood pressure is decreased from 160/90 to 110/70.
B. Pulse is increased from 87 to 95, with an occasional 15. A 77-year-old male client is admitted for elective knee
skipped beat. surgery. Physical examination reveals shallow respirations but
C. The client is oriented when aroused from sleep, and
no sign of respiratory distress. Which of the following is a
goes back to sleep immediately.
normal physiologic change related to aging?
D. The client refuses dinner because of anorexia.
A. Increased elastic recoil of the lungs
Answer: (C) The client is oriented when aroused from sleep, B. Increased number of functional capillaries in the alveoli
and goes back to sleep immediately. This finding suggest that C. Decreased residual volume
the level of consciousness is decreasing. D. Decreased vital capacity

11.Mrs. Cruz, 80 years old is diagnosed with pneumonia. Answer: (D) Decreased vital capacity. Reduction in vital
Which of the following symptoms may appear first? capacity is a normal physiologic changes include decreased
elastic recoil of the lungs, fewer functional capillaries in the
A. Altered mental status and dehydration
alveoli, and an increased in residual volume.
B. Fever and chills
C. Hemoptysis and Dyspnea
16. Nurse John is caring for a male client receiving lidocaine
D. Pleuritic chest pain and cough
I.V. Which factor is the most relevant to administration of this
Answer: (A) Altered mental status and dehydration. Fever, medication?
chills, hemortysis, dyspnea, cough, and pleuritic chest pain A. Decrease in arterial oxygen saturation (SaO2) when
are the common symptoms of pneumonia, but elderly clients measured with a pulse oximeter.
may first appear with only an altered lentil status and B. Increase in systemic blood pressure.
dehydration due to a blunted immune response. C. Presence of premature ventricular contractions (PVCs)
on a cardiac monitor.
D. Increase in intracranial pressure (ICP). diagnose lumps that are cancerous, areas of thickness or
fullness that signal the presence of a malignancy, or masses
Answer: (C) Presence of premature ventricular contractions that are fibrocystic as opposed to malignant.
(PVCs) on a cardiac monitor. Lidocaine drips are commonly
used to treat clients whose arrhythmias haven’t been 21. When caring for a female client who is being treated for
controlled with oral medication and who are having PVCs that hyperthyroidism, it is important to:
are visible on the cardiac monitor. SaO2, blood pressure, and
ICP are important factors but aren’t as significant as PVCs in A. Provide extra blankets and clothing to keep the client
the situation. warm.
B. Monitor the client for signs of restlessness, sweating,
17. Nurse Ron is caring for a male client taking an and excessive weight loss during thyroid replacement
anticoagulant. The nurse should teach the client to: therapy.
C. Balance the client’s periods of activity and rest.
A. Report incidents of diarrhea. D. Encourage the client to be active to prevent
B. Avoid foods high in vitamin K constipation
C. Use a straight razor when shaving.
D. Take aspirin to pain relief. Answer: (C) Balance the client’s periods of activity and
rest. A client with hyperthyroidism needs to be encouraged
Answer: (B) Avoid foods high in vitamin K. The client should to balance periods of activity and rest. Many clients with
avoid consuming large amounts of vitamin K because vitamin hyperthyroidism are hyperactive and complain of feeling
K can interfere with anticoagulation. The client may need to very warm
report diarrhea, but isn’t effect of taking an anticoagulant. An .
electric razor-not a straight razor-should be used to prevent 22. Nurse Kris is teaching a client with history of
cuts that cause bleeding. Aspirin may increase the risk of atherosclerosis. To decrease the risk of atherosclerosis, the
bleeding; acetaminophen should be used to pain relief. nurse should encourage the client to:

18. Nurse Lhynnette is preparing a site for the insertion of an A. Avoid focusing on his weight.
I.V. catheter. The nurse should treat excess hair at the site by: B. Increase his activity level.
C. Follow a regular diet.
A. Leaving the hair intact D. Continue leading a high-stress lifestyle.
B. Shaving the area
C. Clipping the hair in the area Answer: (B) Increase his activity level. The client should be
D. Removing the hair with a depilatory. encouraged to increase his activity level. Maintaining an ideal
weight; following a low-cholesterol, low sodium diet; and
Answer: (C) Clipping the hair in the area. Hair can be a source avoiding stress are all important factors in decreasing the risk
of infection and should be removed by clipping. Shaving the of atherosclerosis.
area can cause skin abrasions and depilatories can irritate the
skin. 23. Nurse Greta is working on a surgical floor. Nurse Greta
must logroll a client following a:
19. Nurse Michelle is caring for an elderly female with
osteoporosis. When teaching the client, the nurse should A. Laminectomy
include information about which major complication: B. Thoracotomy
C. Hemorrhoidectomy
A. Bone fracture D. Cystectomy.
B. Loss of estrogen
C. Negative calcium balance Answer: (A) Laminectomy. The client who has had spinal
D. Dowager’s hump surgery, such as laminectomy, must be log rolled to keep the
spinal column straight when turning. Thoracotomy and
Answer: (A) Bone fracture. Bone fracture is a major cystectomy may turn themselves or may be assisted into a
complication of osteoporosis that results when loss of comfortable position. Under normal
calcium and phosphate increased the fragility of bones. circumstances, hemorrhoidectomy is an outpatient
Estrogen deficiencies result from menopause-not procedure, and the client may resume normal activities
osteoporosis. Calcium and vitamin D supplements may be immediately after surgery.
used to support normal bone metabolism, But a negative
calcium balance isn’t a complication of osteoporosis. 24. A 55-year old client underwent cataract removal with
Dowager’s hump results from bone fractures. It intraocular lens implant. Nurse Oliver is giving the client
develops when repeated vertebral fractures increase spinal discharge instructions. These instructions should include
curvature. which of the following?

20. Nurse Len is teaching a group of women to perform BSE. A. Avoid lifting objects weighing more than 5 lb (2.25 kg).
The nurse should explain that the purpose of performing the B. Lie on your abdomen when in bed
examination is to discover: C. Keep rooms brightly lit.
D. Avoiding straining during bowel movement or bending
A. Cancerous lumps at the waist.
B. Areas of thickness or fullness
C. Changes from previous examinations. Answer: (D) Avoiding straining during bowel movement or
D. Fibrocystic masses bending at the waist. The client should avoid straining, lifting
heavy objects, and coughing harshly because these activities
Answer: (C) Changes from previous examinations. Women increase intraocular pressure. Typically, the client is
are instructed to examine themselves to discover changes instructed to avoid lifting objects weighing more than 15 lb
that have occurred in the breast. Only a physician can (7kg) – not 5lb. instruct the client when lying in bed to lie on
either the side or back. The client should avoid bright light
by wearing sunglasses.
29. The nurse is caring for Kenneth experiencing an acute
25. George should be taught about testicular examinations asthma attack. The client stops wheezing and breath sounds
during: aren’t audible. The reason for this change is that:

A. when sexual activity starts A. The attack is over.


B. After age 69 B. The airways are so swollen that no air cannot get
C. After age 40 through.
D. Before age 20. C. The swelling has decreased.
D. Crackles have replaced wheezes.
Answer: (D) Before age 20. Testicular cancer commonly
occurs in men between ages 20 and 30. A male client should Answer: (B) The airways are so swollen that no air cannot get
be taught how to perform testicular selfexamination before through. During an acute attack, wheezing may stop and
age 20, preferably when he enters his teens. breath sounds become inaudible because the airways are so
swollen that air can’t get through. If the attack is over and
26. A male client undergone a colon resection. While turning swelling has decreased, there would be no more wheezing
him, wound dehiscence with evisceration occurs. Nurse Trish and less emergent concern. Crackles do not replace wheezes
first response is to: during an acute asthma attack.

A. Call the physician 30. Mike with epilepsy is having a seizure. During the active
B. Place a saline-soaked sterile dressing on the wound. seizure phase, the nurse should:
C. Take a blood pressure and pulse.
D. Pull the dehiscence closed. A. Place the client on his back remove dangerous objects,
and insert a bite block.
Answer: (B) Place a saline-soaked sterile dressing on the B. Place the client on his side, remove dangerous objects,
wound. The nurse should first place saline-soaked sterile and insert a bite block.
dressings on the open wound to prevent tissue drying and C. Place the client o his back, remove dangerous objects,
possible infection. Then the nurse should call the physician and hold down his arms.
and take the client’s vital signs. The dehiscence needs to be D. Place the client on his side, remove dangerous objects,
surgically closed, so the nurse should never try to close it. and protect his head.

Answer: (D) Place the client on his side, remove dangerous


27. Nurse Audrey is caring for a client who has suffered a objects, and protect his head. During the active seizure
severe cerebrovascular accident. During routine assessment, phase, initiate precautions by placing the client on his side,
the nurse notices Cheyne- Strokes respirations. Cheyne- removing dangerous objects, and protecting his head from
strokes respirations are: injury. A bite block should never be inserted during the
active seizure phase. Insertion can break the teeth and lead
A. A progressively deeper breaths followed by shallower to aspiration
breaths with apneic periods. 31. After insertion of a cheat tube for a pneumothorax, a
B. Rapid, deep breathing with abrupt pauses between client becomes hypotensive with neck vein distention,
each breath. tracheal shift, absent breath sounds, and diaphoresis. Nurse
C. Rapid, deep breathing and irregular breathing without Amanda suspects a tension pneumothorax has
pauses. occurred. What cause of tension pneumothorax should the
D. Shallow breathing with an increased respiratory rate.
nurse check for?

A. Infection of the lung.


Answer: (A) A progressively deeper breaths followed by B. Kinked or obstructed chest tube
shallower breaths with apneic periods. Cheyne-Strokes C. Excessive water in the water-seal chamber
respirations are breaths that become progressively deeper D. Excessive chest tube drainage
fallowed by shallower respirations with apneas periods. Biot’s
respirations are rapid, deep breathing with abrupt Answer: (B) Kinked or obstructed chest tube. Kinking and
pauses between each breath, and equal depth between each blockage of the chest tube is a common cause of a tension
breath. Kussmaul’s respirations are rapid, deep breathing pneumothorax. Infection and excessive drainage won’t cause
without pauses. Tachypnea is shallow breathing with a tension pneumothorax. Excessive water won’t affect the
increased respiratory rate chest tube drainage.

28. Nurse Bea is assessing a male client with heart failure. The 32. Nurse Maureen is talking to a male client, the client
breath sounds commonly auscultated in clients with heart begins choking on his lunch. He’s coughing forcefully. The
failure are: nurse should:
A. Tracheal A. Stand him up and perform the abdominal thrust
B. Fine crackles maneuver from behind.
C. Coarse crackles B. Lay him down, straddle him, and perform the
D. Friction rubs abdominal thrust maneuver.
C. Leave him to get assistance
Answer: (B) Fine crackles. Fine crackles are caused by fluid in D. Stay with him but not intervene at this time.
the alveoli and commonly occur in clients with heart failure.
Tracheal breath sounds are auscultated over the trachea. Answer: (D) Stay with him but not intervene at this time. If
Coarse crackles are caused by secretion accumulation in the the client is coughing, he should be able to dislodge
airways. Friction rubs occur with pleural inflammation. the object or cause a complete obstruction. If complete
obstruction occurs, the nurse should perform the abdominal A. A 16-year-old female high school student
thrust maneuver with the client standing. If the client is B. A 33-year-old day-care worker
unconscious, she should lay him down. A nurse should never C. A 43-yesr-old homeless man with a history of
leave a choking client alone. alcoholism
D. A 54-year-old businessman
33. Nurse Ron is taking a health history of an 84 year old
client. Which information will be most useful to the nurse for Answer: (C) A 43-yesr-old homeless man with a history of
planning care? alcoholism. Clients who are economically disadvantaged,
malnourished, and have reduced immunity, such as a client
A. General health for the last 10 years. with a history of alcoholism, are at extremely high risk for
B. Current health promotion activities. developing TB. A high school student, daycare worker, and
C. Family history of diseases. businessman probably have a much low risk of contracting TB.
D. Marital status.
37. Virgie with a positive Mantoux test result will be sent for a
Answer: (B) Current health promotion activities. Recognizing chest X-ray. The nurse is aware that which of the following
an individual’s positive health measures is very useful. reasons this is done?
General health in the previous 10 years is important, however,
the current activities of an 84 year old client are most A. To confirm the diagnosis
significant in planning care. Family history of disease for a B. To determine if a repeat skin test is needed
client in later years is of minor significance. Marital status C. To determine the extent of lesions
information may be important for discharge planning but is D. To determine if this is a primary or secondary infection
not as significant for addressing the immediate
medical problem. Answer: (C ) To determine the extent of lesions. If the lesions
are large enough, the chest X-ray will show their presence in
34. When performing oral care on a comatose client, Nurse the lungs. Sputum culture confirms the diagnosis. There
Krina should: can be false-positive and false-negative skin test results. A
chest X-ray can’t determine if this is a primary or secondary
A. Apply lemon glycerin to the client’s lips at least every 2 infection.
hours.
B. Brush the teeth with client lying supine. 38. Kennedy with acute asthma showing inspiratory and
C. Place the client in a side lying position, with the head expiratory wheezes and a decreased forced expiratory
of the bed lowered. volume should be treated with which of the following classes
D. Clean the client’s mouth with hydrogen peroxide.
of medication right away?
Answer: (C) Place the client in a side lying position, with the A. Beta-adrenergic blockers
head of the bed lowered. The client should be positioned in a B. Bronchodilators
side-lying position with the head of the bed lowered to C. Inhaled steroids
prevent aspiration. A small amount of toothpaste should be D. Oral steroids
used and the mouth swabbed or suctioned to remove pooled Answer: (B) Bronchodilators. Bronchodilators are the first line
secretions. Lemon glycerin can be drying if used for extended of treatment for asthma because broncho-constriction is the
periods. Brushing the teeth with the client lying supine may cause of reduced airflow. Beta adrenergic blockers aren’t
lead to aspiration. Hydrogen peroxide is caustic to tissues and used to treat asthma and can cause
should not be used. bronchoconstriction. Inhaled oral steroids may be given to
reduce the inflammation but aren’t used for emergency relief.
35. A 77-year-old male client is admitted with a diagnosis of
dehydration and change in mental status. He’s being 39. Mr. Vasquez 56-year-old client with a 40-year history of
hydrated with L.V. fluids. When the nurse takes his vital signs, smoking one to two packs of cigarettes per day has a chronic
she notes he has a fever of 103°F (39.4°C) a cough producing cough producing thick sputum, peripheral edema and
yellow sputum and pleuritic chest pain. The nurse suspects cyanotic nail beds. Based on this information, he most likely
this client may have which of the following conditions? has which of the following conditions?

A. Adult respiratory distress syndrome (ARDS) A. Adult respiratory distress syndrome (ARDS)
B. Myocardial infarction (MI) B. Asthma
C. Pneumonia C. Chronic obstructive bronchitis
D. Tuberculosis D. Emphysema

Answer: (C) Pneumonia. Fever productive cough and pleuritic Answer: (C) Chronic obstructive bronchitis. Because of this
chest pain are common signs and symptoms of pneumonia. extensive smoking history and symptoms the client most
The client with ARDS has dyspnea and hypoxia with likely has chronic obstructive bronchitis. Client with
worsening hypoxia over time, if not treated ARDS have acute symptoms of hypoxia and typically need
aggressively. Pleuritic chest pain varies with respiration, large amounts of oxygen. Clients with asthma and
unlike the constant chest pain during an MI; so this client emphysema tend not to have chronic cough or peripheral
most likely isn’t having an MI. the client with TB typically has edema
a cough producing blood-tinged sputum. A sputum
culture should be obtained to confirm the nurse’s suspicions. Situation: Francis, age 46 is admitted to the hospital with
diagnosis of Chronic Lymphocytic Leukemia.
36. Nurse Oliver is working in a out patient clinic. He has been 40. The treatment for patients with leukemia is bone marrow
alerted that there is an outbreak of tuberculosis (TB). Which transplantation. Which statement about bone marrow
of the following clients entering the clinic today most likely to transplantation is not correct?
have TB?
A. The patient is under local anesthesia during the Answer: (A) Explain the risks of not having the surgery. The
procedure best initial response is to explain the risks of not having the
B. The aspirated bone marrow is mixed with heparin. surgery. If the client understands the risks but still refuses the
C. The aspiration site is the posterior or anterior iliac crest. nurse should notify the physician and the nurse supervisor
D. The recipient receives cyclophosphamide (Cytoxan) for and then record the client’s refusal in the nurses’ notes.
4 consecutive days before the procedure.
45. During the endorsement, which of the following clients
Answer: (A) The patient is under local anesthesia during the should the on-duty nurse assess first?
procedure. Before the procedure, the patient is administered
with drugs that would help to prevent infection and rejection A. The 58-year-old client who was admitted 2 days ago
of the transplanted cells such as antibiotics, cytotoxic, and with heart failure, blood pressure of 126/76 mm Hg,
corticosteroids. During the transplant, the patient is placed and a respiratory rate of 22 breaths/minute.
under general anesthesia. B. The 89-year-old client with end-stage right-sided heart
failure, blood pressure of 78/50 mm Hg, and a “do not
41. After several days of admission, Francis becomes resuscitate” order
disoriented and complains of frequent headaches. The nurse C. The 62-year-old client who was admitted 1 day ago
in-charge first action would be: with thrombophlebitis and is receiving L.V. heparin
D. The 75-year-old client who was admitted 1 hour ago
A. Call the physician with new-onset atrial fibrillation and is receiving L.V.
B. Document the patient’s status in his charts. dilitiazem (Cardizem)
C. Prepare oxygen treatment
D. Raise the side rails Answer: (D) The 75-year-old client who was admitted 1 hour
ago with new-onset atrial fibrillation and is receiving L.V.
Answer: (D) Raise the side rails. A patient who is disoriented dilitiazem (Cardizem). The client with atrial fibrillation has the
is at risk of falling out of bed. The initial action of the nurse greatest potential to become unstable and is on L.V.
should be raising the side rails to ensure patients safety medication that requires close monitoring. After assessing
this client, the nurse should assess the client
42. During routine care, Francis asks the nurse, “How can I be with thrombophlebitis who is receiving a heparin infusion,
anemic if this disease causes increased my white blood cell and then the 58- year-old client admitted 2 days ago with
production?” The nurse in-charge best response would be heart failure (his signs and symptoms are resolving and don’t
that the increased number of white blood cells (WBC) is: require immediate attention). The lowest priority is the 89-
year-old with end stage right-sided heart failure, who
A. Crowd red blood cells requires time-consuming supportive measures.
B. Are not responsible for the anemia.
C. Uses nutrients from other cells 46. Honey, a 23-year old client complains of substernal chest
D. Have an abnormally short life span of cells. pain and states that her heart feels like “it’s racing out of the
chest”. She reports no history of cardiac disorders. The nurse
Answer: (A) Crowd red blood cells. The excessive production attaches her to a cardiac monitor and notes sinus tachycardia
of white blood cells crowd out red blood cells production with a rate of 136beats/minutes. Breath sounds are clear and
which causes anemia to occur. the respiratory rate is 26 breaths/minutes. Which of the
following drugs should the nurse question the client about
43. Diagnostic assessment of Francis would probably not
using?
reveal:
A. Barbiturates
A. Predominance of lymhoblasts
B. Opioids
B. Leukocytosis
C. Cocaine
C. Abnormal blast cells in the bone marrow
D. Benzodiazepines
D. Elevated thrombocyte counts
Answer: (C) Cocaine. Because of the client’s age and
Answer: (B) Leukocytosis. Chronic Lymphocytic leukemia (CLL)
negative medical history, the nurse should question her
is characterized by increased production of leukocytes and
about cocaine use. Cocaine increases myocardial oxygen
lymphocytes resulting in leukocytosis, and proliferation of
consumption and can cause coronary artery spasm, leading
these cells within the bone marrow, spleen and liver.
to tachycardia, ventricular fibrillation, myocardial ischemia,
and myocardial infarction. Barbiturate overdose may trigger
44. Robert, a 57-year-old client with acute arterial occlusion
respiratory depression and slow pulse. Opioids can cause
of the left leg undergoes an emergency embolectomy. Six marked respiratory depression, while benzodiazepines can
hours later, the nurse isn’t able to obtain pulses in his left cause drowsiness and confusion
foot using Doppler ultrasound. The nurse
immediately notifies the physician, and asks her to prepare 47. A 51-year-old female client tells the nurse in-charge that
the client for surgery. As the nurse enters the client’s room to she has found a painless lump in her right breast during her
prepare him, he states that he won’t have any more surgery. monthly self-examination. Which assessment finding would
Which of the following is the best initial response by the strongly suggest that this client’s lump is cancerous?
nurse?
A. Eversion of the right nipple and mobile mass
A. Explain the risks of not having the surgery B. Nonmobile mass with irregular edges
B. Notifying the physician immediately C. Mobile mass that is soft and easily delineated
C. Notifying the nursing supervisor D. Nonpalpable right axillary lymph nodes
D. Recording the client’s refusal in the nurses’ notes
Answer: (B) Nonmobile mass with irregular edges. Breast
cancer tumors are fixed, hard, and poorly delineated with
irregular edges. A mobile mass that is soft and easily 51. A 37-year-old client with uterine cancer asks the nurse,
delineated is most often a fluid-filled benign cyst. Axillary “Which is the most common type of cancer in women?” The
lymph nodes may or may not be palpable on initial detection nurse replies that it’s breast cancer. Which type of cancer
of a cancerous mass. Nipple retraction — not eversion — causes the most deaths in women?
may be a sign of cancer
A. Breast cancer
48. A 35-year-old client with vaginal cancer asks the nurse, B. Lung cancer
“What is the usual treatment for this type of cancer?” Which C. Brain cancer
treatment should the nurse name? D. Colon and rectal cancer

A. Surgery Answer: (B) Lung cancer. Lung cancer is the most deadly type
B. Chemotherapy of cancer in both women and men. Breast cancer ranks
C. Radiation second in women, followed (in descending order) by colon
D. Immunotherapy and rectal cancer, pancreatic cancer, ovarian cancer, uterine
cancer, lymphoma, leukemia, liver cancer, brain cancer,
Answer: (C) Radiation. The usual treatment for vaginal stomach cancer, and multiple myeloma.
cancer is external or intravaginal radiation therapy. Less
often, surgery is performed. Chemotherapy typically is 52. Antonio with lung cancer develops Horner’s syndrome
prescribed only if vaginal cancer is diagnosed in an early when the tumor invades the ribs and affects the sympathetic
stage, which is rare. Immunotherapy isn’t used to treat nerve ganglia. When assessing for signs and symptoms of this
vaginal cancer. syndrome, the nurse should note:

49. Cristina undergoes a biopsy of a suspicious lesion. The A. miosis, partial eyelid ptosis, and anhidrosis on the
biopsy report classifies the lesion according to the TNM affected side of the face.
staging system as follows: TIS, N0, M0. What does this B. chest pain, dyspnea, cough, weight loss, and fever.
classification mean? C. arm and shoulder pain and atrophy of arm and hand
muscles, both on the affected side.
A. No evidence of primary tumor, no abnormal regional D. hoarseness and dysphagia.
lymph nodes, and no evidence of distant metastasis
B. Carcinoma in situ, no abnormal regional lymph nodes, Answer: (A) miosis, partial eyelid ptosis, and anhidrosis on the
and no evidence of distant metastasis affected side of the face. Horner’s syndrome, which occurs
C. Can’t assess tumor or regional lymph nodes and no when a lung tumor invades the ribs and affects the
evidence of metastasis sympathetic nerve ganglia, is characterized by miosis, partial
D. Carcinoma in situ, no demonstrable metastasis of the eyelid ptosis, and anhidrosis on the affected side of the face.
regional lymph nodes, and ascending degrees of distant Chest pain, dyspnea, cough, weight loss, and fever are
metastasis associated with pleural tumors. Arm and shoulder pain and
atrophy of the arm and hand muscles on the affected side
Answer: (B) Carcinoma in situ, no abnormal regional lymph suggest Pancoast’s tumor, a lung tumor involving the first
nodes, and no evidence of distant metastasis. TIS, N0, M0 thoracic and eighth cervical nerves within the brachial plexus.
denotes carcinoma in situ, no abnormal regional lymph nodes, Hoarseness in a client with lung cancer suggests that
and no evidence of distant metastasis. No evidence the tumor has extended to the recurrent laryngeal nerve;
of primary tumor, no abnormal regional lymph nodes, and no dysphagia suggests that the lung tumor is compressing the
evidence of distant metastasis is classified as T0, N0, M0. If esophagus.
the tumor and regional lymph nodes can’t be assessed and no
evidence of metastasis exists, the lesion is classified as TX, NX, 53. Vic asks the nurse what PSA is. The nurse should reply
M0. A progressive increase in tumor size, no demonstrable that it stands for:
metastasis of the regional lymph nodes, and
ascending degrees of distant metastasis is classified as T1, T2, A. prostate-specific antigen, which is used to screen for
T3, or T4; N0; and M1, M2, or M3. prostate cancer.
B. protein serum antigen, which is used to determine
50. Lydia undergoes a laryngectomy to treat laryngeal cancer. protein levels.
When teaching the client how to care for the neck stoma, the C. pneumococcal strep antigen, which is a bacteria that
nurse should include which instruction? causes pneumonia.
D. Papanicolaou-specific antigen, which is used to screen
A. “Keep the stoma uncovered.” for cervical cancer.
B. “Keep the stoma dry.”
C. “Have a family member perform stoma care initially Answer: (A) prostate-specific antigen, which is used to
until you get used to the procedure.” screen for prostate cancer. PSA stands for prostate-specific
D. “Keep the stoma moist.” antigen, which is used to screen for prostate cancer. The
other answers are incorrect
Answer: (D) “Keep the stoma moist.” The nurse should
instruct the client to keep the stoma moist, such as by 54. What is the most important postoperative instruction that
applying a thin layer of petroleum jelly around the nurse Kate must give a client who has just returned from the
edges, because a dry stoma may become irritated. The operating room after receiving a subarachnoid block?
nurse should recommend placing a stoma bib over the
stoma to filter and warm air before it enters the stoma. The A. “Avoid drinking liquids until the gag reflex returns.”
client should begin performing stoma care B. “Avoid eating milk products for 24 hours.”
without assistance as soon as possible to gain independence C. “Notify a nurse if you experience blood in your urine.”
in self-care activities. D. “Remain supine for the time specified by the physician.”
Answer: (D) “Remain supine for the time specified by the off) to others. The client must lie still during the MRI but can
physician.” The nurse should instruct the client to remain talk to those performing the test by way of the microphone
supine for the time specified by the physician. Local inside the scanner tunnel. The client should hear
anesthetics used in a subarachnoid block don’t alter the gag thumping sounds, which are caused by the sound waves
reflex. No interactions between local anesthetics and food thumping on the magnetic field
occur. Local anesthetics don’t cause hematuria.
59. Nurse Cecile is teaching a female client about preventing
55. A male client suspected of having colorectal cancer will osteoporosis. Which of the following teaching points is
require which diagnostic study to confirm the diagnosis? correct?

A. Stool Hematest A. Obtaining an X-ray of the bones every 3 years is


B. Carcinoembryonic antigen (CEA) recommended to detect bone loss.
C. Sigmoidoscopy B. To avoid fractures, the client should avoid strenuous
D. Abdominal computed tomography (CT) scan exercise.
C. The recommended daily allowance of calcium may be
Answer: (C) Sigmoidoscopy. Used to visualize the lower GI found in a wide variety of foods.
tract, sigmoidoscopy and proctoscopy aid in the detection of D. Obtaining the recommended daily allowance of calcium
two-thirds of all colorectal cancers. Stool Hematest detects requires taking a calcium supplement
blood, which is a sign of colorectal cancer; however, the test
doesn’t confirm the diagnosis. CEA may be elevated Answer: (C) The recommended daily allowance of calcium
in colorectal cancer but isn’t considered a confirming test. may be found in a wide variety of foods. Premenopausal
An abdominal CT scan is used to stage the presence of women require 1,000 mg of calcium per day. Postmenopausal
colorectal cancer. women require 1,500 mg per day. It’s often, though
not always, possible to get the recommended daily
56. During a breast examination, which finding most strongly requirement in the foods we eat. Supplements are available
suggests that the Luz has breast cancer? but not always necessary. Osteoporosis doesn’t show up on
ordinary X-rays until 30% of the bone loss has occurred. Bone
A. Slight asymmetry of the breasts. densitometry can detect bone loss of 3% or less. This test is
B. A fixed nodular mass with dimpling of the overlying sometimes recommended routinely for women over 35
skin who are at risk. Strenuous exercise won’t cause fractures.
C. Bloody discharge from the nipple .
D. Multiple firm, round, freely movable masses that 60. Before Jacob undergoes arthroscopy, the nurse reviews
change with the menstrual cycle the assessment findings for contraindications for this
procedure. Which finding is a contraindication?
Answer: (B) A fixed nodular mass with dimpling of the
overlying skin. A fixed nodular mass with dimpling of the A. Joint pain
overlying skin is common during late stages of breast cancer. B. Joint deformity
Many women have slightly asymmetrical breasts. Bloody C. Joint flexion of less than 50%
nipple discharge is a sign of intraductal papilloma, a benign D. Joint stiffness
condition. Multiple firm, round, freely movable masses that
change with the menstrual cycle indicate fibrocystic breasts, a Answer: (C) Joint flexion of less than 50%. Arthroscopy is
benign condition. contraindicated in clients with joint flexion of less than 50%
because of technical problems in inserting the
57. A female client with cancer is being evaluated for possible instrument into the joint to see it clearly. Other
metastasis. Which of the following is one of the most contraindications for this procedure include skin and wound
common metastasis sites for cancer cells? infections. Joint pain may be an indication, not
a contraindication, for arthroscopy. Joint deformity and joint
A. Liver stiffness aren’t contraindications for this procedure.
B. Colon
C. Reproductive tract 61. Mr. Rodriguez is admitted with severe pain in the knees.
D. White blood cells (WBCs) Which form of arthritis is characterized by urate deposits and
joint pain, usually in the feet and legs, and occurs primarily in
Answer: (A) Liver. The liver is one of the five most common
men over age 30?
cancer metastasis sites. The others are the lymph nodes, lung,
bone, and brain. The colon, reproductive tract, and WBCs are A. Septic arthritis
occasional metastasis sites. B. Traumatic arthritis
C. Intermittent arthritis
58. Nurse Mandy is preparing a client for magnetic resonance D. Gouty arthritis
imaging (MRI) to confirm or rule out a spinal cord lesion.
During the MRI scan, which of the following would pose a Answer: (D) Gouty arthritis. Gouty arthritis, a metabolic
threat to the client? disease, is characterized by urate deposits and pain in the
joints, especially those in the feet and legs. Urate deposits
A. The client lies still. don’t occur in septic or traumatic arthritis. Septic arthritis
B. The client asks questions. results from bacterial invasion of a joint and leads to
C. The client hears thumping sounds. inflammation of the synovial lining. Traumatic arthritis results
D. The client wears a watch and wedding band. from blunt trauma to a joint or ligament. Intermittent
arthritis is a rare, benign condition marked by
Answer: (D) The client wears a watch and wedding regular, recurrent joint effusions, especially in the knees.
band. During an MRI, the client should wear no metal
objects, such as jewelry, because the strong magnetic field
can pull on them, causing injury to the client and (if they fly
62. A heparin infusion at 1,500 unit/hour is ordered for a 64- 66. Mrs. Cruz uses a cane for assistance in walking. Which of
year-old client with stroke in evolution. The infusion contains the following statements is true about a cane or other
25,000 units of heparin in 500 ml of saline solution. How assistive devices?
many milliliters per hour should be given?
A. A walker is a better choice than a cane.
A. 15 ml/hour B. The cane should be used on the affected side
B. 30 ml/hour C. The cane should be used on the unaffected side
C. 45 ml/hour D. A client with osteoarthritis should be encouraged to
D. 50 ml/hour ambulate without the cane

Answer: (B) 30 ml/hour. An infusion prepared with 25,000 Answer: (C) The cane should be used on the unaffected
units of heparin in 500 ml of saline solution yields 50 units of side. A cane should be used on the unaffected side. A client
heparin per milliliter of solution. The equation is set up as 50 with osteoarthritis should be encouraged to ambulate with a
units times X (the unknown quantity) equals 1,500 units/hour, cane, walker, or other assistive device as needed; their use
X equals 30 ml/hour. takes weight and stress off joints.

63. A 76-year-old male client had a thromboembolic right 67. A male client with type 1 diabetes is scheduled to receive
stroke; his left arm is swollen. Which of the following 30 U of 70/30 insulin. There is no 70/30 insulin available. As a
conditions may cause swelling after a stroke? substitution, the nurse may give the client:

A. Elbow contracture secondary to spasticity A. 9 U regular insulin and 21 U neutral protamine


B. Loss of muscle contraction decreasing venous return Hagedorn (NPH).
C. Deep vein thrombosis (DVT) due to immobility of the B. 21 U regular insulin and 9 U NPH.
ipsilateral side C. 10 U regular insulin and 20 U NPH.
D. Hypoalbuminemia due to protein escaping from an D. 20 U regular insulin and 10 U NPH.
inflamed glomerulus
Answer: (A) 9 U regular insulin and 21 U neutral protamine
Answer: (B) Loss of muscle contraction decreasing venous Hagedorn (NPH). A 70/30 insulin preparation is 70% NPH
return. In clients with hemiplegia or hemiparesis loss of and 30% regular insulin. Therefore, a correct substitution
muscle contraction decreases venous return and may cause requires mixing 21 U of NPH and 9 U of regular insulin. The
swelling of the affected extremity. Contractures, or bony other choices are incorrect dosages for the prescribed
calcifications may occur with a stroke, but don’t appear with insulin
swelling. DVT may develop in clients with a stroke but is
more likely to occur in the lower extremities. A stroke 68. Nurse Len should expect to administer which medication
isn’t linked to protein loss to a client with gout?

64. Heberden’s nodes are a common sign of osteoarthritis. A. aspirin


Which of the following statement is correct about this B. furosemide (Lasix)
C. colchicines
deformity?
D. calcium gluconate (Kalcinate)
A. It appears only in men
B. It appears on the distal interphalangeal joint Answer: (C) colchicines. A disease characterized by joint
C. It appears on the proximal interphalangeal joint inflammation (especially in the great toe), gout is
D. It appears on the dorsolateral aspect of the caused by urate crystal deposits in the joints.
interphalangeal joint. The physician prescribes colchicine to reduce these
deposits and thus ease joint inflammation. Although
Answer: (B) It appears on the distal interphalangeal aspirin is used to reduce joint inflammation and pain in
joint. Heberden’s nodes appear on the distal interphalageal clients with osteoarthritis and rheumatoid arthritis, it
joint on both men and women. Bouchard’s node appears on isn’t indicated for gout because it has no effect on urate
the dorsolateral aspect of the proximal interphalangeal joint. crystal formation. Furosemide, a diuretic, doesn’t
relieve gout. Calcium gluconate is used to reverse a
65. Which of the following statements explains the main negative calcium balance and relieve muscle cramps,
difference between rheumatoid arthritis and osteoarthritis? not to treat gout.

A. Osteoarthritis is gender-specific, rheumatoid arthritis 69. Mr. Domingo with a history of hypertension is diagnosed
isn’t with primary hyperaldosteronism. This diagnosis indicates
B. Osteoarthritis is a localized disease rheumatoid that the client’s hypertension is caused by excessive hormone
arthritis is systemic secretion from which of the following glands?
C. Osteoarthritis is a systemic disease, rheumatoid arthritis
is localized A. Adrenal cortex
D. Osteoarthritis has dislocations and subluxations, B. Pancreas
rheumatoid arthritis doesn’t C. Adrenal medulla
D. Parathyroid
Answer: (B) Osteoarthritis is a localized disease rheumatoid
arthritis is systemic. Osteoarthritis is a localized disease, Answer: (A) Adrenal cortex. Excessive secretion of
rheumatoid arthritis is systemic. Osteoarthritis isn’t gender- aldosterone in the adrenal cortex is responsible for the
specific, but rheumatoid arthritis is. Clients have dislocations client’s hypertension. This hormone acts on the renal tubule,
and subluxations in both disorders. where it promotes reabsorption of sodium and excretion
of potassium and hydrogen ions. The pancreas mainly
secretes hormones involved in fuel metabolism. The adrenal
medulla secretes the catecholamines — epinephrine and
norepinephrine. The parathyroids secrete parathyroid span of red blood cells, glycosylated hemoglobin levels
hormone. provide information about blood glucose levels during the
previous 3 months. Fasting blood glucose and urine glucose
70. For a diabetic male client with a foot ulcer, the doctor levels only give information about glucose levels at the point
orders bed rest, a wetto- dry dressing change every shift, and in time when they were obtained. Serum fructosamine levels
blood glucose monitoring before meals and bedtime. Why provide information about blood glucose control over the
are wet-to-dry dressings used for this client? past 2 to 3 weeks.

A. They contain exudate and provide a moist wound 74. Nurse Trinity administered neutral protamine Hagedorn
environment. (NPH) insulin to a diabetic client at 7 a.m. At what time would
B. They protect the wound from mechanical trauma and the nurse expect the client to be most at risk for a
promote healing. hypoglycemic reaction?
C. They debride the wound and promote healing by
secondary intention. A. 10:00 am
D. They prevent the entrance of microorganisms and B. Noon
minimize wound discomfort. C. 4:00 pm
D. 10:00 pm
Answer: (C) They debride the wound and promote healing
by secondary intention. For this client, wet-to-dry dressings Answer: (C) 4:00 pm. NPH is an intermediate-acting insulin
are most appropriate because they clean the foot ulcer by that peaks 8 to 12 hours after administration. Because the
debriding exudate and necrotic tissue, thus promoting nurse administered NPH insulin at 7 a.m., the client is at
healing by secondary intention. Moist, transparent dressings greatest risk for hypoglycemia from 3 p.m. to 7 p.m.
contain exudate and provide a moist wound
environment. Hydrocolloid dressings prevent the entrance 75. The adrenal cortex is responsible for producing which
of microorganisms and minimize wound discomfort. Dry substances?
sterile dressings protect the wound from mechanical trauma
and promote healing. A. Glucocorticoids and androgens
B. Catecholamines and epinephrine
71. Nurse Zeny is caring for a client in acute addisonian crisis. C. Mineralocorticoids and catecholamines
Which laboratory data would the nurse expect to find? D. Norepinephrine and epinephrine

A. Hyperkalemia Answer: (A) Glucocorticoids and androgens. The adrenal


B. Reduced blood urea nitrogen (BUN) glands have two divisions, the cortex and medulla. The
C. Hypernatremia cortex produces three types of hormones:
D. Hyperglycemia glucocorticoids, mineralocorticoids, and androgens. The
medulla produces catecholamines — epinephrine and
Answer: (A) Hyperkalemia. In adrenal insufficiency, the client norepinephrine
has hyperkalemia due to reduced aldosterone secretion. BUN
increases as the glomerular filtration rate is reduced. 76. On the third day after a partial thyroidectomy, Proserfina
Hyponatremia is caused by reduced aldosterone secretion. exhibits muscle twitching and hyperirritability of the nervous
Reduced cortisol secretion leads to impaired system. When questioned, the client reports numbness and
glyconeogenesis and a reduction of glycogen in the liver and tingling of the mouth and fingertips. Suspecting a
muscle, causing hypoglycemia. lifethreatening electrolyte disturbance, the nurse notifies the
surgeon immediately. Which electrolyte disturbance most
72. A client is admitted for treatment of the syndrome of
commonly follows thyroid surgery?
inappropriate antidiuretic hormone (SIADH). Which nursing
intervention is appropriate? A. Hypocalcemia
B. Hyponatremia
A. Infusing I.V. fluids rapidly as ordered C. Hyperkalemia
B. Encouraging increased oral intake D. Hypermagnesemia
C. Restricting fluids
D. Administering glucose-containing I.V. fluids as ordered Answer: (A) Hypocalcemia. Hypocalcemia may follow thyroid
surgery if the parathyroid glands were removed accidentally.
Answer: (C) Restricting fluids. To reduce water retention in a Signs and symptoms of hypocalcemia may be delayed for up
client with the SIADH, the nurse should restrict fluids. to 7 days after surgery. Thyroid surgery doesn’t directly cause
Administering fluids by any route would further increase the serum sodium, potassium, or magnesium
client’s already heightened fluid load. abnormalities. Hyponatremia may occur if the client
inadvertently received too much fluid; however, this can
73. A female client tells nurse Nikki that she has been working happen to any surgical client receiving I.V. fluid therapy, not
hard for the last 3 months to control her type 2 diabetes just one recovering from thyroid surgery. Hyperkalemia
mellitus with diet and exercise. To determine the and hypermagnesemia usually are associated with reduced
effectiveness of the client’s efforts, the nurse should check: renal excretion of potassium and magnesium, not thyroid
surgery.
A. urine glucose level.
B. fasting blood glucose level. 77. Which laboratory test value is elevated in clients who
C. serum fructosamine level.
smoke and can’t be used as a general indicator of cancer?
D. glycosylated hemoglobin level.
A. Acid phosphatase level
Answer: (D) glycosylated hemoglobin level. Because some of B. Serum calcitonin level
the glucose in the bloodstream attaches to some of the C. Alkaline phosphatase level
hemoglobin and stays attached during the 120-day life
D. Carcinoembryonic antigen level 81. Nurse Marie is caring for a 32-year-old client admitted
with pernicious anemia. Which set of findings should the
Answer: (D) Carcinoembryonic antigen level. In clients who nurse expect when assessing the
smoke, the level of carcinoembryonic antigen is elevated.
Therefore, it can’t be used as a general indicator of client?
cancer. However, it is helpful in monitoring cancer
treatment because the level usually falls to normal within 1 A. Pallor, bradycardia, and reduced pulse pressure
month if treatment is successful. An elevated acid B. Pallor, tachycardia, and a sore tongue
phosphatase level may indicate prostate cancer. An elevated C. Sore tongue, dyspnea, and weight gain
alkaline phosphatase level may reflect bone metastasis. D. Angina, double vision, and anorexia
An elevated serum calcitonin level usually signals thyroid
cancer Answer: (B) Pallor, tachycardia, and a sore tongue. Pallor,
tachycardia, and a sore tongue are all characteristic findings
78. Francis with anemia has been admitted to the medical- in pernicious anemia. Other clinical manifestations
include anorexia; weight loss; a smooth, beefy red tongue; a
surgical unit. Which assessment findings are characteristic of
wide pulse pressure; palpitations; angina; weakness; fatigue;
iron-deficiency anemia?
and paresthesia of the hands and feet. Bradycardia, reduced
A. Nights sweats, weight loss, and diarrhea pulse pressure, weight gain, and double vision aren’t
B. Dyspnea, tachycardia, and pallor characteristic findings in pernicious anemia.
C. Nausea, vomiting, and anorexia
D. Itching, rash, and jaundice 82. After receiving a dose of penicillin, a client develops
dyspnea and hypotension. Nurse Celestina suspects the client
Answer: (B) Dyspnea, tachycardia, and pallor. Signs of iron- is experiencing anaphylactic shock. What should the nurse do
deficiency anemia include dyspnea, tachycardia, and pallor as first?
well as fatigue, listlessness, irritability, and headache.
Night sweats, weight loss, and diarrhea may signal acquired A. Page an anesthesiologist immediately and prepare to
immunodeficiency syndrome (AIDS). Nausea, vomiting, and intubate the client.
anorexia may be signs of hepatitis B. Itching, rash, and B. Administer epinephrine, as prescribed, and prepare to
jaundice may result from an allergic or hemolytic reaction. intubate the client if necessary.
C. Administer the antidote for penicillin, as prescribed, and
79. In teaching a female client who is HIV-positive about continue to monitor the client’s vital signs.
pregnancy, the nurse would know more teaching is necessary D. Insert an indwelling urinary catheter and begin to infuse
I.V. fluids as ordered.
when the client says:

A. The baby can get the virus from my placenta.” Answer: (B) Administer epinephrine, as prescribed, and
B. “I’m planning on starting on birth control pills.” prepare to intubate the client if necessary. To reverse
C. “Not everyone who has the virus gives birth to a baby anaphylactic shock, the nurse first should administer
who has the virus.” epinephrine, a potent bronchodilator as prescribed.
D. “I’ll need to have a C-section if I become pregnant and The physician is likely to order additional medications, such
have a baby.” as antihistamines and corticosteroids; if these medications
don’t relieve the respiratory compromise associated with
Ans wer: (D) “I’ll need to have a C-section if I become anaphylaxis, the nurse should prepare to intubate the client.
pregnant and have a baby.” The human immunodeficiency No antidote for penicillin exists; however, the nurse should
virus (HIV) is transmitted from mother to child via the continue to monitor the client’s vital signs. A client who
transplacental route, but a Cesarean section delivery isn’t remains hypotensive may need fluid resuscitation and fluid
necessary when the mother is HIV-positive. The use of intake and output monitoring; however, administering
birth control will prevent the conception of a child who might epinephrine is the first priority.
have HIV. It’s true that a mother who’s HIV positive can give
birth to a baby who’s HIV negative. 83. Mr. Marquez with rheumatoid arthritis is about to begin
aspirin therapy to reduce inflammation. When teaching the
80. When preparing Judy with acquired immunodeficiency client about aspirin, the nurse discusses adverse reactions to
syndrome (AIDS) for discharge to the home, the nurse should prolonged aspirin therapy. These include:
be sure to include which instruction?
A. weight gain.
A. “Put on disposable gloves before bathing.” B. fine motor tremors.
B. “Sterilize all plates and utensils in boiling water.” C. respiratory acidosis.
C. “Avoid sharing such articles as toothbrushes and D. bilateral hearing loss.
razors.”
D. “Avoid eating foods from serving dishes shared by other Answer: (D) bilateral hearing loss. Prolonged use of aspirin
family members.” and other salicylates sometimes causes bilateral hearing loss
of 30 to 40 decibels. Usually, this adverse effect resolves
Answer: (C) “Avoid sharing such articles as toothbrushes and within 2 weeks after the therapy is discontinued.
razors.” The human immunodeficiency virus (HIV), which Aspirin doesn’t lead to weight gain or fine motor tremors.
causes AIDS, is most concentrated in the blood. For this Large or toxic salicylate doses may cause respiratory alkalosis,
reason, the client shouldn’t share personal articles that may not respiratory acidosis.
be blood-contaminated, such as toothbrushes and razors,
with other family members. HIV isn’t transmitted by bathing 84. A 23-year-old client is diagnosed with human
or by eating from plates, utensils, or serving dishes used by immunodeficiency virus (HIV). After recovering from the
a person with AIDS. initial shock of the diagnosis, the client expresses a desire to
learn as much as possible about HIV and acquired
immunodeficiency syndrome (AIDS). When teaching the
client about the immune system, the nurse states that Answer: (D) Western blot test with ELISA. HIV infection is
adaptive immunity is provided by which type of white blood detected by analyzing blood for antibodies to HIV, which form
cell? approximately 2 to 12 weeks after exposure to HIV
and denote infection. The Western blot test —
A. Neutrophil electrophoresis of antibody proteins — is more than 98%
B. Basophil accurate in detecting HIV antibodies when used in
C. Monocyte conjunction with the ELISA. It isn’t specific when used alone.
D. Lymphocyte Erosette immunofluorescence is used to detect viruses in
general; it doesn’t confirm HIV infection. Quantification of T-
Answer: (D) Lymphocyte. The lymphocyte provides adaptive lymphocytes is a useful monitoring test but isn’t diagnostic
immunity — recognition of a foreign antigen and formation of for HIV. The ELISA test detects HIV antibody particles but may
memory cells against the antigen. Adaptive immunity is yield inaccurate results; a positive ELISA result must be
mediated by B and T lymphocytes and can be acquired confirmed by the Western blot test.
actively or passively. The neutrophil is crucial to
phagocytosis. The basophil plays an important role in the 88. A complete blood count is commonly performed before a
release of inflammatory mediators. The monocyte functions Joe goes into surgery. What does this test seek to identify?
in phagocytosis and monokine production.
A. Potential hepatic dysfunction indicated by decreased
85. In an individual with Sjögren’s syndrome, nursing care blood urea nitrogen (BUN) and creatinine levels
should focus on: B. Low levels of urine constituents normally excreted in
the urine
A. moisture replacement. C. Abnormally low hematocrit (HCT) and hemoglobin (Hb)
B. electrolyte balance. levels
C. nutritional supplementation. D. Electrolyte imbalance that could affect the blood’s
D. arrhythmia management. ability to coagulate properly

Answer: (A) moisture replacement. Sjogren’s syndrome is an Answer: (C) Abnormally low hematocrit (HCT) and
autoimmune disorder leading to progressive loss of hemoglobin (Hb) levels. Low preoperative HCT and Hb levels
lubrication of the skin, GI tract, ears, nose, and indicate the client may require a blood transfusion before
vagina. Moisture replacement is the mainstay of therapy. surgery. If the HCT and Hb levels decrease during surgery
Though malnutrition and electrolyte imbalance may occur as because of blood loss, the potential need for a transfusion
a result of Sjogren’s syndrome’s effect on the GI tract, it isn’t increases. Possible renal failure is indicated by elevated
the predominant problem. Arrhythmias aren’t a problem BUN or creatinine levels. Urine constituents aren’t found in
associated with Sjogren’s syndrome. the blood. Coagulation is determined by the presence of
appropriate clotting factors, not electrolytes.
86. During chemotherapy for lymphocytic leukemia, Mathew
develops abdominal pain, fever, and “horse barn” smelling 89. While monitoring a client for the development of
diarrhea. It would be most important for the nurse to advise disseminated intravascular coagulation (DIC), the nurse
the physician to order: should take note of what assessment parameters?

A. enzyme-linked immunosuppressant assay (ELISA) test. A. Platelet count, prothrombin time, and partial
B. electrolyte panel and hemogram. thromboplastin time
C. stool for Clostridium difficile test. B. Platelet count, blood glucose levels, and white blood
D. flat plate X-ray of the abdomen. cell (WBC) count
C. Thrombin time, calcium levels, and potassium levels
Answer: (C) stool for Clostridium difficile D. Fibrinogen level, WBC, and platelet count
test. Immunosuppressed clients — for example, clients
receiving chemotherapy, — are at risk for infection with C. Answer: (A) Platelet count, prothrombin time, and partial
difficile, which causes “horse barn” smelling diarrhea. thromboplastin time. The diagnosis of DIC is based on the
Successful treatment begins with an accurate diagnosis, results of laboratory studies of prothrombin time, platelet
which includes a stool test. The ELISA test is diagnostic for count, thrombin time, partial thromboplastin time, and
human immunodeficiency virus (HIV) and isn’t indicated fibrinogen level as well as client history and
in this case. An electrolyte panel and hemogram may be other assessment factors. Blood glucose levels, WBC count,
useful in the overall evaluation of a client but aren’t calcium levels, and potassium levels aren’t used to confirm a
diagnostic for specific causes of diarrhea. A flat plate of the diagnosis of DIC
abdomen may provide useful information about
bowel function but isn’t indicated in the case of “horse barn” 90. When taking a dietary history from a newly admitted
smelling diarrhea female client, Nurse Len should remember that which of the
following foods is a common allergen?
87. A male client seeks medical evaluation for fatigue, night
sweats, and a 20-lb weight loss in 6 weeks. To confirm that A. Bread
the client has been infected with the human B. Carrots
immunodeficiency virus (HIV), the nurse expects the C. Orange
D. Strawberries
physician to order:

A. E-rosette immunofluorescence. Answer: (D) Strawberries. Common food allergens include


B. quantification of T-lymphocytes. berries, peanuts, Brazil nuts, cashews, shellfish, and eggs.
C. enzyme-linked immunosorbent assay (ELISA). Bread, carrots, and oranges rarely cause allergic reactions.
D. Western blot test with ELISA.
91. Nurse John is caring for clients in the outpatient clinic. A. Assess for a bruit and a thrill.
Which of the following phone calls should the nurse return B. Warm the dialysate solution.
first? C. Position the client on the left side.
D. Insert a Foley catheter
A. A client with hepatitis A who states, “My arms and legs
are itching.” Answer: (B) Warm the dialysate solution. Cold dialysate
B. A client with cast on the right leg who states, “I have a increases discomfort. The solution should be warmed to body
funny feeling in my right leg.” temperature in warmer or heating pad; don’t use microwave
C. A client with osteomyelitis of the spine who states, “I oven.
am so nauseous that I can’t eat.”
D. A client with rheumatoid arthritis who states, “I am 96. Nurse Jannah teaches an elderly client with right-sided
having trouble sleeping.” weakness how to use cane. Which of the following behaviors,
if demonstrated by the client to the nurse, indicates that the
Answer: (B) A client with cast on the right leg who states, “I teaching was effective?
have a funny feeling in my right leg.” It may indicate
neurovascular compromise, requires immediate assessment A. The client holds the cane with his right hand, moves the
can forward followed by the right leg, and then moves
92. Nurse Sarah is caring for clients on the surgical floor and the left leg.
has just received report from the previous shift. Which of the B. The client holds the cane with his right hand, moves the
following clients should the nurse see first? cane forward followed by his left leg, and then moves
the right leg.
A. A 35-year-old admitted three hours ago with a gunshot C. The client holds the cane with his left hand, moves the
wound; 1.5 cm area of dark drainage noted on the cane forward followed by the right leg, and then
dressing. moves the left leg.
B. A 43-year-old who had a mastectomy two days ago; 23 D. The client holds the cane with his left hand, moves the
ml of serosanguinous fluid noted in the Jackson-Pratt cane forward followed by his left leg, and then moves
drain. the right leg.
C. A 59-year-old with a collapsed lung due to an accident;
no drainage noted in the previous eight hours. Answer: (C) The client holds the cane with his left hand,
D. A 62-year-old who had an abdominal-perineal resection moves the cane forward followed by the right leg, and then
three days ago; client complaints of chills. moves the left leg. The cane acts as a support and aids in
weight bearing for the weaker right leg.
Answer: (D) A 62-year-old who had an abdominal-perineal
resection three days ago; client complaints of chills. The client 97. An elderly client is admitted to the nursing home setting.
is at risk for peritonitis; should be assessed for further The client is occasionally confused and her gait is often
symptoms and infection. unsteady. Which of the following actions, if taken by the
nurse, is most appropriate?
93. Nurse Eve is caring for a client who had a thyroidectomy
12 hours ago for treatment of Grave’s disease. The nurse A. Ask the woman’s family to provide personal items
would be most concerned if which of the following was such as photos or mementos.
observed? B. Select a room with a bed by the door so the woman can
look down the hall.
A. Blood pressure 138/82, respirations 16, oral C. Suggest the woman eat her meals in the room with her
temperature 99 degrees Fahrenheit. roommate.
B. The client supports his head and neck when turning his D. Encourage the woman to ambulate in the halls twice a
head to the right. day.
C. The client spontaneously flexes his wrist when the
blood pressure is obtained. Answer: (A) Ask the woman’s family to provide personal
D. The client is drowsy and complains of sore throat. items such as photos or mementos.Photos and mementos
provide visual stimulation to reduce sensory deprivation.
Answer: (C) The client spontaneously flexes his wrist when
the blood pressure is obtained. Carpal spasms indicate 98. Nurse Evangeline teaches an elderly client how to use a
hypocalcemia. standard aluminum walker. Which of the following behaviors,
if demonstrated by the client, indicates that the nurse’s
94. Julius is admitted with complaints of severe pain in the
teaching was effective?
lower right quadrant of the abdomen. To assist with pain
relief, the nurse should take which of the following actions? A. The client slowly pushes the walker forward 12 inches,
then takes small steps forward while leaning on the
A. Encourage the client to change positions frequently in walker.
bed. B. The client lifts the walker, moves it forward 10 inches,
B. Administer Demerol 50 mg IM q 4 hours and PRN. and then takes several small steps forward.
C. Apply warmth to the abdomen with a heating pad. C. The client supports his weight on the walker while
D. Use comfort measures and pillows to position the advancing it forward, then takes small steps while
client. balancing on the walker.
D. The client slides the walker 18 inches forward, then
Answer: (D) Use comfort measures and pillows to position takes small steps while holding onto the walker for
the client.Using comfort measures and pillows to position balance.
the client is a non-pharmacological methods of pain relief

95. Nurse Tina prepares a client for peritoneal dialysis. Which


of the following actions should the nurse take first?
Answer: (B) The client lifts the walker, moves it forward 10 B. Maintain the function of the parathyroid glands.
inches, and then takes several small steps forward. A walker C. Block the formation of thyroxine by the thyroid gland.
needs to be picked up, placed down on all legs. D. Decrease the size and vascularity of the thyroid gland.

99. Nurse Deric is supervising a group of elderly clients in a Answer: (D) Decrease the size and vascularity of the thyroid
residential home setting. The nurse knows that the elderly gland. Lugol’s solution provides iodine, which aids in
are at greater risk of developing sensory deprivation for what decreasing the vascularity of the thyroid gland, which limits
reason? the risk of hemorrhage when surgery is performed.

A. Increased sensitivity to the side effects of medications. 4. Ricardo, was diagnosed with type I diabetes. The nurse is
B. Decreased visual, auditory, and gustatory abilities. aware that acute hypoglycemia also can develop in the client
C. Isolation from their families and familiar surroundings. who is diagnosed with:
D. Decrease musculoskeletal function and mobility. A. Liver disease
B. HypertensionX
Answer: (C) Isolation from their families and familiar C. Type 2 diabetes
surroundings. Gradual loss of sight, hearing, and taste D. Hyperthyroidism
interferes with normal functioning.
Answer: (A) Liver Disease. The client with liver disease has a
100. A male client with emphysema becomes restless and decreased ability to metabolize carbohydrates because of a
confused. What step should nurse Jasmine take next? decreased ability to form glycogen (glycogenesis) and to form
glucose from glycogen.
A. Encourage the client to perform pursed lip breathing.
B. Check the client’s temperature. 5. Tracy is receiving combination chemotherapy for
C. Assess the client’s potassium level. treatment of metastatic carcinoma. Nurse Ruby should
D. Increase the client’s oxygen flow rate. monitor the client for the systemic side effect of:
A. Ascites
Answer: (A) Encourage the client to perform pursed lip B. Nystagmus
breathing. Purse lip breathing prevents the collapse of lung C. Leukopenia
unit and helps client control rate and depth of breathing. D.
E.
F. Polycythemia
PNLE IV for Care of Answer: (C) Leukopenia. Leukopenia, a reduction in WBCs, is
Clients with Physiologic a systemic effect of chemotherapy as a result of
myelosuppression

and Psychosocial 6. Norma, with recent colostomy expresses concern about


the inability to control the passage of gas. Nurse Oliver
Alterations (Part 2) should suggest that the client plan to:
A. Eliminate foods high in cellulose.
B. Decrease fluid intake at meal times.
C. Avoid foods that in the past caused flatus.
D. Adhere to a bland diet prior to social events.
1. Randy has undergone kidney transplant, what assessment
would prompt Nurse Katrina to suspect organ rejection? Answer: (C) Avoid foods that in the past caused flatus. Foods
A. Sudden weight loss that bothered a person preoperatively will continue to do so
B. Polyuria after a colostomy.
C. Hypertension
D. Shock 7. Nurse Ron begins to teach a male client how to perform
colostomy irrigations. The nurse would evaluate that the
Answer: (C) Hypertension. Hypertension, along with fever, instructions were understood when the client states, “I
and tenderness over the grafted kidney, reflects acute should:
rejection. A. Lie on my left side while instilling the irrigating solution.”
B. Keep the irrigating container less than 18 inches above
2. The immediate objective of nursing care for an overweight, the stoma.”
mildly hypertensive male client with ureteral colic and C. Instill a minimum of 1200 ml of irrigating solution to
hematuria is to decrease: stimulate evacuation of the bowel.”
A. Pain D. Insert the irrigating catheter deeper into the stoma if
B. Weight cramping occurs during the procedure.”
C. Hematuria
D. Hypertension Answer: (B) Keep the irrigating container less than 18 inches
above the stoma.” This height permits the solution to flow
Answer: (A) Pain. Sharp, severe pain (renal colic) radiating slowly with little force so that excessive peristalsis is not
toward the genitalia and thigh is caused by uretheral immediately precipitated
distention and smooth muscle spasm; relief form pain is the
priority. 8. Patrick is in the oliguric phase of acute tubular necrosis and
is experiencing fluid and electrolyte imbalances. The client is
3. Matilda, with hyperthyroidism is to receive Lugol’s iodine somewhat confused and complains of nausea and muscle
solution before a subtotal thyroidectomy is performed. The weakness. As part of the prescribed therapy to correct this
nurse is aware that this medication is given to: electrolyte imbalance, the nurse would expect to:
A. Decrease the total basal metabolic rate.
A. Administer Kayexalate A. Oxygen at 1-2L/min is given to maintain the hypoxic
B. Restrict foods high in protein stimulus for breathing.
C. Increase oral intake of cheese and milk. B. Hypoxia stimulates the central chemoreceptors in the
D. Administer large amounts of normal saline via I.V. medulla that makes the client breath.
C. Oxygen is administered best using a non-rebreathing
Answer: (A) Administer Kayexalate. Kayexalate,a potassium mask
exchange resin, permits sodium to be exchanged for D. Blood gases are monitored using a pulse oximeter.
potassium in the intestine, reducing the serum
potassium level. Answer: (A) Oxygen at 1-2L/min is given to maintain the
hypoxic stimulus for breathing. COPD causes a chronic CO2
9. Mario has burn injury. After Forty48 hours, the physician retention that renders the medulla insensitive to the CO2
orders for Mario 2 liters of IV fluid to be administered q12 h. stimulation for breathing. The hypoxic state of the client then
The drop factor of the tubing is 10 gtt/ml. The nurse should becomes the stimulus for breathing. Giving the client oxygen
set the flow to provide: in low concentrations will maintain the client’s hypoxic drive.
A. X18 gtt/min
B. 28 gtt/min 14.Tonny has undergoes a left thoracotomy and a partial
C. 32 gtt/min pneumonectomy. Chest tubes are inserted, and one-bottle
D. 36 gtt/min water-seal drainage is instituted in the operating room. In the
postanesthesia care unit Tonny is placed in Fowler’s position
Answer:(B) 28 gtt/min. This is the correct flow rate; multiply on either his right side or on his back. The nurse is aware that
the amount to be infused (2000 ml) by the drop factor (10) this position:
and divide the result by the amount of time in minutes (12 A. Reduce incisional pain.
hours x 60 minutes B. Facilitate ventilation of the left lung.
C. Equalize pressure in the pleural space.
10.Terence suffered form burn injury. Using the rule of nines, D. Increase venous return
which has the largest percent of burns?
A. Face and neck Answer: (B) Facilitate ventilation of the left lung. Since only a
B. Right upper arm and penis partial pneumonectomy is done, there is a need to promote
C. Right thigh and penis expansion of this remaining Left lung by positioning the
D. Upper trunk client on the opposite unoperated side.
Answer: (D) Upper trunk. The percentage designated for
each burned part of the body using the rule of nines: Head 15.Kristine is scheduled for a bronchoscopy. When teaching
and neck 9%; Right upper extremity 9%; Left upper Kristine what to expect afterward, the nurse’s highest priority
extremity 9%; Anterior trunk 18%; Posterior trunk 18%; of information would be:
Right lower extremity 18%; Left lower extremity 18%; A. Food and fluids will be withheld for at least 2 hours.
Perineum 1%. B. Warm saline gargles will be done q 2h.
C. Coughing and deep-breathing exercises will be done
11. Herbert, a 45 year old construction engineer is brought to q2h.
the hospital unconscious after falling from a 2-story building. D. Only ice chips and cold liquids will be allowed initially.
When assessing the client, the nurse would be most
concerned if the assessment revealed: Answer: (A) Food and fluids will be withheld for at least 2
A. Reactive pupils hours. Prior to bronchoscopy, the doctors sprays the back
B. A depressed fontanel of the throat with anesthetic to minimize the gag reflex and
C. Bleeding from ears thus facilitate the insertion of the bronchoscope. Giving the
D. An elevated temperature client food and drink after the procedure without checking
on the return of the gag reflex can cause the client to
Answer: (C) Bleeding from ears. The nurse needs to perform aspirate. The gag reflex usually returns after two hours
a thorough assessment that could indicate alterations in
cerebral function, increased intracranial pressures, fractures 16.Nurse Tristan is caring for a male client in acute renal
and bleeding. Bleeding from the ears occurs only with basal failure. The nurse should expect hypertonic glucose, insulin
skull fractures that can easily contribute to increased infusions, and sodium bicarbonate to be used to treat:
intracranial pressure and brain herniation A. hypernatremia.
B. hypokalemia.
12. Nurse Sherry is teaching male client regarding his C. hyperkalemia.
permanent artificial pacemaker. Which information given by D. hypercalcemia.
the nurse shows her knowledge deficit about the artificial
cardiac pacemaker? Answer: (C) hyperkalemia. Hyperkalemia is a common
A. take the pulse rate once a day, in the morning upon complication of acute renal failure. It’s life-threatening if
awakening immediate action isn’t taken to reverse it.
B. May be allowed to use electrical appliances The administration of glucose and regular insulin, with
C. Have regular follow up care sodium bicarbonate if necessary, can temporarily prevent
D. May engage in contact sports cardiac arrest by moving potassium into the cells and
temporarily reducing serum potassium
Answer: (D) may engage in contact sports. The client should levels. Hypernatremia, hypokalemia, and hypercalcemia
be advised by the nurse to avoid contact sports. This will don’t usually occur with acute renal failure and aren’t
prevent trauma to the area of the pacemaker generator. treated with glucose, insulin, or sodium bicarbonate

13.The nurse is ware that the most relevant knowledge about 17.Ms. X has just been diagnosed with condylomata
oxygen administration to a male client with COPD is acuminata (genital warts). What information is appropriate to
tell this client?
A. This condition puts her at a higher risk for cervical out of the room, Katrina asks what dysplasia means. Which
cancer; therefore, she should have a Papanicolaou definition should the nurse provide?
(Pap) smear annually. A. Presence of completely undifferentiated tumor cells
B. The most common treatment is metronidazole (Flagyl), that don’t resemble cells of the tissues of their origin.
which should eradicate the problem within 7 to 10 days. B. Increase in the number of normal cells in a normal
C. The potential for transmission to her sexual partner will arrangement in a tissue or an organ.
be eliminated if condoms are used every time they have C. Replacement of one type of fully differentiated cell by
sexual intercourse. another in tissues where the second type normally isn’t
D. The human papillomavirus (HPV), which causes found.
condylomata acuminata, can’t be transmitted during D. Alteration in the size, shape, and organization of
oral sex. differentiated cells.

Answer: (A) This condition puts her at a higher risk for Answer: (D) Alteration in the size, shape, and organization
cervical cancer; therefore, she should have a Papanicolaou of differentiated cells. Dysplasia refers to an alteration in the
(Pap) smear annually. Women with condylomata acuminata size, shape, and organization of differentiated cells. The
are at risk for cancer of the cervix and vulva. Yearly Pap presence of completely undifferentiated tumor cells that
smears are very important for early detection. Because don’t resemble cells of the tissues of their origin is called
condylomata acuminata is a virus, there is no permanent anaplasia. An increase in the number of normal cells in
cure. Because condylomata acuminata can occur on the a normal arrangement in a tissue or an organ is called
vulva, a condom won’t protect sexual partners. HPV can be hyperplasia. Replacement of one type of fully differentiated
transmitted to other parts of the body, such as the mouth, cell by another in tissues where the second type normally
oropharynx, and larynx. isn’t found is called metaplasia.

18.Maritess was recently diagnosed with a genitourinary 21. During a routine checkup, Nurse Mariane assesses a male
problem and is being examined in the emergency department. client with acquired immunodeficiency syndrome (AIDS) for
When palpating the her kidneys, the nurse should keep which signs and symptoms of cancer. What is the most common
anatomical fact in mind? AIDS-related cancer?
A. The left kidney usually is slightly higher than the right A. Squamous cell carcinoma
one. B. Multiple myeloma
B. The kidneys are situated just above the adrenal glands. C. Leukemia
C. The average kidney is approximately 5 cm (2″) long and D. Kaposi’s sarcoma
2 to 3 cm (¾” to 1-1/8″) wide.
D. The kidneys lie between the 10th and 12th thoracic Answer: (D) Kaposi’s sarcoma. Kaposi’s sarcoma is the most
vertebrae. common cancer associated with AIDS. Squamous cell
carcinoma, multiple myeloma, and leukemia may occur in
Answer: (A) The left kidney usually is slightly higher than the anyone and aren’t associated specifically with AIDS.
right one. The left kidney usually is slightly higher than the
right one. An adrenal gland lies atop each kidney. The 22.Ricardo is scheduled for a prostatectomy, and the
average kidney measures approximately 11 cm (4-3/8″) long, anesthesiologist plans to use a spinal (subarachnoid) block
5 to 5.8 cm (2″ to 2¼”) wide, and 2.5 cm (1″) thick. The during surgery. In the operating room, the nurse positions the
kidneys are located retroperitoneally, in the posterior aspect client according to the anesthesiologist’s instructions. Why
of the abdomen, on either side of the vertebral column. They does the client require special positioning for this type of
lie between the 12th thoracic and 3rd lumbar vertebrae. anesthesia?
A. To prevent confusion
19.Jestoni with chronic renal failure (CRF) is admitted to the B. To prevent seizures
urology unit. The nurse is aware that the diagnostic test are C. To prevent cerebrospinal fluid (CSF) leakage
consistent with CRF if the result is: D. To prevent cardiac arrhythmias
A. Increased pH with decreased hydrogen ions.
B. Increased serum levels of potassium, magnesium, and Answer: (C) To prevent cerebrospinal fluid (CSF) leakage. The
calcium. client receiving a subarachnoid block requires
C. Blood urea nitrogen (BUN) 100 mg/dl and serum special positioning to prevent CSF leakage and headache and
creatinine 6.5 mg/ dl. to ensure proper anesthetic distribution. Proper positioning
D. Uric acid analysis 3.5 mg/dl and phenolsulfonphthalein doesn’t help prevent confusion, seizures, or cardiac
(PSP) excretion 75%. arrhythmias.

Answer: (C) Blood urea nitrogen (BUN) 100 mg/dl and serum 23.A male client had a nephrectomy 2 days ago and is now
creatinine 6.5 mg/dl. The normal BUN level ranges 8 to 23 complaining of abdominal pressure and nausea. The first
mg/dl; the normal serum creatinine level ranges from 0.7 to nursing action should be to:
1.5 mg/dl. The test results in option C are abnormally A. Auscultate bowel sounds.
elevated, reflecting CRF and the kidneys’ decreased ability to B. Palpate the abdomen.
remove nonprotein nitrogen waste from the blood. CRF C. Change the client’s position.
causes decreased pH and increased hydrogen ions — not vice D. Insert a rectal tube.
versa. CRF also increases serum levels of potassium,
magnesium, and phosphorous, and decreases serum levels of Answer: (A) Auscultate bowel sounds. If abdominal distention
calcium. A uric acid analysis of 3.5 mg/dl falls within the is accompanied by nausea, the nurse must first auscultate
normal range of 2.7 to 7.7 mg/dl; PSP excretion of 75% bowel sounds. If bowel sounds are absent, the nurse should
also falls with the normal range of 60% to 75%. suspect gastric or small intestine dilation and these findings
must be reported to the physician. Palpation should be
20. Katrina has an abnormal result on a Papanicolaou test. avoided postoperatively with abdominal distention. If
After admitting that she read her chart while the nurse was
peristalsis is absent, changing positions and inserting a rectal are normal. The client’s rectal temperature isn’t significantly
tube won’t relieve the client’s discomfort. elevated and probably results from the fluid volume deficit.

24.Wilfredo with a recent history of rectal bleeding is being 28. Mr. Mendoza who has suffered a cerebrovascular
prepared for a colonoscopy. How should the nurse Patricia accident (CVA) is too weak to move on his own. To help the
position the client for this test initially? client avoid pressure ulcers, Nurse Celia should:
A. Lying on the right side with legs straight A. Turn him frequently.
B. Lying on the left side with knees bent B. Perform passive range-of-motion (ROM) exercises.
C. Prone with the torso elevated C. Reduce the client’s fluid intake.
D. Bent over with hands touching the floor D. Encourage the client to use a footboard.

Answer: (B) Lying on the left side with knees bent. For a Answer: (A) Turn him frequently. The most important
colonoscopy, the nurse initially should position the client on intervention to prevent pressure ulcers is frequent position
the left side with knees bent. Placing the client on the right changes, which relieve pressure on the skin and underlying
side with legs straight, prone with the torso elevated, or bent tissues. If pressure isn’t relieved, capillaries become
over with hands touching the floor wouldn’t allow proper occluded, reducing circulation and oxygenation of the tissues
visualization of the large intestine. and resulting in cell death and ulcer formation. During passive
ROM exercises, the nurse moves each joint through its range
25.A male client with inflammatory bowel disease undergoes of movement, which improves joint mobility and circulation
an ileostomy. On the first day after surgery, Nurse Oliver to the affected area but doesn’t prevent pressure ulcers.
notes that the client’s stoma appears dusky. How should the Adequate hydration is necessary to maintain healthy skin
nurse interpret this finding? and ensure tissue repair. A footboard prevents plantar flexion
A. Blood supply to the stoma has been interrupted. and footdrop by maintaining the foot in a dorsiflexed position.
B. This is a normal finding 1 day after surgery.
C. The ostomy bag should be adjusted. 29.Nurse Maria plans to administer dexamethasone cream to
D. An intestinal obstruction has occurred. a female client who has dermatitis over the anterior chest.
How should the nurse apply this topical agent?
Answer: (A) Blood supply to the stoma has been A. With a circular motion, to enhance absorption.
interrupted. An ileostomy stoma forms as the ileum is B. With an upward motion, to increase blood supply to the
brought through the abdominal wall to the surface skin, affected area
creating an artificial opening for waste elimination. The C. In long, even, outward, and downward strokes in the
stoma should appear cherry red, indicating adequate arterial direction of hair growth
perfusion. A dusky stoma suggests decreased perfusion, D. In long, even, outward, and upward strokes in the
which may result from interruption of the stoma’s blood direction opposite hair growth
supply and may lead to tissue damage or necrosis. A dusky
stoma isn’t a normal finding. Adjusting the ostomy bag Answer: (C) In long, even, outward, and downward strokes in
wouldn’t affect stoma color, which depends on blood supply the direction of hair growth. When applying a topical agent,
to the area. An intestinal obstruction also wouldn’t change the nurse should begin at the midline and use long, even,
stoma color. outward, and downward strokes in the direction of hair
growth. This application pattern reduces the risk of
26.Anthony suffers burns on the legs, which nursing follicle irritation and skin inflammation.
intervention helps prevent contractures?
A. Applying knee splints 30.Nurse Kate is aware that one of the following classes of
B. Elevating the foot of the bed medication protect the ischemic myocardium by blocking
C. Hyperextending the client’s palms catecholamines and sympathetic nerve stimulation is:
D. Performing shoulder range-of-motion exercises A. Beta -adrenergic blockers
B. Calcium channel blocker
Answer: (A) Applying knee splints. Applying knee splints C. Narcotics
prevents leg contractures by holding the joints in a position of D. Nitrates
function. Elevating the foot of the bed can’t
prevent contractures because this action doesn’t hold the Answer: (A) Beta -adrenergic blockers. Beta-adrenergic
joints in a position of function. Hyperextending a body part blockers work by blocking beta receptors in the myocardium,
for an extended time is inappropriate because it can cause reducing the response to catecholamines and sympathetic
contractures. Performing shoulder range-of-motion exercises nerve stimulation. They protect the myocardium, helping
can prevent contractures in the shoulders, but not in the legs. to reduce the risk of another infraction by decreasing
myocardial oxygen demand. Calcium channel blockers reduce
27.Nurse Ron is assessing a client admitted with second- and the workload of the heart by decreasing the heart rate.
third-degree burns on the face, arms, and chest. Which Narcotics reduce myocardial oxygen demand, promote
finding indicates a potential problem? vasodilation, and decrease anxiety. Nitrates reduce
A. Partial pressure of arterial oxygen (PaO2) value of 80 myocardial oxygen consumption bt decreasing left ventricular
mm Hg. end diastolic pressure (preload) and systemic vascular
B. Urine output of 20 ml/hour. resistance (afterload).
C. White pulmonary secretions.
D. Rectal temperature of 100.6° F (38° C). 31.A male client has jugular distention. On what position
should the nurse place the head of the bed to obtain the
Answer: (B) Urine output of 20 ml/hour. A urine output of most accurate reading of jugular vein distention?
less than 40 ml/hour in a client with burns indicates a fluid A. High Fowler’s
volume deficit. This client’s PaO2 value falls within the normal B. Raised 10 degrees
range (80 to 100 mm Hg). White pulmonary secretions also C. Raised 30 degrees
D. Supine position
compressions, the cardiac monitor shows narrow QRS
Answer: (C) Raised 30 degrees. Jugular venous pressure is complexes and a heart rate of beats/minute with a palpable
measured with a centimeter ruler to obtain the vertical pulse. Which of the following actions
distance between the sternal angle and the point of highest should the nurse take first?
pulsation with the head of the bed inclined between 15 to A. Start an L.V. line and administer amiodarone
30 degrees. Increased pressure can’t be seen when the client (Cardarone), 300 mg L.V. over 10 minutes.
is supine or when the head of the bed is raised 10 degrees B. Check endotracheal tube placement.
because the point that marks the pressure level is above the C. Obtain an arterial blood gas (ABG) sample.
jaw (therefore, not visible). In high Fowler’s position, the D. Administer atropine, 1 mg L.V.
veins would be barely discernible above the clavicle.
Answer: (B) Check endotracheal tube placement. ET tube
32.The nurse is aware that one of the following classes of placement should be confirmed as soon as the client arrives
medications maximizes cardiac performance in clients with in the emergency department. Once the airways is
heart failure by increasing ventricular contractility? secured, oxygenation and ventilation should be confirmed
A. Beta-adrenergic blockers using an end-tidal carbon dioxide monitor and pulse oximetry.
B. Calcium channel blocker Next, the nurse should make sure L.V. access is established. If
C. Diuretics the client experiences symptomatic bradycardia, atropine is
D. Inotropic agents administered as ordered 0.5 to 1 mg every 3 to 5 minutes to a
total of 3 mg. Then the nurse should try to find the cause
Answer: (D) Inotropic agents. Inotropic agents are of the client’s arrest by obtaining an ABG sample.
administered to increase the force of the heart’s contractions, Amiodarone is indicated for ventricular tachycardia,
thereby increasing ventricular contractility and ultimately ventricular fibrillation and atrial flutter – not symptomatic
increasing cardiac output. Beta-adrenergic blockers and bradycardia.
calcium channel blockers decrease the heart rate and
ultimately decreased the workload of the heart. Diuretics are 36. After cardiac surgery, a client’s blood pressure measures
administered to decrease the overall vascular volume, also 126/80 mm Hg. Nurse Katrina determines that mean arterial
decreasing the workload of the heart. pressure (MAP) is which of the following?
A. 46 mm Hg
33.A male client has a reduced serum high-density lipoprotein B. 80 mm Hg
(HDL) level and an elevated low-density lipoprotein (LDL) C. 95 mm Hg
level. Which of the following dietary modifications is not D. 90 mm Hg
appropriate for this client?
A. Fiber intake of 25 to 30 g daily Answer: (C) 95 mm Hg. Use the following formula to calculate
B. Less than 30% of calories form fat MAP
C. Cholesterol intake of less than 300 mg daily  MAP = systolic + 2 (diastolic) /3
D. Less than 10% of calories from saturated fat  MAP=[126 mm Hg + 2 (80 mm Hg) ]/3
 MAP=286 mm HG/ 3
Answer: (B) Less than 30% of calories form fat. A client with  MAP=95 mm Hg
low serum HDL and high serum LDL levels should get less than
30% of daily calories from fat. The other modifications 37. A female client arrives at the emergency department with
are appropriate for this client. chest and stomach pain and a report of black tarry stool for
several months. Which of the following order should the
34. A 37-year-old male client was admitted to the coronary nurse Oliver anticipate?
care unit (CCU) 2 days ago with an acute myocardial A. Cardiac monitor, oxygen, creatine kinase and lactate
infarction. Which of the following actions would breach the dehydrogenase levels
client confidentiality? B. Prothrombin time, partial thromboplastin time,
A. The CCU nurse gives a verbal report to the nurse on the fibrinogen and fibrin split product values.
telemetry unit before transferring the client to that unit C. Electrocardiogram, complete blood count, testing for
B. The CCU nurse notifies the on-call physician about a occult blood, comprehensive serum metabolic panel.
change in the client’s condition D. Electroencephalogram, alkaline phosphatase and
C. The emergency department nurse calls up the latest aspartate aminotransferase levels, basic serum
electrocardiogram results to check the client’s progress. metabolic panel
D. At the client’s request, the CCU nurse updates the
client’s wife on his condition Answer: (C) Electrocardiogram, complete blood count, testing
for occult blood, comprehensive serum metabolic panel. An
Answer: (C) The emergency department nurse calls up the electrocardiogram evaluates the complaints of chest
latest electrocardiogram results to check the client’s pain, laboratory tests determines anemia, and the stool test
progress. The emergency department nurse is no longer for occult blood determines blood in the stool. Cardiac
directly involved with the client’s care and thus has no legal monitoring, oxygen, and creatine kinase and lactate
right to information about his present condition. Anyone dehydrogenase levels are appropriate for a cardiac primary
directly involved in his care (such as the telemetry nurse and problem. A basic metabolic panel and alkaline phosphatase
the on-call physician) has the right to information about his and aspartate aminotransferase levels assess liver function.
condition. Because the client requested that the nurse Prothrombin time, partial thromboplastin time, fibrinogen
update his wife on his condition, doing so doesn’t breach and fibrin split products are measured to verify bleeding
confidentiality dyscrasias, An electroencephalogram evaluates brain
electrical activity.
35. A male client arriving in the emergency department is
receiving cardiopulmonary resuscitation from paramedics 38. Macario had coronary artery bypass graft (CABG) surgery
who are giving ventilations through an endotracheal (ET) tube 3 days ago. Which of the following conditions is suspected by
that they placed in the client’s home. During a pause in
the nurse when a decrease in platelet count from 230,000 ul as von Willebrand’s disease is a type of hemophilia and isn’t
to 5,000 ul is noted? linked to SLE. Moderate to severe anemia is associated with
A. Pancytopenia SLE, not polycythermia. Dressler’s syndrome is pericarditis
B. Idiopathic thrombocytopemic purpura (ITP) that occurs after a myocardial infarction and isn’t linked to
C. Disseminated intravascular coagulation (DIC) SLE.
D. Heparin-associated thrombosis and thrombocytopenia
(HATT) 43. The nurse is aware that the following symptoms is most
commonly an early indication of stage 1 Hodgkin’s disease?
Answer: (D) Heparin-associated thrombosis and A. Pericarditis
thrombocytopenia (HATT). HATT may occur after CABG B. Night sweat
surgery due to heparin use during surgery. Although DIC and C. Splenomegaly
ITP cause platelet aggregation and bleeding, neither is D. Persistent hypothermia
common in a client after revascularization
surgery. Pancytopenia is a reduction in all blood cells. Answer: (B) Night sweat. In stage 1, symptoms include a
single enlarged lymph node (usually), unexplained fever,
39. Which of the following drugs would be ordered by the night sweats, malaise, and generalized pruritis. Although
physician to improve the platelet count in a male client with splenomegaly may be present in some clients, night sweats
idiopathic thrombocytopenic purpura (ITP)? are generally more prevalent. Pericarditis isn’t associated
A. Acetylsalicylic acid (ASA) with Hodgkin’s disease, nor is hypothermia. Moreover,
B. Corticosteroids splenomegaly and pericarditis aren’t symptoms. Persistent
C. Methotrezate hypothermia is associated with Hodgkin’s but isn’t an early
D. Vitamin K sign of the disease.

Answer: (B) Corticosteroids. Corticosteroid therapy can 44. Francis with leukemia has neutropenia. Which of the
decrease antibody production and phagocytosis of the following functions must frequently assessed?
antibody-coated platelets, retaining more A. Blood pressure
functioning platelets. Methotrexate can cause B. Bowel sounds
thrombocytopenia. Vitamin K is used to treat an excessive C. Heart sounds
anticoagulate state from warfarin overload, and D. Breath sounds
ASA decreases platelet aggregation.
Answer: (D) Breath sounds. Pneumonia, both viral and
40. A female client is scheduled to receive a heart valve fungal, is a common cause of death in clients with
replacement with a porcine valve. Which of the following neutropenia, so frequent assessment of respiratory rate
types of transplant is this? and breath sounds is required. Although assessing blood
A. Allogeneic pressure, bowel sounds, and heart sounds is important, it
B. Autologous won’t help detect pneumonia
C. Syngeneic
D. Xenogeneic 45. The nurse knows that neurologic complications of
multiple myeloma (MM) usually involve which of the
Answer: (D) Xenogeneic. An xenogeneic transplant is following body system?
between is between human and another species. A syngeneic A. Brain
transplant is between identical twins, allogeneic transplant is B. Muscle spasm
between two humans, and autologous is a transplant from C. Renal dysfunction
the same individual. D. Myocardial irritability

41. Marco falls off his bicycle and injuries his ankle. Which of Answer: (B) Muscle spasm. Back pain or paresthesia in the
the following actions shows the initial response to the injury lower extremities may indicate impending spinal cord
in the extrinsic pathway? compression from a spinal tumor. This should be recognized
A. Release of Calcium and treated promptly as progression of the tumor may result
B. Release of tissue thromboplastin in paraplegia. The other options, which reflect parts of the
C. Conversion of factors XII to factor XIIa nervous system, aren’t usually affected by MM.
D. Conversion of factor VIII to factor VIIIa
46. Nurse Patricia is aware that the average length of time
Answer: (B). Tissue thromboplastin is released when from human immunodeficiency virus (HIV) infection to the
damaged tissue comes in contact with clotting factors. development of acquired immunodeficiency syndrome (AIDS)?
Calcium is released to assist the conversion of factors X to Xa. A. Less than 5 years
Conversion of factors XII to XIIa and VIII to VIII a are part of B. 5 to 7 years
the intrinsic pathway C. 10 years
D. More than 10 years
42. Instructions for a client with systemic lupus
erythematosus (SLE) would include information about which Answer: (C)10 years. Epidermiologic studies show the
of the following blood dyscrasias? average time from initial contact with HIV to the
A. Dressler’s syndrome development of AIDS is 10 years
B. Polycythemia
C. Essential thrombocytopenia 47. An 18-year-old male client admitted with heat stroke
D. Von Willebrand’s disease begins to show signs of disseminated intravascular
coagulation (DIC). Which of the following laboratory findings
Answer: (C) Essential thrombocytopenia. Essential is most consistent with DIC?
thrombocytopenia is linked to immunologic disorders, such as A. Low platelet count
SLE and human immunodeficiency vitus. The disorder known B. Elevated fibrinogen levels
C. Low levels of fibrin degradation products B. “You should not worry about her hair, just be glad that
D. Reduced prothrombin time she is alive”.
C. “Yes it is upsetting. But try to cover up your feelings
Answer: (A) Low platelet count. In DIC, platelets and clotting when you are with her or else she may be upset”.
factors are consumed, resulting in microthrombi and D. “This is only temporary; Stacy will re-grow new hair in
excessive bleeding. As clots form, fibrinogen levels decrease 3-6 months, but may be different in texture”.
and the prothrombin time increases. Fibrin
degeneration products increase as fibrinolysis takes places. Answer: (D) “This is only temporary; Stacy will re-grow new
hair in 3-6 months, but may be different in texture”. This is
48. Mario comes to the clinic complaining of fever, drenching the appropriate response. The nurse should help the mother
night sweats, and unexplained weight loss over the past 3 how to cope with her own feelings regarding the child’s
months. Physical examination reveals a single enlarged disease so as not to affect the child negatively. When the hair
supraclavicular lymph node. Which of the following is the grows back, it is still of the same color and texture.
most probable diagnosis?
A. Influenza 52. Stacy has beginning stomatitis. To promote oral hygiene
B. Sickle cell anemia and comfort, the nurse in-charge should:
C. Leukemia A. Provide frequent mouthwash with normal saline.
D. Hodgkin’s disease B. Apply viscous Lidocaine to oral ulcers as needed.
C. Use lemon glycerine swabs every 2 hours.
Answer: (D) Hodgkin’s disease. Hodgkin’s disease typically D. Rinse mouth with Hydrogen Peroxide.
causes fever night sweats, weight loss, and lymph mode
enlargement. Influenza doesn’t last for months. Clients with Answer: (B) Apply viscous Lidocaine to oral ulcers as
sickle cell anemia manifest signs and symptoms of needed. Stomatitis can cause pain and this can be relieved by
chronic anemia with pallor of the mucous membrane, applying topical anesthetics such as lidocaine before mouth
fatigue, and decreased tolerance for exercise; they don’t care. When the patient is already comfortable, the nurse can
show fever, night sweats, weight loss or lymph node proceed with providing the patient with oral rinses of saline
enlargement. Leukemia doesn’t cause lymph solution mixed with equal part of water or hydrogen peroxide
node enlargement mixed water in 1:3 concentrations to promote oral hygiene.
49. A male client with a gunshot wound requires an Every 2-4 hours.
emergency blood transfusion. His blood type is AB negative.
Which blood type would be the safest for him to receive?
A. AB Rh-Xpositive
B. A Rh-positive 53. During the administration of chemotherapy agents, Nurse
C. A Rh-negative Oliver observed that the IV site is red and swollen, when the
D. O Rh-positive IV is touched Stacy shouts in pain. The first nursing action to
take is:
Answer: (C) A Rh-negative. Human blood can sometimes A. Notify the physician
contain an inherited D antigen. Persons with the D antigen B. Flush the IV line with saline solution
have Rh-positive blood type; those lacking the antigen have C. Immediately discontinue the infusion
Rh-negative blood. It’s important that a person with D. Apply an ice pack to the site, followed by warm
Rhnegative blood receives Rh-negative blood. If Rh-positive compress.
blood is administered to an Rh-negative person, the
recipient develops anti-Rh agglutinins, and sub sequent Answer: (C) Immediately discontinue the infusion. Edema or
transfusions with Rh-positive blood may cause serious swelling at the IV site is a sign that the needle has been
reactions with clumping and hemolysis of red blood cells dislodged and the IV solution is leaking into the tissues
Situation: Stacy is diagnosed with acute lymphoid leukemia causing the edema. The patient feels pain as the nerves are
(ALL) and beginning chemotherapy. irritated by pressure and the IV solution. The first action of
the nurse would be to discontinue the infusion right away to
50. Stacy is discharged from the hospital following her prevent further edema and other complication
chemotherapy treatments. Which statement of Stacy’s
mother indicated that she understands when she will contact 54. The term “blue bloater” refers to a male client which of
the physician? the following conditions?
A. “I should contact the physician if Stacy has difficulty in A. Adult respiratory distress syndrome (ARDS)
sleeping”. B. Asthma
B. “I will call my doctor if Stacy has persistent vomiting C. Chronic obstructive bronchitis
and diarrhea”. D. Emphysema
C. “My physician should be called if Stacy is irritable and
unhappy”. Answer: (C) Chronic obstructive bronchitis. Clients with
D. “Should Stacy have continued hair loss, I need to call chronic obstructive bronchitis appear bloated; they have
the doctor”. large barrel chest and peripheral edema, cyanotic nail beds,
and at times, circumoral cyanosis. Clients with ARDS are
Answer: (B) “I will call my doctor if Stacy has persistent acutely short of breath and frequently need intubation for
vomiting and diarrhea”. Persistent (more than 24 hours) mechanical ventilation and large amount of oxygen. Clients
vomiting, anorexia, and diarrhea are signs of toxicity and the with asthma don’t exhibit characteristics of chronic disease,
patient should stop the medication and notify the health and clients with emphysema appear pink and cachectic.
care provider. The other manifestations are expected side
effects of chemotherapy 55. The term “pink puffer” refers to the female client with
which of the following conditions?
51. Stacy’s mother states to the nurse that it is hard to see A. Adult respiratory distress syndrome (ARDS)
Stacy with no hair. The best response for the nurse is: B. Asthma
A. “Stacy looks very nice wearing a hat”.
C. Chronic obstructive bronchitis Liver cirrhosis is a chronic and irreversible disease of the liver
D. Emphysema characterized by generalized inflammation and fibrosis of the
liver tissues.
Answer: (D) Emphysema. Because of the large amount of
energy it takes to breathe, clients with emphysema are 60.The biopsy of Mr. Gonzales confirms the diagnosis of
usually cachectic. They’re pink and usually breathe through cirrhosis. Mr. Gonzales is at increased risk for excessive
pursed lips, hence the term “puffer.” Clients with ARDS are bleeding primarily because of:
usually acutely short of breath. Clients with asthma don’t A. Impaired clotting mechanism
have any particular characteristics, and clients with chronic B. Varix formation
obstructive bronchitis are bloated and cyanotic in appearance. C. Inadequate nutrition
D. Trauma of invasive procedure
56. Jose is in danger of respiratory arrest following the
administration of a narcotic analgesic. An arterial blood gas Answer: (A) Impaired clotting mechanism. Cirrhosis of the
value is obtained. Nurse Oliver would expect the paco2 to be liver results in decreased Vitamin K absorption and formation
which of the following values? of clotting factors resulting in impaired clotting mechanism.
A. 15 mm Hg
B. 30 mm Hg 61. Mr. Gonzales develops hepatic encephalopathy. Which
C. 40 mm Hg clinical manifestation is most common with this condition?
D. 80 mm Hg A. Increased urine output
B. Altered level of consciousness
Answer: D 80 mm Hg. A client about to go into respiratory C. Decreased tendon reflex
arrest will have inefficient ventilation and will be retaining D. Hypotension
carbon dioxide. The value expected would be around 80 mm
Hg. All other values are lower than expected. Answer: (B) Altered level of consciousness. Changes in
behavior and level of consciousness are the first sins of
57. Timothy’s arterial blood gas (ABG) results are as follows; hepatic encephalopathy. Hepatic encephalopathy is caused
pH 7.16; Paco2 80 mm Hg; Pao2 46 mm Hg; HCO3- 24mEq/L; by liver failure and develops when the liver is unable to
Sao2 81%. This ABG result represents which of the following convert protein metabolic product ammonia to urea. This
conditions? results in accumulation of ammonia and other toxic in the
A. Metabolic acidosis blood that damages the cells.
B. Metabolic alkalosis
C. Respiratory acidosis 62. When Mr. Gonzales regained consciousness, the physician
D. Respirator y alkalosis orders 50 ml of Lactose p.o. every 2 hours. Mr. Gozales
develops diarrhea. The nurse best action would be:
Answer: (C) Respiratory acidosis. Because Paco2 is high at 80 A. “I’ll see if your physician is in the hospital”.
mm Hg and the metabolic measure, HCO3- is normal, the B. “Maybe your reacting to the drug; I will withhold the
client has respiratory acidosis. The pH is less than 7.35, next dose”.
academic, which eliminates metabolic and C. “I’ll lower the dosage as ordered so the drug causes
respiratory alkalosis as possibilities. If the HCO3- was below only 2 to 4 stools a day”.
22 mEq/L the client would have metabolic acidosis. D. “Frequently, bowel movements are needed to reduce
sodium level”.
58. Norma has started a new drug for hypertension. Thirty
minutes after she takes the drug, she develops chest Answer: (C) “I’ll lower the dosage as ordered so the drug
tightness and becomes short of breath and tachypneic. She causes only 2 to 4 stools a day”. Lactulose is given to a
has a decreased level of consciousness. These signs indicate patients with hepatic encephalopathy to reduce absorption of
which of the following conditions? ammonia in the intestines by binding with ammonia and
A. Asthma attack promoting more frequent bowel movements. If the patient
B. Pulmonary embolism experience diarrhea, it indicates over dosage and the nurse
C. Respiratory failure must reduce the amount of medication given to the patient.
D. Rheumatoid arthritis The stool will be mashy or soft. Lactulose is also very sweet
and may cause cramping and bloating.
Answer: (C) Respiratory failure. The client was reacting to the
drug with respiratory signs of impending anaphylaxis, which 63. Which of the following groups of symptoms indicates a
could lead to eventually respiratory failure. Although the ruptured abdominal aortic aneurysm?
signs are also related to an asthma attack or a A. Lower back pain, increased blood pressure, decreased
pulmonary embolism, consider the new drug first. re blood cell (RBC) count, increased white blood (WBC)
Rheumatoid arthritis doesn’t manifest these signs. count.
B. Severe lower back pain, decreased blood pressure,
Situation: Mr. Gonzales was admitted to the hospital with decreased RBC count, increased WBC count.
ascites and jaundice. To rule out cirrhosis of the liver: C. Severe lower back pain, decreased blood pressure,
decreased RBC count, decreased RBC count, decreased
59. Which laboratory test indicates liver cirrhosis? WBC count.
A. Decreased red blood cell count D. Intermitted lower back pain, decreased blood pressure,
B. Decreased serum acid phosphate level decreased RBC count, increased WBC count.
C. Elevated white blood cell count
D. Elevated serum aminotransferase Answer: (B) Severe lower back pain, decreased blood
pressure, decreased RBC count, increased WBC count.Severe
Answer: (D) Elevated serum aminotransferase. Hepatic cell lower back pain indicates an aneurysm rupture, secondary to
death causes release of liver enzymes pressure being applied within the abdominal cavity.
alanine aminotransferase (ALT), aspartate aminotransferase When ruptured occurs, the pain is constant because it can’t
(AST) and lactate dehydrogenase (LDH) into the circulation.
be alleviated until the aneurysm is repaired. Blood pressure 68. Nurse Rose is aware that the statement that best explains
decreases due to the loss of blood. After the aneurysm why furosemide (Lasix) is administered to treat hypertension
ruptures, the vasculature is interrupted and blood volume is is:
lost, so blood pressure wouldn’t increase. For the A. It dilates peripheral blood vessels.
same reason, the RBC count is decreased – not increased. The B. It decreases sympathetic cardioacceleration.
WBC count increases as cell migrate to the site of injury. C. It inhibits the angiotensin-coverting enzymes
D. It inhibits reabsorption of sodium and water in the
64. After undergoing a cardiac catheterization, Tracy has a loop of Henle
large puddle of blood under his buttocks. Which of the
following steps should the nurse take first? Answer: (D) It inhibits reabsorption of sodium and water in
A. Call for help. the loop of Henle. Furosemide is a loop diuretic that inhibits
B. Obtain vital signs sodium and water reabsorption in the loop Henle, thereby
C. Ask the client to “lift up” causing a decrease in blood pressure. Vasodilators cause
D. Apply gloves and assess the groin site dilation of peripheral blood vessels, directly relaxing vascular
smooth muscle and decreasing blood pressure. Adrenergic
Answer: (D) Apply gloves and assess the groin site. Observing blockers decrease sympathetic cardioacceleration
standard precautions is the first priority when dealing with and decrease blood pressure. Angiotensin-converting enzyme
any blood fluid. Assessment of the groin site is the inhibitors decrease blood pressure due to their action on
second priority. This establishes where the blood is coming angiotensin.
from and determineshow much blood has been lost. The goal .
in this situation is to stop the bleeding. The nurse would call 69. Nurse Nikki knows that laboratory results supports the
for help if it were warranted after the assessment of the diagnosis of systemic lupus erythematosus (SLE) is:
situation. After determining the extent of the bleeding, vital A. Elavated serum complement level
signs assessment is important. The nurse should never move B. Thrombocytosis, elevated sedimentation rate
the client, in case a clot has formed. Moving can disturb the C. Pancytopenia, elevated antinuclear antibody (ANA)
clot and cause rebleeding. titer
D. Leukocysis, elevated blood urea nitrogen (BUN) and
65. Which of the following treatment is a suitable surgical creatinine levels
intervention for a client with unstable angina? Answer: (C) Pancytopenia, elevated antinuclear antibody
A. Cardiac catheterization (ANA) titer. Laboratory findings for clients with SLE usually
B. Echocardiogram show pancytopenia, elevated ANA titer, and decreased
C. Nitroglycerin serum complement levels. Clients may have elevated BUN
D. Percutaneous transluminal coronary angioplasty (PTCA) and creatinine levels from nephritis, but the increase does
not indicate SLE
Answer: (D) Percutaneous transluminal coronary angioplasty
(PTCA). PTCA can alleviate the blockage and restore blood 70. Arnold, a 19-year-old client with a mild concussion is
flow and oxygenation. An echocardiogram is a noninvasive discharged from the emergency department. Before
diagnosis test. Nitroglycerin is an oral sublingual medication. discharge, he complains of a headache. When offered
Cardiac catheterization is a diagnostic tool – not a treatment acetaminophen, his mother tells the nurse the headache is
severe and she would like her son to have something stronger.
66. The nurse is aware that the following terms used to Which of the following responses by the nurse is appropriate?
describe reduced cardiac output and perfusion impairment A. “Your son had a mild concussion, acetaminophen is
due to ineffective pumping of the heart is: strong enough.”
A. Anaphylactic shock B. “Aspirin is avoided because of the danger of Reye’s
B. Cardiogenic shock syndrome in children or young adults.”
C. Distributive shock C. “Narcotics are avoided after a head injury because
D. Myocardial infarction (MI) they may hide a worsening condition.”
D. Stronger medications may lead to vomiting, which
Answer: (B) Cardiogenic shock. Cardiogenic shock is shock increases the intracarnial pressure (ICP).”
related to ineffective pumping of the heart. Anaphylactic
shock results from an allergic reaction. Distributive shock Answer: (C) Narcotics are avoided after a head injury because
results from changes in the intravascular volume distribution they may hide a worsening condition. Narcotics may mask
and is usually associated with increased cardiac output. MI changes in the level of consciousness that indicate increased
isn’t a shock state, though a severe MI can lead to shock. ICP and shouldn’t acetaminophen is strong enough ignores
the mother’s question and therefore isn’t appropriate.
67. A client with hypertension ask the nurse which factors can Aspirin is contraindicated in conditions that may have
cause blood pressure to drop to normal levels? bleeding, such as trauma, and for children or young adults
A. Kidneys’ excretion to sodium only. with viral illnesses due to the danger of Reye’s syndrome.
B. Kidneys’ retention of sodium and water Stronger medications may not necessarily lead to vomiting
C. Kidneys’ excretion of sodium and water but will sedate the client, thereby masking changes in his
D. Kidneys’ retention of sodium and excretion of water level of consciousness.

Answer: (C) Kidneys’ excretion of sodium and water. The 71. When evaluating an arterial blood gas from a male client
kidneys respond to rise in blood pressure by excreting sodium with a subdural hematoma, the nurse notes the Paco2 is 30
and excess water. This response ultimately affects sysmolic mm Hg. Which of the following responses best describes the
blood pressure by regulating blood volume. Sodium or water result?
retention would only further increase blood pressure. Sodium A. Appropriate; lowering carbon dioxide (CO2) reduces
and water travel together across the membrane in the intracranial pressure (ICP)
kidneys; one can’t travel without the other. B. Emergent; the client is poorly oxygenated
C. Normal
D. Significant; the client has alveolar hypoventilation is caused by severe hyperthyroidism. Tibial myxedema,
peripheral mucinous edema involving the lower leg,
Answer: (A) Appropriate; lowering carbon dioxide (CO2) is associated with hypothyroidism but isn’t life-threatening
reduces intracranial pressure (ICP). A normal Paco2 value is
35 to 45 mm Hg CO2 has vasodilating properties; therefore, 76. Nurse Sugar is assessing a client with Cushing’s syndrome.
lowering Paco2 through hyperventilation will lower ICP Which observation should the nurse report to the physician
caused by dilated cerebral vessels. Oxygenation is evaluated immediately?
through Pao2 and oxygen saturation. Alveolar A. Pitting edema of the legs
hypoventilation would be reflected in an increased Paco2. B. An irregular apical pulse
C. Dry mucous membranes
72. When prioritizing care, which of the following clients D. Frequent urination
should the nurse Olivia assess first?
A. A 17-year-old clients 24-hours postappendectomy Answer: (B) An irregular apical pulse. Because Cushing’s
B. A 33-year-old client with a recent diagnosis of Guillain- syndrome causes aldosterone overproduction, which
Barre syndrome increases urinary potassium loss, the disorder may lead to
C. A 50-year-old client 3 days postmyocardial infarction hypokalemia. Therefore, the nurse should immediately report
D. A 50-year-old client with diverticulitis signs and symptoms of hypokalemia, such as an irregular
apical pulse, to the physician. Edema is an expected finding
Answer: (B) A 33-year-old client with a recent diagnosis of because aldosterone overproduction causes sodium and fluid
Guillain-Barre syndrome . Guillain-Barre syndrome is retention. Dry mucous membranes and frequent urination
characterized by ascending paralysis and potential respiratory signal dehydration, which isn’t associated with Cushing’s
failure. The order of client assessment should follow client syndrome.
priorities, with disorder of airways, breathing, and
then circulation. There’s no information to suggest the 77. Cyrill with severe head trauma sustained in a car accident
postmyocardial infarction client has an arrhythmia or other is admitted to the intensive care unit. Thirty-six hours later,
complication. There’s no evidence to suggest hemorrhage or the client’s urine output suddenly rises above 200 ml/hour,
perforation for the remaining clients as a priority of care. leading the nurse to suspect diabetes insipidus. Which
laboratory findings support the nurse’s suspicion of diabetes
73. JP has been diagnosed with gout and wants to know why insipidus?
colchicine is used in the treatment of gout. Which of the A. Above-normal urine and serum osmolality levels
following actions of colchicines explains why it’s effective for B. Below-normal urine and serum osmolality levels
gout? C. Above-normal urine osmolality level, below-normal
A. Replaces estrogen serum osmolality level
B. Decreases infection D. Below-normal urine osmolality level, above-normal
C. Decreases inflammation serum osmolality level
D. Decreases bone demineralization
Answer: (D) Below-normal urine osmolality level, above-
Answer: (C) Decreases inflammation. Then action of normal serum osmolality level. In diabetes insipidus,
colchicines is to decrease inflammation by reducing the excessive polyuria causes dilute urine, resulting in a below-
migration of leukocytes to synovial fluid. Colchicine normal urine osmolality level. At the same time, polyuria
doesn’t replace estrogen, decrease infection, or decrease depletes the body of water, causing dehydration that leads to
bone demineralization. an above-normal serum osmolality level. For the same
reasons, diabetes insipidus doesn’t cause above-normal urine
74. Norma asks for information about osteoarthritis. Which of osmolality or below-normal serum osmolality levels.
the following statements about osteoarthritis is correct?
A. Osteoarthritis is rarely debilitating 78. Jomari is diagnosed with hyperosmolar hyperglycemic
B. Osteoarthritis is a rare form of arthritis nonketotic syndrome (HHNS) is stabilized and prepared for
C. Osteoarthritis is the most common form of arthritis discharge. When preparing the client for discharge and home
D. Osteoarthritis afflicts people over 60 management, which of the following statements indicates
that the client understands her condition and how to control
Answer: (C) Osteoarthritis is the most common form of it?
arthritis. Osteoarthritis is the most common form of arthritis A. “I can avoid getting sick by not becoming dehydrated
and can be extremely debilitating. It can afflict people of any and by paying attention to my need to urinate, drink,
age, although most are elderly or eat more than usual.”
B. “If I experience trembling, weakness, and headache, I
75. Ruby is receiving thyroid replacement therapy develops should drink a glass of soda that contains sugar.”
the flu and forgets to take her thyroid replacement medicine. C. “I will have to monitor my blood glucose level closely
The nurse understands that skipping this medication will put and notify the physician if it’s constantly elevated.”
the client at risk for developing which of the following D. “If I begin to feel especially hungry and thirsty, I’ll eat a
lifethreatening complications? snack high in carbohydrates.”
A. Exophthalmos
B. Thyroid storm Answer: (A) “I can avoid getting sick by not becoming
C. Myxedema coma dehydrated and by paying attention to my need to urinate,
D. Tibial myxedema drink, or eat more than usual.” Inadequate fluid intake during
hyperglycemic episodes often leads to HHNS. By recognizing
Answer: (C) Myxedema coma. Myxedema coma, severe the signs of hyperglycemia (polyuria, polydipsia, and
hypothyroidism, is a life-threatening condition that may polyphagia) and increasing fluid intake, the client
develop if thyroid replacement medication isn’t may prevent HHNS. Drinking a glass of nondiet soda would be
taken. Exophthalmos, protrusion of the eyeballs, is seen appropriate for hypoglycemia. A client whose diabetes is
with hyperthyroidism. Thyroid storm is life-threatening but controlled with oral antidiabetic agents usually doesn’t need
to monitor blood glucose levels. A highcarbohydrate diet Answer: (D) Performing capillary glucose testing every 4
would exacerbate the client’s condition, particularly if fluid hours. The nurse should perform capillary glucose testing
intake is low. every 4 hours because excess cortisol may cause insulin
resistance, placing the client at risk for hyperglycemia. Urine
79. A 66-year-old client has been complaining of sleeping ketone testing isn’t indicated because the client does secrete
more, increased urination, anorexia, weakness, irritability, insulin and, therefore, isn’t at risk for ketosis. Urine specific
depression, and bone pain that interferes with her going gravity isn’t indicated because although fluid balance can be
outdoors. Based on these assessment findings, the nurse compromised, it usually isn’t dangerously
would suspect which of the following disorders? imbalanced. Temperature regulation may be affected by
A. Diabetes mellitus excess cortisol and isn’t an accurate indicator of infection.
B. Diabetes insipidus
C. Hypoparathyroidism 83. Capillary glucose monitoring is being performed every 4
D. Hyperparathyroidism hours for a client diagnosed with diabetic ketoacidosis. Insulin
is administered using a scale of regular insulin according to
Answer: (D) Hyperparathyroidism. Hyperparathyroidism is glucose results. At 2 p.m., the client has a capillary glucose
most common in older women and is characterized by bone level of 250 mg/dl for which he receives 8 U of regular insulin.
pain and weakness from excess parathyroid hormone (PTH). Nurse Mariner should expect the dose’s:
Clients also exhibit hypercaliuria-causing polyuria. A. onset to be at 2 p.m. and its peak to be at 3 p.m.
While clients with diabetes mellitus and diabetes insipidus B. onset to be at 2:15 p.m. and its peak to be at 3 p.m.
also have polyuria, they don’t have bone pain and increased C. onset to be at 2:30 p.m. and its peak to be at 4 p.m.
sleeping. Hypoparathyroidism is characterized by urinary D. onset to be at 4 p.m. and its peak to be at 6 p.m.
frequency rather than polyuria.
Answer: (C) onset to be at 2:30 p.m. and its peak to be at 4
80. Nurse Lourdes is teaching a client recovering from p.m.. Regular insulin, which is a short-acting insulin, has an
addisonian crisis about the need to take fludrocortisone onset of 15 to 30 minutes and a peak of 2 to 4 hours. Because
acetate and hydrocortisone at home. Which statement by the the nurse gave the insulin at 2 p.m., the expected onset
client indicates an understanding of the instructions? would be from 2:15 p.m. to 2:30 p.m. and the peak from 4
A. “I’ll take my hydrocortisone in the late afternoon, p.m. to 6 p.m.
before dinner.”
B. “I’ll take all of my hydrocortisone in the morning, right 84. The physician orders laboratory tests to confirm
after I wake up.” hyperthyroidism in a female client with classic signs and
C. “I’ll take two-thirds of the dose when I wake up and symptoms of this disorder. Which test result would confirm
one-third in the late afternoon.” the diagnosis?
D. “I’ll take the entire dose at bedtime.” A. No increase in the thyroid-stimulating hormone (TSH)
level after 30 minutes during the TSH stimulation test
Answer: (C) “I’ll take two-thirds of the dose when I wake up B. A decreased TSH level
and one-third in the late afternoon.” Hydrocortisone, a C. An increase in the TSH level after 30 minutes during the
glucocorticoid, should be administered according to a TSH stimulation test
schedule that closely reflects the body’s own secretion of this D. Below-normal levels of serum triiodothyronine (T3) and
hormone; therefore, two-thirds of the dose of hydrocortisone serum thyroxine (T4) as detected by radioimmunoassay
should be taken in the morning and one-third in the late
afternoon. This dosage schedule reduces adverse effects. Answer: (A) No increase in the thyroid-stimulating hormone
(TSH) level after 30 minutes during the TSH stimulation
81. Which of the following laboratory test results would test. In the TSH test, failure of the TSH level to rise after
suggest to the nurse Len that a client has a corticotropin- 30 minutes confirms hyperthyroidism. A decreased TSH level
secreting pituitary adenoma? indicates a pituitary deficiency of this hormone. Below-
A. High corticotropin and low cortisol levels normal levels of T3 and T4, as detected by radioimmunoassay,
B. Low corticotropin and high cortisol levels signal hypothyroidism. A below-normal T4 level also occurs in
C. High corticotropin and high cortisol levels malnutrition and liver disease and may result
D. Low corticotropin and low cortisol levels from administration of phenytoin and certain other drugs.

Answer: (C) High corticotropin and high cortisol levels. A 85. Rico with diabetes mellitus must learn how to self-
corticotropin-secreting pituitary tumor would cause administer insulin. The physician has prescribed 10 U of U-
high corticotropin and high cortisol levels. A high 100 regular insulin and 35 U of U-100 isophane insulin
corticotropin level with a low cortisol level and a low suspension (NPH) to be taken before breakfast. When
corticotropin level with a low cortisol level would teaching the client how to select and rotate insulin injection
be associated with hypocortisolism. Low corticotropin and sites, the nurse should provide which instruction?
high cortisol levels would be seen if there was a primary A. “Inject insulin into healthy tissue with large blood
defect in the adrenal glands vessels and nerves.”
B. “Rotate injection sites within the same anatomic
82. A male client is scheduled for a transsphenoidal region, not among different regions.”
hypophysectomy to remove a pituitary tumor. Preoperatively, C. “Administer insulin into areas of scar tissue or
the nurse should assess for potential complications by doing hypotrophy whenever possible.”
which of the following? D. “Administer insulin into sites above muscles that you
A. Testing for ketones in the urine plan to exercise heavily later that day.”
B. Testing urine specific gravity
C. Checking temperature every 4 hours Answer: (B) “Rotate injection sites within the same anatomic
D. Performing capillary glucose testing every 4 hours region, not among different regions.” The nurse should
instruct the client to rotate injection sites within the same
anatomic region. Rotating sites among different regions may
cause excessive day-to-day variations in the blood glucose
level; also, insulin absorption differs from one region to the Answer: (B) Calcium and phosphorous. In osteoporosis, bones
next. Insulin should be injected only into healthy tissue lose calcium and phosphate salts, becoming porous, brittle,
lacking large blood vessels, nerves, or scar tissue or other and abnormally vulnerable to fracture. Sodium and potassium
deviations. Injecting insulin into areas of hypertrophy may aren’t involved in the development of osteoporosis.
delay absorption. The client shouldn’t inject insulin into areas
of lipodystrophy (such as hypertrophy or atrophy); to prevent 90. Johnny a firefighter was involved in extinguishing a house
lipodystrophy, the client should rotate injection sites fire and is being treated to smoke inhalation. He develops
systematically. Exercise speeds drug absorption, so the client severe hypoxia 48 hours after the incident, requiring
shouldn’t inject insulin into sites above muscles that will be intubation and mechanical ventilation. He most likely has
exercised heavily. developed which of the following conditions?
A. Adult respiratory distress syndrome (ARDS)
86. Nurse Sarah expects to note an elevated serum glucose B. Atelectasis
level in a client with hyperosmolar hyperglycemic nonketotic C. Bronchitis
syndrome (HHNS). Which other laboratory finding should the D. Pneumonia
nurse anticipate?
A. Elevated serum acetone level Answer: (A) Adult respiratory distress syndrome
B. Serum ketone bodies (ARDS). Severe hypoxia after smoke inhalation is typically
C. Serum alkalosis related to ARDS. The other conditions listed aren’t typically
D. Below-normal serum potassium level associated with smoke inhalation and severe hypoxia.

Answer: (D) Below-normal serum potassium level. A client 91. A 67-year-old client develops acute shortness of breath
with HHNS has an overall body deficit of potassium resulting and progressive hypoxia requiring right femur. The hypoxia
from diuresis, which occurs secondary to the was probably caused by which of the following conditions?
hyperosmolar, hyperglycemic state caused by the relative A. Asthma attack
insulin deficiency. An elevated serum acetone level and B. Atelectasis
serum ketone bodies are characteristic of diabetic C. Bronchitis
ketoacidosis. Metabolic acidosis, not serum alkalosis, may D. Fat embolism
occur in HHNS.
Answer: (D) Fat embolism. Long bone fractures are correlated
87. For a client with Graves’ disease, which nursing with fat emboli, whichcause shortness of breath and hypoxia.
intervention promotes comfort? It’s unlikely the client has developed asthma or bronchitis
A. Restricting intake of oral fluids without a previous history. He could develop atelectasis but it
B. Placing extra blankets on the client’s bed typically doesn’t produce progressive hypoxia.
C. Limiting intake of high-carbohydrate foods
D. Maintaining room temperature in the low-normal range 92. A client with shortness of breath has decreased to absent
breath sounds on the right side, from the apex to the base.
Answer: (D) Maintaining room temperature in the low- Which of the following conditions would best explain this?
normal range. Graves’ disease causes signs and symptoms A. Acute asthma
of hypermetabolism, such as heat intolerance, diaphoresis, B. Chronic bronchitis
excessive thirst and appetite, and weight loss. To reduce heat C. Pneumonia
intolerance and diaphoresis, the nurse should keep the D. Spontaneous pneumothorax
client’s room temperature in the low-normal range. To
replace fluids lost via diaphoresis, the nurse Answer: (D) Spontaneous pneumothorax. A spontaneous
should encourage, not restrict, intake of oral fluids. Placing pneumothorax occurs when the client’s lung collapses,
extra blankets on the bed of a client with heat intolerance causing an acute decreased in the amount of functional
would cause discomfort. To provide needed energy and lung used in oxygenation. The sudden collapse was the
calories, the nurse should encourage the client to eat high- cause of his chest pain and shortness of breath. An asthma
carbohydrate foods. attack would show wheezing breath sounds, and bronchitis
would have rhonchi. Pneumonia would have bronchial
88. Patrick is treated in the emergency department for a breath sounds over the area of consolidation
Colles’ fracture sustained during a fall. What is a Colles’
fracture? 93. A 62-year-old male client was in a motor vehicle accident
A. Fracture of the distal radius as an unrestrained driver. He’s now in the emergency
B. Fracture of the olecranon department complaining of difficulty of breathing and chest
C. Fracture of the humerus pain. On auscultation of his lung field, no breath sounds are
D. Fracture of the carpal scaphoid present in the upper lobe. This client may have which of the
following conditions?
Answer: (A) Fracture of the distal radius. Colles’ fracture is a A. Bronchitis
fracture of the distal radius, such as from a fall on an B. Pneumonia
outstretched hand. It’s most common in women. C. Pneumothorax
Colles’ fracture doesn’t refer to a fracture of the olecranon, D. Tuberculosis (TB)
humerus, or carpal scaphoid
Answer: (C) Pneumothorax. From the trauma the client
89. Cleo is diagnosed with osteoporosis. Which electrolytes experienced, it’s unlikely he has bronchitis, pneumonia, or
are involved in the development of this disorder? TB; rhonchi with bronchitis, bronchial breath sounds with TB
A. Calcium and sodium would be heard.
B. Calcium and phosphorous
C. Phosphorous and potassium 94. If a client requires a pneumonectomy, what fills the area
D. Potassium and sodium of the thoracic cavity?
A. The space remains filled with air only Answer: (B) 21. 3000 x 10 divided by 24 x 60
B. The surgeon fills the space with a gel
C. Serous fluids fills the space and consolidates the region 99. Mickey, a 6-year-old child with a congenital heart disorder
D. The tissue from the other lung grows over to the other is admitted with congestive heart failure. Digoxin (lanoxin)
side 0.12 mg is ordered for the child. The bottle of Lanoxin
contains .05 mg of Lanoxin in 1 ml of solution. What amount
Answer: (C) Serous fluids fills the space and consolidates the should the nurse administer to the child?
region. Serous fluid fills the space and eventually A. 1.2 ml
consolidates, preventing extensive mediastinal shift of the B. 2.4 ml
heart and remaining lung. Air can’t be left in the space. C. 3.5 ml
There’s no gel that can be placed in the pleural space. The D. 4.2 ml
tissue from the other lung can’t cross the
mediastinum, although a temporary mediastinal shift exits Answer: (B) 2.4 ml. .05 mg/ 1 ml = .12mg/ x ml, .05x = .12, x =
until the space is filled. 2.4 ml.

95. Hemoptysis may be present in the client with a 100. Nurse Alexandra teaches a client about elastic stockings.
pulmonary embolism because of which of the following Which of the following statements, if made by the client,
reasons? indicates to the nurse that the teaching was successful?
A. Alveolar damage in the infracted area A. “I will wear the stockings until the physician tells me to
B. Involvement of major blood vessels in the occluded area remove them.”
C. Loss of lung parenchyma B. “I should wear the stockings even when I am sleep.”
D. Loss of lung tissue C. “Every four hours I should remove the stockings for a
half hour.”
Answer: (A) Alveolar damage in the infracted area. The D. “I should put on the stockings before getting out of
infracted area produces alveolar damage that can lead bed in the morning.”
to the production of bloody sputum, sometimes in massive Answer: (D) “I should put on the stockings before getting out
amounts. Clot formation usually occurs in the legs. There’s a of bed in the morning. Promote venous return by applying
loss of lung parenchyma and subsequent scar tissue external pressure on veins.
formation
96. Aldo with a massive pulmonary embolism will have an
arterial blood gas analysis performed to determine the extent PNLE V for Care of Clients with
of hypoxia. The acid-base disorder that may be present is?
A. Metabolic acidosis
Physiologic and Psychosocial
B. Metabolic alkalosis Alterations (Part 3)
C. Respiratory acidosis
D. Respiratory alkalosis
1. Mr. Marquez reports of losing his job, not being able to
Answer: (D) Respiratory alkalosis. A client with massive
sleep at night, and feeling upset with his wife. Nurse John
pulmonary embolism will have a large region and blow off
responds to the client, “You may want to talk about your
large amount of carbon dioxide, which crosses the unaffected
employment situation in group today.” The Nurse is using
alveolar-capillary membrane more readily than does
which therapeutic technique?
oxygen and results in respiratory alkalosis.

97. After a motor vehicle accident, Armand an 22-year-old A. Observations


client is admitted with a pneumothorax. The surgeon inserts a B. Restating
chest tube and attaches it to a chest drainage system. C. Exploring
Bubbling soon appears in the water seal chamber. Which of D. Focusing\
the following is the most likely cause of the bubbling?
A. Air leak Answer: (D) Focusing. The nurse is using focusing by
B. Adequate suction suggesting that the client discuss a specific issue. The nurse
C. Inadequate suction didn’t restate the question, make observation, or ask further
D. Kinked chest tube question (exploring).

Answer: (A) Air leak. Bubbling in the water seal chamber of a 2. Tony refuses his evening dose of Haloperidol (Haldol), then
chest drainage system stems from an air leak. In becomes extremely agitated in the dayroom while other
pneumothorax an air leak can occur as air is pulled from the clients are watching television. He begins cursing and
pleural space. Bubbling doesn’t normally occur with throwing furniture. Nurse Oliver first action is to:
either adequate or inadequate suction or any preexisting
bubbling in the water seal chamber. A. Check the client’s medical record for an order for an as-
needed I.M. dose of medication for agitation.
98. Nurse Michelle calculates the IV flow rate for a B. Place the client in full leather restraints.
postoperative client. The client receives 3,000 ml of Ringer’s C. Call the attending physician and report the behavior.
lactate solution IV to run over 24 hours. The IV infusion set D. Remove all other clients from the dayroom.
has a drop factor of 10 drops per milliliter. The nurse should
regulate the client’s IV to deliver how many drops per minute? Answer: (D) Remove all other clients from the dayroom. The
A. 18 nurse’s first priority is to consider the safety of the clients in
B. 21 the therapeutic setting. The other actions are appropriate
C. 35 responses after ensuring the safety of other clients.
D. 40
3. Tina who is manic, but not yet on medication, comes to the C. Superego
drug treatment center. The nurse would not let this client join D. Oedipal complex
the group session because:
Answer: (C) Superego. This behavior shows a weak sense of
A. The client is disruptive. moral consciousness. According to Freudian theory,
B. The client is harmful to self. personality disorders stem from a weak superego.
C. The client is harmful to others.
D. The client needs to be on medication first. 8. In preparing a female client for electroconvulsive therapy
(ECT), Nurse Michelle knows that succinylcoline (Anectine)
Answer: (A) The client is disruptive. Group activity provides will be administered for which therapeutic effect?
too much stimulation, which the client will not be able to
handle (harmful to self) and as a result will be disruptive to A. Short-acting anesthesia
others. B. Decreased oral and respiratory secretions.
C. Skeletal muscle paralysis.
4. Dervid, an adolescent boy was admitted for substance D. Analgesia.
abuse and hallucinations. The client’s mother asks Nurse
Armando to talk with his husband when he arrives at the Answer: (C) Skeletal muscle paralysis. Anectine is a
hospital. The mother says that she is afraid of what the father depolarizing muscle relaxant causing paralysis. It is used to
might say to the boy. The most appropriate nursing reduce the intensity of muscle contractions during
intervention would be to: the convulsive stage, thereby reducing the risk of bone
fractures or dislocation
A. Inform the mother that she and the father can work
through this problem themselves. 9. Nurse Gina is aware that the dietary implications for a
B. Refer the mother to the hospital social worker. client in manic phase of bipolar disorder is:
C. Agree to talk with the mother and the father together.
D. Suggest that the father and son work things out. A. Serve the client a bowl of soup, buttered French bread,
and apple slices.
Answer: (C) Agree to talk with the mother and the father B. Increase calories, decrease fat, and decrease protein.
together. By agreeing to talk with both parents, the nurse can C. Give the client pieces of cut-up steak, carrots, and an
provide emotional support and further assess and validate apple.
the family’s needs. D. Increase calories, carbohydrates, and protein.

5. What is Nurse John likely to note in a male client being Answer: (D) Increase calories, carbohydrates, and
admitted for alcohol withdrawal? protein.This client increased protein for tissue building and
increased calories to replace what is burned up (usually via
A. Perceptual disorders. carbohydrates).
B. Impending coma.
C. Recent alcohol intake. 10.What parental behavior toward a child during an
D. Depression with mutism. admission procedure should cause Nurse Ron to suspect child
abuse?
Answer: (A) Perceptual disorders. Frightening visual
hallucinations are especially common in clients experiencing A. Flat affect
alcohol withdrawal. B. Expressing guilt
C. Acting overly solicitous toward the child.
6. Aira has taken amitriptyline HCL (Elavil) for 3 days, but now D. Ignoring the child.
complains that it “doesn’t help” and refuses to take it. What
should the nurse say or do? Answer: (C) Acting overly solicitous toward the child. This
behavior is an example of reaction formation, a
A. Withhold the drug. coping mechanism.
B. Record the client’s response.
C. Encourage the client to tell the doctor. 11.Nurse Lynnette notices that a female client with
D. Suggest that it takes awhile before seeing the results. obsessive-compulsive disorder washes her hands for long
periods each day. How should the nurse respond to this
Answer: (D) Suggest that it takes awhile before seeing the compulsive behavior?
results. The client needs a specific response; that it takes 2
to 3 weeks (a delayed effect) until the therapeutic blood A. By designating times during which the client can focus
level is reached on the behavior.
B. By urging the client to reduce the frequency of the
7. Dervid, an adolescent has a history of truancy from school, behavior as rapidly as possible.
running away from home and “barrowing” other people’s C. By calling attention to or attempting to prevent the
things without their permission. The adolescent denies behavior.
stealing, rationalizing instead that as long as no one was using D. By discouraging the client from verbalizing anxieties.
the items, it was all right to borrow them. It is important for
the nurse to understand the psychodynamically, this behavior Answer: (A) By designating times during which the client can
may be largely attributed to a developmental defect related focus on the behavior. The nurse should designate times
to the: during which the client can focus on the compulsive behavior
or obsessive thoughts. The nurse should urge the client to
A. Id reduce the frequency of the compulsive behavior gradually,
B. Ego not rapidly. She shouldn’t call attention to or try to prevent
the behavior. Trying to prevent the behavior may cause pain that her personality caused her disorder wouldn’t help her
and terror in the client. The nurse should encourage the understand and resolve the underlying conflict.
client to verbalize anxieties to help distract attention from
the compulsive behavior. 14.Nurse Krina knows that the following drugs have been
known to be effective in treating obsessive-compulsive
12.After seeking help at an outpatient mental health clinic, disorder (OCD):
Ruby who was raped while walking her dog is diagnosed with
posttraumatic stress disorder (PTSD). Three months later, A. benztropine (Cogentin) and diphenhydramine
Ruby returns to the clinic, complaining of fear, loss of control, (Benadryl).
and helpless feelings. Which nursing intervention is most B. chlordiazepoxide (Librium) and diazepam (Valium)
appropriate for Ruby? C. fluvoxamine (Luvox) and clomipramine (Anafranil)
D. divalproex (Depakote) and lithium (Lithobid)
A. Recommending a high-protein, low-fat diet.
B. Giving sleep medication, as prescribed, to restore a Answer: (C) fluvoxamine (Luvox) and clomipramine
normal sleepwake cycle. (Anafranil). The antidepressants fluvoxamine and
C. Allowing the client time to heal. clomipramine have been effective in the treatment of OCD.
D. Exploring the meaning of the traumatic event with the Librium and Valium may be helpful in treating anxiety related
client. to OCD but aren’t drugs of choice to treat the illness. The
other medications mentioned aren’t effective in the
Answer: (D) Exploring the meaning of the traumatic event treatment of OCD.
with the client. The client with PTSD needs encouragement to
examine and understand the meaning of the traumatic event 15.Alfred was newly diagnosed with anxiety disorder. The
and consequent losses. Otherwise, symptoms may worsen physician prescribed buspirone (BuSpar). The nurse is aware
and the client may become depressed or engage in self- that the teaching instructions for newly prescribed buspirone
destructive behavior such as substance abuse. The client should include which of the following?
must explore the meaning of the event and won’t heal
without this, no matter how much time passes. Behavioral A. A warning about the drugs delayed therapeutic effect,
techniques, such as relaxation therapy, may help decrease which is from 14 to 30 days.
the client’s anxiety and induce sleep. The physician may B. A warning about the incidence of neuroleptic malignant
prescribe antianxiety agents or antidepressants cautiously to syndrome (NMS).
avoid dependence; sleep medication is rarely appropriate. C. A reminder of the need to schedule blood work in 1
A special diet isn’t indicated unless the client also has an week to check blood levels of the drug.
eating disorder or a nutritional problem. D. A warning that immediate sedation can occur with a
resultant drop in pulse.
13.Meryl, age 19, is highly dependent on her parents and
fears leaving home to go away to college. Shortly before the Answer: (A) A warning about the drugs delayed therapeutic
semester starts, she complains that her legs are paralyzed effect, which is from 14 to 30 days. The client should be
and is rushed to the emergency department. When physical informed that the drug’s therapeutic effect might not be
examination rules out a physical cause for her paralysis, the reached for 14 to 30 days. The client must be instructed
physician admits her to the psychiatric unit where she is to continue taking the drug as directed. Blood level checks
diagnosed with conversion disorder. Meryl asks the nurse, aren’t necessary. NMS hasn’t been reported with this drug,
“Why has this happened to me?” What is the nurse’s best but tachycardia is frequently reported
response?
16.Richard with agoraphobia has been symptom-free for 4
A. “You’ve developed this paralysis so you can stay with months. Classic signs and symptoms of phobias include:
your parents. You must deal with this conflict if you
want to walk again.” A. Insomnia and an inability to concentrate.
B. “It must be awful not to be able to move your legs. You B. Severe anxiety and fear.
may feel better if you realize the problem is C. Depression and weight loss.
psychological, not physical.” D. Withdrawal and failure to distinguish reality from
C. “Your problem is real but there is no physical basis for fantasy.
it. We’ll work on what is going on in your life to find
out why it’s happened.” Answer: (B) Severe anxiety and fear. Phobias cause severe
D. “It isn’t uncommon for someone with your personality anxiety (such as a panic attack) that is out of proportion to
to develop a conversion disorder during times of stress.” the threat of the feared object or situation. Physical signs and
symptoms of phobias include profuse sweating, poor
Answer: (C) “Your problem is real but there is no physical motor control, tachycardia, and elevated blood pressure.
basis for it. We’ll work on what is going on in your life to find Insomnia, an inability to concentrate, and weight loss are
out why it’s happened.” The nurse must be honest with the common in depression. Withdrawal and failure to distinguish
client by telling her that the paralysis has no physiologic reality from fantasy occur in schizophrenia.
cause while also conveying empathy and acknowledging that
her symptoms are real. The client will benefit from psychiatric 17.Which medications have been found to help reduce or
treatment, which will help her understand the underlying eliminate panic attacks?
cause of her symptoms. After the psychological conflict is
resolved, her symptoms will disappear. Saying that it must be
A. Antidepressants
awful not to be able to move her legs wouldn’t answer the
B. Anticholinergics
client’s question; knowing that the cause is psychological
C. Antipsychotics
wouldn’t necessarily make her feel better. Telling her that she
D. Mood stabilizers
has developed paralysis to avoid leaving her parents or
Answer: (A) Antidepressants. Tricyclic and monoamine can produce suspiciousness, dilated pupils, and increased
oxidase (MAO) inhibitor antidepressants have been found to blood pressure.
be effective in treating clients with panic attacks. Why these
drugs help control panic attacks isn’t clearly understood. 21.The nurse is caring for a client diagnosed with antisocial
Anticholinergic agents, which are smooth-muscle personality disorder. The client has a history of fighting,
relaxants, relieve physical symptoms of anxiety but don’t cruelty to animals, and stealing. Which of the following traits
relieve the anxiety itself. Antipsychotic drugs are would the nurse be most likely to uncover during assessment?
inappropriate because clients who experience panic attacks
aren’t psychotic. Mood stabilizers aren’t indicated A. History of gainful employment
because panic attacks are rarely associated with mood B. Frequent expression of guilt regarding antisocial
changes. behavior
C. Demonstrated ability to maintain close, stable
18.A client seeks care because she feels depressed and has relationships
gained weight. To treat her atypical depression, the physician D. d. A low tolerance for frustration
prescribes tranylcypromine sulfate (Parnate), 10 mg by
mouth twice per day. When this drug is used to treat atypical Answer: (D) A low tolerance for frustration. Clients with an
depression, what is its onset of action? antisocial personality disorder exhibit a low tolerance for
frustration, emotional immaturity, and a lack of
A. 1 to 2 days impulse control. They commonly have a history of
B. 3 to 5 days unemployment, miss work repeatedly, and quit work without
C. 6 to 8 days other plans for employment. They don’t feel guilt about their
D. 10 to 14 days behavior and commonly perceive themselves as victims. They
also display a lack of responsibility for the outcome of
Answer: (B) 3 to 5 days. Monoamine oxidase inhibitors, their actions. Because of a lack of trust in others, clients with
such as tranylcypromine, have an onset of action of antisocial personality disorder commonly have difficulty
approximately 3 to 5 days. A full clinical response may be developing stable, close relationships.
delayed for 3 to 4 weeks. The therapeutic effects may
continue for 1 to 2 weeks after discontinuation 22.Nurse Amy is providing care for a male client undergoing
opiate withdrawal. Opiate withdrawal causes severe physical
19. A 65 years old client is in the first stage of Alzheimer’s discomfort and can be life-threatening. To minimize these
disease. Nurse Patricia should plan to focus this client’s care effects, opiate users are commonly detoxified with:
on:
A. Barbiturates
A. Offering nourishing finger foods to help maintain the B. Amphetamines
client’s nutritional status. C. Methadone
B. Providing emotional support and individual counseling. D. Benzodiazepines
C. Monitoring the client to prevent minor illnesses from
turning into major problems. Answer: (C) Methadone. Methadone is used to detoxify
D. Suggesting new activities for the client and family to do opiate users because it binds with opioid receptors at many
together. sites in the central nervous system but doesn’t have the same
deterious effects as other opiates, such as cocaine, heroin,
Answer: (B) Providing emotional support and individual and morphine. Barbiturates, amphetamines,
counseling. Clients in the first stage of Alzheimer’s disease are and benzodiazepines are highly addictive and would require
aware that something is happening to them and may become detoxification treatment.
overwhelmed and frightened. Therefore, nursing care
typically focuses on providing emotional support and 23.Nurse Cristina is caring for a client who experiences false
individual counseling. The other options are appropriate sensory perceptions with no basis in reality. These
during the second stage of Alzheimer’s disease, when perceptions are known as:
the client needs continuous monitoring to prevent minor
illnesses from progressing into major problems and when A. Delusions
maintaining adequate nutrition may become a challenge. B. Hallucinations
During this stage, offering nourishing finger foods helps C. Loose associations
clients to feed themselves and maintain adequate nutrition. D. Neologisms

20.The nurse is assessing a client who has just been admitted Answer: (B) Hallucinations. Hallucinations are visual, auditory,
to the emergency department. Which signs would suggest an gustatory, tactile, or olfactory perceptions that have no basis
overdose of an antianxiety agent? in reality. Delusions are false beliefs, rather than perceptions,
that the client accepts as real. Loose associations are rapid
A. Combativeness, sweating, and confusion shifts among unrelated ideas. Neologisms are bizarre words
B. Agitation, hyperactivity, and grandiose ideation that have meaning only to the client.
C. Emotional lability, euphoria, and impaired memory
D. Suspiciousness, dilated pupils, and increased blood 24. Nurse Marco is developing a plan of care for a client with
pressure anorexia nervosa. Which action should the nurse include in
the plan?
Answer: (C) Emotional lability, euphoria, and impaired
memory. Signs of antianxiety agent overdose include A. Restricts visits with the family and friends until the
emotional lability, euphoria, and impaired memory. client begins to eat.
Phencyclidine overdose can cause combativeness, sweating, B. Provide privacy during meals.
and confusion. Amphetamine overdose can result in agitation, C. Set up a strict eating plan for the client.
hyperactivity, and grandiose ideation. Hallucinogen overdose
D. Encourage the client to exercise, which will reduce her the problem and seeking help. Repression is suppressing past
anxiety. events from the consciousness because of guilty association.

Answer: (C) Set up a strict eating plan for the 28.Richard is admitted with a diagnosis of schizotypal
client. Establishing a consistent eating plan and monitoring personality disorder. Which signs would this client exhibit
the client’s weight are very important in this disorder. The during social situations?
family and friends should be included in the client’s care. The
client should be monitored during meals-not given privacy. A. Aggressive behavior
Exercise must be limited and supervised. B. Paranoid thoughts
C. Emotional affect
25.Tim is admitted with a diagnosis of delusions of grandeur. D. Independence needs
The nurse is aware that this diagnosis reflects a belief that
one is: Answer: (B) Paranoid thoughts. Clients with schizotypal
personality disorder experience excessive social anxiety that
A. Highly important or famous. can lead to paranoid thoughts. Aggressive behavior is
B. Being persecuted uncommon, although these clients may experience
C. Connected to events unrelated to oneself agitation with anxiety. Their behavior is emotionally cold with
D. Responsible for the evil in the world. a flattened affect, regardless of the situation. These clients
demonstrate a reduced capacity for close or dependent
Answer: (A) Highly important or famous. A delusion of relationships.
grandeur is a false belief that one is highly important or
famous. A delusion of persecution is a false belief that one 29. Nurse Mickey is caring for a client diagnosed with bulimia.
is being persecuted. A delusion of reference is a false belief The most appropriate initial goal for a client diagnosed with
that one is connected to events unrelated to oneself or a bulimia is to:
belief that one is responsible for the evil in the world
A. Avoid shopping for large amounts of food.
26.Nurse Jen is caring for a male client with manic depression. B. Control eating impulses.
The plan of care for a client in a manic state would include: C. Identify anxiety-causing situations
D. Eat only three meals per day.
A. Offering a high-calorie meals and strongly encouraging Answer: (C) Identify anxiety-causing situations. Bulimic
the client to finish all food. behavior is generally a maladaptive coping response to stress
B. Insisting that the client remain active through the day and underlying issues. The client must identify anxiety-
so that he’ll sleep at night. causing situations that stimulate the bulimic behavior and
C. Allowing the client to exhibit hyperactive, demanding, then learn new ways of coping with the anxiety.
manipulative behavior without setting limits.
D. Listening attentively with a neutral attitude and 30.Rudolf is admitted for an overdose of amphetamines.
avoiding power struggles. When assessing the client, the nurse should expect to see:

Answer: (D) Listening attentively with a neutral attitude and A. Tension and irritability
avoiding power struggles. The nurse should listen to the B. Slow pulse
client’s requests, express willingness to seriously consider the C. Hypotension
request, and respond later. The nurse should encourage the D. Constipation
client to take short daytime naps because he expends so
much energy. The nurse shouldn’t try to restrain the Answer: (A) Tension and irritability. An amphetamine is a
client when he feels the need to move around as long as his nervous system stimulant that is subject to abuse because of
activity isn’t harmful. High calorie finger foods should be its ability to produce wakefulness and euphoria.
offered to supplement the client’s diet, if he can’t remain An overdose increases tension and irritability. Options B and
seated long enough to eat a complete meal. The nurse C are incorrect because amphetamines stimulate
shouldn’t be forced to stay seated at the table to finish norepinephrine, which increase the heart rate and blood
a meal. The nurse should set limits in a calm, clear, and self- flow. Diarrhea is a common adverse effect so option D in is
confident tone of voice. incorrect

27.Ramon is admitted for detoxification after a cocaine 31.Nicolas is experiencing hallucinations tells the nurse, “The
overdose. The client tells the nurse that he frequently uses voices are telling me I’m no good.” The client asks if the nurse
cocaine but that he can control his use if he chooses. Which hears the voices. The most appropriate response by the nurse
coping mechanism is he using? would be:

A. Withdrawal A. “It is the voice of your conscience, which only you can
B. Logical thinking control.”
C. Repression B. “No, I do not hear your voices, but I believe you can
D. Denial hear them”.
C. “The voices are coming from within you and only you
Answer: (D) Denial. Denial is unconscious defense mechanism can hear them.”
in which emotional conflict and anxiety is avoided by refusing D. “Oh, the voices are a symptom of your illness; don’t pay
to acknowledge feelings, desires, impulses, or external facts any attention to them.”
that are consciously intolerable. Withdrawal is a common
response to stress, characterized by apathy. Logical thinking Answer: (B) “No, I do not hear your voices, but I believe you
is the ability to think rationally and make can hear them”. The nurse, demonstrating knowledge and
responsible decisions, which would lead the client admitting
understanding, accepts the client’s perceptions even though
they are hallucinatory 37.When assessing a premorbid personality characteristics of
a client with a major depression, it would be unusual for the
32.The nurse is aware that the side effect of nurse to find that this client demonstrated:
electroconvulsive therapy that a client may experience:
A. Rigidity
A. Loss of appetite B. Stubbornness
B. Postural hypotension C. Diverse interest
C. Confusion for a time after treatment D. Over meticulousness
D. Complete loss of memory for a time
Answer: (C) Diverse interest. Before onset of depression,
Answer: (C) Confusion for a time after treatment. The these clients usually have very narrow, limited interest.
electrical energy passing through the cerebral cortex during
ECT results in a temporary state of confusion after treatment. 38.Nurse Krina recognizes that the suicidal risk for depressed
client is greatest:
33.A dying male client gradually moves toward resolution of
feelings regarding impending death. Basing care on the A. As their depression begins to improve
theory of Kubler-Ross, Nurse Trish plans to use nonverbal B. When their depression is most severe
interventions when assessment reveals that the client is in C. Before nay type of treatment is started
the: D. As they lose interest in the environment

A. Anger stage Answer: (A) As their depression begins to improve. At this


B. Denial stage point the client may have enough energy to plan
C. Bargaining stage and execute an attempt
D. Acceptance stage
39.Nurse Kate would expect that a client with vascular
Answer: (D) Acceptance stage. Communication and dementis would experience:
intervention during this stage are mainly nonverbal, as when
the client gestures to hold the nurse’s hand A. Loss of remote memory related to anoxia
34.The outcome that is unrelated to a crisis state is: B. Loss of abstract thinking related to emotional state
C. Inability to concentrate related to decreased stimuli
A. Learning more constructive coping skills D. Disturbance in recalling recent events related to
B. Decompensation to a lower level of functioning. cerebral hypoxia.
C. Adaptation and a return to a prior level of functioning.
D. A higher level of anxiety continuing for more than 3 Answer: (D) Disturbance in recalling recent events related to
months. cerebral hypoxia. Cell damage seems to interfere with
registering input stimuli, which affects the ability to register
Answer: (D) A higher level of anxiety continuing for more and recall recent events; vascular dementia is related to
than 3 months. This is not an expected outcome of a crisis multiple vascular lesions of the cerebral cortex
because by definition a crisis would be resolved in 6 weeks. and subcortical structure.

35.Miranda a psychiatric client is to be discharged with orders 40.Josefina is to be discharged on a regimen of lithium
for haloperidol (haldol) therapy. When developing a teaching carbonate. In the teaching plan for discharge the nurse
plan for discharge, the nurse should include cautioning the should include:
client against:
A. Advising the client to watch the diet carefully
A. Driving at night B. Suggesting that the client take the pills with milk
B. Staying in the sun C. Reminding the client that a CBC must be done once a
C. Ingesting wines and cheeses month.
D. Taking medications containing aspirin D. Encouraging the client to have blood levels checked as
ordered.
Answer: (B) Staying in the sun. Haldol causes
photosensitivity. Severe sunburn can occur on exposure to Answer: (D) Encouraging the client to have blood levels
the sun checked as ordered. Blood levels must be checked monthly
or bimonthly when the client is on maintenance therapy
36.Jen a nursing student is anxious about the upcoming board because there is only a small range between therapeutic and
examination but is able to study intently and does not toxic levels.
become distracted by a roommate’s talking and loud music.
The student’s ability to ignore distractions and to focus on 41.The psychiatrist orders lithium carbonate 600 mg p.o t.i.d
studying demonstrates: for a female client. Nurse Katrina would be aware that the
teaching about the side effects of this drug were understood
A. Mild-level anxiety when the client state, “I will call my doctor immediately if I
B. Panic-level anxiety notice any:
C. Severe-level anxiety
D. Moderate-level anxiety A. Sensitivity to bright light or sun
B. Fine hand tremors or slurred speech
Answer: (D) Moderate-level anxiety. A moderately anxious C. Sexual dysfunction or breast enlargement
person can ignore peripheral events and focuses on central D. d. Inability to urinate or difficulty when urinating
concerns
Answer: (B) Fine hand tremors or slurred speech. These are 47. Dervid with paranoid schizophrenia repeatedly uses
common side effects of lithium carbonate. profanity during an activity therapy session. Which response
by the nurse would be most appropriate?
42.Nurse Mylene recognizes that the most important factor
necessary for the establishment of trust in a critical care area A. “Your behavior won’t be tolerated. Go to your room
is: immediately.”
B. “You’re just doing this to get back at me for making you
A. Privacy come to therapy.”
B. Respect C. “Your cursing is interrupting the activity. Take time out
C. Empathy in your room for 10 minutes.”
D. Presence D. “I’m disappointed in you. You can’t control yourself
even for a few minutes.”
Answer: (D) Presence. The constant presence of a nurse
provides emotional support because the client knows that Answer: (C) “Your cursing is interrupting the activity. Take
someone is attentive and available in case of an emergency time out in your room for 10 minutes.” The nurse should set
limits on client behavior to ensure a comfortable
43.When establishing an initial nurse-client relationship, environment for all clients. The nurse should accept hostile
Nurse Hazel should explore with the client the: or quarrelsome client outbursts within limits without
becoming personally offended, as in option A. Option B is
A. Client’s perception of the presenting problem. incorrect because it implies that the client’s actions reflect
B. Occurrence of fantasies the client may experience. feelings toward the staff instead of the client’s own misery.
C. Details of any ritualistic acts carried out by the client Judgmental remarks, such as option D, may decrease the
D. Client’s feelings when external; controls are instituted. client’s self-esteem

Answer: (A) Client’s perception of the presenting 48.Nurse Maureen knows that the nonantipsychotic
problem. The nurse can be most therapeutic by starting medication used to treat some clients with schizoaffective
where the client is, because it is the client’s concept of the disorder is:
problem that serves as the starting point of the relationship.
A. phenelzine (Nardil)
44.Tranylcypromine sulfate (Parnate) is prescribed for a B. chlordiazepoxide (Librium)
depressed client who has not responded to the tricyclic C. lithium carbonate (Lithane)
antidepressants. After teaching the client about the D. imipramine (Tofranil)
medication, Nurse Marian evaluates that learning has
occurred when the client states, “I will avoid: Answer: (C) lithium carbonate (Lithane). Lithium carbonate,
an antimania drug, is used to treat clients with cyclical
A. Citrus fruit, tuna, and yellow vegetables.” schizoaffective disorder, a psychotic disorder once
B. Chocolate milk, aged cheese, and yogurt’” classified under schizophrenia that causes affective
C. Green leafy vegetables, chicken, and milk.” symptoms, including maniclike activity. Lithium helps control
D. Whole grains, red meats, and carbonated soda.” the affective component of this disorder. Phenelzine is a
monoamine oxidase inhibitor prescribed for clients
Answer: (B) Chocolate milk, aged cheese, and yogurt’. These who don’t respond to other antidepressant drugs such as
high-tyramine foods, when ingested in the presence of an imipramine. Chlordiazepoxide, an antianxiety agent,
MAO inhibitor, cause a severe hypertensive response. generally is contraindicated in psychotic clients. Imipramine,
primarily considered an antidepressant agent, is also used to
45.Nurse John is a aware that most crisis situations should treat clients with agoraphobia and that undergoing cocaine
resolve in about: detoxification

49.Which information is most important for the nurse Trinity


A. 1 to 2 weeks
to include in a teaching plan for a male schizophrenic client
B. 4 to 6 weeks
taking clozapine (Clozaril)?
C. 4 to 6 months
D. 6 to 12 months
A. Monthly blood tests will be necessary.
Answer: (B) 4 to 6 weeks. Crisis is self-limiting and lasts from B. Report a sore throat or fever to the physician
4 to 6 weeks. immediately.
C. Blood pressure must be monitored for hypertension.
46. Nurse Judy knows that statistics show that in adolescent D. Stop the medication when symptoms subside.
suicide behavior:
Answer: (B) Report a sore throat or fever to the physician
immediately. A sore throat and fever are indications of an
A. Females use more dramatic methods than males
infection caused by agranulocytosis, a potentially life-
B. Males account for more attempts than do females
threatening complication of clozapine. Because of the risk of
C. Females talk more about suicide before attempting it
agranulocytosis, white blood cell (WBC) counts are necessary
D. Males are more likely to use lethal methods than are
weekly, not monthly. If the WBC count drops below
females
3,000/μl, the medication must be stopped. Hypotension may
occur in clients taking this medication. Warn the client to
Answer: (D) Males are more likely to use lethal methods than
stand up slowly to avoid dizziness from orthostatic
are females. This finding is supported by research; females
hypotension. The medication should be continued, even
account for 90% of suicide attempts but males are three
when symptoms have been controlled. If the medication
times more successful because of methods used.
must be stopped, it should be slowly tapered over 1 to 2
weeks and only under the supervision of a physician.
of interest or pleasure in almost all activities, with signs and
50.Ricky with chronic schizophrenia takes neuroleptic symptoms recurring for at least 2 weeks.
medication is admitted to the psychiatric unit. Nursing
assessment reveals rigidity, fever, hypertension, and 53. After taking an overdose of phenobarbital (Barbita),
diaphoresis. These findings suggest which Mario is admitted to the emergency department. Dr. Trinidad
lifethreatening reaction: prescribes activated charcoal (Charcocaps) to be
administered by mouth immediately. Before administering
A. Tardive dyskinesia. the dose, the nurse verifies the dosage ordered. What is
B. Dystonia. the usual minimum dose of activated charcoal?
C. Neuroleptic malignant syndrome.
D. Akathisia. A. 5 g mixed in 250 ml of water
B. 15 g mixed in 500 ml of water
Answer: (C) Neuroleptic malignant syndrome. The client’s C. 30 g mixed in 250 ml of water
signs and symptoms suggest neuroleptic malignant syndrome, D. 60 g mixed in 500 ml of water
a life-threatening reaction to neuroleptic medication that
requires immediate treatment. Tardive dyskinesia causes Answer: (C) 30 g mixed in 250 ml of water. The usual adult
involuntary movements of the tongue, mouth, facial muscles, dosage of activated charcoal is 5 to 10 times the estimated
and arm and leg muscles. Dystonia is characterized by cramps weight of the drug or chemical ingested, or a minimum
and rigidity of the tongue, face, neck, and back muscles. dose of 30 g, mixed in 250 ml of water. Doses less than this
Akathisia causes restlessness, anxiety, and jitteriness. will be ineffective; doses greater than this can increase the
risk of adverse reactions, although toxicity doesn’t occur with
51.Which nursing intervention would be most appropriate if a activated charcoal, even at the maximum dose.
male client develop orthostatic hypotension while taking
amitriptyline (Elavil)? 54.What herbal medication for depression, widely used in
Europe, is now being prescribed in the United States?
A. Consulting with the physician about substituting a
different type of antidepressant. A. Ginkgo biloba
B. Advising the client to sit up for 1 minute before getting B. Echinacea
out of bed. C. St. John’s wort
C. Instructing the client to double the dosage until the D. Ephedra
problem resolves.
D. Informing the client that this adverse reaction should Answer: (C) St. John’s wort. St. John’s wort has been found to
disappear within 1 week. have serotonin-elevating properties, similar to prescription
antidepressants. Ginkgo biloba is prescribed to enhance
Answer: (B) Advising the client to sit up for 1 minute before mental acuity. Echinacea has immune-stimulating properties.
getting out of bed. To minimize the effects of amitriptyline- Ephedra is a naturally occurring stimulant that is similar
induced orthostatic hypotension, the nurse should advise the to ephedrine.
client to sit up for 1 minute before getting out of bed.
Orthostatic hypotension commonly occurs with tricyclic 55.Cely with manic episodes is taking lithium. Which
antidepressant therapy. In these cases, the dosage may electrolyte level should the nurse check before administering
be reduced or the physician may prescribe nortriptyline, this medication?
another tricyclic antidepressant. Orthostatic hypotension
disappears only when the drug is discontinued. A. Calcium
B. Sodium
52.Mr. Cruz visits the physician’s office to seek treatment for C. Chloride
depression, feelings of hopelessness, poor appetite, insomnia, D. Potassium
fatigue, low selfesteem, poor concentration, and difficulty
making decisions. The client states that these symptoms Answer: (B) Sodium. Lithium is chemically similar to sodium. If
began at least 2 years ago. Based on this report, the nurse sodium levels are reduced, such as from sweating or diuresis,
Tyfany suspects: lithium will be reabsorbed by the kidneys, increasing the risk
of toxicity. Clients taking lithium shouldn’t restrict their intake
A. Cyclothymic disorder. of sodium and should drink adequate amounts of fluid each
B. Atypical affective disorder. day. The other electrolytes are important for normal body
C. Major depression. functions but sodium is most important to the absorption of
D. Dysthymic disorder. lithium.

Answer: (D) Dysthymic disorder. Dysthymic disorder is 56.Nurse Josefina is caring for a client who has been
marked by feelings of depression lasting at least 2 years, diagnosed with delirium. Which statement about delirium is
accompanied by at least two of the following true?
symptoms: sleep disturbance, appetite disturbance, low
energy or fatigue, low selfesteem, poor concentration, A. It’s characterized by an acute onset and lasts about 1
difficulty making decisions, and hopelessness. These month.
symptoms may be relatively continuous or separated by B. It’s characterized by a slowly evolving onset and lasts
intervening periods of normal mood that last a few days to about 1 week.
a few weeks. Cyclothymic disorder is a chronic mood C. It’s characterized by a slowly evolving onset and lasts
disturbance of at least 2 years’ duration marked by numerous about 1 month.
periods of depression and hypomania. Atypical affective D. It’s characterized by an acute onset and lasts hours to
disorder is characterized by manic signs and symptoms. a number of days.
Major depression is a recurring, persistent sadness or loss
Answer: (D) It’s characterized by an acute onset and lasts hospitalization is in poor physical condition from starvation
hours to a number of days. Delirium has an acute onset and and may die as a result of arrhythmias, hypothermia,
typically can last from several hours to several days. malnutrition, infection, or cardiac abnormalities secondary
to electrolyte imbalances. Therefore, monitoring the client’s
57.Edward, a 66 year old client with slight memory vital signs, serum electrolyte level, and acid base balance is
impairment and poor concentration is diagnosed with crucial. Option A may worsen anxiety. Option B is incorrect
primary degenerative dementia of the Alzheimer’s type. Early because a weight obtained after breakfast is more accurate
signs of this dementia include subtle personality changes and than one obtained after the evening meal. Option D
withdrawal from social interactions. To assess for progression would reward the client with attention for not eating and
to the middle stage of Alzheimer’s disease, the nurse reinforce the control issues that are central to the underlying
should observe the client for: psychological problem; also, the client may record food and
fluid intake inaccurately.
A. Occasional irritable outbursts.
B. Impaired communication. 60.Celia with a history of polysubstance abuse is admitted to
C. Lack of spontaneity. the facility. She complains of nausea and vomiting 24 hours
D. Inability to perform self-care activities. after admission. The nurse assesses the client and notes
piloerection, pupillary dilation, and lacrimation. The nurse
Answer: (B) Impaired communication. Initially, memory suspects that the client is going through which of the
impairment may be the only cognitive deficit in a client with following withdrawals?
Alzheimer’s disease. During the early stage of this
disease, subtle personality changes may also be present. A. Alcohol withdrawal
However, other than occasional irritable outbursts and lack B. Cannibis withdrawal
of spontaneity, the client is usually cooperative and exhibits C. Cocaine withdrawal
socially appropriate behavior. Signs of advancement to the D. Opioid withdrawal
middle stage of Alzheimer’s disease include exacerbated
cognitive impairment with obvious personality changes Answer: (D) Opioid withdrawal. The symptoms listed are
and impaired communication, such as inappropriate specific to opioid withdrawal. Alcohol withdrawal would show
conversation, actions, and responses. During the late stage, elevated vital signs. There is no real withdrawal from cannibis.
the client can’t perform self-care activities and may become Symptoms of cocaine withdrawal include depression, anxiety,
mute and agitation.
58.Isabel with a diagnosis of depression is started on
imipramine (Tofranil), 75 mg by mouth at bedtime. The nurse 61.Mr. Garcia, an attorney who throws books and furniture
should tell the client that: around the office after losing a case is referred to the
psychiatric nurse in the law firm’s employee assistance
A. This medication may be habit forming and will be program. Nurse Beatriz knows that the client’s behavior most
discontinued as soon as the client feels better. likely represents the use of which defense mechanism?
B. This medication has no serious adverse effects.
C. The client should avoid eating such foods as aged A. Regression
cheeses, yogurt, and chicken livers while taking the B. Projection
medication. C. Reaction-formation
D. This medication may initially cause tiredness, which D. Intellectualization
should become less bothersome over time.
Answer: (A) Regression. An adult who throws temper
Answer: (D) This medication may initially cause tiredness, tantrums, such as this one, is displaying regressive behavior,
which should become less bothersome over time. Sedation is or behavior that is appropriate at a younger age. In projection,
a common early adverse effect of imipramine, a tricyclic the client blames someone or something other than the
antidepressant, and usually decreases as tolerance source. In reaction formation, the client acts in opposition to
develops. Antidepressants aren’t habit forming and don’t his feelings. In intellectualization, the client overuses rational
cause physical or psychological dependence. However, after a explanations orabstract thinking to decrease the significance
long course of high-dose therapy, the dosage should be of a feeling or event.
decreased gradually to avoid mild withdrawal symptoms.
Serious adverse effects, although rare, include myocardial 62.Nurse Anne is caring for a client who has been treated
infarction, heart failure, and tachycardia. Dietary long term with antipsychotic medication. During the
restrictions, such as avoiding aged cheeses, yogurt, and assessment, Nurse Anne checks the client for tardive
chicken livers, are necessary for a client taking a monoamine dyskinesia. If tardive dyskinesia is present, Nurse Anne would
oxidase inhibitor, not a tricyclic antidepressant. most likely observe:

59.Kathleen is admitted to the psychiatric clinic for treatment A. Abnormal movements and involuntary movements of
of anorexia nervosa. To promote the client’s physical health, the mouth, tongue, and face.
the nurse should plan to: B. Abnormal breathing through the nostrils accompanied
by a “thrill.”
A. Severely restrict the client’s physical activities. C. Severe headache, flushing, tremors, and ataxia.
B. Weigh the client daily, after the evening meal. D. Severe hypertension, migraine headache,
C. Monitor vital signs, serum electrolyte levels, and acid-
base balance. Answer: (A) Abnormal movements and involuntary
D. Instruct the client to keep an accurate record of food movements of the mouth, tongue, and face. Tardive
and fluid intake. dyskinesia is a severe reaction associated with long term use
of antipsychotic medication. The clinical manifestations
Answer: (C) Monitor vital signs, serum electrolyte levels, and include abnormal movements (dyskinesia) and involuntary
acid-base balance. An anorexic client who requires
movements of the mouth, tongue (fly catcher tongue), and has an I.Q. of below 20; Mild mental retardation 50-70 and
face. Severe mental retardation has an I.Q. of 20-35.

63.Dennis has a lithium level of 2.4 mEq/L. The nurse 67.The therapeutic approach in the care of Armand an
immediately would assess the client for which of the autistic child include the following EXCEPT:
following signs or symptoms?
A. Engage in diversionary activities when acting -out
A. Weakness B. Provide an atmosphere of acceptance
B. Diarrhea C. Provide safety measures
C. Blurred vision D. Rearrange the environment to activate the child
D. Fecal incontinence
Answer: (D) Rearrange the environment to activate the
Answer: (C) Blurred vision. At lithium levels of 2 to 2.5 mEq/L child. The child with autistic disorder does not want
the client will experienced blurred vision, muscle twitching, change. Maintaining a consistent environment is therapeutic.
severe hypotension, and persistent nausea and vomiting. A. Angry outburst can be re-channeling through safe
With levels between 1.5 and 2 mEq/L the client experiencing activities. B. Acceptance enhances a trusting relationship. C.
vomiting, diarrhea, muscle weakness, ataxia, Ensure safety from self-destructive behaviors like head
dizziness, slurred speech, and confusion. At lithium levels of banging and hair pulling
2.5 to 3 mEq/L or higher, urinary and fecal incontinence
occurs, as well as seizures, cardiac dysrythmias, peripheral 68.Jeremy is brought to the emergency room by friends who
vascular collapse, and death. state that he took something an hour ago. He is actively
hallucinating, agitated, with irritated nasal septum.
64.Nurse Jannah is monitoring a male client who has been
placed inrestraints because of violent behavior. Nurse A. Heroin
determines that it will be safe to remove the restraints when: B. Cocaine
C. LSD
A. The client verbalizes the reasons for the violent D. Marijuana
behavior.
B. The client apologizes and tells the nurse that it will Answer: (B) cocaine. The manifestations indicate intoxication
never happen again. with cocaine, a CNS stimulant. A. Intoxication with heroine is
C. No acts of aggression have been observed within 1 manifested by euphoria then impairment in judgment,
hour after the release of two of the extremity attention and the presence of papillary constriction. C.
restraints. Intoxication with hallucinogen like LSD is manifested
D. The administered medication has taken effect. by grandiosity, hallucinations, synesthesia and increase in
vital signs D. Intoxication with Marijuana, a cannabinoid is
Answer: (C) No acts of aggression have been observed manifested by sensation of slowed time, conjunctival redness,
within 1 hour after the release of two of the extremity social withdrawal, impaired judgment and hallucinations.
restraints. The best indicator that the behavior is controlled,
if the client exhibits no signs of aggression after partial 69.Nurse Pauline is aware that Dementia unlike delirium is
release of restraints. Options A, B, and D do not ensure that characterized by:
the client has controlled the behavior.
A. Slurred speech
65.Nurse Irish is aware that Ritalin is the drug of choice for a B. Insidious onset
child with ADHD. The side effects of the following may be C. Clouding of consciousness
noted by the nurse: D. Sensory perceptual change

A. Increased attention span and concentration Answer: (B) insidious onset. Dementia has a gradual onset
B. Increase in appetite and progressive deterioration. It causes pronounced memory
C. Sleepiness and lethargy and cognitive disturbances. A,C and D are all characteristics of
D. Bradycardia and diarrhea delirium.

Answer: (A) increased attention span and concentration. The 70.A 35 year old female has intense fear of riding an elevator.
medication has a paradoxic effect that decrease hyperactivity She claims “ As if I will die inside.” The client is suffering from:
and impulsivity among children with ADHD. B, C, D.
Side effects of Ritalin include anorexia, insomnia, diarrhea A. Agoraphobia
and irritability. B. Social phobia
C. Claustrophobia
66.Kitty, a 9 year old child has very limited vocabulary and D. Xenophobia
interaction skills. She has an I.Q. of 45. She is diagnosed to
have Mental retardation of this classification: Answer: (C) Claustrophobia. Claustrophobia is fear of closed
space. A. Agoraphobia is fear of open space or being a
A. Profound situation where escape is difficult. B. Social phobia is fear of
B. Mild performing in the presence of others in a way that will
C. Moderate be humiliating or embarrassing. D. Xenophobia is fear of
D. Severe strangers.

Answer: (C) Moderate. The child with moderate mental 71.Nurse Myrna develops a counter-transference reaction.
retardation has an I.Q. of 35- 50 Profound Mental retardation This is evidenced by:
A. Revealing personal information to the client someone in her past to the patient D. Resistance is the
B. Focusing on the feelings of the client. client’s refusal to submit himself to the care of the nurse
C. Confronting the client about discrepancies in verbal or
non-verbal behavior 75.Marielle, 17 years old was sexually attacked while on her
D. The client feels angry towards the nurse who resembles way home from school. She is brought to the hospital by her
his mother. mother. Rape is an example of which type of crisis:

Answer: (A) Revealing personal information to the A. Situational


client. Counter-transference is an emotional reaction of the B. Adventitious
nurse on the client based on her unconscious needs and C. Developmental
conflicts. B and C. These are therapeutic approaches. D. This D. Internal
is transference reaction where a client has an emotional
reaction towards the nurse based on her past. Answer: (B) Adventitious. Adventitious crisis is a crisis
involving a traumatic event. It is not part of everyday life. A.
72.Tristan is on Lithium has suffered from diarrhea and Situational crisis is from an external source that upset ones
vomiting. What should the nurse in-charge do first: psychological equilibrium C and D. Are the same. They are
transitional or developmental periods in life
A. Recognize this as a drug interaction
B. Give the client Cogentin 76. Nurse Greta is aware that the following is classified as an
C. Reassure the client that these are common side effects Axis I disorder by the Diagnosis and Statistical Manual of
of lithium therapy Mental Disorders, Text Revision (DSM-IV-TR) is:
D. Hold the next dose and obtain an order for a stat
serum lithium level A. Obesity
B. Borderline personality disorder
Answer: (D) Hold the next dose and obtain an order for a stat C. Major depression
serum lithium level. Diarrhea and vomiting are manifestations D. Hypertension
of Lithium toxicity. The next dose of lithium should be
withheld and test is done to validate the observation. A. The Answer: (C) Major depression. The DSM-IV-TR classifies major
manifestations are not due to drug interaction. B. Cogentin is depression as an Axis I disorder. Borderline personality
used to manage the extra pyramidal symptom side effects disorder as an Axis II; obesity and hypertension, Axis III.
of antipsychotics. C. The common side effects of Lithium are
fine hand tremors, nausea, polyuria and polydipsia. 77.Katrina, a newly admitted is extremely hostile toward a
staff member she has just met, without apparent reason.
73.Nurse Sarah ensures a therapeutic environment for all the According to Freudian theory, the nurse should suspect that
client. Which of the following best describes a therapeutic the client is experiencing which of the following phenomena?
milieu?
A. Intellectualization
A. A therapy that rewards adaptive behavior B. Transference
B. A cognitive approach to change behavior C. Triangulation
C. A living, learning or working environment. D. Splitting
D. A permissive and congenial environment
Answer: (B) Transference. Transference is the unconscious
Answer: (C) A living, learning or working environment. A assignment of negative or positive feelings evoked by a
therapeutic milieu refers to a broad conceptual approach significant person in the client’s past to another person.
in which all aspects of the environment are channeled to Intellectualization is a defense mechanism in which the client
provide a therapeutic environment for the client. The six avoids dealing with emotions by focusing on facts.
environmental elements include structure, safety, norms; Triangulation refers to conflicts involving three family
limit setting, balance and unit modification. A. Behavioral members. Splitting is a defense mechanism commonly seen
approach in psychiatric care is based on the premise that in clients with personality disorder in which the world is
behavior can be learned or unlearned through the use perceived as all good or all bad.
of reward and punishment. B. Cognitive approach to change
behavior is done by correcting distorted perceptions and 78.An 83year-old male client is in extended care facility is
irrational beliefs to correct maladaptive behaviors. D. This is anxious most of the time and frequently complains of a
not congruent with therapeutic milieu. number of vague symptoms that interfere with his ability to
eat. These symptoms indicate which of the following
74.Anthony is very hostile toward one of the staff for no disorders?
apparent reason. He is manifesting:
A. Conversion disorder
A. Splitting B. Hypochondriasis
B. Transference C. Severe anxiety
C. Countertransference D. Sublimation
D. Resistance
Answer: (B) Hypochondriasis. Complains of vague physical
Answer: (B) Transference. Transference is a positive or symptoms that have no apparent medical causes are
negative feeling associated with a significant person in the characteristic of clients with hypochondriasis. In many cases,
client’s past that are unconsciously assigned to another A. the GI system is affected. Conversion disorders are
Splitting is a defense mechanism commonly seen in a characterized by one or more neurologic symptoms. The
client with personality disorder in which the world is client’s symptoms don’t suggest severe anxiety. A client
perceived as all good or all bad C. Countert-transference is a
phenomenon where the nurse shifts feelings assigned to
experiencing sublimation channels maladaptive feelings or A. “I went to the mall with my friends last Saturday”
impulses into socially acceptable behavior B. “I’m hyperventilating only when I have a panic attack”
C. “Today I decided that I can stop taking my medication”
79. Charina, a college student who frequently visited the D. “Last night I decided to eat more than a bowl of cereal”
health center during the past year with multiple vague
complaints of GI symptoms before course examinations. Answer: (A) “I went to the mall with my friends last
Although physical causes have been eliminated, the Saturday”. Clients with panic disorder tent to be socially
student continues to express her belief that she has a serious withdrawn. Going to the mall is a sign of working on
illness. These symptoms are typically of which of the avoidance behaviors. Hyperventilating is a key symptom of
following disorders? panic disorder. Teaching breathing control is a
major intervention for clients with panic disorder. The client
A. Conversion disorder taking medications for panic disorder; such as tricylic
B. Depersonalization antidepressants and benzodiazepines, must be weaned off
C. Hypochondriasis these drugs. Most clients with panic disorder
D. Somatization disorder with agoraphobia don’t have nutritional problems.

Answer: (C) Hypochondriasis. Hypochodriasis in this case is 83. The effectiveness of monoamine oxidase (MAO) inhibitor
shown by the client’s belief that she has a serious illness, drug therapy in client with posttraumatic stress disorder can
although pathologic causes have been eliminated. The be demonstrated by which of the following client self –
disturbance usually lasts at lease 6 with identifiable reports?
life stressor such as, in this case, course examinations.
Conversion disorders are characterized by one or more A. “I’m sleeping better and don’t have nightmares”
neurologic symptoms. Depersonalization refers to persistent B. “I’m not losing my temper as much”
recurrent episodes of feeling detached from one’s self or C. “I’ve lost my craving for alcohol”
body. Somatoform disorders generally have a chronic course D. “I’ve lost my phobia for water”
with few remissions
Answer: (A) “I’m sleeping better and don’t have
80. Nurse Daisy is aware that the following pharmacologic nightmares” MAO inhibitors are used to treat sleep problems,
agents are sedative hypnotic medication is used to induce nightmares, and intrusive daytime thoughts in individual with
sleep for a client experiencing a sleep disorder is: posttraumatic stress disorder. MAO inhibitors aren’t used to
help control flashbacks or phobias or to decrease the craving
A. Triazolam (Halcion) for alcohol.
B. Paroxetine (Paxil)\
C. Fluoxetine (Prozac) 84. Mark, with a diagnosis of generalized anxiety disorder
D. Risperidone (Risperdal) wants to stop taking his lorazepam (Ativan). Which of the
following important facts should nurse Betty discuss with the
Answer: (A) Triazolam (Halcion). Triazolam is one of a group client about discontinuing the medication?
of sedative hypnotic medication that can be used for a limited
time because of the risk of dependence. Paroxetine is a A. Stopping the drug may cause depression
scrotonin-specific reutake inhibitor used for treatment B. Stopping the drug increases cognitive abilities
of depression panic disorder, and obsessive-compulsive C. Stopping the drug decreases sleeping difficulties
disorder. Fluoxetine is a scrotonin-specific reuptake inhibitor D. Stopping the drug can cause withdrawal symptoms
used for depressive disorders and obsessive-compulsive
disorders. Risperidome is indicated for psychotic disorders. Answer: (D) Stopping the drug can cause withdrawal
symptoms. Stopping antianxiety drugs such as
81. Aldo, with a somatoform pain disorder may obtain benzodiazepines can cause the client to have withdrawal
secondary gain. Which of the following statement refers to a symptoms. Stopping a benzodiazepine doesn’t tend to cause
secondary gain? depression, increase cognitive abilities, or decrease sleeping
difficulties.
A. It brings some stability to the family
B. It decreases the preoccupation with the physical illness 85. Jennifer, an adolescent who is depressed and reported by
C. It enables the client to avoid some unpleasant activity his parents as having difficulty in school is brought to the
D. It promotes emotional support or attention for the community mental health center to be evaluated. Which of
client the following other health problems would the nurse suspect?

Answer: (D) It promotes emotional support or attention for A. Anxiety disorder


the client. Secondary gain refers to the benefits of the illness B. Behavioral difficulties
that allow the client to receive emotional support or C. Cognitive impairment
attention. Primary gain enables the client to avoid some D. Labile moods
unpleasant activity. A dysfunctional family may disregard the
real issue, although some conflict is relieved. Answer: (B) Behavioral difficulties. Adolescents tend to
Somatoform pain disorder is a preoccupation with pain in the demonstrate severe irritability and behavioral problems
absence of physical disease. rather than simply a depressed mood. Anxiety disorder is
more commonly associated with small children rather than
82. Dervid is diagnosed with panic disorder with agoraphobia with adolescents. Cognitive impairment is typically
is talking with the nurse in-charge about the progress made in associated with delirium or dementia. Labile mood is more
treatment. Which of the following statements indicates a characteristic of a client with cognitive impairment or
positive client response? bipolar disorder
86. Ricardo, an outpatient in psychiatric facility is diagnosed Dysarthria is difficulty in speech production. Flight of ideas is
with dysthymic disorder. Which of the following statement rapid shifting from one topic to another
about dysthymic disorder is true?
90. Which of the following descriptions of a client’s
A. It involves a mood range from moderate depression to experience and behavior can be assessed as an illusion?
hypomania
B. It involves a single manic depression A. The client tries to hit the nurse when vital signs must be
C. It’s a form of depression that occurs in the fall and taken
winter B. The client says, “I keep hearing a voice telling me to run
D. It’s a mood disorder similar to major depression but of away”
mild to moderate severity C. The client becomes anxious whenever the nurse leaves
the bedside
Answer: (D) It’s a mood disorder similar to major depression D. The client looks at the shadow on a wall and tells the
but of mild to moderate severity. Dysthymic disorder is a nurse she sees frightening faces on the wall.
mood disorder similar to major depression but it remains Answer: (D) The client looks at the shadow on a wall and tells
mild to moderate in severity. Cyclothymic disorder is a mood the nurse she sees frightening faces on the wall. Minor
disorder characterized by a mood range from memory problems are distinguished from dementia by their
moderate depression to hypomania. Bipolar I disorder is minor severity and their lack of significant interference with
characterized by a single manic episode with no past major the client’s social or occupational lifestyle. Other options
depressive episodes. Seasonalaffective disorder is a form of would be included in the history data but don’t directly
depression occurring in the fall and winter. correlate with the client’s lifestyle.

87. The nurse is aware that the following ways in vascular 91. During conversation of Nurse John with a client, he
dementia different from Alzheimer’s disease is: observes that the client shift from one topic to the next on a
regular basis. Which of the following terms describes this
A. Vascular dementia has more abrupt onset disorder?
B. The duration of vascular dementia is usually brief
C. Personality change is common in vascular dementia A. Flight of ideas
D. The inability to perform motor activities occurs in B. Concrete thinking
vascular dementia C. Ideas of reference
D. Loose association
Answer: (A) Vascular dementia has more abrupt
onset. Vascular dementia differs from Alzheimer’s disease Answer: (D) Loose association. Loose associations are
in that it has a more abrupt onset and runs a highly variable conversations that constantly shift in topic. Concrete
course. Personally change is common in Alzheimer’s disease. thinking implies highly definitive thought processes. Flight of
The duration of delirium is usually brief. The inability to ideas is characterized by conversation that’s disorganized
carry out motor activities is common in Alzheimer’s disease from the onset. Loose associations don’t necessarily start in
a cogently, then becomes loose
88. Loretta, a newly admitted client was diagnosed with
delirium and has history of hypertension and anxiety. She had 92. Francis tells the nurse that her coworkers are sabotaging
been taking digoxin, furosemide (Lasix), and diazepam the computer. When the nurse asks questions, the client
(Valium) for anxiety. This client’s impairment may be related becomes argumentative. This behavior shows personality
to which of the following conditions? traits associated with which of the following personality
disorder?
A. Infection
B. Metabolic acidosis A. Antisocial
C. Drug intoxication B. Histrionic
D. Hepatic encephalopathy C. Paranoid
D. Schizotypal
Answer: (C) Drug intoxication. This client was taking several
medications that have a propensity for producing delirium; Answer: (C) Paranoid. Because of their suspiciousness,
digoxin (a digitalis glycoxide), furosemide (a thiazide diuretic), paranoid personalities ascribe malevolent activities to others
and diazepam (a benzodiazepine). Sufficient supporting data and tent to be defensive, becoming quarrelsome and
don’t exist to suspect the other options as causes. argumentative. Clients with antisocial personality disorder
can also be antagonistic and argumentative but are
89. Nurse Ron enters a client’s room, the client says, “They’re less suspicious than paranoid personalities. Clients with
crawling on my sheets! Get them off my bed!” Which of the histrionic personality disorder are dramatic, not suspicious
following assessment is the most accurate? and argumentative. Clients with schizoid personality disorder
are usually detached from other and tend to have eccentric
A. The client is experiencing aphasia behavior.
B. The client is experiencing dysarthria
C. The client is experiencing a flight of ideas 93. Which of the following interventions is important for a
D. The client is experiencing visual hallucination Cely experiencing with paranoid personality disorder taking
olanzapine (Zyprexa)?
Answer: (D) The client is experiencing visual
hallucination. The presence of a sensory stimulus correlates A. Explain effects of serotonin syndrome
with the definition of a hallucination, which is a false sensory B. Teach the client to watch for extrapyramidal adverse
perception. Aphasia refers to a communication problem. reaction
C. Explain that the drug is less affective if the client world. They need to have as in-depth assessment of physical
smokes complaints that may spill over into their delusional symptoms.
D. Discuss the need to report paradoxical effects such as Talking with the client won’t provide as assessment of his
euphoria itching, and itching isn’t as adverse reaction of antipsychotic
drugs, calling the physician to get the client’s
Answer: (C) Explain that the drug is less affective if the client medication increased doesn’t address his physical complaints.
smokes. Olanzapine (Zyprexa) is less effective for clients who
smoke cigarettes. Serotonin syndrome occurs with clients 97. Ivy, who is on the psychiatric unit is copying and imitating
who take a combination of antidepressant medications. the movements of her primary nurse. During recovery, she
Olanzapine doesn’t cause euphoria, and extrapyramidal says, “I thought the nurse was my mirror. I felt connected
adverse reactions aren’t a problem. However, the client only when I saw my nurse.” This behavior is known by which
should be aware of adverse effects such as tardive dyskinesia. of the following terms?

94. Nurse Alexandra notices other clients on the unit avoiding A. Modeling
a client diagnosed with antisocial personality disorder. When B. Echopraxia
discussing appropriate behavior in group therapy, which of C. Ego-syntonicity
the following comments is expected about this client by his D. Ritualism
peers?
Answer: (B) Echopraxia. Echopraxia is the copying of
A. Lack of honesty another’s behaviors and is the result of the loss of ego
B. Belief in superstition boundaries. Modeling is the conscious copying of someone’s
C. Show of temper tantrums behaviors. Ego-syntonicity refers to behaviors that
D. Constant need for attention correspond with the individual’s sense of self. Ritualism
behaviors are repetitive and compulsive
Answer: (A) Lack of honesty. Clients with antisocial
personality disorder tent to engage in acts of dishonesty, 98. Jun approaches the nurse and tells that he hears a voice
shown by lying. Clients with schizotypal personality disorder telling him that he’s evil and deserves to die. Which of the
tend to be superstitious. Clients with histrionic following terms describes the client’s perception?
personality disorders tend to overreact to frustrations and
disappointments, have temper tantrums, and seek attention. A. Delusion
B. Disorganized speech
95. Tommy, with dependent personality disorder is working C. Hallucination
to increase his selfesteem. Which of the following statements D. Idea of reference
by the Tommy shows teaching was successful?
Answer: (C) Hallucination. Hallucinations are sensory
A. “I’m not going to look just at the negative things about experiences that are misrepresentations of reality or have no
myself” basis in reality. Delusions are beliefs not based in reality.
B. “I’m most concerned about my level of competence and Disorganized speech is characterized by jumping from one
progress” topic to the next or using unrelated words. An idea
C. “I’m not as envious of the things other people have as I of reference is a belief that an unrelated situation holds
used to be” special meaning for the client.
D. “I find I can’t stop myself from taking over things other
should be doing” 99. Mike is admitted to a psychiatric unit with a diagnosis of
undifferentiated schizophrenia. Which of the following
Answer: (A) “I’m not going to look just at the negative things defense mechanisms is probably used by mike?
about myself”. As the clients makes progress on improving
self-esteem, selfblame and negative self evaluation will A. Projection
decrease. Clients with dependent personality disorder tend to B. Rationalization
feel fragile and inadequate and would be extremely unlikely C. Regression
to discuss their level of competence and progress. These D. Repression
clients focus on self and aren’t envious or jealous. Individuals
with dependent personality disorders don’t take over Answer: (C) Regression. Regression, a return to earlier
situations because they see themselves as inept and behavior to reduce anxiety, is the basic defense mechanism
inadequate. in schizophrenia. Projection is a defense mechanism in
which one blames others and attempts to justify actions;
96. Norma, a 42-year-old client with a diagnosis of chronic it’s used primarily by people with paranoid schizophrenia
undifferentiated schizophrenia lives in a rooming house that and delusional disorder. Rationalization is a defense
has a weekly nursing clinic. She scratches while she tells the mechanism used to justify one’s action. Repression is the
nurse she feels creatures eating away at her skin. Which of basic defense mechanism in the neuroses; it’s an involuntary
the following interventions should be done first? exclusion of painful thoughts, feelings, or experiences
from awareness
A. Talk about his hallucinations and fears
B. Refer him for anticholinergic adverse reactions 100. Rocky has started taking haloperidol (Haldol). Which of
C. Assess for possible physical problems such as rash the following instructions is most appropriate for Ricky before
D. Call his physician to get his medication increased to taking haloperidol?
control his psychosis
A. Should report feelings of restlessness or agitation at
Answer: (C) Assess for possible physical problems such as once
rash. Clients with schizophrenia generally have poor B. Use a sunscreen outdoors on a year-round basis
visceral recognition because they live so fully in their fantasy
C. Be aware you’ll feel increased energy taking this drug 4. The registered nurse is planning to delegate task to a
D. This drug will indirectly control essential hypertension certified nursing assistant. Which of the following clients
should not be assigned to a CAN?
Answer: (A) Should report feelings of restlessness or agitation
at once. Agitation and restlessness are adverse effect of A. A client diagnosed with diabetes and who has an
haloperidol and can be treated with antocholinergic drugs. infected toe
Haloperidol isn’t likely to cause photosensitivity or control B. A client who had a CVA in the past two months
essential hypertension. Although the client may experience C. A client with Chronic renal failure
increased concentration and activity, these effects are due to D. A client with chronic venous insufficiency
a decreased in symptoms, not the drug itself.
Answer: A. The patient is experiencing a potentially serious
SET 2 complication related to diabetes and needs ongoing
assessment by an RN

5. The nurse in the medication unit passes the medications for


all the clients on the nursing unit. The head nurse is making
PNLE I Nursing Practice rounds with the physician and coordinates clients’ activities
with other departments. The nurse assistant changes the bed
Scope of this Nursing Test I is parallel to the NP1 NLE
lines and answers call lights. A second nurse is assigned for
Coverage:
changing wound dressings; a licensed practitioner nurse takes
 Foundation of Nursing
vital signs and bathes theclients. This illustrates of what
 Nursing Research
method of nursing care?
 Professional Adjustment
 Leadership and Management
1. The registered nurse is planning to delegate tasks to A. Case management method
unlicensed assistive personnel (UAP). Which of the following B. Primary nursing method
task could the registered nurse safely assigned to a UAP? C. Team method
D. Functional method
A. Monitor the I&O of a comatose toddler client with
Answer: D. It describes functional nursing. Staff is assigned
salicylate poisoning
to specific task rather than specific clients.
B. Perform a complete bed bath on a 2-year-old with
multiple injuries from a serious fall
6. A registered nurse has been assigned to six clients on the
C. Check the IV of a preschooler with Kawasaki disease
12-hour shift. The RN is responsible for every aspect of care
D. Give an outmeal bath to an infant with eczema
such as formulating the care of plan, intervention and
evaluating the care during her shift. At the end of her shift, the
Answer: D. Bathing an infant with eczema can be safely
RN will pass this same task to the next RN in charge. This
delegated to an aide; this task is basic and can competently
nursing care illustrates of what kind of method?
performed by an aid.

2. A nurse manager assigned a registered nurse from telemetry A. primary nursing method
unit to the pediatrics unit. There were three patients assigned B. case method
to the RN. Which of the following patients should not be C. team method
assigned to the floated nurse? D. functional method

Answer: B. Case management. The nurse assumes total


A. A 9-year-old child diagnosed with rheumatic fever
responsibility for meeting the needs of the client during her
B. A young infant after pyloromyotomy
entire duty.
C. A 4-year-old with VSD following cardiac catheterization
D. A 5-month-old with Kawasaki disease
7. A newly hired nurse on an adult medicine unit with 3
months experience was asked to float to pediatrics. The nurse
Answer: B. The RN floated from the telemetry unit would be
hesitates to perform pediatric skills and receive an interesting
least prepared to care for a young infant who has just had GI
assignment that feels overwhelming. The nurse should:
surgery and requires a specific feeding regimen.

3. A nurse in charge in the pediatric unit is absent. The nurse A. resign on the spot from the nursing position and apply
manager decided to assign the nurse in the obstetrics unit to for a position that does not require floating
the pediatrics unit. Which of the following patients could the B. Inform the nursing supervisor and the charge nurse
nurse manager safely assign to the float nurse? on the pediatric floor about the nurse’s lack of skill
and feelings of hesitations and request assistance
C. Ask several other nurses how they feel about pediatrics
A. A child who had multiple injuries from a serious vehicle
and find someone else who is willing to accept the
accident
assignment
B. A child diagnosed with Kawasaki disease and with
D. Refuse the assignment and leave the unit requesting a
cardiac complications
vacation a day
C. A child who has had a nephrectomy for Wilm’s
tumor
Answer: B. The nurse is ethically obligated to inform the
D. A child receiving an IV chelating therapy for lead
person responsible for the assignment and the person
poisoning
responsible for the unit about the nurse’s skill level. The
nurse therefore avoids a situation of abandoningclients and
Answer: C. RN floated from the obstetrics unit should be
exposing them to greater risks
able to care for a client with major abdominal surgery,
because this nurse has experienced caring for clients with
8. An experienced nurse who voluntarily trained a less
cesarean births.
experienced nurse with the intention of enhancing the skills
and knowledge and promoting professional advancement to
the nurse is called a:
A. mentor B. Obtaining consent is not the responsibility of the
B. team leader physician
C. case manager C. A physician will not subject himself to liability if he
D. change agent withholds any facts that are necessary to form the basis
of an intelligent consent
Answer: A. This describes a mentor D. If the nurse witnesses a consent for surgery, the nurse
is, in effect, indicating that the signature is that of the
9. The pediatrics unit is understaffed and the nurse manager purported person and that the person’s condition is
informs the nurses in the obstetrics unit that she is going to as indicated at the time of signing
assign one nurse to float in the pediatric units. Which
statement by the designated float nurse may put her job at risk? Answer: D. The nurse who witness a consent for treatment
or surgery is witnessing only that the client signed the form
A. “I do not get along with one of the nurses on the and that the client’s condition is as indicated at the time of
pediatrics unit” signing. The nurse is not witnessing that the client is
B. “I have a vacation day coming and would like to take “informed”.
that now”
C. “I do not feel competent to go and work on that area” 14. A mother in labor told the nurse that she was expecting
D. “ I am afraid I will get the most serious clients in the unit” that her baby has no chance to survive and expects that the
baby will be born dead. The mother accepts the fate of the
Answer:B. This action demonstrates a lack of responsibility baby and informs the nurse that when the baby is born and
and the nurse should attempt negotiation with the nurse requires resuscitation, the mother refuses any treatment to her
manager baby and expresses hostility toward the nurse while the
pediatric team is taking care of the baby. The nurse is legally
10. The newly hired staff nurse has been working on a medical obligated to:
unit for 3 weeks. The nurse manager has posted the team
leader assignments for the following week. The new staff A. Notify the pediatric team that the mother has refused
knows that a major responsibility of the team leader is to: resuscitation and any treatment for the baby and take the
baby to the mother
A. Provide care to the most acutely ill client on the team B. Get a court order making the baby a ward of the court
B. Know the condition and needs of all the patients on C. Record the statement of the mother, notify the
the team pediatric team, and observe carefully for signs of
C. Document the assessments completed by the team impaired bonding and neglect as a reasonable
members suspicion of child abuse
D. Supervise direct care by nursing assistants D. Do nothing except record the mother’s statement in the
medical record
Answer: B. The team leader is responsible for the overall
management of all clients and staff on the team, and this Answer: C. Although the statements by the mother may not
information is essential in order to accomplish this create a suspicion of neglect, when they are coupled with
observations about impaired bonding and maternal
11. A 15-year-old girl just gave birth to a baby boy who needs attachment, they may impose the obligation to report child
emergency surgery. The nurse prepared the consent form and neglect. The nurse is further obligated to notify caregivers of
it should be signed by: refusal to consent to treatment

15. The hospitalized client with a chronic cough is scheduled


A. The Physician for bronchoscopy. The nurse is tasks to bring the informed
B. The Registered Nurse caring for the client consent document into the client’s room for a signature. The
C. The 15-year-old mother of the baby boy client asks the nurse for details of the procedure and demands
D. The mother of the girl an explanation why the process of informed consent is
necessary. The nurse responds that informed consent means:
Answer: C. Even though the mother is a minor, she is legally
able to sign consent for her own child.
A. The patient releases the physician from all responsibility
12. A nurse caring to a client with Alzheimer’s disease for the procedure.
overheard a family member say to the client, “if you pee one B. The immediate family may make decision against the
more time, I won’t give you any more food and drinks”. What patient’s will.
initial action is best for the nurse to take? C. The physician must give the client or surrogates
enough information to make health care judgments
consistent with their values and goals.
A. Take no action because it is the family member saying D. The patient agrees to a procedure ordered by the
that to the client physician even if the client does not understand what the
B. Talk to the family member and explain that what outcome will be.
she/he has said is not appropriate for the client
C. Give the family member the number for an Elder Abuse 1. Answer: C. It best explains what informed consent is and
Hot line provides for legal rights of the patient
D. Document what the family member has said
16. A hospitalized client with severe necrotizing ulcer of the
Answer: B. This response is the most direct and immediate. lower leg is schedule for an amputation. The client tells the
This is a case of potential need for advocacy and patient’s nurse that he will not sign the consent form and he does not
rights. want any surgery or treatment because of religious beliefs
about reincarnation. What is the role of the RN?
13. Which is true about informed consent?
A. call a family meeting
A. A nurse may accept responsibility signing a consent B. discuss the religious beliefs with the physician
form if the client is unable C. encourage the client to have the surgery
D. inform the client of other options
Answer: A. Phenytoin (Dilantin) can cause venous irritation
Answer: B. The physician may not be aware of the role that due to its alkalinity, therefore it should be mixed with normal
religious beliefs play in making a decision about surgery. saline.

17. While in the hospital lobby, the RN overhears the three 22. The nurse is caring to a client who is hypotensive.
staff discussing the health condition of her client. What would Following a large hematemesis, how should the nurse position
be the appropriate nursing action for the RN to take? the client?

A. Tell them it is not appropriate to discuss the A. Feet and legs elevated 20 degrees, trunk horizontal,
condition of the client head on small pillow
B. Ignore them, because it is their right to discuss anything B. Low Fowler’s with knees gatched at 30 degrees
they want to C. Supine with the head turned to the left
C. Join in the conversation, giving them supportive input D. Bed sloped at a 45 degree angle with the head lowest and
about the case of the client the legs highest
D. Report this incident to the nursing supervisor
Answer: A. This position increases venous return, improves
Answer: A. The behavior should be stopped. The first step is cardiac volume, and promotes adequate ventilation and
to remind the staff that confidentiality may be violated cerebral perfusion

18. A staff nurse has had a serious issue with her colleague. In 23. The client is brought to the emergency department after a
this situation, it is best to: serious accident. What would be the initial nursing action of
the nurse to the client?
A. Discuss this with the supervisor
B. Not discuss the issue with anyone. It will probably A. assess the level of consciousness and circulation
resolve itself B. check respirations, circulation, neurological response
C. Try to discuss with the colleague about the issue and C. align the spine, check pupils, check for hemorrhage
resolve it when both are calmer D. check respiration, stabilize spine, check circulation
D. Tell other members of the network what the team
member did Answer: D. Checking the airway would be a priority, and a
neck injury should be suspected
Answer: C. Waiting for emotions to dissipate and sitting
down with the colleague is the first rule of conflict resolution. 24. A nurse is assigned to care to a client with Parkinson’s
disease. What interventions are important if the nurse wants to
19. The nurse is caring to a client who just gave birth to a improve nutrition and promote effective swallowing of the
healthy baby boy. The nurse may not disclose confidential client?
information when:
A. Eat solid food
A. The nurse discusses the condition of the client in a B. Give liquids with meals
clinical conference with other nurses C. Feed the client
B. The client asks the nurse to discuss the her condition D. Sit in an upright position to eat
with the family
C. The father of a woman who just delivered a baby is Answer: D. Client with Parkinson’s disease are at a high risk
on the phone to find out the sex of the baby for aspiration and undernutrition. Sitting upright promotes
D. A researcher from an institutionally approved research more effective swallowing.
study reviews the medical record of a patient
25. During tracheal suctioning, the nurse should implement
Answer: C. The nurse has no idea who the person is on the safety measures. Which of the following should the nurse
phone and therefore may not share the information even if implements?
the patient gives permission
A. limit suction pressure to 150-180 mmHg
20. A 17-year-old married client is scheduled for surgery. The B. suction for 15-20 seconds
nurse taking care of the client realizes that consent has not C. wear eye goggles
been signed after preoperative medications were given. What D. remove the inner cannula
should the nurse do?
Answer: C. It is important to protect the RN’s eyes from the
A. Call the surgeon possible contamination of coughed-up secretions
B. Ask the spouse to sign the consent
C. Obtain a consent from the client as soon as possible 26. The nurse is conducting a discharge instructions to a client
D. Get a verbal consent from the parents of the client diagnosed with diabetes. What sign of hypoglycemia should
be taught to a client?
Answer: A. The priority is to let the surgeon know, who in
turn may ask the husband to sign the consent A. warm, flushed skin
B. hunger and thirst
21. A 12-year-old client is admitted to the hospital. The C. increase urinary output
physician ordered Dilantin to the client. In administering IV D. palpitation and weakness
phenytoin (Dilantin) to a child, the nurse would be most
correct in mixing it with: Answer: D. There has been too little food or too much insulin.
Glucose levels can be markedly decreased (less than 50
A. Normal Saline mg/dl). Severe hypoglycemia may be fatal if not detected
B. Heparinized normal saline
C. 5% dextrose in water 27. A client admitted to the hospital and diagnosed with
D. Lactated Ringer’s solution Addison’s disease. What would be the appropriate nursing
action to the client?
A. administering insulin-replacement therapy D. complete a gastrointestinal and neurological
B. providing a low-sodium diet assessment
C. restricting fluids to 1500 ml/day
D. reducing physical and emotional stress Answer: D. Assessment and more data collection are needed.
The client may have gastrointestinal or neurological
Answer: D. Because the client’s ability to react to stress is problems that account for the symptoms. The anorexia could
decreased, maintaining a quiet environment becomes a result from medications, poor dentition, or indigestion, and
nursing priority. Dehydration is a common problem in the bruises may be attributed to ataxia, frequent falls, vertigo
Addison’s disease, so close observation of the client’s or medication.
hydration level is crucial.
32. The night shift nurse is making rounds. When the nurse
28. The nurse is to perform tracheal suctioning. During enters a client’s room, the client is on the floor next to the bed.
tracheal suctioning, which nursing action is essential to What would be the initial action of the nurse?
prevent hypoxemia?
A. chart that the patient fell
A. aucultating the lungs to determine the baseline data to B. call the physician
assess the effectiveness of suctioning C. chart that the client was found on the floor next to the
B. removing oral and nasal secretions bed
C. encouraging the patient to deep breathe and cough to D. fill out an incident report
facilitate removal of upper-airway secretions
D. administering 100% oxygen to reduce the effects of Answer: B. This is closest to suggesting action-assessment,
airway obstruction during suctioning. rather than paperwork- and is therefore the best of the four.

Answer: D. Presuctioning and postsuctioning ventilation


with 100% oxygen is important in reducing hypoxemia 33. The nurse on the night shift is about to administer
which occurs when the flow of gases in the airway is medication to a preschooler client and notes that the child has
obstructed by the suctioning catheter. no ID bracelet. The best way for the nurse to identify the client
is to ask:
29. An infant is admitted and diagnosed with pneumonia and
suspicious-looking red marks on the swollen face resembling a A. The adult visiting, “The child’s name is
handprint. The nurse does further assessment to the client. ____________________?”
How would the nurse document the finding? B. The child, “Is your name____________?”
C. Another staff nurse to identify this child
A. Facial edema with ecchymosis and handprint mark: D. The other children in the room what the child’s name is
crackles and wheezes
B. Facial edema, with red marks; crackles in the lung Answer: C. The only acceptable way to identify a
C. Facial edema with ecchymosis that looks like a preschooler client is to have a parent or another staff member
handprint identify the client.
D. Red bruise mark and ecchymosis on face
34. The nurse caring to a client has completed the assessment.
Answer: B. This is an example of objective data of both Which of the following will be considered to be the most
pulmonary status and direct observation on the skin by the accurate charting of a lump felt in the right breast?
nurse.
A. “abnormally felt area in the right breast, drainage noted”
30. On the evening shift, the triage nurse evaluates several B. “hard nodular mass in right breast nipple”
clients who were brought to the emergency department. Which C. “firm mass at five ‘ clock, outer quadrant, 1cm from
in the following clients should receive highest priority? right nipple’
D. “mass in the right breast 4cmx1cm
A. an elderly woman complaining of a loss of appetite and
fatigue for the past week Answer: C. It describes the mass in the greatest detail.
B. A football player limping and complaining of pain and
swelling in the right ankle 35. The physician instructed the nurse that intravenous
C. A 50-year-old man, diaphoretic and complaining of pyelogram will be done to the client. The client asks the nurse
severe chest pain radiating to his jaw what is the purpose of the procedure. The appropriate nursing
D. A mother with a 5-year-old boy who says her son has response is to:
been complaining of nausea and vomited once since
noon A. outline the kidney vasculature
B. determine the size, shape, and placement of the kidneys
Answer: C. These are likely signs of an acute myocardial C. test renal tubular function and the patency of the
infarction (MI). An acute MI is a cardiovascular emergency urinary tract
requiring immediate attention. Acute MI is potentially fatal if D. measure renal blood flow
not treated immediately.
Answer: C. Intravenous pyelogram tests both the function
31. A 80-year-old female client is brought to the emergency and patency of the kidneys. After the intravenous injection of
department by her caregiver, on the nurse’s assessment; the a radiopaque contrast medium, the size, location, and patency
following are the manifestations of the client: anorexia, of the kidneys can be observed by roentgenogram, as well as
cachexia and multiple bruises. What would be the best nursing the patency of the urethra and bladder as the kidneys function
intervention? to excrete the contrast medium.

A. check the laboratory data for serum albumin, hematocrit, 36. A client visits the clinic for screening of scoliosis. The
and hemoglobin nurse should ask the client to:
B. talk to the client about the caregiver and support system
C. complete a police report on elder abuse A. bend all the way over and touch the toes
B. stand up as straight and tall as possible
C. bend over at a 90-degree angle from the waist Answer: D. Parents should be taught to feel the area that is
D. bend over at a 45-degree angle from the waist raised and measure only that

Answer: C. This is the recommended position for screening 41. A community health nurse is schedule to do home visit.
for scoliosis. It allows the nurse to inspect the alignment of She visits to an elderly person living alone. Which of the
the spine, as well as to compare both shoulders and both hips. following observation would be a concern?

37. A client with tuberculosis is admitted in the hospital for 2 A. Picture windows
weeks. When a client’s family members come to visit, they B. Unwashed dishes in the sink
would be adhering to respiratory isolation precautions when C. Clear and shiny floors
they: D. Brightly lit rooms

A. wash their hands when leaving Answer: C. It is a safety hazard to have shiny floors because
B. put on gowns, gloves and masks they can cause falls.
C. avoid contact with the client’s roommate
D. keep the client’s room door open 42. After a birth, the physician cut the cord of the baby, and
before the baby is given to the mother, what would be the
Answer: A. Handwashing is the best method for reducing initial nursing action of the nurse?
cross-contamination. Gowns and gloves are not always
required when entering a client’s room. A. examine the infant for any observable abnormalities
B. confirm identification of the infant and apply bracelet to
38. An infant is brought to the emergency department and mother and infant
diagnosed with pyloric stenosis. The parents of the client ask C. instill prophylactic medication in the infant’s eyes
the nurse, “Why does my baby continue to vomit?” Which of D. wrap the infant in a prewarmed blanket and cover
the following would be the best nursing response of the nurse? the head

A. “Your baby eats too rapidly and overfills the stomach, Answer:D. The first priority, beside maintaining a newborn’s
which causes vomiting patent airway, is body temperature.
B. “Your baby can’t empty the formula that is in the
stomach into the bowel” 43. A 2-year-old client is admitted to the hospital with severe
C. “The vomiting is due to the nausea that accompanies eczema lesions on the scalp, face, neck and arms. The client is
pyloric stenosis” scratching the affected areas. What would be the best nursing
D. “Your baby needs to be burped more thoroughly after intervention to prevent the client from scratching the affected
feeding” areas?

Answer: B. Pyloric stenosis is an anomaly of the upper A. elbow restraints to the arms
gastrointestinal tract. The condition involves a thickening, or B. Mittens to the hands
hypertrophy, of the pyloric sphincter located at the distal end C. Clove-hitch restraints to the hands
of the stomach. This causes a mechanical intestinal D. A posey jacket to the torso
obstruction, which leads to vomiting after feeding the infant.
The vomiting associated with pyloric stenosis is described as Answer: B. The purpose of restraints for this child is to keep
being projectile in nature. This is due to the increasing the child from scratching the affected areas. Mittens restraint
amounts of formula the infant begins to consume coupled would prevent scratching, while allowing the most movement
with the increasing thickening of the pyloric sphincter. permissible.
39. A 70-year-old client with suspected tuberculosis is brought 44. The parents of the hospitalized client ask the nurse how
to the geriatric care facilities. An intradermal tuberculosis test their baby might have gotten pyloric stenosis. The appropriate
is schedule to be done. The client asks the nurse what is the nursing response would be:
purpose of the test. Which of the following would be the best
rationale for this?
A. There is no way to determine this preoperatively
B. Their baby was born with this condition
A. reactivation of an old tuberculosis infection C. Their baby developed this condition during the first
B. increased incidence of new cases of tuberculosis in few weeks of life
persons over 65 years old D. Their baby acquired it due to a formula allergy
C. greater exposure to diverse health care workers
D. respiratory problems are characteristic in this population Answer: C. Pyloric stenosis is not a congenital anatomical
defect, but the precise etiology is unknown. It develops
Answer: B. Increased incidence of TB has been seen in the during the first few weeks of life.
general population with a high incidence reported in
hospitalized elderly clients. Immunosuppression and lack of 45. A male client comes to the clinic for check-up. In doing a
classic manifestations because of the aging process are just physical assessment, the nurse should report to the physician
two of the contributing factors of tuberculosis in the elderly. the most common symptom of gonorrhea, which is:
40. The nurse is making a health teaching to the parents of the
client. In teaching parents how to measure the area of A. pruritus
induration in response to a PPD test, the nurse would be most B. pus in the urine
accurate in advising the parents to measure: C. WBC in the urine
D. Dysuria
A. both the areas that look red and feel raised Answer: B. Pus is usually the first symptom, because the
B. The entire area that feels itchy to the child bacteria reproduce in the bladder.
C. Only the area that looks reddened
D. Only the area that feels raised
46. Which of the following would be the most important goal Answer: D. The tumor infiltrates nearby tissue, it can cause
in the nursing care of an infant client with eczema? retraction of the overlying skin and create a dimpling
appearance.
A. preventing infection
B.
C.
maintaining the comfort level
providing for adequate nutrition
PNLE II Nursing Practice
D. decreasing the itching

Answer:A. Preventing infection in the infant with eczema is The scope of this Nursing Test II is parallel to the NP2 NLE
the nurse’s most important goal. The infant with eczema is at Coverage:
high risk for infection due to numerous breaks in the skin’s  Maternal and Child Health
integrity. Intact skin is always the infant’s first line of  Community Health Nursing
defense against infection.  Communicable Diseases
 Integrated Management of Childhood Illness
47. The nurse is making a discharge instruction to a client 1. The student nurse is assigned to take the vital signs of the
receiving chemotherapy. The client is at risk for bone marrow clients in the pediatric ward. The student nurse reports to the
depression. The nurse gives instructions to the client about
staff nurse that the parent of a toddler who is 2 days
how to prevent infection at home. Which of the following
postoperative after a cleft palate repair has given the toddler
health teaching would be included?
a pacifier. What would be the best immediate action of the
nurse?
A. “Get a weekly WBC count”
B. “Do not share a bathroom with children or pregnant
woman” A. Notify the pediatrician of this finding
C. “Avoid contact with others while receiving B. Reassure the student that this is an acceptable action
chemotherapy” on the parent’s part
D. “Do frequent hand washing and maintain good C. Discuss this action with the parents
hygiene” D. Ask the student nurse to remove the pacifier from the
toddler’s mouth
Answer: D. Frequent hand washing and good hygiene are the
best means of preventing infection. Answer: C. Nothing must be placed in the mouth of a
toddler who just undergone a cleft palate repair until the
48. The nurse is assigned to care the client with infectious suture line has completely healed. It is the nurse’s
disease. The best antimicrobial agent for the nurse to use in responsibility to inform the parent of the client. Spoon, forks,
handwashing is: straws, and tongue blades are other unacceptable items to
place in the mouth of a toddler who just undergone cleft
A. Isopropyl alcohol palate repair. The general principle of care is that nothing
B. Hexachlorophene (Phisohex) should enter the mouth until the suture line has completely
C. Soap and water healed.
D. Chlorhexidine gluconate (CHG) (Hibiclens)
2. The nurse is providing a health teaching to the mother of
Answer: D. CHG is a highly effective antimicrobial
an 8-year-old child with cystic fibrosis. Which of the following
ingredient, especially when it is used consistently over time.
statement if made by the mother would indicate to the nurse
49. The mother of the client tells the nurse, “ I’m not going to the need for further teaching about the medication regimen
have my baby get any immunization”. What would be the best of the child?
nursing response to the mother?
A. “My child might need an extra capsule if the meal is
A. “You and I need to review your rationale for this high in fat”
decision” B. “I’ll give the enzyme capsule before every snack”
B. “Your baby will not be able to attend day care without C. “I’ll give the enzyme capsule before every meal”
immunizations” D. “My child hates to take pills, so I’ll mix the capsule into
C. “Your decision can be viewed as a form of child abuse a cup of hot chocolate
and neglect”
D. “You are needlessly placing other people at risk for Answer: D. The pancreatic capsules contain pancreatic
communicable diseases” enzyme that should be administered in a cold, not a hot,
medium (example: chilled applesauce versus hot chocolate)
Answer: A. The mother may have many reasons for such a to maintain the medication’s integrity.
decision. It is the nurse’s responsibility to review this
decision with the mother and clarify any misconceptions
3. The mother brought her child to the clinic for follow-up
regarding immunizations that may exist.
check up. The mother tells the nurse that 14 days after
50. The nurse is teaching the client about breast self- starting an oral iron supplement, her child’s stools are black.
examination. Which observation should the client be taught to Which of the following is the best nursing response to the
recognize when doing the examination for detection of breast mother?
cancer?
A. “I will notify the physician, who will probably decrease
A. tender, movable lump the dosage slightly”
B. pain on breast self-examination B. “This is a normal side effect and means the medication
C. round, well-defined lump is working”
D. dimpling of the breast tissue C. “You sound quite concerned. Would you like to talk
about this further?”
D. “I will need a specimen to check the stool for possible
bleeding”
baseline renal function must be evaluated before initiating
Answer: B. When oral iron preparations are given correctly, either medication.
the stools normally turn dark green or black. Parents of
children receiving this medication should be advised that 8. Which of the following is the suited size of the needle
this side effect indicates the medication is being absorbed would the nurse select to administer the IM injection to a
and is working well. preschool child?

4. An 8-year-old boy with asthma is brought to the clinic for A. 18 G, 1-1/2 inch
check up. The mother asks the nurse if the treatment given to B. 25 G, 5/8 inch
her son is effective. What would be the appropriate response C. 21 G, 1 inch
of the nurse? D. 18 G, 1inch

A. I will review first the child’s height on a growth chart to Answer: C. In selecting the correct needle to administer an
know if the treatment is working IM injection to a preschooler, the nurse should always look
B. I will review first the child’s weight on a growth chart to at the child and use judgment in evaluating muscle mass and
know if the treatment is working amount of subcutaneous fat. In this case, in the absence of
C. I will review first the number of prescriptions refills the further data, the nurse would be most correct in selecting a
child has required over the last 6 months to give you needle gauge and length appropriate for the “average’
an accurate answer preschool child. A medium-gauge needle (21G) that is 1 inch
D. I will review first the number of times the child has seen long would be most appropriate.
the pediatrician during the last 6 months to give you an
accurate answer 9. A 9-year-old boy is admitted to the hospital. The boy is
being treated with salicylates for the migratory polyarthritis
Answer: C. Reviewing the number of prescription refills the accompanying the diagnosis of rheumatic fever. Which of the
child has required over the last 6 months would be the best following activities performed by the child would give a best
indicator of how well controlled and thus how effective the sign that the medication is effective?
child’s asthma treatment is. Breakthrough wheezing,
shortness of breath, and upper respiratory infections would A. Listening to story of his mother
require that the child take additional medication. This would B. Listening to the music in the radio
be reflected in the number of prescription refills. C. Playing mini piano
D. Watching movie in the dvd mini player
5. The nurse is caring to a child client who is receiving
tetracycline. The nurse is aware that in taking this medication, Answer: C. The purpose of the salicylate therapy is to relieve
it is very important to: the pain associated with the migratory polyarthritis
accompanying the rheumatic fever. Playing mini piano
A. Administer the drug between meals would require movement of the child’s joints and would
B. Monitor the child’s hearing provide the nurse with a means of evaluating the child’s
C. Give the drug through a straw level of pain.
D. Keep the child out of the sunlight
10. The physician decided to schedule the 4-year-old client
Answer: D. Tetracycline may cause a phototoxic reaction. for repair of left undescended testicle. The Injection of a
hormone, HCG finds it less successful for treatment. To
6. A 14 day-old infant with a cyanotic heart defects and mild administer a pentobarbital sodium (Nembutal) suppository
congestive heart failure is brought to the emergency preoperatively to this client, in which position should the
department. During assessment, the nurse checks the apical nurse place him?
pulse rate of the infant. The apical pulse rate is 130 beats per
minute. Which of the following is the appropriate nursing A. Supine with foot of bed elevated
action? B. Prone with legs abducted
C. Sitting with foot of bed elevated
A. Retake the apical pulse in 15 minutes D. Side-lying with upper leg flexed
B. Retake the apical pulse in 30 minutes
C. Notify the pediatrician immediately Answer: D. The recommended position to administer rectal
D. Administer the medication as scheduled medications to children is side-lying with the upper leg
flexed. This position allows the nurse to safely and
Answer: D. The normal heart rate of an infant is 120-160 effectively administer the medication while promoting
beats per minute comfort for the child.

7. The physician prescribed gentamicin (Garamycin) to a child 11. The nurse is caring to a 24-month-old child diagnosed
who is also receiving chemotherapy. Before administering the with congenital heart defect. The physician prescribed digoxin
drug, the nurse should check the results of the child’s: (Lanoxin) to the client. Before the administration of the drug,
the nurse checks the apical pulse rate to be 110 beats per
A. CBC and platelet count minute and regular. What would be the next nursing action?
B. Auditory tests
C. Renal Function tests A. Check the other vital signs and level of consciousness
D. Abdominal and chest x-rays B. Withhold the digoxin and notify the physician
C. Give the digoxin as prescribed
Answer: C. Both gentamicin and chemotherapeutic agents D. Check the apical and radial simultaneously, and if they
can cause renal impairment and acute renal failure; thus are the same, give the digoxin.
Answer:C. For a 12month-old child, 110 apical pulse rate is accomplish this, the toddler must be able to explore and
normal and therefore it is safe to give the digoxin. A manipulate the environment.
toddler’s normal pulse rate is slightly lower than an infant’s
(120). 16. A mother who gave birth to her second daughter is so
concerned about her 2-year old daughter. She tells the nurse,
12. An 8-year-old client with cystic fibrosis is admitted to the “I am afraid that my 2-year-old daughter may not accept her
hospital and will undergo a chest physiotherapy treatment. newly born sister”. It is appropriate to the nurse to response
The therapy should be properly coordinated by the nurse that:
with the respiratory therapy department so that treatments
occur during: A. The older daughter be given more responsibility and
assure her “that she is a big girl now, and doesn’t need
A. After meals Mommy as much”
B. Between meals B. The older daughter not have interaction with the baby
C. After medication at the hospital, because she may harm her new sibling
D. Around the child’s play schedule C. The older daughter stay with her grandmother for a few
days until the parents and new baby are settled at
Answer: B. Chest physiotherapy treatments are scheduled home
between meals to prevent aspiration of stomach contents, D. The mother spend time alone with her older daughter
because the child is placed in a variety of positions during when the baby is sleeping
the treatment process.
Answer: D. The introduction of a baby into a family with one
13. The nurse is providing health teaching about the or more children challenges parent to promote acceptance
breastfeeding and family planning to the client who gave of the baby by siblings. The parent’s attitudes toward the
birth to a healthy baby girl. Which of the following statement arrival of the baby can set the stage for the other children’s
would alert the nurse that the client needs further teaching? reaction. Spending time with the older siblings alone will
also reassure them of their place in the family, even though
A. “I understand that the hormones for breastfeeding may the older children will have to eventually assume new
affect when my periods come” positions within the family hierarchy.
B. “Breastfeeding causes my womb to tighten and bleed
less after birth” 17. A 2-year-old client with cystic fibrosis is confined to bed
C. “I may not have periods while I am breastfeeding, so I and is not allowed to go to the playroom. Which of the
don’t need family planning” following is an appropriate toy would the nurse select for the
D. “I can get pregnant as early as one month after my baby child:
was born”
A. Puzzle
Answer: C. It is common misconception that breastfeeding B. Musical automobile
may prevent pregnancy. C. Arranging stickers in the album
D. Pounding board and hammer
14. A toddler is brought to the hospital because of severe
diarrhea and vomiting. The nurse assigned to the client enters Answer: D. The autonomous toddler would be frustrated by
the client’s room and finds out that the client is using a soiled being confined to be. The pounding board and hammer is
blanket brought in from home. The nurse attempts to remove developmentally appropriate and an excellent way for the
the blanket and replace it with a new and clean blanket. The toddler to release frustration.
toddler refuses to give the soiled blanket. The nurse realizes
that the best explanation for the toddler’s behavior is: 18. Which of the following clients is at high risk for
developmental problem?
A. The toddler did not bond well with the maternal figure
B. The blanket is an important transitional object A. A toddler with acute Glomerulonephritis on
C. The toddler is anxious about the hospital experience antihypertensive and antibiotics
D. The toddler is resistive to nursing interventions B. A 5-year-old with asthma on cromolyn sodium
C. A preschooler with tonsillitis
Answer: B. The “security blanket” is an important transitional D. A 2 1/2 –year old boy with cystic fibrosis
object for the toddler. It provides a feeling of comfort and
safety when the maternal figure is not present or when in a Answer: D. It is the developmental task of an 18-month-old
new situation for which the toddler was not prepared. toddler to explore and learn about the environment. The
Virtually any object (stuffed animal, doll, book etc) can respiratory complications associated with cystic fibrosis
become a security blanket for the toddler (which are present in almost all children with cystic fibrosis)
could prevent this development task from occurring.
15. The nurse has knowledge about the developmental task
of the child. In caring a 3-year-old-client, the nurse knows 19. Which of the following would be the best divesionary
that the suited developmental task of this child is to: activity for the nurse to select for a 2 weeks hospitalized 3-
year-old girl?
A. Learn to play with other children
B. Able to trust others A. Crayons and coloring books
C. Express all needs through speaking B. doll
D. Explore and manipulate the environment C. xylophone toy
D. Puzzles
Answer: D. Toddlers need to meet the developmental
milestone of autonomy versus shame and doubt. In order to
Answer: C. The best diversion for a hospitalized child aged 2- pediatric unit would allow this. This activity also provides
3 years old would be anything that makes noise or makes a the school-age child a needed opportunity to interact with
mess; xylophone which certainly makes noise or music others in the absence of school and personal friends.
would be the best choice.
24. The parent of a 16-year-old boy tells the nurse that his
20. A nurse is providing safety instructions to the parents of son is driving a motorbike very fast and with one hand. “It is
the 11-month-old child. Which of the following will the nurse making me crazy!” What would be the best explanation of
includes in the instructions? the nurse to the behavior of the boy?

A. Plugging all electrical outlets in the house A. The adolescent might have an unconscious death wish
B. Installing a gate at the top and bottom of any stairs in B. The adolescent feels indestructible
the home C. The adolescent lacks life experience to realize how
C. Purchasing an infant car seat as soon as possible dangerous the behavior is
D. Begin to teach the child not to place small objects in the D. The adolescent has found a way to act out hostility
mouth toward the parent

Answer: B. An 11-month-old child stands alone and can walk Answer: B. Adolescents do feel indestructible, and this is
holding onto people or objects. Therefore the installation of a reflected in many risk-taking behaviors.
gate at the top and bottom of any stairs in the house is crucial
for the child’s safety 25. An 8-month-old infant is admitted to the hospital due to
diarrhea. The nurse caring for the client tells the mother to
21. An 8-year-old girl is in second grade and the parents stay beside the infant while making assessment. Which of the
decided to enroll her to a new school. While the child is following developmental milestones the infant has reached?
focusing on adjusting to new environment and peers, her
grades suffer. The child’s father severely punishes the child A. Has a three-word vocabulary
and forces her daughter to study after school. The father B. Interacts with other infants
does not allow also her daughter to play with other children. C. Stands alone
These data indicate to the nurse that this child is deprived of D. Recognizes but is fearful of strangers
forming which normal phase of development?
Answer: D. An 8-month-old infant both recognizes and is
A. Heterosexual relationships fearful of strangers. This developmental milestone is known
B. A love relationship with the father as “stranger anxiety”.
C. A dependency relationship with the father
D. Close relationship with peers 26. The community nurse is conducting a health teaching in
the group of married women. When teaching a woman about
Answer: D. In second grade a child needs to form a close fertility awareness, the nurse should emphasize that the basal
relationships with peers. body temperature:

22. A 5-year-old boy client is scheduled for hernia surgery. A. Should be recorded each morning before any activity
The nurse is preparing to do preoperative teaching with the B. Is the average temperature taken each morning
child. The nurse should knows that the 5-year-old would: C. Can be done with a mercury thermometer but not a
digital one
A. Expect a simple yet logical explanation regarding the D. Has a lower degree of accuracy in predicting ovulation
surgery than the cervical mucus test
B. Asks many questions regarding the condition and the
procedure Answer: A. The basal body temperature (BBT) is the lowest
C. Worry over the impending surgery body temperature of a healthy person that is taken
D. Be uninterested in the upcoming surgery immediately after waking and before getting out of bed. The
BBT usually varies from 36.2 – 36.3 degree Celsius during
Answer: B. A 5-year-old is highly concerned with body menses and for about 5-7 days afterward. About the time of
integrity. The preschool-age child normally asks many ovulation, a slight drop approximately 0.05 degree Celsius in
questions and in a situation such as this, could be expected to temperature may be seen; after ovulation, in concert with
ask even more the increasing progesterone levels of the early luteal phase,
the BBT rises 0.2-0.4 degree Celsius. This elevation remains
23. The nine-year-old client is admitted in the hospital for until 2-3 days before menstruation, or if pregnancy has
almost 1 week and is on bed rest. The child complains of occurred.
being bored and it seems tiresome to stay on bed and doing
nothing. What activity selected by the nurse would the child 27. The community nurse is providing an instruction to the
most likely find stimulating? clients in the health center about the use of diaphragm for
family planning. To evaluate the understanding of the woman,
A. Watching a video the nurse asks her to demonstrate the use of the diaphragm.
B. Putting together a puzzle Which of following statement indicates a need for further
C. Assembling handouts with the nurse for an upcoming health teaching?
staff development meeting
D. Listening to a compact disc A. “I should check the diaphragm carefully for holes every
time I use it.”
Answer: C. A 9-year-old enjoys working and feeling a sense B. “The diaphragm must be left in place for at least 6 hours
of accomplishment. The school-age child also enjoys after intercourse.”
“showing off,” and doing something with the nurse on the
C. “I really need to use the diaphragm and jelly most infant has eczema. Which of the following is the most
during the middle of my menstrual cycle important nursing goal:
D. “I may need a different size diaphragm if I gain or lose
more than 20 pounds” A. Preventing infection
B. Providing for adequate nutrition
Answer: C. The woman must understand that, although the C. Decreasing the itching
“fertile” period is approximately midcycle, hormonal D. Maintaining the comfort level
variations do occur and can result in early or late ovulations.
To be effective, the diaphragm should be inserted before Answer: A. Preventing infection in the infant with eczema is
every intercourse. the nurse’s most important goal. The infant with eczema is
at high risk for infection due to numerous breaks in the
28. The client visits the clinic for prenatal check-up. While skin’s integrity. Intact skin is always the infant’s first line of
waiting for the physician, the nurse decided to conduct health defense against infection.
teaching to the client. The nurse informed the client that
primigravida mother should go to the hospital when which 32. The nurse in the health center is providing immunization
patter is evident? to the children. The nurse is carefully assessing the condition
of the children before giving the vaccines. Which of the
A. Contractions are 2-3 minutes apart, lasting 90 seconds, following would the nurse note to withhold the infant’s
and membranes have ruptured scheduled immunizations?
B. Contractions are 5-10 minutes apart, lasting 30 seconds,
and are felt as strong menstrual cramps A. a dry cough
C. Contractions are 3-5 minutes apart, accompanied by B. a skin rash
rectal pressure and bloody show C. a low-grade fever
D. Contractions are 5 minutes apart, lasting 60 seconds, D. a runny nose
and increasing in intensity
Answer: B. A skin rash could indicate a concurrent
Answer: D. Although instructions vary among birth centers, infectious disease process in the infant. The scheduled
primigravidas should seek care when regular contractions immunizations should be withheld until the status of the
are felt about 5 minutes apart, becoming longer and infant’s health can be determined. Fevers above 38.5
stronger. degrees Celsius, alteration in skin integrity, and infectious-
appearing secretions are indications to withhold
29. A nurse is planning a home visit program to a new mother immunizations.
who is 2 weeks postpartum and breastfeeding, the nurse
includes in her health teaching about the resumption of 33. A mother brought her child in the health center for
fertility, contraception and sexual activity. Which of the hepatitis B vaccination in a series. The mother informs the
following statement indicates that the mother has nurse that the child missed an appointment last month to
understood the teaching? have the third hepatitis B vaccination. Which of the following
statements is the appropriate nursing response to the mother?
A. “Because breastfeeding speeds the healing process
after birth, I can have sex right away and not worry A. “I will examine the child for symptoms of hepatitis B”
about infection” B. “Your child will start the series again”
B. “Because I am breastfeeding and my hormones are C. “Your child will get the next dose as soon as possible”
decreased, I may need to use a vaginal lubricant when D. “Your child will have a hepatitis titer done to determine
I have sex” if immunization has taken place.”
C. “After birth, you have to have a period before you can
get pregnant again’ Answer: C. Continuity is essential to promote active
D. “Breastfeeding protects me from pregnancy because it immunity and give hepatitis B lifelong prophylaxis. Optimally,
keeps my hormones down, so I don’t need any the third vaccination is given 6 months after the first.
contraception until I stop breastfeeding”
34. The community health nurse implemented a new
Answer: B. Prolactin suppresses estrogen, which is needed program about effective breast cancer screening technique
to stimulate vaginal lubrication during arousal. for the female personnel of the health department of
Valenzuela. Which of the following technique should the
30. A community nurse enters the home of the client for nurse consider to be of the lowest priority?
follow-up visit. Which of the following is the most
appropriate area to place the nursing bag of the nurse when
A. Yearly breast exam by a trained professional
conducting a home visit?
B. Detailed health history to identify women at risk
C. Screening mammogram every year for women over age
A. cushioned footstool 50
B. bedside wood table D. Screening mammogram every 1-2 years for women over
C. kitchen countertop age of 40.
D. living room sofa
Answer: B. Because of the high incidence of breast cancer,
Answer: B. A wood surface provides the least chance for all women are considered to be at risk regardless of health
organisms to be present. history.

31. The nurse in the health center is making an assessment to 35. Which of the following technique is considered an aseptic
the infant client. The nurse notes some rashes and small practice during the home visit of the community health nurse?
fluid-filled bumps in the skin. The nurse suspects that the
A. Wrapping used dressing in a plastic bag before placing C. Red beans, green leafy vegetables, and fish for iron
them in the nursing bag and calcium needs plus prenatal vitamins and iron
B. Washing hands before removing equipment from the supplements
nursing bag D. Red meat, milk and eggs for iron and calcium needs plus
C. Using the client’s soap and cloth towel for hand washing prenatal vitamins and iron supplements
D. Placing the contaminated needles and syringes in a
labeled container inside the nursing bag Answer: C. This is appropriate foods that are high in iron and
calcium but would not affect lactose intolerance.
Answer: B. Handwashing is the best way to prevent the
spread of infection. 40. A woman with active tuberculosis (TB) and has visited the
health center for regular therapy for five months wants to
36. The nurse is planning to conduct a home visit in a small become pregnant. The nurse knows that further information
community. Which of the following is the most important is necessary when the woman states:
factor when planning the best time for a home care visit?
A. “Spontaneous abortion may occur in one out of five
A. Purpose of the home visit women who are infected”
B. Preference of the patient’s family B. “Pulmonary TB may jeopardize my pregnancy”
C. Location of the patient’s home C. “I know that I may not be able to have close contact
D. Length of time of the visit will take with my baby until contagious is no longer a problem
D. “I can get pregnant after I have been free of TB for 6
Answer: A. The purpose of the visit takes priority. months”

37. The nurse assigned in the health center is counseling a 30- Answer: D. Intervention is needed when the woman thinks
year-old client requesting oral contraceptives. The client tells that she needs to wait only 6 months after being free of TB
the nurse that she has an active yeast infection that has before she can get pregnant. She needs to wait 1.5-2years
recurred several times in the past year. Which statement by after she is declared to be free of TB before she should
the nurse is inaccurate concerning health promotion actions attempt pregnancy.
to prevent recurring yeast infection?
41. The Department of Health is alarmed that almost 33
A. “During treatment for yeast, avoid vaginal intercourse million people suffer from food poisoning every year.
for one week” Salmonella enteritis is responsible for almost 4 million cases
B. “Wear loose-fitting cotton underwear” of food poisoning. One of the major goals is to promote
C. “Avoid eating large amounts of sugar or sugar-bingeing” proper food preparation. The community health nurse is
D. “Douche once a day with a mild vinegar and water tasks to conduct health teaching about the prevention of
solution” food poisoning to a group of mother everyday. The nurse can
help identify signs and symptoms of specific organisms to
Answer: D. Frequent douching interferes with the natural help patients get appropriate treatment. Typical symptoms of
protective barriers in the vagina that resist yeast infection salmonella include:
and should be avoided.
A. Nausea, vomiting and paralysis
38. During immunization week in the health center, the B. Bloody diarrhea
parent of a 6-month-old infant asks the health nurse, “Why is C. Diarrhea and abdominal cramps
our baby going to receive so many immunizations over a long D. Nausea, vomiting and headache
time period?” The best nursing response would be:
Answer: C. Salmonella organisms cause lower GI symptoms
A. “The number of immunizations your baby will receive
shows how many pediatric communicable and 42. A community health nurse makes a home visit to an
infectious diseases can now be prevented.” elderly person living alone in a small house. Which of the
B. “You need to ask the physician” following observation would be a great concern?
C. “The number of immunizations your baby will receive is
determined by your baby’s health history and age” A. Big mirror in a wall
D. “It is easier on your baby to receive several B. Scattered and unwashed dishes in the sink
immunizations rather than one at a time” C. Shiny floors with scattered rugs
D. Brightly lit rooms
Answer: A. Completion for the recommended schedule of
infant immunizations does not require a large number of Answer: C. It is a safety hazard to have shiny floors and
immunizations, but it also provides protection against scattered rugs because they can cause falls and rugs should
multiple pediatric communicable and infectious diseases. be removed.

39. The community health nurse is conducting a health 43. The health nurse is conducting health teaching about
teaching about nutrition to a group of pregnant women who “safe” sex to a group of high school students. Which of the
are anemic and are lactose intolerant. Which of the following following statement about the use of condoms should the
foods should the nurse especially encourage during the third nurse avoid making?
trimester?
A. “Condoms should be used because they can prevent
A. Cheese, yogurt, and fish for protein and calcium needs infection and because they may prevent pregnancy”
plus prenatal vitamins and iron supplements B. “Condoms should be used even if you have recently
B. Prenatal iron and calcium supplements plus a regular tested negative for HIV”
adult diet
C. “Condoms should be used every time you have sex B. Calf pain when the foot is dorsiflexed
because condoms prevent all forms of sexually C. Inability of the child to extend the legs fully when lying
transmitted diseases” supine
D. “Condoms should be used every time you have sex even D. Pain when the chin is pulled down to the chest
if you are taking the pill because condoms can prevent
the spread of HIV and gonorrhea” Answer: C. Kernig’s sign is the inability of the child to extend
the legs fully when lying supine. This sign is frequently
Answer: C. Condoms do not prevent ALL forms of sexually present in bacterial meningitis. Nuchal rigidity is also
transmitted diseases present in bacterial meningitis and occurs when pain
prevents the child from touching the chin to the chest.
44. The department of health is promoting the breastfeeding
program to all newly mothers. The nurse is formulating a plan 48. A community health nurse makes a home visit to a child
of care to a woman who gave birth to a baby girl. The nursing with an infectious and communicable disease. In planning
care plan for a breast-feeding mother takes into account that care for the child, the nurse must determine that the primary
breast-feeding is contraindicated when the woman: goal is that the:

A. Is pregnant A. Child will experience mild discomfort


B. Has genital herpes infection B. Child will experience only minor complications
C. Develops mastitis C. Child will not spread the infection to others
D. Has inverted nipples D. Public health department will be notified

Answer: A. Pregnancy is one contraindication to breast- Answer: C. The primary goal is to prevent the spread of the
feeding. Milk secretion is inhibited and the baby’s sucking disease to others. The child should experience no
may stimulate uterine contractions. complication. Although the health department may need to
be notified at some point, it is no the primary goal. It is also
45. The City health department conducted a medical mission important to prevent discomfort as much as possible
in Barangay Marulas. Majority of the children in the Barangay
Marulas were diagnosed with pinworms. The community 49. The mother brings her daughter to the health care clinic.
health nurse should anticipate that the children’s chief The child was diagnosed with conjunctivitis. The nurse
complaint would be: provides health teaching to the mother about the proper care
of her daughter while at home. Which statement by the
A. Lack of appetite mother indicates a need for additional information?
B. Severe itching of the scalp
C. Perianal itching A. “I do not need to be concerned about the spreading of
D. Severe abdominal pain this infection to others in my family”
B. “I should apply warm compresses before instilling
Answer: C. Perianal itching is the child’s chief complaint antibiotic drops if purulent discharge is present in my
associated with the diagnosis of pinworms. The itching, in this daughter’s eye”
instance, is often described as being “intense” in nature. C. “I can use an ophthalmic analgesic ointment at
Pinworms infestation usually occurs because the child is in nighttime if I have eye discomfort”
the anus-to-mouth stage of development (child uses the D. “I should perform a saline eye irrigation before instilling,
toilet, does not wash hands, places hands and pinworm eggs the antibiotic drops into my daughter’s eye if purulent
in mouth). Teaching the child hand washing before eating and discharge is present”
after using the toilet can assist in breaking the cycle
Answer: A. Conjunctivitis is highly contagious. Antibiotic
46. The mother brought her daughter to the health center. drops are usually administered four times a day. When
The child has head lice. The nurse anticipates that the nursing purulent discharge is present, saline eye irrigations or eye
diagnosis most closely correlated with this is: applications of warm compresses may be necessary before
instilling the medication. Ophthalmic analgesic ointment or
A. Fluid volume deficit related to vomiting drops may be instilled, especially at bedtime, because
B. Altered body image related to alopecia discomfort becomes more noticeable when the eyelids are
C. Altered comfort related to itching closed.
D. Diversional activity deficit related to hospitalization
50. A community health nurse is caring for a group of flood
Answer: C. Severe itching of the scalp is the classic sign and victims in Marikina area. In planning for the potential needs
symptom of head lice in a child. In turn, this would lead to of this group, which is the most immediate concern?
the nursing diagnosis of “altered comfort”.
A. Finding affordable housing for the group
47. The mother brings a child to the health care clinic because B. Peer support through structured groups
of severe headache and vomiting. During the assessment of C. Setting up a 24-hour crisis center and hotline
the health care nurse, the temperature of the child is 40 D. Meeting the basic needs to ensure that adequate food,
degree Celsius, and the nurse notes the presence of nuchal shelter and clothing are available
rigidity. The nurse is suspecting that the child might be
suffering from bacterial meningitis. The nurse continues to Answer: D. The question asks about the immediate concern.
assess the child for the presence of Kernig’s sign. Which The ABCs of community health care are always attending to
finding would indicate the presence of this sign? people’s basic needs of food, shelter, and clothing

A. Flexion of the hips when the neck is flexed from a lying


position
5. The physician has given instruction to the nurse that the
PNLE III Nursing Practice client can be ambulated on crutches, with no weight bearing
on the affected limb. The nurse is aware that the appropriate
The scope of this Nursing Test III is parallel to the NP3 NLE
crutch gait for the nurse to teach the client would be:
Coverage:
 Medical Surgical Nursing
1. The nurse is going to replace the Pleur-O-Vac attached to A. Tripod gait
the client with a small, persistent left upper lobe B. Two-point gait
pneumothorax with a Heimlich Flutter Valve. Which of the C. Four-point gait
following is the best rationale for this? D. Three-point gait

Answer: D. The three-point gait is appropriate when weight


A. Promote air and pleural drainage
bearing is not allowed on the affected limb. The swing-to
B. Prevent kinking of the tube
and swing-through crutch gaits may also be used when only
C. Eliminate the need for a dressing
one leg can be used for weight bearing
D. Eliminate the need for a water-seal drainage
6. The client is transferred to the nursing care unit from the
Answer: D. The Heimlich flutter valve has a one-way valve
operating room after a transurethral resection of the prostate.
that allows air and fluid to drain. Underwater seal drainage
The client is complaining of pain in the abdomen area. The
is not necessary. This can be connected to a drainage bag for
nurse suspects of bladder spasms, which of the following is
the patient’s mobility. The absence of a long drainage tubing
the best nursing action to minimize the pain felt by the client?
and the presence of a one-way valve promote effective
therapy
A. Advising the client not to urinate around catheter
2. The client with acute pancreatitis and fluid volume deficit is B. Intermittent catheter irrigation with saline
transferred from the ward to the ICU. Which of the following C. Giving prescribed narcotics every 4 hour
will alert the nurse? D. Repositioning catheter to relieve pressure

Answer: A. The client needs to be told before surgery that


A. Decreased pain in the fetal position
the catheter causes the urge to void. Attempts to void
B. Urine output of 35mL/hr
around the catheter cause the bladder muscles to contract
C. CVP of 12 mmHg
and result in painful spasms.
D. Cardiac output of 5L/min
7. A client is diagnosed with peptic ulcer. The nurse caring for
Answer: C. C = the normal CVP is 0-8 mmHg. This value
the client expects the physician to order which diet?
reflects hypervolemia. The right ventricular function of this
client reflects fluid volume overload, and the physician
should be notified. A. NPO
B. Small feedings of bland food
3. The nurse in the morning shift is making rounds in the ward. C. A regular diet given frequently in small amounts
The nurse enters the client’s room and found the client in D. Frequent feedings of clear liquids
discomfort condition. The client complains of stiffness in the
joints. To reduce the early morning stiffness of the joints of Answer; B. Bland feedings should be given in small amounts
the client,the nurse can encourage the client to: on a frequent basis to neutralize the hydrochloric acid and
to prevent overload
A. Sleep with a hot pad
8. The nurse is going to insert a Miller-Abbott tube to the
B. Take to aspirins before arising, and wait 15 minutes
client. Before insertion of the tube, the balloon is tested for
before attempting locomotion
patency and capacity and then deflated. Which of the
C. Take a hot tub bath or shower in the morning
following nursing measure will ease the insertion to the tube?
D. Put joints through passive ROM before trying to move
them actively
A. Positioning the client in Semi-Fowler’s position
Answer: C. A hot tub bath or shower in the morning helps B. Administering a sedative to reduce anxiety
many patients limber up and reduces the symptoms of early C. Chilling the tube before insertion
morning stiffness. Cold and ice packs are used to a lesser D. Warming the tube before insertion
degree, though some clients state that cold decreases
localized pain, particularly during acute attacks. Answer: C. Chilling the tube before insertion assists in
relieving some of the nasal discomfort. Water-soluble
4. The nurse is planning of care to a client with peptic ulcer lubricants along with viscous lidocaine (Xylocaine) may also
disease. To avoid the worsening condition of the client, the be used. It is usually only lightly lubricated before insertion
nurse should carefully plan the diet of the client. Which of the
following will be included in the diet regime of the client? 9. The physician ordered a low-sodium diet to the client.
Which of the following food will the nurse avoid to give to the
client?
A. Eating mainly bland food and milk or dairy products
B. Reducing intake of high-fiber foods
C. Eating small, frequent meals and a bedtime snack A. Orange juice.
D. Eliminating intake of alcohol and coffee B. Whole milk.
C. Ginger ale.
Answer: D. These substances stimulate the production of D. Black coffee.
hydrochloric acid, which is detrimental in peptic ulcer
disease.
Answer: B. Whole milk should be avoided to include in the small pillow under thehead and neck. Neck muscles have
client’s diet because it has 120 mg of sodium in 8 0z of milk been affected during a thyroidectomy, support essential for
comfort and incisional support.
10. Mr. Bean, a 70-year-old client is admitted in the hospital
for almost one month. The nurse understands that prolonged 15. What would be the recommended diet the nurse will
immobilization could lead to decubitus ulcers. Which of the implement to a client with burns of the head, face, neck and
following would be the least appropriate nursing intervention anterior chest?
in the prevention of decubitus?
A. Serve a high-protein, high-carbohydrate diet
A. Giving backrubs with alcohol B. Encourage full liquid diet
B. Use of a bed cradle C. Serve a high-fat diet, high-fiber diet
C. Frequent assessment of the skin D. Monitor intake to prevent weight gain
D. Encouraging a high-protein diet
Answer: A. A positive nitrogen balance is important for
Answer: A. Alcohol is extremely drying and contributes to meeting metabolic needs, tissue repair, and resistance to
skin break down. An emollient lotion should be used. infection. Caloric goals may be as high as 5000 calories per
day.
11. The physician prescribed digoxin 0.125 mg PO qd to a
client and instructed the nurse that the client is on high- 16. A client with multiple fractures of both lower extremities
potassium diet. High potassium foods are recommended in is admitted for 3 days ago and is on skeletal traction. The
the diet of a client taking digitalis preparations because a low client is complaining of having difficulty in bowel movement.
serum potassium has which of the following effects? Which of the following would be the most appropriate
nursing intervention?
A. Potentiates the action of digoxin
B. Promotes calcium retention A. Administer an enema
C. Promotes sodium excretion B. Perform range-of-motion exercise to all extremities
D. Puts the client at risk for digitalis toxicity C. Ensure maximum fluid intake (3000ml/day)
D. Put the client on the bedpan every 2 hours
Answer: D. Potassium influences the excitability of nerves
and muscles. When potassium is low and the client is on Answer: C. The best early intervention would be to increase
digoxin, the risk of digoxin toxicity is increased. fluid intake, because constipation is common when activity
is decreased or usual routines have been interrupted.
12. The nurse is caring for a client who is transferred from the
operating room for pneumonectomy. The nurse knows that 17. John is diagnosed with Addison’s disease and admitted in
immediately following pneumonectomy; the client should be the hospital. What would be the appropriate nursing care for
in what position? John?

A. Supine on the unaffected side A. Reducing physical and emotional stress


B. Low-Fowler’s on the back B. Providing a low-sodium diet
C. Semi-Fowler’s on the affected side C. Restricting fluids to 1500ml/day
D. Semi-Fowler’s on the unaffected side D. Administering insulin-replacement therapy

Answer: C. This position allows maximum expansion, Answer: A. Because the client’s ability is to react to stress is
ventilation, and perfusion of the remaining lung. decreased, maintaining a quiet environment becomes A
nursing priority. Dehydration is a common problem in
13. A client is placed on digoxin, high potassium foods are Addison’s disease, so close observation of the client’s
recommended in the diet of the client. Which of the following hydration level is crucial. To promote optimal hydration and
foods willthe nurse give to the client? sodium intake, fluid intake is increased, particularly fluid
containing electrolytes, such as broths, carbonated
A. Whole grain cereal, orange juice, and apricots beverages, and juices.
B. Turkey, green bean, and Italian bread
C. Cottage cheese, cooked broccoli, and roast beef 18. Mr. Smith is scheduled for an above-the-knee amputation.
D. Fish, green beans and cherry pie After the surgery he was transferred to the nursing care unit.
The nurse assigned to him knows that 72 hours after the
Answer: C. This position allows maximum expansion, procedure the client should be positioned properly to prevent
ventilation, and perfusion of the remaining lung. contractures. Which of the following is the best position to
the client?
14. The nurse is assigned to care to a client who undergone
thyroidectomy. What nursing intervention is important A. Side-lying, alternating left and right sides
during the immediate postoperative period following a B. Sitting in a reclining chair twice a day
thyroidectomy? C. Lying on abdomen several times daily
D. Supine with stump elevated at least 30 degrees
A. Assess extremities for weakness and flaccidity
B. Support the head and neck during position changes Answer: C. At about 48-72 hours, the client must be turned
C. Position the client in high Fowler’s onto the abdomen to prevent flexion contractures.
D. Medicate for restlessness and anxiety
19. A client is scheduled to have an inguinal herniorraphy in
Answer: B. Stress on the suture line should be avoided. the outpatient surgical department. The nurse is providing
Prevent flexion or hyperextension of the neck, and provide a health teaching about post surgical care to the client. Which
of the following statement if made by the client would reflect 23. Following a needle biopsy of the kidney, which
the need for more teaching? assessment is an indication that the client is bleeding?

A. “I should call the physician if I have a cough or cold A. Slow, irregular pulse
before surgery” B. Dull, abdominal discomfort
B. “I will be able to drive soon after surgery” C. Urinary frequency
C. “I will not be able to do any heavy lifting for 3-6 weeks D. Throbbing headache
after surgery”
D. “I should support my incision if I have to cough or turn” Answer: B. An accumulation of blood from the kidney into
the abdomen would manifest itself with these symptoms
Answer:B. The client should not drive for 2 weeks after
surgery to avoid stress on the incision. This reflects a need 24. A client with acute bronchitis is admitted in the hospital.
for additional teaching. The nurse assigned to the client is making a plan of care
regarding expectoration of thick sputum. Which nursing
20. Ms Jones is brought to the emergency room and is action is most effective?
complaining of muscle spasms, numbness, tremors and
weakness in the arms and legs. The client was diagnosed with A. Place the client in a lateral position every 2 hour
multiple sclerosis. The nurse assigned to Ms. Jones is aware B. Splint the patient’s chest with pillows when coughing
that she has to prevent fatigue to the client to alleviate the C. Use humified oxygen
discomfort. Which of the following teaching is necessary to D. Offer fluids at regular intervals
prevent fatigue?
Answer: D. Fluids liquefy secretions and therefore make it
A. Avoid extremes in temperature easier to expectorate
B. Install safety devices in the home
C. Attend support group meetings 25. The nurse is going to assess the bowel sound of the client.
D. Avoid physical exercise For accurate assessment of the bowel sound, the nurse
should listen for at least:
Answer: A. Extremes in heat and cold will exacerbate
symptoms. Heat delays transmission of impulses and A. 5 minutes
increases fatigue. B. 60 seconds
C. 30 seconds
D. 2 minutes
21. Mr. Stewart is in sickle cell crisis and complaining pain in
the joints and difficulty of breathing. On the assessment of Answer: D. Physical assessment guidelines recommend
the nurse, his temperature is 38.1 ºC. The physician ordered listening for atleast 2 minutes in each quadrant (and up to 5
Morphine sulfate via patient-controlled analgesia (PCA), and minutes, not at least 5 minutes).
oxygen at 4L/min. A priority nursing diagnosis to Mr. Stewart
is risk for infection. A nursing intervention to assist in 26. The nurse encourages the client to wear compression
preventing infection is: stockings. What is the rationale behind in using compression
stockings?
A. Using standard precautions and medical asepsis
B. Enforcing a “no visitors” rule A. Compression stockings promote venous return
C. Using moist heat on painful joints B. Compression stockings divert blood to major vessels
D. Monitoring a vital signs every 2 hour C. Compression stockings decreases workload on the heart
D. Compression stockings improve arterial circulation
Answer: A. Vigilant implementation of standard precautions
and medical asepsis is an effective means of preventing Answer: A. Compression stockings promote venous return
infection and prevent peripheral pooling.

22. Mrs. Maupin is a professor in a prestigious university for 27. Mr. Whitman is a stroke client and is having difficulty in
30 years. After lecture, she experience blurring of vision and swallowing. Which is the best nursing intervention is most
tiredness. Mrs. Maupin is brought to the emergency likely to assist the client?
department. On assessment, the nurse notes that the blood
pressure of the client is 139/90. Mrs. Maupin has been
A. Placing food in the unaffected side of the mouth
diagnosed with essential hypertension and placed on
B. Increasing fiber in the diet
medication to control her BP. Which potential nursing
C. Asking the patient to speak slowly
diagnosis will be a priority for discharge teaching?
D. Increasing fluid intake

A. Sleep Pattern disturbance Answer: A. Placing food in the unaffected side of the mouth
B. Impaired physical mobility assists in the swallowing process because the client has
C. Noncompliance sensation on that side and will have more control over the
D. Fluid volume excess swallowing process.

Answer: C. Noncompliance is a major problem in the 28. Following nephrectomy, the nurse closely monitors the
management of chronic disease. In hypertension, the client urinary output of the client. Which assessment finding is an
often does not feel ill and thus does not see a need to follow early indicator of fluid retention in the postoperative period?
a treatment regimen.
A. Periorbital edema
B. Increased specific gravity of urine
C. A urinary output of 50mL/hr that the priority nursing intervention in the immediate post
D. Daily weight gain of 2 lb or more operative care of this client is:

Answer: D. Daily weights are taken following nephrectomy. A. Elevate the stump on a pillow for the first 24 hours
Daily increases of 2 lb or more are indicative of fluid B. Encourage use of trapeze
retention and should be reported to the physician. Intake C. Position the client prone periodically
and output records may also reflect this imbalance. D. Apply a cone-shaped dressing

29. A nurse is completing an assessment to a client with Answer: A. The elevation of the stump on a pillow for the
cirrhosis. Which of the following nursing assessment is first 24 hours decreases edema and increases venous return.
important to notify the physician?
34. A client with a diagnosis of gastric ulcer is complaining of
A. Expanding ecchymosis syncope and vertigo. What would be the initial nursing
B. Ascites and serum albumin of 3.2 g/dl intervention by the nurse?
C. Slurred speech
D. Hematocrit of 37% and hemoglobin of 12g/dl A. Monitor the client’s vital signs
B. Keep the client on bed rest
Answer: A. Clients with cirrhosis have already coagulation C. Keep the patient on bed rest
due to thrombocytopenia and vitamin K deficiency. This D. Give a stat dose of Sucralfate (Carafate)
could be a sign of bleeding
Answer: B. The priority is to maintain client’s safety. With
30. Mr. Park is 32-year-old, a badminton player and has a syncope and vertigo, the client is at high risk for falling.
type 1 diabetes mellitus. After the game, the client complains
of becoming diaphoretic and light-headedness. The client 35. After a right lower lobectomy on a 55-year-old client,
asks the nurse how to avoid this reaction. The nurse will which action should the nurse initiate when the client is
recommend to: transferred from the post anesthesia care unit?

A. Allow plenty of time after the insulin injection and A. Notify the family to report the client’s condition
before beginning the match B. Immediately administer the narcotic as ordered
B. Eat a carbohydrate snack before and during the C. Keep client on right side supported by pillows
badminton match D. Encourage coughing and deep breathing every 2 hours
C. Drink plenty of fluids before, during, and after bed time
D. Take insulin just before starting the badminton match Answer: D. Coughing and deep breathing are essential for
re-expansion of the lung
Answer: B. Exercise enhances glucose uptake, and the client
is at risk for an insulin reaction. Snacks with carbohydrates 36. The nurse is providing a discharge instruction about the
will help. prevention of urinary stasis to a client with frequent bladder
infection. Which of the following will the nurse include in the
31. A client is rushed to the emergency room due to serious instruction?
vehicle accident. The nurse is suspecting of head injury.
Which of the following assessment findings would the nurse
A. Drink 3-4 quarts of fluid every day
report to the physician?
B. Empty the bladder every 2-4 hours while awake
C. Encourage the use of coffee, tea, and colas for their
A. CVP of 5mmHa diuretic effect
B. Glasgow Coma Scale score of 13 D. Teach Kegel exercises to control bladder flow
C. Polyuria and dilute urinary output
D. Insomnia Answer:B. Avoiding stasis of urine by emptying the bladder
every 2-4 hours will prevent overdistention of the bladder
Answer: C. These are symptoms of diabetes insipidus. The and future urinary tract infections.
patient can become hypovolemic and vasopressin may
reverse the Polyuria. 37. A male client visits the clinic for check-up. The client tells
the nurse that there is a yellow discharge from his penis. He
32. Mrs. Moore, 62-year-old, with diabetes is in the also experiences a burning sensation when urinating. The
emergency department. She stepped on a sharp sea shells nurse is suspecting of gonorrhea. What teaching is necessary
while walking barefoot along the beach. Mrs. Moore did not for this client?
notice that the object pierced the skin until later that evening.
What problem does the client most probably have?
A. Sex partner of 3 months ago must be treated
B. Women with gonorrhea are symptomatic
A. Nephropathy C. Use a condom for sexual activity
B. Macroangiopathy D. Sex partner needs to be evaluated
C. Carpal tunnel syndrome
D. Peripheral neuropathy Answer: D. If infected, the sex partner must be evaluated
and treated
Answer: D. Peripheral neuropathy refers to nerve damage of
the hands and feet. The client did not notice that the object 38. A client with AIDS is admitted in the hospital. He is
pierced the skin. receiving intravenous therapy. While the nurse is assessing
the IV site, the client becomes confused and restless and the
33. A client with gangrenous foot has undergone a below- intravenous catheter becomes disconnected and minimal
knee amputation. The nurse in the nursing care unit knows
amount of the client’s blood spills onto the floor. Which B. “Eating six meals a day can be a bother, can’t it?”
action will the nurse take to remove the blood spill? C. “Some clients can tolerate three meals a day by the
time they leave the hospital. Maybe it will be a little
A. Promptly clean with a 1:10 solution of household longer for you.”
bleach and water D. “ It varies from client to client, but generally in 6-12
B. Promptly clean up the blood spill with full-strength months most clients can return to their previous meal
antimicrobial cleaning solution patterns”
C. Immediately mop the floor with boiling water
D. Allow the blood to dry before cleaning to decrease the Answer:D. In response to the question of the client, the nurse
possibility of cross-contamination needs to provide brief, accurate information. Some clients
who have had gastrectomies are able to tolerate three meals
Answer: A. A 1:10 solution of household bleach and water is a day before discharge from the hospital. However, for the
recommended by the Centers for Disease Control and majority of clients, it takes 6-12 months before their
Prevention to kill the human immunodeficiency virus (HIV). surgically reduced stomach has stretched enough to
accommodate a larger meal
39. Before surgery, the physician ordered pentobarbital
sodium (Nembutal) for the client to sleep. The night before 43. A male client with cirrhosis is complaining of belly pain,
the scheduled surgery, the nurse gave the pre-medication. itchiness and his breasts are getting larger and also the
One hour later the client is still unable to sleep. The nurse abdomen. The client is so upset because of the discomfort
review the client’s chart and note the physician’s prescription and asks the nurse why his breast and abdomen are getting
with an order to repeat. What should the nurse do next? larger. Which of the following is the appropriate nursing
response?
A. Rub the client’s back until relaxed
B. Prepare a glass of warm milk A. “How much of a difference have you noticed”
C. Give the second dose of pentobarbital sodium B. “It’s part of the swelling your body is experiencing”
D. Explore the client’s feelings about surgery C. “It’s probably because you have been less physically
active”
Answer: D. Given the data, presurgical anxiety is suspected. D. “Your liver is not destroying estrogen hormones that all
The client needs an opportunity to talk about concerns men produce”
related to surgery before further actions (which may mask
the anxiety). Answer: A. This allows the client to elaborate his concern
and provides the nurse a baseline of assessment
40. The nurse on the night shift is making rounds in the
nursing care unit. The nurse is about to enter to the client’s 44. A client is diagnosed with detached retina and scheduled
room when a ventilator alarm sounds, what is the first action for surgery. Preoperative teaching of the nurse to the client
the nurse should do? includes:

A. Assess the lung sounds A. No eye pain is expected postoperatively


B. Suction the client right away B. Semi-fowler’s position will be used to reduce pressure
C. Look at the client in the eye.
D. Turn and position the client C. Eye patches may be used postoperatively
D. Return of normal vision is expected following surgery
Answer: C. A quick look at the client can help identify the
type and cause of the ventilator alarm. Disconnection of the Answer: C. Use of eye patches may be continued
tube from the ventilator, bronchospasm, and anxiety are postoperatively, depending on surgeon preference. This is
some of the obvious reasons that could trigger an alarm. done to achieve >90% success rate of the surgery.

41. What effective precautions should the nurse use to 45. A 70-year-old client is brought to the emergency
control the transmission of methicillin-resistant department with a caregiver. The client has manifestations of
Staphylococcus aureus (MRSA)? anorexia, wasting of muscles and multiple bruises. What
nursing interventions would the nurse implement?
A. Use gloves and handwashing before and after client
contact A. Talk to the client about the caregiver and support
B. Do nasal cultures on healthcare providers system
C. Place the client on total isolation B. Complete a gastrointestinal and neurological
D. Use mask and gown during care of the MRSA client assessment
C. Check the lab data for serum albumin, hematocrit and
Answer: A. Contact isolation has been advised by the hemoglobin
Centers for Disease Control and Prevention (CDC) to control D. Complete a police report on elder abuse
transmission of MRSA, which includes gloves and
handwashing. Answer: B. Assessment and more data collection are needed.
The client may have gastrointestinal or neurological
42. The postoperative gastrectomy client is scheduled for problems that account for the symptoms. The anorexia
discharge. The client asks the nurse, “When I will be allowed could result from medications, poor dentition, or indigestion,
to eat three meals a day like the rest of my family?”. The the bruises may be attributed to ataxia, frequent falls,
appropriate nursing response is: vertigo, or medication.

46. A nurse is providing a discharge instruction to the client


A. “You will probably have to eat six meals a day for the
about the self-catheterization at home. Which of the
rest of your life.”
following instructions would the nurse include?
A. Wash the catheter with soap and water after each use Answer: B. Toxoplasmosis is an opportunistic infection and a
B. Lubricate the catheter with Vaseline parasite of birds and mammals. The oocysts remain infectious
C. Perform the Valsalva maneuver to promote insertion in moist soil for about 1 year
D. Replace the catheter with a new one every 24 hour

Answer: A. The catheter should be washed with soap and


PNLE IV Nursing Practice
water after withdrawal and placed in a clean container. It
The scope of this Nursing Test IV is parallel to the NP4 NLE
can be reused until it is too hard or too soft for insertion.
Coverage:
Self-care, prevention of complications, and cost-
 Medical Surgical Nursing
effectiveness are important in home management.
1. Following spinal injury, the nurse should encourage the
client to drink fluids to avoid:
47. The nurse in the nursing care unit is assigned to care to a
client who is Immunocompromised. The client tells the nurse
that his chest is painful and the blisters are itchy. What would A. Urinary tract infection.
be the nursing intervention to this client? B. Fluid and electrolyte imbalance.
C. Dehydration.
D. Skin breakdown.
A. Call the physician
B. Give a prn pain medication
Answer: A. Clients in the early stage of spinal cord damage
C. Clarify if the client is on a new medication
experience an atonic bladder, which is characterized by the
D. Use gown and gloves while assessing the lesions
absence of muscle tone, an enlarged capacity, no feeling of
discomfort with distention, and overflow with a large
Answer: D. The client may have herpes zoster (shingles), a
residual. This leads to urinary stasis and infection. High fluid
viral infection. The nurse should use standard precautions in
intake limits urinary stasis and infection by diluting the urine
assessing the lesions. Immunocompromised clients are at
and increasing urinary output.
risk for infection.
2. The client is transferred from the operating room to
48. A client is admitted and has been diagnosed with bacterial
recovery room after an open-heart surgery. The nurse
(meningococcal) meningitis. The infection control registered
assigned is taking the vital signs of the client. The nurse
nurse visits the staff nurse caring to the client. What
notified the physician when the temperature of the client
statement made by the nurse reflects an understanding of
rises to 38.8 ºC or 102 ºF because elevated temperatures:
the management of this client?

A. May be a forerunner of hemorrhage.


A. speech pattern may be altered
B. Are related to diaphoresis and possible chilling.
B. Respiratory isolation is necessary for 24 hours after
C. May indicate cerebral edema.
antibiotics are started
D. Increase the cardiac output.
C. Perform skin culture on the macular popular rash
D. Expect abnormal general muscle contractions
Answer: D. The temperature of 102 ºF (38.8ºC) or greater
lead to an increased metabolism and cardiac workload.
Answer: B. After a minimum of 24 hours of IV antibiotics, the
client is no longer considered communicable. Evaluation of
3. After radiation therapy for cancer of the prostate, the
the nurse’s knowledge is needed for safe care and continuity
client experienced irritation in the bladder. Which of the
of care
following sign of bladder irritability is correct?
49. A 18-year-old male client had sustained a head injury
from a motorbike accident. It is uncertain whether the client A. Hematuria
may have minimal but permanent disability. The family is B. Dysuria
concerned regarding the client’s difficulty accepting the C. Polyuria
possibility of long term effects. Which nursing diagnosis is D. Dribbling
best for this situation?
ANSWER: B. Dysuria, nocturia, and urgency are all signs an
irritable bladder after radiation therapy.
A. Nutrition, less than body requirements
B. Injury, potential for sensory-perceptual alterations
4. A client is diagnosed with a brain tumor in the occipital
C. Impaired mobility, related to muscle weakness
lobe. Which of the following will the client most likely
D. Anticipatory grieving, due to the loss of independence
experience?
Answer: D. Stem of the question supports this choice by
stating that the client has difficulty accepting the potential A. Visual hallucinations.
disability. B. Receptive aphasia.
C. Hemiparesis.
50. A client with AIDS is scheduled for discharge. The client D. Personality changes.
tells the nurse that one of his hobbies at home is gardening.
What will be the discharge instruction of the nurse to the ANSWER: A. The occipital lobe is involve with visual
client knowing that the client is prone to toxoplasmosis? interpretation.

5. A client with Addison’s disease has a blood pressure of


A. Wash all vegetables before cooking
65/60. The nurse understands that decreased blood pressure
B. Wear gloves when gardening
of the client with Addison’s disease involves a disturbance in
C. Wear a mask when travelling to foreign countries
the production of:
D. Avoid contact with cats and birds

A. Androgens
B. Glucocorticoids 10. The physician advised the client with Hemiparesis to use a
C. Mineralocorticoids cane. The client asks the nurse why cane will be needed. The
D. Estrogen nurse explains to the client that cane is advised specifically to:

ANSWER: C. Mineralocorticoids such as aldosterone cause A. Aid in controlling involuntary muscle movements.
the kidneys to retain sodium ions. With sodium, water is B. Relieve pressure on weight-bearing joints.
also retained, elevating blood pressure. Absence of this C. Maintain balance and improve stability.
hormone thus causes hypotension. D. Prevent further injury to weakened muscles.

6. The nurse is planning to teach the client about a ANSWER: C. Hemiparesis creates instability. Using a cane
spontaneous pneumothorax. The nurse would base the provides a wider base of support and, therefore greater
teaching on the understanding that: stability.

A. Inspired air will move from the lung into the pleural 11. The nurse is conducting a discharge teaching regarding
space. the prevention of further problems to a client who
B. There is greater negative pressure within the chest undergone surgery for carpal tunnel syndrome of the right
cavity. hand. Which of the following instruction will the nurse
C. The heart and great vessels shift to the affected side. includes?
D. The other lung will collapse if not treated immediately.
A. Learn to type using your left hand only.
ANSWER: B. As a person with a tear in the lung inhales, air B. Avoid typing in a long period of time.
moves through that opening into the intrapleural and causes C. Avoid carrying heavy things using the right hand.
partial or complete collapse of the lungs. D. Do manual stretching exercise during breaks.

7. During an assessment, the nurse recognizes that the client ANSWER: D. Manual stretching exercises will assist in
has an increased risk for developing cancer of the tongue. keeping the muscles and tendons supple and pliable,
Which of the following health history will be a concern? reducing the traumatic consequences of repetitive activity.

A. Heavy consumption of alcohol. 12. A female client is admitted because of recurrent urinary
B. Frequent gum chewing. tract infections. The client asks the nurse why she is prone to
C. Nail biting. this disease. The nurse states that the client is most
D. Poor dental habits. susceptible because of:

ANSWER: A. Heavy alcohol ingestion predisposes an A. Continuity of the mucous membrane.


individual to the development of oral cancer. B. Inadequate fluid intake.
C. The length of the urethra.
8. The client in the orthopedic unit asks the nurse the reason D. Poor hygienic practices.
behind why compact bone is stronger than cancellous bone.
Which of the following is the correct response of the nurse? ANSWER: C. The length of the urethra is shorter in females
than in males; therefore microorganisms have a shorter
A. Compact bone is stronger than cancellous bone because distance to travel to reach the bladder. The proximity of the
of its greater size. meatus to the anus in females also increases this incidence.
B. Compact bone is stronger than cancellous bone because
of its greater weight. 13. A 55-year-old client is admitted with chest pain that
C. Compact bone is stronger than cancellous bone because radiates to the neck, jaw and shoulders that occurs at rest,
of its greater volume. with high body temperature, weak with generalized sweating
D. Compact bone is stronger than cancellous bone and with decreased blood pressure. A myocardial infarction is
because of its greater density. diagnosed. The nurse knows that the most accurate
explanation for one of these presenting adaptations is:
ANSWER: D. The greater the density of compact bone makes
it stronger than the cancellous bone. Compact bone forms A. Catecholamines released at the site of the infarction
from cancellous bone by the addition of concentric rings of causes intermittent localized pain.
bones substances to the marrow spaces of cancellous bone. B. Parasympathetic reflexes from the infarcted
The large marrow spaces are reduced to haversian canals. myocardium causes diaphoresis.
C. Constriction of central and peripheral blood vessels
9. The nurse is reviewing the laboratory results of the client. causes a decrease in blood pressure.
In reviewing the results of the RBC count, the nurse D. Inflammation in the myocardium causes a rise in the
understands that the higher the red blood cell count, the : systemic body temperature.

A. Greater the blood viscosity. ANSWER: D. Temperature may increase within the first 24
B. Higher the blood pH. hours and persist as long as a week.
C. Less it contributes to immunity.
D. Lower the hematocrit. 14. Following an amputation of a lower limb to a male client,
the nurse provides an instruction on how to prevent a hip
ANSWER: A. Viscosity, a measure of a fluid’s internal flexion contracture. The nurse should instruct the client to:.
resistance to flow, is increased as the number of red cells
suspended in plasma. A. Perform quadriceps muscle setting exercises twice a day.
B. Sit in a chair for 30 minutes three times a day.
C. Lie on the abdomen 30 minutes every four hours.
D. Turn from side to side every 2 hours. 19. A 38-year-old client with severe hypertension is
hospitalized. The physician prescribed a Captopril (Capoten)
ANSWER: C. The hips are in extension when the client is and Alprazolam (Xanax) for treatment. The client tells the
prone; this keeps the hips from flexing. nurse that there is something wrong with the medication and
nursing care. The nurse recognizes this behavior is probably a
15. The physician scheduled the client with rheumatoid manifestation of the client’s:
arthritis for the injection of hydrocortisone into the knee joint.
The client asks the nurse why there is a need for this injection. A. Reaction to hypertensive medications.
The nurse explains that the most important reason for doing B. Denial of illness.
this is to: C. Response to cerebral anoxia.
D. Fear of the health problem.
A. Lubricate the joint.
B. Prevent ankylosis of the joint. ANSWER: D. Clients adapting to illness frequently feel afraid
C. Reduce inflammation. and helpless and strike out at health team members as a
D. Provide physiotherapy. way of maintaining control or denying their fear.

ANSWER: C. Steroids have an anti-inflammatory effect that 20. Before discharge, the nurse scheduled the client who had
can reduce arthritic pannus formation. a colostomy for colorectal cancer for discharge instruction
about resuming activities. The nurse should plan to help the
16. The nurse is assigned to care for a 57-year-old female client understands that:
client who had a cataract surgery an hour ago. The nurse
should: A. After surgery, changes in activities must be made to
accommodate for the physiologic changes caused by
A. Advise the client to refrain from vigorous brushing of the operation.
teeth and hair. B. Most sports activities, except for swimming, can be
B. Instruct the client to avoid driving for 2 weeks. resumed based on the client’s overall physical condition.
C. Encourage eye exercises to strengthen the ocular C. With counseling and medical guidance, a near normal
musculature. lifestyle, including complete sexual function is possible.
D. Teach the client coughing and deep-breathing D. Activities of daily living should be resumed as quickly as
techniques. possible to avoid depression and further dependency.

ANSWER: A. Activities such as rigorous brushing of hair and ANSWER: C. There are few physical restraints on activity
teeth cause increased intraocular pressure and may lead to postoperatively, but the client may have emotional
hemorrhage in the anterior chamber. problems resulting from the body image changes.

17. A client with AIDS develops bacterial pneumonia is 21. A client is scheduled for bariatric surgery. Preoperative
admitted in the emergency department. The client’s arterial teaching is done. Which of the following statement would
blood gases is drawn and the result is PaO2 80mmHg. then alert the nurse that further teaching to the client is necessary?
arterial blood gases are drawn again and the level is reduced
from 80 mmHg to 65 mmHg. The nurse should; A. “I will be limiting my intake to 600 to 800 calories a day
once I start eating again.”
A. Have arterial blood gases performed again to check for B. “I’m going to have a figure like a model in about a
accuracy. year.”
B. Increase the oxygen flow rate. C. “I need to eat more high-protein foods.”
C. Notify the physician. D. “I will be going to be out of bed and sitting in a chair the
D. Decrease the tension of oxygen in the plasma. first day after surgery.”.

ANSWER: C. This decrease in PaO2 indicates respiratory ANSWER: B. Clients need to be prepared emotionally for the
failure; it warrants immediate medical evaluation. body image changes that occur after bariatric surgery.
Clients generally experience excessive abdominal skin folds
18. An 18-year-old college student is brought to the after weight stabilizes, which may require a panniculectomy.
emergency department due to serious motor vehicle accident. Body image disturbance often occurs in response to
Right above-knee-amputation is done. Upon awakening from incorrectly estimating one’s size; it is not uncommon for the
surgery the client tells the nurse, “What happened to me? I client to still feel fat no matter how much weight is lost.
cannot remember anything?” Which of the following would
be the appropriate initial nursing response? 22. The client who had transverse colostomy asks the nurse
about the possible effect of the surgery on future sexual
A. “You sound concerned; You’ll probably remember more relationship. What would be the best nursing response?
as you wake up.”
B. “Tell me what you think happened.” A. The surgery will temporarily decrease the client’s sexual
C. “You were in a car accident this morning.” impulses.
D. “An amputation of your right leg was necessary because B. Sexual relationships must be curtailed for several weeks.
of an accident.” C. The partner should be told about the surgery before any
sexual activity.
ANSWER: C. This is truthful and provides basic information D. The client will be able to resume normal sexual
that may prompt recollection of what happened; it is a relationships.
starting point.
ANSWER: D. Surgery on the bowel has no direct anatomic or 27. The physician prescribed Albuterol (Proventil) to the
physiologic effect on sexual performance. However, the client with severe asthma. After the administration of the
nurse should encourage verbalization. medication the nurse should monitor the client for:

23. A 75-year-old male client tells the nurse that his wife has A. Palpitation
osteoporosis and asks what chances he had of getting also B. Visual disturbance
osteoporosis like his wife. Which of the following is the C. Decreased pulse rate
correct response of the nurse? D. Lethargy

A. “This is only a problem for women.” ANSWER: A. Albuterol’s sympathomimetic effect causes
B. “You are not at risk because of your small frame.” cardiac stimulation that may cause tachycardia and
C. “You might think about having a bone density test,” palpitation.
D. “Exercise is a good way to prevent this problem.”
28. A client is receiving diltiazem (Cardizem). What should the
ANSWER: C. Osteoporosis is not restricted to women; it is a nurse include in a teaching plan aimed at reducing the side
potential major health problem of all older adults; estimates effects of this medication?
indicate that half of all women have at least one osteoporitic
fracture and the risk in men is estimated between 13% and A. Take the drug with an antacid.
25%; a bone mineral density measurement assesses the B. Lie down after meals.
mass of bone per unit volume or how tightly the bone is C. Avoid dairy products in diet.
packed. D. Change positions slowly.

24. An older adult client with acute pain is admitted in the ANSWER: D. Changing positions slowly will help prevent the
hospital. The nurse understands that in managing acute pain side effect of orthostatic hypotension.
of the client during the first 24 hours, the nurse should
ensure that: 29. A client is receiving simvastatin (Zocor). The nurse is
aware that this medication is effective when there is decrease
A. Ordered PRN analgesics are administered on a in:
scheduled basis.
B. Patient controlled analgesia is avoided in this A. The triglycerides
population. B. The INR
C. Pain medication is ordered via the intramuscular route. C. Chest pain
D. An order for meperidine (Demerol) is secured for pain D. Blood pressure
relief.
ANSWER: A. Therapeutic effects of simvastatin include
ANSWER: A. Around-the-clock administration of analgesics is decreased serum triglyceries, LDL and cholesterol.
recommended for acute pain in the older adult population;
this help to maintain a therapeutic blood level of pain 30. A client is taking nitroglycerine tablets, the nurse should
medication. teach the client the importance of:

25. A nurse is caring to an older adult with presbycusis. In


A. Increasing the number of tablets if dizziness or
formulating nursing care plan for this client, the nurse should
hypertension occurs.
expect that hearing loss of the client that is caused by aging
B. Limiting the number of tablets to 4 per day.
to have:
C. Making certain the medication is stored in a dark
container.
A. Overgrowth of the epithelial auditory lining. D. Discontinuing the medication if a headache develops.
B. Copious, moist cerumen.
C. Difficulty hearing women’s voices. ANSWER: C. Nitroglycerine is sensitive to light and moisture
D. Tears in the tympanic membrane. ad must be stored in a dark, airtight container.

ANSWER: C. Generally, female voices have a higher pitch 31. The physician prescribes Ibuprofen (Motrin) and
than male voices; older adults with presbycusis (hearing loss hydroxychloroquine sulfate (Plaquenil) for a 58-year-old male
caused by the aging process) have more difficulty hearing client with arthritis. The nurse provides information about
higher-pitched sounds. toxicity of the hydroxychloroquine. The nurse can determine
if the information is clearly understood if the client states:
26. The nurse is reviewing the client’s chart about the
ordered medication. The nurse must observe for signs of
A. “I will contact the physician immediately if I develop
hyperkalemia when administering:
blurred vision.”
B. “I will contact the physician immediately if I develop
A. Furosemide (Lasix) urinary retention.”
B. Hydrochlorothiazide (HydroDIURIL) C. “I will contact the physician immediately if I develop
C. Metolazone (Zaroxolyn) swallowing difficulty.”
D. Spironolactone (Aldactone) D. “I will contact the physician immediately if I develop
feelings of irritability.”
ANSWER: D. Aldactone is a potassium-sparing diuretic;
hyperkalemia is an adverse effect. ANSWER: A. Visual disturbance are a sign of toxicity because
retinopathy can occur with this drug.
32. The client with an acute myocardial infarction is
hospitalized for almost one week. The client experiences Answer: D. Carbamazepine ( Tegretol) is administered to
nausea and loss of appetite. The nurse caring for the client control pain by reducing the transmission of nerve impulses
recognizes that these symptoms may indicate the: in clients with trigeminal neuralgia.

A. Adverse effects of spironolactone (Aldactone) 37. Administration of potassium iodide solution is ordered to
B. Adverse effects of digoxin (Lanoxin) the client who will undergo a subtotal thyroidectomy. The
C. Therapeutic effects of propranolol (Indiral) nurse understands that this medication is given to:
D. Therapeutic effects of furosemide (Lasix)
A. Ablate the cells of the thyroid gland that produce T4.
ANSWER: B. Toxic levels of Lanoxin stimulate the medullary B. Decrease the total basal metabolic rate.
chemoreceptor trigger zone, resulting in nausea and C. Decrease the size and vascularity of the thyroid.
subsequent anorexia. D. Maintain function of the parathyroid gland.

33. A client with a partial occlusion of the left common Answer: C. Potassium iodide, which aids in decreasing the
carotid artery is scheduled for discharge. The client is still vascularity of the thyroid gland, decreases the risk for
receiving Coumadin. The nurse provided a discharge hemorrhage.
instruction to the client regarding adverse effects of
Coumadin. The nurse should tell the client to consult with the 38. A client with Addison’s disease is scheduled for discharge.
physician if: Before the discharge, the physician prescribes hydrocortisone
and fludrocortisone. The nurse expects the hydrocortisone to:
A. Swelling of the ankles increases.
B. Blood appears in the urine. A. Increase amounts of angiotensin II to raise the client’s
C. Increased transient Ischemic attacks occur. blood pressure.
D. The ability to concentrate diminishes. B. Control excessive loss of potassium salts.
C. Prevent hypoglycemia and permit the client to
Answer: B. Warfarin derivatives cause an increase in the respond to stress.
prothrombin time and INR, leading to an increased risk for D. Decrease cardiac dysrhythmias and dyspnea.
bleeding. Any abnormal or excessive bleeding must be
reported, because it may indicate toxic levels of the drug. Answer: C. Hydrocortisone is a glucocorticoid that has anti-
inflammatory action and aids in metabolism of carbohydrate,
34. Levodopa is ordered for a client with Parkinson’s disease. fat, and protein, causing elevation of blood glucose. Thus it
Before starting the medication, the nurse should know that: enables the body to adapt to stress.

A. Levodopa is inadequately absorbed if given with meals. 39. A client with diabetes insipidus is taking Desmopressin
B. Levodopa may cause the side effects of orthostatic acetate (DDAVP). To determine if the drug is effective, the
hypotension. nurse should monitor the client’s:
C. Levodopa must be monitored by weekly laboratory
tests. A. Arterial blood pH
D. Levodopa causes an initial euphoria followed by B. Pulse rate
depression. C. Serum glucose
D. Intake and output
Answer: B. Levodopa is the metabolic precursor of
dopamine. It reduces sympathetic outflow by limiting Answer: D. DDAVP replaces the ADH, facilitating
vasoconstriction, which may result in orthostatic reabsorption of water and consequent return of normal
hypotension. urine output and thirst.

35. In making a diagnosis of myasthenia gravis Edrophonium 40. A client with recurrent urinary tract infections is to be
HCI (Tensilon) is used. The nurse knows that this drug will discharged. The client will be taking nitrofurantoin (Macrobid)
cause a temporary increase in: 50 mg po every evening at home. The nurse provides
discharge instructions to the client. Which of the following
A. Muscle strength instructions will be correct?
B. Symptoms
C. Blood pressure A. Strain urine for crystals and stones
D. Consciousness B. Increase fluid intake.
C. Stop the drug if the urinary output increases
Answer: A. Tensilon, an anticholinesterase drug, causes D. Maintain the exact time schedule for drug taking.
temporary relief of symptoms of myasthenia gravis in client
who have the disease and is therefore an effective Answer: B. To prevent crystal formation, the client should
diagnostic aid. have sufficient intake to produce 1000 to 1500 mL of urine
daily while taking this drug.
36. The nurse can determine the effectiveness of
carbamazepine (Tegretol) in the management of trigeminal 41. A client with cancer of the lung is receiving chemotherapy.
neuralgia by monitoring the client’s: The physician orders antibiotic therapy for the client. The
nurse understands that chemotherapy destroys rapidly
A. Seizure activity growing leukocytes in the:
B. Liver function
C. Cardiac output A. Bone marrow
D. Pain relief B. Liver
C. Lymph nodes D. Obtain an ECG strip and have lidocaine available
D. Blood
Answer: C. Vital signs monitor cardiorespiratory status;
Answer: A. Prolonged chemotherapy may slow the hyperkalemia causes serious cardiac dysrhythmias.
production of leukocytes in bone marrow, thus suppressing
the activity of the immune system. Antibiotics may be 47. Potassium chloride, 20 mEq, is ordered and to be added in
required to help counter infections that the body can no the IV solution of a client in a diabetic ketoacidosis. The
longer handle easily. primary reason for administering this drug is:

42. The physician reduced the client’s Dexamethasone A. Replacement of excessive losses
(Decadron) dosage gradually and to continue a lower B. Treatment of hyperpnea
maintenance dosage. The client asks the nurse about the C. Prevention of flaccid paralysis
change of dosage. The nurse explains to the client that the D. Treatment of cardiac dysrhythmias
purpose of gradual dosage reduction is to allow:
Answer: A. Once treatment with insulin for diabetic
A. Return of cortisone production by the adrenal glands. ketoacidosis is begun, potassium ions reenter the cell,
B. Production of antibodies by the immune system causing hypokalemia; therefore potassium, along with the
C. Building of glycogen and protein stores in liver and replacement fluid, is generally supplied.
muscle
D. Time to observe for return of increases intracranial 48. A female client is brought to the emergency unit. The
pressure client is complaining of abdominal cramps. On assessment,
client is experiencing anorexia and weight is reduced. The
Answer: A. Any hormone normally produced by the body physician’s diagnosis is colitis. Which of the following
must be withdrawn slowly to allow the appropriate organ to symptoms of fluid and electrolyte imbalance should the nurse
adjust and resume production. report immediately?

43. The nurse is assigned to care for a client with diarrhea. A. Skin rash, diarrhea, and diplopia
Excessive fluid loss is expected. The nurse is aware that fluid B. Development of tetaniy with muscles spasms
deficit can most accurately be assessed by: C. Extreme muscle weakness and tachycardia
D. Nausea, vomiting, and leg and stomach cramps.
A. The presence of dry skin
B. A change in body weight Answer: C. Potassium, the major intracellular cation,
C. An altered general appearance functions with sodium and calcium to regulate
D. A decrease in blood pressure neuromuscular activity and contraction of muscle fibers,
particularly the heart muscle. In hypokalemia these
Answer: B. Dehydration is most readily and accurately symptoms develop.
measured by serial assessment of body weight; 1 L of fluid
weighs 2.2 pounds. 49. The client is to receive an IV piggyback medication. When
preparing the medication the nurse should be aware that it is
44. Which of the following is the most important electrolyte very important to:
of intracellular fluid?
A. Use strict sterile technique
A. Potassium B. Use exactly 100mL of fluid to mix the medication
B. Sodium C. Change the needle just before adding the medication
C. Chloride D. Rotate the bag after adding the medication
D. Calcium
Answer: A. Because IV solutions enter the body’s internal
Answer: A. The concentration of potassium is greater inside environment, all solutions and medications utilizing this
the cell and is important in establishing a membrane route must be sterile to prevent the introduction of
potential, a critical factor in the cell’s ability to function. microbes.

45. Which of the following client has a high risk for 50. The nurse is reviewing the laboratory result of the client.
developing hyperkalemia? An arterial blood gas report indicates the client’s pH is 7.20,
PCO2 35 mmHg and HCO3 is 19 mEq/L. The results are
A. Crohn’s disease consistent with:
B. End-Stage renal disease
C. Cushing’s syndrome A. Metabolic acidosis
D. Chronic heart failure B. Metabolic alkalosis
C. Respiratory acidosis
Answer: B. The kidneys normally eliminate potassium from D. Respiratory alkalosis
the body; hyperkalemia may necessitate dialysis.
Answer: A. A low pH and bicarbonate level are consistent
46. The nurse is reviewing the laboratory result of the client. with metabolic acidosis.
The client’s serum potassium level is 5.8 mEq/L. Which of the
following is the initial nursing action?
PNLE V Nursing Practice
A. Call the cardiac arrest team to alert them The scope of this Nursing Test V is parallel to the NP5
B. Call the laboratory and repeat the test NLE Coverage:
C. Take the client’s vital signs and notify the physician  Psychiatric Nursing
1. A 17-year-old client has a record of being absent in the 6. A 16-year-old girl was diagnosed with anorexia. What
class without permission, and “borrowing” other people’s would be the first assessment of the nurse?
things without asking permission. The client denies stealing;
rationalizing instead that as long as no one was using the items, A. What food she likes.
there is no problem to use it by other people. It is important B. Her desired weight.
for the nurse to understand that psychodynamically, the C. Her body image.
behavior of the client may be largely attributed to a D. What causes her behavior.
development defect related to the:
Answer: A. Although all options may appear correct. A is the
A. Oedipal complex best because it focuses on a range of possible positive
B. Superego reinforcers, a basis for an effective behavior modification
C. Id program. It can lead to concrete, specific nursing
D. Ego interventions right away and provides a therapeutic use of
“control” for the 16-year-old.
Answer: B. This shows a weak sense of moral consciousness.
According to Freudian theory, personality disorders stem 7. On an adolescent unit, a nurse caring to a client was
from a weak superego. informed that her client’s closest roommate dies at night.
What would be the most appropriate nursing action?
2. A client tells the nurse, “Yesterday, I was planning to kill
myself.” What is the best nursing response to this cient? A. Do not bring it up unless the client asks.
B. Tell the client that her roommate went home.
A. “What are you going to do this time?” C. Tell the client, if asked, “You should ask the doctor.”
B. Say nothing. Wait for the client’s next comment D. Tell the client that her closest roommate died.
C. “You seem upset. I am going to be here with you;
perhaps you will want to talk about it” Answer: A. The nurse needs to wait and see: do not “jump
D. “Have you felt this way before?” the gun”; do not assume that the client wants to know now.

Answer: C. The client needs to have his or her feelings 8. A woman gave birth to an unhealthy infant, and with some
acknowledged, with encouragement to discuss feelings, and body defects. The nurse should expect the woman’s initial
be reassured about the nurse’s presence. reactions to include:

3. In crisis intervention therapy, which of the following A. Depression


principle that the nurse will use to plan her/his goals? B. Withdrawal
C. Apathy
A. Crises are related to deep, underlying problems D. Anger
B. Crises seldom occur in normal people’s lives
C. Crises may go on indefinitely. Answer: D. The woman is experiencing an actual loss and
D. Crises usually resolved in 4-6 weeks. will probably exhibit many of the same symptoms as a
person who has lost someone to death.
Answer: D. Part of the definition of a crisis is a time span of
4-6 weeks. 9. A client in the psychiatric unit is shouting out loud and tells
the nurse, “Please, help me. They are coming to get me.” What
4. The nurse enters the room of the male client and found out would be the appropriate nursing response?
that the client urinates on the floor. The client hides when the
nurse is about to talk to him. Which of the following is the A. “ I won’t let anyone get you.”
best nursing intervention? B. “Who are they?”
C. “I don’t see anyone coming.”
A. Place restriction on the client’s activities when his D. “You look frightened.”
behavior occurs.
B. Ask the client to clean the soiled floor. Answer: C. This option is an example of pointing out reality-
C. Take the client to the bathroom at regular intervals. the nurse’s perception.
D. Limit fluid intake.
10. A client who is severely obese tells the nurse, “My
Answer; C. The client is most likely confused, rather than therapist told me that I eat a lot because I didn’t get any
exhibiting acting-out, hostile behavior. Frequent toileting will attention and love from my mother. What does the therapist
allow urination in an appropriate place. mean?” What is the best nursing response?

5. A young lady with a diagnosis of schizophrenic reaction is A. “What do you think is the connection between your not
admitted to the psychiatric unit. In the past two months, the getting enough love and overeating?”
client has poor appetite, experienced difficulty in sleeping, B. “Tell me what you think the therapist means.”
was mute for long periods of time, just stayed in her room, C. “You need to ask your therapist.”
grinning and pointing at things. What would be the initial D. “ We are here to deal with your diet, not with your
nursing action on admitting the client to the unit? psychological problems.”

A. Assure the client that “ You will be well cared for.” Answer: B. This response asks information that the nurse can
B. Introduce the client to some of the other clients. use. If the client understands the statement, the nurse can
C. Ask “Do you know where you are?” support the therapist when focusing on connection between
D. Take the client to the assigned room. food, love, and mother. If the client does not understand
thestatement, the nurse can help get clarification from the
Answer: D. The client needs basic, simple orientation that therapist.
directly relates to the here-and-now, and does not require
verbal interaction. 11. After the discussion about the procedure the physician
scheduled the client for mastectomy. The client tells the nurse,
“If my breasts will be removed, I’m afraid my husband will marvelous visit.” Which of the following coping mechanism
not love me anymore and maybe he will never touch me.” can be described to thestatement of the client?
What should the nurse’s response?
A. Identification.
A. “I doubt that he feels that way.” B. Rationalization.
B. “What makes you feel that way?” C. Denial.
C. “Have you discussed your feelings with your D. Compensation.
husband?”
D. Ask the husband, in front of the wife, how he feels about Answer: C. Denial is the act of avoiding disagreeable
this. realities by ignoring them.

Answer: C. This option redirects the client to talk to her 17. A male client is quiet when the physician told him that he
husband. has stage IV cancer and has 4 months to live. The nurse
determines that this reaction may be an example of:
12. The child is brought to the hospital by the parents. During
assessment of the nurse, what parental behavior toward a child A. Indifference
should alert the nurse to suspect child abuse? B. Denial
C. Resignation
A. Ignoring the child. D. Anger
B. Flat affect.
C. Expressions of guilt. Answer: B. Reactions when told of a life-threatening illness
D. Acting overly solicitous toward the child stem from Kübler-Ross’ ideas on death and dying. Denial is a
typical grief response, and usually is a first reaction.
Answer: D. This is an example of reaction formation, a
coping mechanism. 18. A nurse is caring to a female client with five young
children. The family member told the client that her ex-
13. A nurse is caring to a client with manic disorder in the husband has died 2 days ago. The reaction of the client is
psychiatric ward. On the morning shift, the nurse is talking stunned silence, followed by anger that the ex-husband left no
with the client who is now exhibiting a manic episode with insurance money for their young children. The nurse should
flight of ideas. The nurse primarily needs to: understand that:

A. Focus on the feelings conveyed rather than the A. The children and the injustice done to them by their
thoughts expressed. father’s death are the woman’s main concern.
B. Speak loudly and rapidly to keep the client’s attention, B. To explain the woman’s reaction, the nurse needs more
because the client is easily distracted. information about the relationship and breakup.
C. Allow the client to talk freely. C. The woman is not reacting normally to the news.
D. Encourage the client to complete one thought at a time. D. The woman is experiencing a normal bereavement
reaction.
Answer: A. Often the verbalized ideas are jumbled, but the
underlying feelings are discernible and must be Answer: D. Shock and anger are commonly the primary
acknowledged. initial reactions.

14. The nurse is caring to an autistic child. Which of the 19. A client who is manic comes to the outpatient department.
following play behavior would the nurse expect to see in a The nurse is assigning an activity for the client. What activity
child? is best for the nurse to encourage for a client in a manic phase?

A. competitive play A. Solitary activity, such as walking with the nurse, to


B. nonverbal play decrease stimulation.
C. cooperative play B. Competitive activity, such as bingo, to increase the
D. solitary play client’s self-esteem.
C. Group activity, such as basketball, to decrease isolation.
Answer: D. Autistic children do best with solitary play D. Intellectual activity, such as scrabble, to increase
because they typically do not interact with others in a socially concentration.
comprehensible and acceptable way.
Answer: A. This option avoids external stimuli, yet channels
15. The client is telling the nurse in the psychiatric ward, “I the excess motor activity that is often part of the manic phase.
hate them.” Which of the following is the most appropriate
nursing response to the client? 20. The nurse is about to administer Imipramine HCI (Tofranil)
to the client, the client says, “Why should I take this?” The
A. “Tell me about your hate.” doctor started me on this 10days ago; it didn’t help me at all.”
B. “I will stay with you as long as you feel this way.” Which of the following is the best nursing response:
C. “For whom do you have these feelings?”
D. “I understand how you can feel this way.” A. “What were you expecting to happen?”
B. “It usually takes 2-3 weeks to be effective.”
Answer: A. The nurse is asking the client to clarify and C. “Do you want to refuse this medication? You have the
further discuss feelings. right.”
D. “That’s a long time wait when you feel so depressed.”
16. The mother visits her son with major depression in the
psychiatric unit. After the conversation of the client and the Answer: B. The patient needs a brief, factual answer.
mother, the nurse asks the mother how it is talking to her son.
The mother tells the nurse that it was a stressful time. During 21. Which of the following drugs the nurse should choose to
an interview with the client, the client says, “we had a administer to a client to prevent pseudoparkinsonism?
A. Isocarboxazid (Marplan)
B. Chlorpromazine HCI (Thorazine) Answer: A. It is important to acknowledge a statement, even
C. Trihexyphenidyl HCI (Artane) if it is not understood.
D. Trifluoperazine HCI (Stelazine)
27. A male client diagnosed with depression tells the nurse, “I
Answer: C. Trihexyphenidyl HCI (Artane) is often used to don’t want to look weak and I don’t even cry because my wife
counteract side effect of pseudoparkinsonism, which often and my kids can’t bear it.” The nurse understands that this is
accompanies the use of phenothiazine, such as an example of:
chlorpromazine HCI (Thorazine or Trifluoperazine HCI
(Stelazine). A. Repression.
B. Suppression.
22. The nurse is caring to an 80-year-old client with dementia? C. Undoing.
What is the most important psychosocial need for this client? D. Rationalization.

A. Focus on the there-and-then rather the here-and-now. Answer: D. Rationalization is the process of constructing
B. Limit in the number of visitors, to minimize confusion. plausible reasons for one’s responses
C. Variety in their daily life, to decrease depression.
D. A structured environment, to minimize regressive 28. A female client tells the nurse that she is afraid to go out
behaviors. from her room because she thinks that the other client might
kill her. The nurse is aware that this behavior is related to:
Answer: D. Persons with dementia needs sameness,
consistency, structure, routine, and predictability. A. Hallucination.
B. Ideas of reference.
23. A client tells the nurse, “I don’t want to eat any meals C. Delusion of persecution.
offered in this hospital because the food is poisoned.” The D. Illusion.
nurse is aware that the client is expressing an example of:
Answer: C. The client has ideas that someone is out to kill
A. Delusion. her.
B. Hallucination.
C. Negativism. 29. A female client is taking Imipramine HCI (Tofranil) for
D. Illusion. almost 1 week and shows less awareness of the physical body.
What problem would the nurse be most concerned?
Answer: A. This is a false belief developed in response to an
emotional need. A. Nausea.
B. Gait disturbances.
24. A client is admitted in the hospital. On assessment, the C. Bowel movements.
nurse found out that the client had several suicidal attempts. D. Voiding.
Which of the following is the most important nursing action?
Answer: D. A serious side effect of Imipramine HCI
A. Ignore the client as long as he or she is talking about (Tofranil) is urinary retention (voiding problems)
suicide, because suicide attempt is unlikely.
B. Administer medication. 30. A 6-year-old client dies in the nursing unit. The parents
C. Relax vigilance when the client seems to be recovering want to see the child. What is the most appropriate nursing
from depression. action?
D. Maintain constant awareness of the client’s
whereabouts. A. Give the parents time alone with the body.
B. Ask the physician for permission.
Answer:D. The client must be constantly observed. C. Complete the postmortem care and quietly accompany
the family to the child’s room.
25. The nurse suspects that the client is suffering from D. Suggest the parents to wait until the funeral service to
depression. During assessment, what are the most say “good-bye.”
characteristic signs and symptoms of depression the nurse
would note? Answer: A. This allows the parents/family to grieve over the
loss of the child, by going through the steps of leave taking.
A. Constipation, increased appetite.
B. Anorexia, insomnia. 31. A 20-year-old female client is diagnosed with anxiety
C. Diarrhea, anger. disorder. The physician prescribed Flouxetine (Prozac). What
D. Verbosity, increased social interaction is the most important side effects should a nurse be concerned?

Answer: B. The appetite is diminished and sleeping is A. Tremor, drowsiness.


affected to a client with depression. B. Seizures, suicidal tendencies.
. C. Visual disturbance, headache.
26. The client in the psychiatric unit states that, “The goodas D. Excessive diaphoresis, diarrhea.
are coming! I must be ready.” In response to this neologism,
the nurse’s initial response is to: Answer: B. Assess for suicidal tendencies, especially during
early therapy. There is an increased risk of seizures in
A. Acknowledge that the word has some special meaning debilitated client and those with a history of seizures.
for the client.
B. Try to interpret what the client means. 32. A nurse is assigned to activate a client who is withdrawn,
C. Divert the client’s attention to an aspect of reality. hears voices and negativistic. What would be the best nursing
D. State that what the client is saying has not been approach?
understood and then divert attention to something that is
really bound.
A. Mention that the “voices” would want the client to
participate. 37. A nurse is completing the routine physical examination to
B. Demand that the client must join a group activity. a healthy 16-year-old male client. The client shares to the
C. Give the client a long explanation of the benefits of nurse that he feels like killing his girlfriend because he found
activity. out that her girlfriend had another boyfriend. He then laughs,
D. Tell the client that the nurse needs a partner for an and asks the nurse to keep this a secret just between the two of
activity. them. The nurse reviews his chart and notes that there is no
previously history of violence or psychiatric illness. Which of
Answer: D. The nurse helps to activate by doing something the following would be the best action of the nurse to take at
with the client. this time?

33. A nurse is going to give a rectal suppository as a A. Suggest the teen meet with a counselor to discuss his
preoperative medication to a 4-year-old boy. The boy is very feelings about his girlfriend.
anxious and frightened. Which of the following statement by B. Tell the teen that his feelings are normal, and
the nurse would be most appropriate to gain the child’s recommend that he find another girlfriend to take his
cooperation? mind off the problem.
C. Recall the teenage boys often say things they really do
A. “Be a big kid! Everyone’s waiting for you.” not mean and ignore the comment.
B. “Lie still now and I’ll let you have one of your presents D. Regard the comment seriously and notify the teen’s
before you even have your operation.” primary health care provider and parents
C. “Take a nice, big, deep breath and then let me hear
you count to five.” Answer: D. Any threat to the safety of oneself or other
D. “You look so scared. Want to know a secret? This won’t should always be taken seriously and never disregarded by
hurt a bit!” the nurse.
Answer: C. Preschool children commonly experience fears
and fantasies regarding invasive procedures. The nurse 38. Which of the following person will be at highest risk for
should attempts to momentarily distract the child with a suicide?
simple task that can be easily accomplished while the child
remains in the side-lying position. The suppository can be A. A student at exam time
slipped into place while the child is counting, and then the B. A married woman, age 40, with 6 children.
nurse can praise the child for cooperating, while holding the C. A person who is an alcoholic.
buttocks together to prevent expulsion of the suppository. D. A person who made a previous suicide attempt.
34. A depressed client is on an MAO inhibitor? What should Answer: C. The likelihood of multiple contributing factors
the nurse watch out for? may make this person at higher risk for suicide. Some factors
that may exist are physical illness related to alcoholism,
A. Hypertensive crisis. emotional factors ( anxiety, guilt, remorse), social isolation
B. Diet restrictions. due to impaired relationships and economic problems related
C. Taking medication with meals. to employment.
D. Exposure to sunlight.
39. A male client is repetitively doing the handwashing every
Answer: A. This is the more inclusive answer, although diet time he touches things. It is important for a nurse to
restrictions (answer1) are important, their purpose is to understand that the client’s behavior is probably an attempt to:
prevent hypertensive crisis (answer 2).
A. Seek attention from the staff.
35. A 16-year-old girl is admitted for treatment of a fracture. B. Control unacceptable impulses or feelings.
The client shares to the nurse caring to her that her step-father C. Do what the voices the patient hears tell him or her to do.
has made sexual advances to her. She got the chance to tell it D. Punish himself or herself for guilt feeling.
to her mother but refuses to believe. What is the most
therapeutic action of the nurse would be: Answer: B. A ritual, such as compulsive handwashing, is an
attempt to allay anxiety caused by unconscious impulses that
A. Tell the client to work it out with her father. are frightening.
B. Tell the client to discuss it with her mother.
C. Ask the father about it. 40. In a mental health settings, the basic goal of nursing is to:
D. Ask the mother what she thinks.
A. Advance the science of psychiatry by initiating research
Answer: D. This comes closest to beginning to focus on and gathering data for current statistics on emotional
family-centered approach to intervene in the “conspiracy of illness.
silence”. This is therefore the best among the options. B. Plan activity programs for clients.
C. Understand various types of family therapy and
36. A client with a diagnosis of paranoid disorder is admitted psychological tests and how to interpret them.
in the psychiatric hospital. The client tells the nurse, “the FBI D. Maintain a therapeutic environment.
is following me. These people are plotting against me.” With
this statement the nurse will need to: Answer: D. This is the most neutral answer by process of
elimination.
A. Acknowledge that this is the client’s belief but not the
nurse’s belief. 41. A 3-year-old boy is brought to the emergency department.
B. Ask how that makes the client feel. After an hour, the boy dies of respiratory failure. The mother
C. Show the client that no one is behind. of the boy becomes upset, shouting and abusive, saying to the
D. Use logic to help the client doubt this belief. nurse, “If it had been your son, they would have done more to
save it. “What should the nurse say or do?
Answer: A. The nurse should neither challenge nor use logic
to dispel an irrational belief.
A. Touch her and tell her exactly what was done for her B. Use restraints while the client is in bed to keep him or
baby. her from wandering off during the night.
B. Allow the mother to continue her present behavior C. Use restraints while the client is sitting in a chair to keep
while sitting quietly with her. him or her from wandering off during the day.
C. “No, all clients are given the same good care.” D. Provide a night-light and a big clock.
D. “Yes, you’re probably right. Your son did not get better
care.” Answer: D. This option is best to decrease confusion and
disorientation to place and time
Answer: B. This option allows a normal grief response
(anger). 47. A 30-year-old married woman comes to the hospital for
treatment of fractures. The woman tells the nurse that she was
42. The nurse is interacting to a client with an antisocial physically abused by her husband. The woman receives a call
personality disorder. What would be the most therapeutic from her husband telling her to get home and things will be
approach of the nurse to an antisocial behavior? different. He felt sorry of what he did. What can the nurse
advise her?
A. Gratify the client’s inner needs.
B. Give the client opportunities to test reality. A. “Do you think so?”
C. Provide external controls. B. “It’s not likely.”
D. Reinforce the client’s self-concept. C. “What will be different?”
D. “I hope so, for your sake.”
Answer: C. Personality disorders stem from a weak superego,
implying a lack of adequate controls. Answer: C. This option helps the woman to think through
and elaborate on her own thoughts and prognosis.
43. A 55-year-old male client tells the nurse that he needs his
glasses and hearing aid with him in the recovery room after 48. A female client was diagnosed with breast cancer. It is
the surgery, or he will be upset for not granting his request. found to be stage IV, and a modified mastectomy is performed.
What is the appropriate nursing response? After the procedure, what behaviors could the nurse expects
the client to display?
A. “Do you get upset and confused often?”
B. “You won’t need your glasses or hearing aid. The nurses A. Denial of the possibility of carcinoma.
will take care of you.” B. Signs of grief reaction.
C. “I understand. You will be able to cooperate best if C. Relief that the operation is over.
you know what is going on, so I will find out how I D. Signs of deep depression.
can arrange to have your glasses and hearing aid
available to you in the recovery room.” Answer:B. It is mostly likely that grief would be expressed
D. I understand you might be more cooperative if you have because of object loss.
your aid and glasses, but that is just not possible. Rules,
you know.” 49. A client is withdrawn and does not want to interact to
anybody even to the nurse. What is the best initial nursing
Answer: C. The client will be easier to care for if he has his approach to encourage communication with this client?
hearing aid and glasses.
A. Use simple questions that call for a response.
44. The male client had fight with his roommates in the B. Encourage discussion of feelings.
psychiatric unit. The client agitated client is placed in isolation C. Look through a photo album together.
for seclusion. The nurse knows it is essential that: D. Bring up neutral topics.

A. A staff member has frequent contacts with the client. Answer: D. Neutral, nonthreatening topics are best in
B. Restraints are applied. attempting to encourage a response.
C. The client is allowed to come out after 4 hours.
D. All the furniture is removed form the isolation room. 50. Which of the following nursing approach is most
important in a client with depression?
Answer: A. Frequent contacts at times of stress are important,
especially when a client is isolated. A. Deemphasizing preoccupation with elimination,
nourishment, and sleep.
45. A medical representative comes to the hospital unit for the B. Protecting against harm to others.
promotion of a new product. A female client, admitted for C. Providing motor outlets for aggressive, hostile
hysterical behavior, is found embracing him. What should the feelings.
nurse say? D. Reducing interpersonal contacts.

A. “Have you considered birth control?” Answer: C. It is important to externalize the anger away from
B. “This isn’t the purpose of either of you being here.” self.
C. “I see you’ve made a new friend.”
D. “Think about what you are doing.”
SET 3
Answer: B. This response is aimed at redirecting the
inappropriate behavior.

46. A client with dementia is for discharge. The nurse is PNLE I for Foundation of Nursing
providing a discharge instruction to the family member
1. Which element in the circular chain of infection can be
regarding safety measures at home. What suggestion can the
eliminated by preserving skin integrity?
nurse make to the family members?
A. Host
A. Avoid stairs without banisters. B. Reservoir
C. Mode of transmission Answer: B. The urinary system is normally free of
D. Portal of entry microorganisms except at the urinary meatus. Any procedure
that involves entering this system must use surgically aseptic
Answer: D. In the circular chain of infection, pathogens must measures to maintain a bacteria-free state.
be able to leave their reservoir and be transmitted to a
susceptible host through a portal of entry, such as broken 7. Sterile technique is used whenever:
skin.
A. Strict isolation is required
2. Which of the following will probably result in a break in B. Terminal disinfection is performed
sterile technique for respiratory isolation? C. Invasive procedures are performed
D. Protective isolation is necessary
A. Opening the patient’s window to the outside
environment Answer: C. All invasive procedures, including surgery,
B. Turning on the patient’s room ventilator catheter insertion, and administration of parenteral therapy,
C. Opening the door of the patient’s room leading into require sterile technique to maintain a sterile environment.
the hospital corridor All equipment must be sterile, and the nurse and
D. Failing to wear gloves when administering a bed bath the physician must wear sterile gloves and maintain surgical
asepsis. In the operating room, the nurse and physician are
Answer: C. Respiratory isolation, like strict isolation, required to wear sterile gowns, gloves, masks, hair covers,
requires that the door to the door patient’s room remain and shoe covers for all invasive procedures. Strict isolation
closed. However, the patient’s room should be well requires the use of clean gloves, masks, gowns and
ventilated, so opening the window or turning on the equipment to prevent the transmission of highly
ventricular is desirable. The nurse does not need to wear communicable diseases by contact or by airborne routes.
gloves for respiratory isolation, but good hand washing is Terminal disinfection is the disinfection of all contaminated
important for all types of isolation. supplies and equipment after a patient has been discharged to
prepare them for reuse by another patient. The purpose of
3. Which of the following patients is at greater risk for protective (reverse) isolation is to prevent a person
contracting an infection? with seriously impaired resistance from coming into contact
who potentially pathogenic organisms.
A. A patient with leukopenia
B. A patient receiving broad-spectrum antibiotics 8. Which of the following constitutes a break in sterile
C. A postoperative patient who has undergone orthopedic technique while preparing a sterile field for a dressing change?
surgery
D. A newly diagnosed diabetic patient A. Using sterile forceps, rather than sterile gloves, to handle
a sterile item
Answer: A. Leukopenia is a decreased number of leukocytes B. Touching the outside wrapper of sterilized material
(white blood cells), which are important in resisting infection. without sterile gloves
None of the other situations would put the patient at risk for C. Placing a sterile object on the edge of the sterile field
contracting an infection; taking broadspectrum antibiotics D. Pouring out a small amount of solution (15 to 30 ml)
might actually reduce the infection risk. before pouring the solution into a sterile container

4. Effective hand washing requires the use of: Answer: C. The edges of a sterile field are considered
contaminated. When sterile items are allowed to come in
A. Soap or detergent to promote emulsification contact with the edges of the field, the sterile items also
B. Hot water to destroy bacteria become contaminated.
C. A disinfectant to increase surface tension
D. All of the above 9. A natural body defense that plays an active role in
preventing infection is:
Answer: A. Soaps and detergents are used to help remove
bacteria because of their ability to lower the surface tension A. Yawning
of water and act as emulsifying agents. Hot water may lead to B. Body hair
skin irritation or burns. C. Hiccupping
D. Rapid eye movements
5. After routine patient contact, hand washing should last at
least: Answer: B. Hair on or within body areas, such as the nose,
traps and holds particles that contain microorganisms.
A. 30 seconds Yawning and hiccupping do not prevent microorganisms
B. 1 minute from entering or leaving the body. Rapid eye movement
C. 2 minute marks the stage of sleep during which dreaming occurs.
D. 3 minutes
10. All of the following statement are true about donning
Answer: A. Depending on the degree of exposure to sterile gloves except:
pathogens, hand washing may last from 10 seconds to 4
minutes. After routine patient contact, hand washing for 30 A. The first glove should be picked up by grasping the
seconds effectively minimizes the risk of inside of the cuff.
pathogen transmission. B. The second glove should be picked up by inserting the
gloved fingers under the cuff outside the glove.
6. Which of the following procedures always requires surgical C. The gloves should be adjusted by sliding the gloved
asepsis? fingers under the sterile cuff and pulling the glove over
the wrist
A. Vaginal instillation of conjugated estrogen D. The inside of the glove is considered sterile
B. Urinary catheterization
C. Nasogastric tube insertion Answer: D. The inside of the glove is always considered to
D. Colostomy irrigation be clean, but not sterile.
11.When removing a contaminated gown, the nurse should be Answer: A. Platelets are disk-shaped cells that are essential
careful that the first thing she touches is the: for blood coagulation. A platelet count determines the
number of thrombocytes in blood available for promoting
A. Waist tie and neck tie at the back of the gown hemostasis and assisting with blood coagulation after
B. Waist tie in front of the gown injury. It also is used to evaluate the patient’s potential for
C. Cuffs of the gown bleeding; however, this is not its primary purpose. The
D. Inside of the gown normal count ranges from 150,000 to 350,000/mm3. A count
of 100,000/mm3 or less indicates a potential for bleeding;
Answer: A. The back of the gown is considered clean, the count of less than 20,000/mm3 is associated with
front is contaminated. So, after removing gloves and washing spontaneous bleeding.
hands, the nurse should untie the back of the gown; slowly
move backward away from the gown, holding the inside of 16.Which of the following white blood cell (WBC) counts
the gown and keeping the edges off the floor; turn and fold clearly indicates leukocytosis?
the gown inside out; discard it in a contaminated linen
container; then wash her hands again. A. 4,500/mm³
B. 7,000/mm³
12.Which of the following nursing interventions is considered C. 10,000/mm³
the most effective form or universal precautions? D. 25,000/mm³

A. Cap all used needles before removing them from their Answer: D. Leukocytosis is any transient increase in the
syringes number of white blood cells (leukocytes) in the blood.
B. Discard all used uncapped needles and syringes in Normal WBC counts range from 5,000 to 100,000/mm3.
an impenetrable protective container Thus, a count of 25,000/mm3 indicates leukocytosis.
C. Wear gloves when administering IM injections
D. Follow enteric precautions 17. After 5 days of diuretic therapy with 20mg of furosemide
(Lasix) daily, a patient begins to exhibit fatigue, muscle
Answer: B. According to the Centers for Disease Control cramping and muscle weakness. These symptoms probably
(CDC), blood-to-blood contact occurs most commonly when indicate that the patient is experiencing:
a health care worker attempts to cap a used needle. Therefore,
used needles should never be recapped; instead they should A. Hypokalemia
be inserted in a specially designed puncture resistant, labeled B. Hyperkalemia
container. Wearing gloves is not always necessary C. Anorexia
when administering an I.M. injection. Enteric precautions D. Dysphagia
prevent the transfer of pathogens via feces.
Answer: A. Fatigue, muscle cramping, and muscle
13.All of the following measures are recommended to prevent weaknesses are symptoms of hypokalemia (an inadequate
pressure ulcers except: potassium level), which is a potential side effect of diuretic
therapy. The physician usually orders
A. Massaging the reddened are with lotion supplemental potassium to prevent hypokalemia in patients
B. Using a water or air mattress receiving diuretics. Anorexia is another symptom of
C. Adhering to a schedule for positioning and turning hypokalemia. Dysphagia means difficulty swallowing.
D. Providing meticulous skin care
18.Which of the following statements about chest X-ray is
Answer: A. Nurses and other health care professionals false?
previously believed that massaging a reddened area with
lotion would promote venous return and reduce edema to the A. No contradictions exist for this test
area. However, research has shown that massage only B. Before the procedure, the patient should remove all
increases the likelihood of cellular ischemia and necrosis to jewelry, metallic objects, and buttons above the waist
the area. C. A signed consent is not required
D. Eating, drinking, and medications are allowed before this
14.Which of the following blood tests should be performed test
before a blood transfusion?
Answer: A. Pregnancy or suspected pregnancy is the only
A. Prothrombin and coagulation time contraindication for a chest X-ray. However, if a chest X-ray
B. Blood typing and cross-matching is necessary, the patient can wear a lead apron to protect the
C. Bleeding and clotting time pelvic region from radiation. Jewelry, metallic objects, and
D. Complete blood count (CBC) and electrolyte levels. buttons would interfere with the X-ray and thus should not
be worn above the waist. A signed consent is not required
Answer: B. Before a blood transfusion is performed, the because a chest X-ray is not an invasive examination. Eating,
blood of the donor and recipient must be checked for drinking and medications are allowed because the X-ray is of
compatibility. This is done by blood typing (a test that the chest, not the abdominal region.
determines a person’s blood type) and cross-matching
(a procedure that determines the compatibility of the donor’s 19.The most appropriate time for the nurse to obtain a sputum
and recipient’s blood after the blood types has been matched). specimen for culture is:
If the blood specimens are incompatible, hemolysis and
antigen-antibody reactions will occur. A. Early in the morning
B. After the patient eats a light breakfast
15.The primary purpose of a platelet count is to evaluate the: C. After aerosol therapy
D. After chest physiotherapy
A. Potential for clot formation
B. Potential for bleeding Answer: A. Obtaining a sputum specimen early in this
C. Presence of an antigen-antibody response morning ensures an adequate supply of bacteria for culturing
D. Presence of cardiac enzymes and decreases the risk of contamination from food or
medication.
20.A patient with no known allergies is to receive penicillin A. 18G, 1 ½” long
every 6 hours. When administering the medication, the nurse B. 22G, 1” long
observes a fine rash on the C. 22G, 1 ½” long
D. 25G, 5/8” long
patient’s skin. The most appropriate nursing action would be
to: Answer: D. A 25G, 5/8” needle is the recommended size for
insulin injection because insulin is administered by the
A. Withhold the moderation and notify the physician subcutaneous route. An 18G, 1 ½” needle is usually used for
B. Administer the medication and notify the physician I.M. injections in children, typically in the vastus lateralis. A
C. Administer the medication with an antihistamine 22G, 1 ½” needle is usually used for adult I.M. injections,
D. Apply corn starch soaks to the rash which are typically administered in the vastus lateralis or
ventrogluteal site.
Answer: A. Initial sensitivity to penicillin is commonly
manifested by a skin rash, even in individuals who have not 25.The appropriate needle gauge for intradermal injection is:
been allergic to it previously. Because of the danger of
anaphylactic shock, he nurse should withhold the drug A. 20G
and notify the physician, who may choose to substitute B. 22G
another drug. Administering an antihistamine is a dependent C. 25G
nursing intervention that requires a written physician’s order. D. 26G
Although applying corn starch to the rash may relieve
discomfort, it is not the nurse’s top priority in such Answer:D. Because an intradermal injection does not
a potentially life-threatening situation. penetrate deeply into the skin, a small-bore 25G needle is
recommended. This type of injection is used primarily to
21.All of the following nursing interventions are correct when administer antigens to evaluate reactions for allergy
using the Ztrack method of drug injection except: or sensitivity studies. A 20G needle is usually used for I.M.
injections of oilbased medications; a 22G needle for I.M.
A. Prepare the injection site with alcohol injections; and a 25G needle, for I.M. injections; and a 25G
B. Use a needle that’s a least 1” long needle, for subcutaneous insulin injections.
C. Aspirate for blood before injection
D. Rub the site vigorously after the injection to promote 26.Parenteral penicillin can be administered as an:
absorption
A. IM injection or an IV solution
Answer: D. The Z-track method is an I.M. injection B. IV or an intradermal injection
technique in which the patient’s skin is pulled in such a way C. Intradermal or subcutaneous injection
that the needle track is sealed off after the injection. This D. IM or a subcutaneous injection
procedure seals medication deep into the muscle,
thereby minimizing skin staining and irritation. Rubbing the Answer: A. Parenteral penicillin can be administered I.M. or
injection site is contraindicated because it may cause the added to a solution and given I.V. It cannot be administered
medication to extravasate into the skin. subcutaneously or intradermally.

22.The correct method for determining the vastus lateralis site 27.The physician orders gr 10 of aspirin for a patient. The
for I.M. injection is to: equivalent dose in milligrams is:

A. Locate the upper aspect of the upper outer quadrant of A. 0.6 mg


the buttock about 5 to 8 cm below the iliac crest B. 10 mg
B. Palpate the lower edge of the acromion process and the C. 60 mg
midpoint lateral aspect of the arm D. 600 mg
C. Palpate a 1” circular area anterior to the umbilicus
D. Divide the area between the greater femoral Answer:D. gr 10 x 60mg/gr 1 = 600 mg
trochanter and the lateral femoral condyle into thirds,
and select the middle third on the anterior of the 28.The physician orders an IV solution of dextrose 5% in
thigh water at 100ml/hour. What would the flow rate be if the drop
factor is 15 gtt = 1 ml?
Answer: D. The vastus lateralis, a long, thick muscle that
extends the full length of the thigh, is viewed by many A. 5 gtt/minute
clinicians as the site of choice for I.M. injections because it B. 13 gtt/minute
has relatively few major nerves and blood vessels. The C. 25 gtt/minute
middle third of the muscle is recommended as the injection D. 50 gtt/minute
site. The patient can be in a supine or sitting position for an
injection into this site. Answer: C. 100ml/60 min X 15 gtt/ 1 ml = 25 gtt/minute

23.The mid-deltoid injection site is seldom used for I.M. 29.Which of the following is a sign or symptom of a
injections because it: hemolytic reaction to blood transfusion?

A. Can accommodate only 1 ml or less of medication A. Hemoglobinuria


B. Bruises too easily B. Chest pain
C. Can be used only when the patient is lying down C. Urticaria
D. Does not readily parenteral medication D. Distended neck veins

Answer: A. The mid-deltoid injection site can accommodate Answer: A. Hemoglobinuria, the abnormal presence of
only 1 ml or less of medication because of its size and hemoglobin in the urine, indicates a hemolytic reaction
location (on the deltoid muscle of the arm, close to the (incompatibility of the donor’s and recipient’s blood). In this
brachial artery and radial nerve). reaction, antibodies in the recipient’s plasma combine rapidly
with donor RBC’s; the cells are hemolyzed in
24.The appropriate needle size for insulin injection is: either circulatory or reticuloendothelial system. Hemolysis
occurs more rapidly in ABO incompatibilities than in Rh
incompatibilities. Chest pain and urticaria may be symptoms Answer: D. A drug-allergy is an adverse reaction resulting
of impending anaphylaxis. Distended neck veins are from an immunologic response following a previous
an indication of hypervolemia. sensitizing exposure to the drug. The reaction can range from
a rash or hives to anaphylactic shock. Tolerance to a drug
30.Which of the following conditions may require fluid means that the patient experiences a decreasing
restriction? physiologic response to repeated administration of the drug
in the same dosage. Idiosyncrasy is an individual’s unique
A. Fever hypersensitivity to a drug, food, or other substance; it appears
B. Chronic Obstructive Pulmonary Disease to be genetically determined. Synergism, is a drug interaction
C. Renal Failure in which the sum of the drug’s combined effects is
D. Dehydration greater than that of their separate effects.

Answer: C. In real failure, the kidney loses their ability to 35.A patient has returned to his room after femoral
effectively eliminate wastes and fluids. Because of this, arteriography. All of the following are appropriate nursing
limiting the patient’s intake of oral and I.V. fluids may be interventions except:
necessary. Fever, chronic obstructive pulmonary disease, and
dehydration are conditions for which fluids should A. Assess femoral, popliteal, and pedal pulses every 15
be encouraged. minutes for 2 hours
B. Check the pressure dressing for sanguineous drainage
31.All of the following are common signs and symptoms of C. Assess a vital signs every 15 minutes for 2 hours
phlebitis except: D. Order a hemoglobin and hematocrit count 1 hour
after the arteriography
A. Pain or discomfort at the IV insertion site
B. Edema and warmth at the IV insertion site Answer: D. A hemoglobin and hematocrit count would be
C. A red streak exiting the IV insertion site ordered by the physician if bleeding were suspected. The
D. Frank bleeding at the insertion site other answers are appropriate nursing interventions for a
patient who has undergone femoral arteriography.
Answer: D. Phlebitis, the inflammation of a vein, can be
caused by chemical irritants (I.V. solutions or medications), 36.The nurse explains to a patient that a cough:
mechanical irritants (the needle or catheter used during
venipuncture or cannulation), or a localized allergic reaction A. Is a protective response to clear the respiratory tract
to the needle or catheter. Signs and symptoms of of irritants
phlebitis include pain or discomfort, edema and heat at the B. Is primarily a voluntary action
I.V. insertion site, and a red streak going up the arm or leg C. Is induced by the administration of an antitussive drug
from the I.V. insertion site. D. Can be inhibited by “splinting” the abdomen

32.The best way of determining whether a patient has learned Answer: A. Coughing, a protective response that clears the
to instill ear medication properly is for the nurse to: respiratory tract of irritants, usually is involuntary; however
it can be voluntary, as when a patient is taught to perform
A. Ask the patient if he/she has used ear drops before coughing exercises. An antitussive drug inhibits coughing.
B. Have the patient repeat the nurse’s instructions using her Splinting the abdomen supports the abdominal muscles when
own words a patient coughs.
C. Demonstrate the procedure to the patient and encourage
to ask questions 37.An infected patient has chills and begins shivering. The
D. Ask the patient to demonstrate the procedure best nursing intervention is to:

Answer: D. Return demonstration provides the most certain A. Apply iced alcohol sponges
evidence for evaluating the effectiveness of patient teaching. B. Provide increased cool liquids
C. Provide additional bedclothes
33.Which of the following types of medications can be D. Provide increased ventilation
administered via gastrostomy tube?
Answer: C. In an infected patient, shivering results from the
A. Any oral medications body’s attempt to increase heat production and the
B. Capsules whole contents are dissolve in water production of neutrophils and phagocytotic action through
C. Enteric-coated tablets that are thoroughly dissolved in increased skeletal muscle tension and contractions. Initial
water vasoconstriction may cause skin to feel cold to the touch.
D. Most tablets designed for oral use, except for Applying additional bed clothes helps to equalize the
extended-duration compounds body temperature and stop the chills. Attempts to cool the
body result in further shivering, increased metabloism, and
Answer: D. Capsules, enteric-coated tablets, and most thus increased heat production.
extended duration or sustained release products should not be
dissolved for use in a gastrostomy tube. They are 38.A clinical nurse specialist is a nurse who has:
pharmaceutically manufactured in these forms for valid
reasons, and altering them destroys their purpose. The A. Been certified by the National League for Nursing
nurse should seek an alternate physician’s order when an B. Received credentials from the Philippine Nurses’
ordered medication is inappropriate for delivery by tube. Association
C. Graduated from an associate degree program and is a
34.A patient who develops hives after receiving an antibiotic registered professional nurse
is exhibiting drug: D. Completed a master’s degree in the prescribed
clinical area and is a registered professional nurse
A. Tolerance
B. Idiosyncrasy Answer: .D. A clinical nurse specialist must have completed
C. Synergism a master’s degree in a clinical specialty and be a registered
D. Allergy professional nurse. The National League of Nursing accredits
educational programs in nursing and provides a testing D. Maintain the drainage tubing and collection bag
service to evaluate student nursing competence but it does below bladder level to facilitate drainage by gravity
not certify nurses. The American Nurses Association
identifies requirements for certification and offers Answer: D. Maintaing the drainage tubing and collection bag
examinations for certification in many areas of nursing., such level with the patient’s bladder could result in reflux of urine
as medical surgical nursing. These certification (credentialing) into the kidney. Irrigating the bladder with Neosporin and
demonstrates that the nurse has the knowledge and the ability clamping the catheter for 1 hour every 4 hours must be
to provide high quality nursing care in the area of her prescribed by a physician.
certification. A graduate of an associate degree program is
not a clinical nurse specialist: however, she is prepared to 44.The ELISA test is used to:
provide bed side nursing with a high degree of knowledge
and skill. She must successfully complete the A. Screen blood donors for antibodies to human
licensing examination to become a registered professional immunodeficiency virus (HIV)
nurse. B. Test blood to be used for transfusion for HIV antibodies
C. Aid in diagnosing a patient with AIDS
39.The purpose of increasing urine acidity through dietary D. All of the above
means is to:
Answer: D. The ELISA test of venous blood is used to assess
A. Decrease burning sensations blood and potential blood donors to human
B. Change the urine’s color immunodeficiency virus (HIV). A positive ELISA test
C. Change the urine’s concentration combined with various signs and symptoms helps to
D. Inhibit the growth of microorganisms diagnose acquired immunodeficiency syndrome (AIDS)

Answer: D. Microorganisms usually do not grow in an acidic 45.The two blood vessels most commonly used for TPN
environment. infusion are the:

40.Clay colored stools indicate: A. Subclavian and jugular veins


B. Brachial and subclavian veins
A. Upper GI bleeding C. Femoral and subclavian veins
B. Impending constipation D. Brachial and femoral veins
C. An effect of medication
D. Bile obstruction : D. Tachypnea (an abnormally rapid rate of breathing) would
indicate that the patient was still hypoxic (deficient in
Answer: D. Bile colors the stool brown. Any inflammation or oxygen).The partial pressures of arterial oxygen and carbon
obstruction that impairs bile flow will affect the stool dioxide listed are within the normal range. Eupnea refers to
pigment, yielding light, clay-colored stool. Upper GI normal respiration.
bleeding results in black or tarry stool. Constipation
is characterized by small, hard masses. Many medications 46.Effective skin disinfection before a surgical procedure
and foods will discolor stool – for example, drugs containing includes which of the following methods?
iron turn stool black.; beets turn stool red.
A. Shaving the site on the day before surgery
41.In which step of the nursing process would the nurse ask a B. Applying a topical antiseptic to the skin on the evening
patient if the medication she administered relieved his pain? before surgery
C. Having the patient take a tub bath on the morning of
A. Assessment surgery
B. Analysis D. Having the patient shower with an antiseptic soap on
C. Planning the evening v=before and the morning of surgery
D. Evaluation
Answer: D. Studies have shown that showering with an
Answer: D. In the evaluation step of the nursing process, the antiseptic soap before surgery is the most effective method of
nurse must decide whether the patient has achieved the removing microorganisms from the skin. Shaving the site of
expected outcome that was identified in the planning phase. the intended surgery might cause breaks in the skin, thereby
increasing the risk of infection; however, if
42.All of the following are good sources of vitamin A except: indicated, shaving, should be done immediately before
surgery, not the day before. A topical antiseptic would not
A. White potatoes remove microorganisms and would be beneficial only after
B. Carrots proper cleaning and rinsing. Tub bathing might transfer
C. Apricots organisms to another body site rather than rinse them away.
D. Egg yolks
47.When transferring a patient from a bed to a chair, the nurse
Answer: A. The main sources of vitamin A are yellow and should use which muscles to avoid back injury?
green vegetables (such as carrots, sweet potatoes, squash,
spinach, collard greens, broccoli, and cabbage) and yellow A. Abdominal muscles
fruits (such as apricots, and cantaloupe). Animal sources B. Back muscles
include liver, kidneys, cream, butter, and egg yolks. C. Leg muscles
D. Upper arm muscles
43.Which of the following is a primary nursing intervention
necessary for all patients with a Foley Catheter in place? Answer: C. The leg muscles are the strongest muscles in the
body and should bear the greatest stress when lifting. Muscles
A. Maintain the drainage tubing and collection bag level of the abdomen, back, and upper arms may be easily injured
with the patient’s bladder
B. Irrigate the patient with 1% Neosporin solution three 48.Thrombophlebitis typically develops in patients with which
times a daily of the following conditions?
C. Clamp the catheter for 1 hour every 4 hours to maintain
the bladder’s elasticity A. Increases partial thromboplastin time
B. Acute pulsus paradoxus Answer: B . Regular timely ingestion of oral
C. An impaired or traumatized blood vessel wall
D. Chronic Obstructive Pulmonary Disease (COPD) contraceptives is necessary to
maintain hormonal levels of the drugs to
Answer: C. The factors, known as Virchow’s triad, suppress the action of the hypothalamus and
collectively predispose a patient to thromboplebitis; impaired
venous return to the heart, blood hypercoagulability, and anterior pituitary leading to inappropriate
injury to a blood vessel wall. Increased secretion of FSH and LH. Therefore, follicles
partial thromboplastin time indicates a prolonged bleeding do not mature, ovulation is inhibited, and
time during fibrin clot formation, commonly the result of
anticoagulant (heparin) therapy. Arterial blood disorders pregnancy is prevented. The estrogen content
(such as pulsus paradoxus) and lung diseases (such as COPD) of the oral site contraceptive may cause the
do not necessarily impede venous return of injure vessel nausea, regardless of when the pill is taken.
walls.
Side effects and drug interactions may occur
49.In a recumbent, immobilized patient, lung ventilation can with oral contraceptives regardless of the
become altered, leading to such respiratory complications as: time the pill is taken.
A. Respiratory acidosis, ateclectasis, and hypostatic
pneumonia 2. When teaching a client about contraception.
B. Appneustic breathing, atypical pneumonia and
respiratory alkalosis Which of the following would the nurse
C. Cheyne-Strokes respirations and spontaneous include as the most effective method for
pneumothorax
D. Kussmail’s respirations and hypoventilation preventing sexually transmitted infections?
Answer: A. Because of restricted respiratory movement, a A. Spermicides
recumbent, immobilize patient is at particular risk for B. Diaphragm
respiratory acidosis from poor gas exchange; atelectasis from
reduced surfactant and accumulated mucus in the bronchioles, C. Condoms
and hypostatic pneumonia from bacterial growth caused by D. Vasectomy
stasis of mucus secretions.

50.Immobility impairs bladder elimination, resulting in such 1. Answer:C . Condoms, when used
disorders as correctly and consistently, are the most
A. Increased urine acidity and relaxation of the perineal
effective contraceptive method or barrier
muscles, causing incontinence against bacterial and viral
B. Urine retention, bladder distention, and infection sexually transmitted infections. Although
C. Diuresis, natriuresis, and decreased urine specific gravity
D. Decreased calcium and phosphate levels in the urine
spermicides kill sperm, they do
not provide reliable protection against the
Answer: B. The immobilized patient commonly suffers from spread of sexually transmitted infections,
urine retention caused by decreased muscle tone in the
especially intracellular organisms such as
perineum. This leads to bladder distention and urine stagnation,
which provide an excellent medium for bacterial growth HIV. Insertion and removal of the
leading to infection. Immobility also results in more alkaline diaphragm along with the use of the
urine with excessive amounts of calcium, sodium and spermicides may cause vaginal irritations,
phosphate, a gradual decrease in urine production, and an
which could place the client at risk for
increased specific gravity.
infection transmission. Male sterilization
eliminates spermatozoa from the
PNLE II for Maternal ejaculate, but it does not eliminate
bacterial and/or viral microorganisms that
can cause sexually transmitted infections.
and Child Health
1. For the client who is using oral 3. When preparing a woman who is 2 days
contraceptives, the nurse informs the postpartum for discharge, recommendations
client about the need to take the pill at the for which of the following contraceptive
same time each day to accomplish which of methods would be avoided?
the following? A. Diaphragm
A. Decrease the incidence of nausea B. Female condom
B. Maintain hormonal levels C. Oral contraceptives
C. Reduce side effects D. Rhythm method
D. Prevent drug interactions
Answer: A . The diaphragm must be fitted
individually to ensure effectiveness. Because
of the changes to the reproductive structures C. Increased fiber intake
during pregnancy and following delivery, the D. Decreased fluid intake
diaphragm must be refitted, usually at the
6 weeks’ examination following childbirth or Answer: C . During the third trimester, the
after a weight loss of 15 lbs or more. In enlarging uterus places pressure on
addition, for maximum effectiveness, the intestines. This coupled with the effect of
spermicidal jelly should be placed in the dome hormones on smooth muscle relaxation
and around the rim. However, spermicidal causes decreased intestinal motility
jelly should not be inserted into the vagina (peristalsis). Increasing fiber in the diet will
until involution is completed at approximately help fecal matter pass more quickly through
6 weeks. Use of a female condom protects the the intestinal tract, thus decreasing the
reproductive system from the introduction of amount of water that is absorbed. As a result,
semen or spermicides into the vagina and may stool is softer and easier to pass. Enemas
be used after childbirth. Oral contraceptives could precipitate preterm labor and/or
may be started within the first postpartum electrolyte loss and should be avoided.
week to ensure suppression of ovulation. For Laxatives may cause preterm labor by
the couple who has determined the female’s stimulating peristalsis and may interfere with
fertile period, using the rhythm the absorption of nutrients. Use for more than
method, avoidance of intercourse during this 1 week can also lead to laxative dependency.
period, is safe and effective Liquid in the diet helps provide a semisolid,
soft consistency to the stool. Eight to ten
4. For which of the following clients would glasses of fluid per day are essential to
the nurse expect that an intrauterine device maintain hydration and promote stool
would not be recommended? evacuation.

A. Woman over age 35 6. Which of the following would the nurse use
B. Nulliparous woman as the basis for the teaching plan when caring
C. Promiscuous young adult for a pregnant teenager concerned about
D. Postpartum client
gaining too much weight during pregnancy?
Answer: C . An IUD may increase the risk of A. 10 pounds per trimester
pelvic inflammatory disease, especially in B. 1 pound per week for 40 weeks
women with more than one sexual partner, C. ½ pound per week for 40 weeks
because of the increased risk of sexually D. A total gain of 25 to 30 pounds
transmitted infections. An UID should not be
used if the woman has an active or chronic Answer: D . To ensure adequate fetal growth
pelvic infection, postpartum and development during the 40 weeks of a
infection, endometrial hyperplasia or pregnancy, a total weight gain 25 to 30
carcinoma, or uterine abnormalities. Age is pounds is recommended: 1.5 pounds in the
not a factor in determining the risks first 10 weeks; 9 pounds by 30 weeks; and
associated with IUD use. Most IUD users are 27.5 pounds by 40 weeks. The pregnant
over the age of 30. Although there is a woman should gain less weight in the first
slightly higher risk for infertility in women and second trimester than in the third. During
who have never been pregnant, the IUD is an the first trimester, the client should only gain
acceptable option as long as the risk-benefit 1.5 pounds in the first 10 weeks, not 1 pound
ratio is discussed. IUDs may be per week. A weight gain of ½ pound per
inserted immediately after delivery, but this week would be 20 pounds for the
is not recommended because of the increased total pregnancy, less than the recommended
risk and rate of expulsion at this time. amount.

5. A client in her third trimester tells the nurse, 7. The client tells the nurse that her last
“I’m constipated all the time!” Which of the menstrual period started on January 14 and
following should the nurse recommend? ended on January 20. Using Nagele’s rule, the
nurse determines her EDD to be which of the
A. Daily enemas
B. Laxatives following?
A. September 27 D. External electronic fetal monitor placed at
B. October 21 the umbilicus
C. November 7
D. December 27 Answer: B. At 12 weeks gestation, the uterus
rises out of the pelvis and is palpable above
Answer: B . To calculate the EDD by the symphysis pubis. The Doppler intensifies
Nagele’s rule, add 7 days to the first day of the sound of the fetal pulse rate so it is
the last menstrual period and count back 3 audible. The uterus has merely risen out of
months, changing the year appropriately. To the pelvis into the abdominal cavity and is
obtain a date of September 27, 7 days have not at the level of the umbilicus. The
been added to the last day of the LMP (rather fetal heart rate at this age is not audible with
than the first day of the LMP), plus 4 months a stethoscope. The uterus at 12 weeks is just
(instead of 3 months) were counted back. To above the symphysis pubis in the abdominal
obtain the date of November 7, 7 days have cavity, not midway between the umbilicus
been subtracted (instead of added) from and the xiphoid process. At 12 weeks
the first day of LMP plus November the FHR would be difficult to auscultate with
indicates counting back 2 months (instead of a fetoscope. Although the external electronic
3 months) from January. To obtain the date fetal monitor would project the FHR, the
of December 27, 7 days were added to the uterus has not risen to the umbilicus at 12
last day of the LMP (rather than the first day weeks.
of the LMP) and December indicates
counting back only 1 month (instead of 10.When developing a plan of care for a client
3 months) from January. newly diagnosed with gestational diabetes,
which of the following instructions would be
8. When taking an obstetrical history on a the priority?
pregnant client who states, “I had a son born
at 38 weeks gestation, a daughter born at 30 A. Dietary intake
weeks gestation and I lost a baby at about 8 B. Medication
C. Exercise
weeks,” the nurse should record
D. Glucose monitoring
her obstetrical history as which of the
following? Answer: A . Although all of the choices are
A. G2 T2 P0 A0 L2 important in the management of diabetes,
B. G3 T1 P1 A0 L2 diet therapy is the mainstay of the treatment
C. G3 T2 P0 A0 L2 plan and should always be the priority.
D. G4 T1 P1 A1 L2 Women diagnosed with gestational
diabetes generally need only diet therapy
Answer: D. The client has been pregnant four without medication to control their
times, including current pregnancy (G). Birth blood sugar levels. Exercise, is important for
at 38 weeks’ gestation is considered full term all pregnant women and especially for
(T), while birth form 20 weeks to 38 weeks is diabetic women, because it burns up glucose,
considered preterm (P). A thus decreasing blood sugar. However,
spontaneous abortion occurred at 8 weeks dietary intake, not exercise, is the priority.
(A). She has two living children (L). All pregnant women with diabetes should
have periodic monitoring of serum
9. When preparing to listen to the fetal heart glucose. However, those with gestational
rate at 12 weeks’ gestation, the nurse would diabetes generally do not need daily glucose
use which of the following? monitoring. The standard of care
recommends a fasting and 2- hour
A. Stethoscope placed midline at the postprandial blood sugar level every 2 weeks.
umbilicus
B. Doppler placed midline at the 11.A client at 24 weeks gestation has gained 6
suprapubic region pounds in 4 weeks. Which of the following
C. Fetoscope placed midway between the would be the priority when assessing the
umbilicus and the xiphoid process client?
A. Glucosuria B. Pain
B. Depression C. Knowledge Deficit
C. Hand/face edema D. Anticipatory Grieving
D. Dietary intake
Answer:B . For the client with an ectopic
Answer: C. After 20 weeks’ gestation, when pregnancy, lower abdominal pain,
there is a rapid weight gain, preeclampsia usually unilateral, is the primary symptom.
should be suspected, which may be caused Thus, pain is the priority. Although
by fluid retention manifested by edema, the potential for infection is always present,
especially of the hands and face. The three the risk is low in ectopic pregnancy because
classic signs of preeclampsia are pathogenic microorganisms have not been
hypertension, edema, and proteinuria. introduced from external sources. The client
Although urine is checked for glucose at each may have a limited knowledge of
clinic visit, this is not the priority. Depression the pathology and treatment of the condition
may cause either anorexia or excessive food and will most likely experience grieving, but
intake, leading to excessive weight gain or this is not the priority at this time.
loss. This is not, however, the priority
consideration at this time. Weight gain 14.Before assessing the postpartum client’s
thought to be caused by excessive food uterus for firmness and position in relation to
intake would require a 24-hour diet recall. the umbilicus and midline, which of the
However, excessive intake would not be the following should the nurse do first?
primary consideration for this client at this
time. A. Assess the vital signs
B. Administer analgesia
12. A client 12 weeks’ pregnant come to the C. Ambulate her in the hall
emergency department with abdominal D. Assist her to urinate
cramping and moderate vaginal bleeding.
Speculum examination reveals 2 to 3 cms Answer: D. Before uterine assessment is
performed, it is essential that the
cervical dilation. The nurse would document
woman empty her bladder. A full bladder
these findings as which of the following? will interfere with the accuracy of
A. Threatened abortion the assessment by elevating the uterus and
B. Imminent abortion displacing to the side of the midline. Vital
C. Complete abortion sign assessment is not necessary unless an
D. Missed abortion abnormality in uterine assessment is
identified. Uterine assessment should not
Answer:B. Cramping and vaginal bleeding cause acute pain that requires administration
coupled with cervical dilation signifies that of analgesia. Ambulating the client is an
termination of the pregnancy is inevitable and essential component of postpartum care, but
cannot be prevented. Thus, the nurse would is not necessary prior to assessment of the
document an imminent abortion. In a uterus.
threatened abortion, cramping and vaginal
bleeding are present, but there is no cervical 15.Which of the following should the nurse do
dilation. The symptoms may subside or when a primipara who is lactating tells the
progress to abortion. In a complete abortion nurse that she has sore nipples?
all the products of conception are expelled. A
missed abortion is early fetal intrauterine A. Tell her to breast feed more frequently
death without expulsion of the products of B. Administer a narcotic before breast
conception. feeding
C. Encourage her to wear a nursing brassiere
13.Which of the following would be the D. Use soap and water to clean the nipples
priority nursing diagnosis for a client with an
Answer:A. Feeding more frequently, about
ectopic pregnancy?
every 2 hours, will decrease the
A. Risk for infection infant’s frantic, vigorous sucking from
hunger and will decrease breast engorgement,
soften the breast, and promote ease of correct A. A dark red discharge on a 2-day
latching-on for feeding. Narcotics postpartum client
administered prior to breast feeding are B. A pink to brownish discharge on a client
passed through the breast milk to the infant, who is 5 days postpartum
causing excessive sleepiness. Nipple soreness C. Almost colorless to creamy discharge on
is not severe enough to warrant narcotic a client 2 weeks after delivery
analgesia. All postpartum clients, especially D. A bright red discharge 5 days after
lactating mothers, should wear a delivery
supportive brassiere with wide cotton straps.
This does not, however, prevent or reduce Answer: D. Any bright red vaginal discharge
nipple soreness. Soaps are drying to the skin would be considered abnormal,
of the nipples and should not be used on the but especially 5 days after delivery, when the
breasts of lactating mothers. Dry nipple lochia is typically pink to brownish. Lochia
skin predisposes to cracks and fissures, rubra, a dark red discharge, is present for 2 to
which can become sore and painful. 3 days after delivery. Bright red vaginal
bleeding at this time suggests late postpartum
16.The nurse assesses the vital signs of a hemorrhage, which occurs after the first 24
client, 4 hours’ postpartum that are as follows: hours following delivery and is generally
BP 90/60; temperature 100.4ºF; pulse 100 caused by retained placental fragments
weak, thready; R 20 per minute. Which of the or bleeding disorders. Lochia rubra is the
following should the nurse do first? normal dark red discharge occurring in the
first 2 to 3 days after delivery, containing
A. Report the temperature to the physician epithelial cells, erythrocyes, leukocytes and
B. Recheck the blood pressure with another decidua. Lochia serosa is a pink to
cuff brownish serosanguineous discharge
C. Assess the uterus for firmness and occurring from 3 to 10 days after delivery
position that contains decidua, erythrocytes,
D. Determine the amount of lochia leukocytes, cervical mucus,
and microorganisms. Lochia alba is an
Answer: D. A weak, thready pulse elevated almost colorless to yellowish
to 100 BPM may indicate discharge occurring from 10 days to 3 weeks
impending hemorrhagic shock. An increased after delivery and containing leukocytes,
pulse is a compensatory mechanism of the decidua, epithelial cells, fat, cervical mucus,
body in response to decreased fluid volume. cholesterol crystals, and bacteria.
Thus, the nurse should check the amount of
lochia present. Temperatures up to 100.48F 18.A postpartum client has a temperature of
in the first 24 hours after birth are related to 101.4ºF, with a uterus that is tender when
the dehydrating effects of labor and are palpated, remains unusually large, and not
considered normal. Although rechecking the descending as normally expected. Which of
blood pressure may be a correct choice of the following should the nurse assess next?
action, it is not the first action that should
be implemented in light of the other data. A. Lochia
The data indicate a potential impending B. Breasts
hemorrhage. Assessing the uterus for C. Incision
firmness and position in relation to the D. Urine
umbilicus and midline is important, but the
nurse should check the extent of vaginal Answer: A. The data suggests an infection of
bleeding first. Then it would be appropriate the endometrial lining of the uterus. The
to check the uterus, which may be a possible lochia may be decreased or copious, dark
cause of the hemorrhage. brown in appearance, and foul smelling,
providing further evidence of a possible
17.The nurse assesses the postpartum vaginal infection. All the client’s data indicate a
discharge (lochia) on four clients. Which of uterine problem, not a breast problem.
the following assessments would warrant Typically, transient fever, usually 101ºF, may
notification of the physician? be present with breast
engorgement. Symptoms of mastitis include Covering the scale with a warmed blanket
influenza-like manifestations. prior to weighing prevents heat loss through
Localized infection of an episiotomy or C- conduction. A knit cap prevents heat loss
section incision rarely causes from the head a large head, a large
systemic symptoms, and uterine involution body surface area of the newborn’s body.
would not be affected. The client data do not
include dysuria, frequency, or urgency, 21.A newborn who has an asymmetrical Moro
symptoms of urinary tract infections, which reflex response should be further assessed for
would necessitate assessing the client’s urine. which of the following?

19.Which of the following is the priority focus A. Talipes equinovarus


of nursing practice with the current early B. Fractured clavicle
postpartum discharge? C. Congenital hypothyroidism
D. Increased intracranial pressure
A. Promoting comfort and restoration of
health Answer: B. A fractured clavicle would
B. Exploring the emotional status of the prevent the normal Moro response
family of symmetrical sequential extension and
C. Facilitating safe and effective self-and abduction of the arms followed by flexion
newborn care and adduction. In talipes equinovarus
D. Teaching about the importance of family (clubfoot) the foot is turned medially, and in
planning plantar flexion, with the heel elevated. The
feet are not involved with the Moro reflex.
Answer: C. Because of early postpartum Hypothyroiddism has no effect on
discharge and limited time for teaching, the the primitive reflexes. Absence of the Moror
nurse’s priority is to facilitate the safe and reflex is the most significant single indicator
effective care of the client and newborn. of central nervous system status, but it is not
Although promoting comfort and restoration a sign of increased intracranial pressure.
of health, exploring the family’s emotional
status, and teaching about family 22.During the first 4 hours after a male
planning are important in circumcision, assessing for which of the
postpartum/newborn nursing care, they are following is the priority?
not the priority focus in the limited time
presented by early post-partum discharge. A. Infection
B. Hemorrhage
20. Which of the following actions would be C. Discomfort
least effective in maintaining a neutral thermal D. Dehydration
environment for the newborn?
Answer:B. Hemorrhage is a potential risk
A. Placing infant under radiant warmer after following any surgical procedure. Although
bathing the infant has been given vitamin K to
B. Covering the scale with a warmed blanket facilitate clotting, the prophylactic dose is
prior to weighing often not sufficient to prevent bleeding.
C. Placing crib close to nursery window Although infection is a possibility, signs will
for family viewing not appear within 4 hours after the surgical
D. Covering the infant’s head with a knit procedure. The primary discomfort of
stockinette circumcision occurs during the surgical
procedure, not afterward. Although feedings
Answer: C. Heat loss by radiation occurs are withheld prior to the circumcision, the
when the infant’s crib is placed too near cold chances of dehydration are minimal.
walls or windows. Thus placing the
newborn’s crib close to the viewing window 23.The mother asks the nurse. “What’s wrong
would be least effective. Body heat is lost with my son’s breasts? Why are they so
through evaporation during bathing. Placing enlarged?” Whish of the following would be
the infant under the radiant warmer after the best response by the nurse?
bathing will assist the infant to be rewarmed.
A. “The breast tissue is inflamed from the Which of the following statements by the
trauma experienced with birth” mother indicates effective teaching?
B. “A decrease in material hormones
present before birth causes enlargement,” A. “Daily soap and water cleansing is best”
C. “You should discuss this with your doctor. B. ‘Alcohol helps it dry and kills germs”
It could be a malignancy” C. “An antibiotic ointment applied daily
D. “The tissue has hypertrophied while the prevents infection”
baby was in the uterus” D. “He can have a tub bath each day”

Answer: B . The presence of excessive Answer: B. Application of 70% isopropyl


estrogen and progesterone in the maternal alcohol to the cord
fetal blood followed by prompt withdrawal at minimizes microorganisms (germicidal) and
birth precipitates breast engorgement, which promotes drying. The cord should be kept
will spontaneously resolve in 4 to 5 days dry until it falls off and the stump has healed.
after birth. The trauma of the birth process Antibiotic ointment should only be used to
does not cause inflammation of treat an infection, not as a prophylaxis.
the newborn’s breast tissue. Newborns do not Infants should not be submerged in a tub of
have breast malignancy. This reply by the water until the cord falls off and the stump
nurse would cause the mother to have undue has completely healed.
anxiety. Breast tissue does not hypertrophy in
the fetus or newborns. 26.A newborn weighing 3000 grams and
feeding every 4 hours needs 120 calories/kg
24. Immediately after birth the nurse notes the of body weight every 24 hours for proper
following on a male newborn: respirations 78; growth and development. How many ounces
apical hearth rate 160 BPM, nostril flaring; of 20 cal/oz formula should this
mild intercostal newborn receive at each feeding to meet
nutritional needs?
retractions; and grunting at the end of
expiration. Which of the following should the A. 2 ounces
nurse do? B. 3 ounces
C. 4 ounces
A. Call the assessment data to the D. 6 ounces
physician’s attention
B. Start oxygen per nasal cannula at 2 L/min. Answer:B. To determine the amount of
C. Suction the infant’s mouth and nares formula needed, do the
D. Recognize this as normal first period of following mathematical calculation. 3 kg x
reactivity 120 cal/kg per day = 360 calories/day feeding
q 4 hours = 6 feedings per day = 60 calories
Answer: D . The first 15 minutes to 1 hour per feeding: 60 calories per feeding; 60
after birth is the first period of calories per feeding with formula 20 cal/oz =
reactivity involving respiratory and 3 ounces per feeding. Based on the
circulatory adaptation to extrauterine life. calculation. 2, 4 or 6 ounces are incorrect.
The data given reflect the normal changes
during this time period. The 27.The postterm neonate with meconium-
infant’s assessment data reflect normal stained amniotic fluid needs care designed to
adaptation. Thus, the physician does not need especially monitor for which of the following?
to be notified and oxygen is not needed. The
data do not indicate the presence of choking, A. Respiratory problems
gagging or coughing, which are signs of B. Gastrointestinal problems
excessive secretions. Suctioning is not C. Integumentary problems
necessary. D. Elimination problems

25.The nurse hears a mother telling a friend Answer: A. Intrauterine anoxia may cause
on the telephone about umbilical cord care. relaxation of the anal sphincter and emptying
of meconium into the amniotic fluid. At birth
some of the meconium fluid may be aspirated,
causing mechanical obstruction or chemical facilitate coping and a sense of control, but
pneumonitis. The infant is not at increased seizure precautions are the priority.
risk for gastrointestinal problems. Even
though the skin is stained with meconium, it 30. A postpartum primipara asks the nurse,
is noninfectious (sterile) and nonirritating. “When can we have sexual intercourse again?”
The postterm meconiumstained infant is not Which of the following would be the nurse’s
at additional risk for bowel or urinary best response?
problems.
A. “Anytime you both want to.”
28.When measuring a client’s fundal height, B. “As soon as choose a contraceptive
which of the following techniques denotes the method.”
correct method of measurement used by the C. “When the discharge has stopped and
nurse? the incision is healed.”
D. “After your 6 weeks examination.”
A. From the xiphoid process to the umbilicus
B. From the symphysis pubis to the xiphoid Answer: C. Cessation of the lochial
process discharge signifies healing of the
C. From the symphysis pubis to the endometrium. Risk of hemorrhage and
fundus infection are minimal 3 weeks after a
D. From the fundus to the umbilicus normal vaginal delivery. Telling the client
anytime is inappropriate because
Answer: C . The nurse should use a this response does not provide the client with
nonelastic, flexible, paper measuring the specific information she is requesting.
tape, placing the zero point on the superior Choice of a contraceptive method is
border of the symphysis pubis and stretching important, but not the specific criteria for
the tape across the abdomen at the midline to safe resumption of sexual activity. Culturally,
the top of the fundus. The xiphoid and the 6- weeks’ examination has been used as
umbilicus are not appropriate landmarks to the time frame for resuming sexual activity,
use when measuring the height of the fundus but it may be resumed earlier.
(McDonald’s measurement).
31.When preparing to administer the vitamin
29.A client with severe preeclampsia is K injection to a neonate, the nurse would
admitted with of BP 160/110, proteinuria, and select which of the following sites as
severe pitting edema. Which of the following appropriate for the injection?
would be most important to include in the
A. Deltoid muscle
client’s plan of care?
B. Anterior femoris muscle
A. Daily weights C. Vastus lateralis muscle
B. Seizure precautions D. Gluteus maximus muscle
C. Right lateral positioning
D. Stress reduction Answer: C . The middle third of the vastus
lateralis is the preferred injection site
Answer: B . Women hospitalized with severe for vitamin K administration because it is
preeclampsia need decreased free of blood vessels and nerves and is large
CNS stimulation to prevent a seizure. Seizure enough to absorb the medication. The deltoid
precautions provide environmental safety muscle of a newborn is not large enough for
should a seizure occur. Because of edema, a newborn IM injection. Injections into
daily weight is important but not the priority. this muscle in a small child might cause
Preclampsia causes vasospasm and therefore damage to the radial nerve. The anterior
can reduce utero-placental perfusion. The femoris muscle is the next safest muscle to
client should be placed on her left side to use in a newborn but is not the safest.
maximize blood flow, reduce blood pressure, Because of the proximity of the sciatic nerve,
and promote diuresis. Interventions to reduce the gluteus maximus muscle should not be
stress and anxiety are very important to until the child has been walking 2 years.
32.When performing a pelvic examination, Eating low-sodium crackers would
the nurse observes a red swollen area on the be appropriate. Since liquids can increase
right side of the vaginal orifice. The nurse nausea avoiding them in the morning hours
would document this as enlargement of which when nausea is usually the strongest is
of the following? appropriate. Eating six small meals a day
would keep the stomach full, which often
A. Clitoris decrease nausea.
B. Parotid gland
C. Skene’s gland 35.The nurse documents positive ballottement
D. Bartholin’s gland in the client’s prenatal record. The nurse
understands that this indicates which of the
Answer: D . Bartholin’s glands are the glands following?
on either side of the vaginal orifice. The
clitoris is female erectile tissue found in the A. Palpable contractions on the abdomen
perineal area above the urethra. The parotid B. Passive movement of the unengaged
glands are open into the mouth. Skene’s fetus
glands open into the posterior wall of the C. Fetal kicking felt by the client
female urinary meatus. D. Enlargement and softening of the uterus

33.To differentiate as a female, the hormonal Answer: B . Ballottement indicates passive


stimulation of the embryo that must occur movement of the unengaged
involves which of the following? fetus. Ballottement is not a contraction. Fetal
kicking felt by the client
A. Increase in maternal estrogen secretion represents quickening. Enlargement and
B. Decrease in maternal androgen secretion softening of the uterus is known
C. Secretion of androgen by the fetal gonad as Piskacek’s sign.
D. Secretion of estrogen by the fetal gonad
36.During a pelvic exam the nurse notes a
Answer: D . The fetal gonad must secrete purple-blue tinge of the cervix. The nurse
estrogen for the embryo to differentiate as a documents this as which of the following?
female. An increase in maternal estrogen
secretion does not effect differentiation of the A. Braxton-Hicks sign
embryo, and maternal estrogen secretion B. Chadwick’s sign
occurs in every pregnancy. Maternal C. Goodell’s sign
androgen secretion remains the same D. McDonald’s sign
as before pregnancy and does not effect
differentiation. Secretion of androgen by the Answer: B . Chadwick’s sign refers to the
fetal gonad would produce a male fetus. purple-blue tinge of the cervix.
Braxton Hicks contractions are painless
34.A client at 8 weeks’ gestation calls contractions beginning around the 4th month.
complaining of slight nausea in the morning Goodell’s sign indicates softening of the
hours. Which of the following client cervix. Flexibility of the uterus against the
interventions should the nurse question? cervix is known as McDonald’s sign.

A. Taking 1 teaspoon of bicarbonate of 37.During a prenatal class, the nurse explains


soda in an 8-ounce glass of water the rationale for breathing techniques during
B. Eating a few low-sodium crackers before preparation for labor based on the
getting out of bed understanding that breathing techniques are
C. Avoiding the intake of liquids in the most important in achieving which of
morning hours
the following?
D. Eating six small meals a day instead of
thee large meals A. Eliminate pain and give the expectant
parents something to do
Answer: A . Using bicarbonate would B. Reduce the risk of fetal distress by
increase the amount of sodium increasing uteroplacental perfusion
ingested, which can cause complications.
C. Facilitate relaxation, possibly reducing done because it could cause
the perception of pain hemorrhage. Assessing maternal vital signs
D. Eliminate pain so that less analgesia and can help determine maternal
anesthesia are needed physiologic status. Fetal heart rate is
important to assess fetal well-being and
Answer: C . Breathing techniques can raise should be done. Monitoring the contractions
the pain threshold and reduce the perception will help evaluate the progress of labor.
of pain. They also promote relaxation.
Breathing techniques do not eliminate pain, 40.Which of the following would be the
but they can reduce it. Positioning, not nurse’s most appropriate response to a client
breathing, increases uteroplacental perfusion. who asks why she must have a cesarean
delivery if she has a complete placenta previa?
38.After 4 hours of active labor, the nurse
notes that the contractions of a primigravida A. “You will have to ask your physician
client are not strong enough to dilate the when he returns.”
cervix. Which of the B. “You need a cesarean to prevent
hemorrhage.”
following would the nurse anticipate doing? C. “The placenta is covering most of your
cervix.”
A. Obtaining an order to begin IV D. “The placenta is covering the opening
oxytocin infusion of the uterus and blocking your baby.”
B. Administering a light sedative to allow
the patient to rest for several hour Answer: D . A complete placenta previa
C. Preparing for a cesarean section for occurs when the placenta covers the opening
failure to progress of the uterus, thus blocking the passageway
D. Increasing the encouragement to the for the baby. This response explains what a
patient when pushing begins complete previa is and the reason the
baby cannot come out except by cesarean
Answer:A . The client’s labor is hypotonic. delivery. Telling the client to ask
The nurse should call the physical and obtain the physician is a poor response and would
an order for an infusion of oxytocin, which increase the patient’s anxiety. Although a
will assist the uterus to contact more cesarean would help to prevent hemorrhage,
forcefully in an attempt to dilate the cervix. the statement does not explain why the
Administering light sedative would be done hemorrhage could occur. With a
for hypertonic uterine contractions. complete previa, the placenta is covering all
Preparing for cesarean section is unnecessary the cervix, not just most of it.
at this time. Oxytocin would increase the
uterine contractions and hopefully progress 41.The nurse understands that the fetal head is
labor before a cesarean would be necessary.
in which of the following positions with a face
It is too early to anticipate client pushing
with contractions. presentation?
A. Completely flexed
39.A multigravida at 38 weeks’ gestation is B. Completely extended
admitted with painless, bright red bleeding C. Partially extended
and mild contractions every 7 to 10 minutes. D. Partially flexed
Which of the following assessments should be
avoided? Answer: B . With a face presentation, the
head is completely extended. With a vertex
A. Maternal vital sign presentation, the head is completely or
B. Fetal heart rate partially flexed. With a brow (forehead)
C. Contraction monitoring presentation, the head would be partially
D. Cervical dilation extended.

Answer: D . The signs indicate placenta 42.With a fetus in the left-anterior breech
previa and vaginal exam to presentation, the nurse would expect the fetal
determine cervical dilation would not be
heart rate would be most audible in which of 45.When describing dizygotic twins to a
the following areas? couple, on which of the following would the
nurse base the explanation?
A. Above the maternal umbilicus and to the
right of midline A. Two ova fertilized by separate sperm
B. In the lower-left maternal abdominal B. Sharing of a common placenta
quadrant C. Each ova with the same genotype
C. In the lower-right maternal abdominal D. Sharing of a common chorion
quadrant
D. Above the maternal umbilicus and to Answer: A . Dizygotic (fraternal) twins
the left of midline involve two ova fertilized by separate
sperm. Monozygotic (identical) twins involve
Answer: D . With this presentation, the fetal a common placenta, same genotype, and
upper torso and back face the left common chorion.
upper maternal abdominal wall. The fetal
heart rate would be most audible above the 46.Which of the following refers to the single
maternal umbilicus and to the left of the cell that reproduces itself after conception?
middle.A The other positions would be
incorrect. A. Chromosome
B. Blastocyst
43.The amniotic fluid of a client has a C. Zygote
greenish tint. The nurse interprets this to be D. Trophoblast
the result of which of the following?
Answer: C . The zygote is the single cell that
A. Lanugo reproduces itself after conception.
B. Hydramnio The chromosome is the material that makes
C. Meconium up the cell and is gained from each parent.
D. Vernix Blastocyst and trophoblast are later terms for
the embryo after zygote.
Answer: C. The greenish tint is due to the
presence of meconium. Lanugo is the soft, 47.In the late 1950s, consumers and health
downy hair on the shoulders and back of the care professionals began challenging the
fetus. Hydramnios represents excessive routine use of analgesics and anesthetics
amniotic fluid. Vernix is the white, cheesy during childbirth. Which of the following was
substance covering the fetus. an outgrowth of this concept?
44.A patient is in labor and has just been told A. Labor, delivery, recovery, postpartum
she has a breech presentation. The nurse (LDRP)
should be particularly alert for which of the B. Nurse-midwifery
following? C. Clinical nurse specialist
D. Prepared childbirth
A. Quickening
B. Ophthalmia neonatorum Answer: D . Prepared childbirth was the
C. Pica direct result of the 1950’s challenging of
D. Prolapsed umbilical cord the routine use of analgesic and anesthetics
during childbirth. The LDRP was a much
Answer: D . In a breech position, because of later concept and was not a direct result of
the space between the presenting part and the the challenging of routine use of analgesics
cervix, prolapse of the umbilical cord is and anesthetics during childbirth. Roles
common. Quickening is the woman’s first for nurse midwives and clinical nurse
perception of fetal movement. Ophthalmia specialists did not develop from
neonatorum usually results from maternal this challenge.
gonorrhea and is conjunctivitis. Pica refers
to the oral intake of nonfood substances. 48.A client has a midpelvic contracture from a
previous pelvic injury due to a motor vehicle
accident as a teenager. The nurse is aware that
this could PNLE III for
prevent a fetus from passing through or
around which structure during childbirth?
Medical Surgical
A. Symphysis pubis Nursing
B. Sacral promontory
C. Ischial spines 1. Marco who was diagnosed with
D. Pubic arch brain tumor was scheduled for craniotomy. In
preventing the development of cerebral edema
Answer: C . The ischial spines are located in after surgery, the nurse should expect the use
the mid-pelvic region and could be narrowed of:
due to the previous pelvic injury. The
symphysis pubis, sacral promontory, and A. Diuretics
pubic arch are not part of the mid-pelvis. B. Antihypertensive
C. Steroids
49.When teaching a group of adolescents D. Anticonvulsants
about variations in the length of the menstrual
cycle, the nurse understands that the Answer: C . Glucocorticoids (steroids) are
underlying mechanism is used for their anti-inflammatory action,
due to variations in which of the following which decreases the development of edema.
phases?
2. Halfway through the administration of
A. Menstrual phase blood, the female client complains of lumbar
B. Proliferative phase pain. After stopping the infusion Nurse Hazel
C. Secretory phase should:
D. Ischemic phase
A. Increase the flow of normal saline
Answer: B . Variations in the length of the B. Assess the pain further
menstrual cycle are due to variations in the C. Notify the blood bank
proliferative phase. The menstrual, secretory D. Obtain vital signs.
and ischemic phases do not contribute to this
variation. Answer: A . The blood must be stopped at
once, and then normal saline should
50.When teaching a group of adolescents be infused to keep the line patent and
about male hormone production, which of the maintain blood volume.
following would the nurse include as being
produced by the Leydig cells? 3. Nurse Maureen knows that the positive
diagnosis for HIV infection is made based on
A. Follicle-stimulating hormone which of the following:
B. Testosterone
C. Leuteinizing hormone A. A history of high risk sexual behaviors.
D. Gonadotropin releasing hormone B. Positive ELISA and western blot tests
C. Identification of an associated
Answer: B . Testosterone is produced by the opportunistic infection
Leyding cells in the seminiferous tubules. D. Evidence of extreme weight loss and high
Follicle-stimulating hormone and leuteinzing fever
hormone are released by the anterior pituitary
gland. The hypothalamus is responsible for Answer:B . These tests confirm the presence
releasing gonadotropin-releasing hormone. of HIV antibodies that occur in response to
the presence of the human immunodeficiency
virus (HIV).
4. Nurse Maureen is aware that a client who Answer: C . Elevation increases lymphatic
has been diagnosed with chronic renal failure drainage, reducing edema and pain.
recognizes an adequate amount of high-
biologic-value protein when the food the 8. Nurse hazel receives emergency laboratory
client selected from the menu was: results for a client with chest pain and
immediately informs the physician. An
A. Raw carrots increased myoglobin level suggests which of
B. Apple juice the following?
C. Whole wheat bread
D. Cottage cheese A. Liver disease
B. Myocardial damage
Answer: D . One cup of cottage cheese C. Hypertension
contains approximately 225 calories, 27 g D. Cancer
of protein, 9 g of fat, 30 mg cholesterol, and
6 g of carbohydrate. Proteins of high biologic Answer: B . Detection of myoglobin is a
value (HBV) contain optimal levels of amino diagnostic tool to determine
acids essential for life. whether myocardial damage has occurred.

5. Kenneth who has diagnosed with uremic 9. Nurse Maureen would expect the a client
syndrome has the potential to develop with mitral stenosis would demonstrate
complications. Which among the following symptoms associated with congestion in the:
complications should the nurse anticipates:
A. Right atrium
A. Flapping hand tremors B. Superior vena cava
B. An elevated hematocrit level C. Aorta
C. Hypotension D. Pulmonary
D. Hypokalemia
Answer: D . When mitral stenosis is present,
Answer: A . Elevation of uremic waste the left atrium has difficulty emptying
products causes irritation of the nerves, its contents into the left ventricle because
resulting in flapping hand tremors. there is no valve to prevent back ward flow
into the pulmonary vein, the pulmonary
6. A client is admitted to the hospital with circulation is under pressure.
benign prostatic hyperplasia, the nurse most
relevant assessment would be: 10. A client has been diagnosed with
hypertension. The nurse priority nursing
A. Flank pain radiating in the groin diagnosis would be:
B. Distention of the lower abdomen
C. Perineal edema A. Ineffective health maintenance
D. Urethral discharge B. Impaired skin integrity
C. Deficient fluid volume
Answer: B . This indicates that the bladder is D. Pain
distended with urine, therefore palpable.
Answer: A . Managing hypertension is the
7. A client has undergone with penile implant. priority for the client with
After 24 hrs of surgery, the client’s scrotum hypertension. Clients with hypertension
was edematous and painful. The nurse should: frequently do not experience pain, deficient
volume, or impaired skin integrity. It is the
A. Assist the client with sitz bath asymptomatic nature of hypertension
B. Apply war soaks in the scrotum that makes it so difficult to treat.
C. Elevate the scrotum using a soft
support 11. Nurse Hazel teaches the client with angina
D. Prepare for a possible incision and about common expected side effects of
drainage. nitroglycerin including:
A. high blood pressure Answer: A . Good source of vitamin B12 are
B. stomach cramps dairy products and meats.
C. headache
D. shortness of breath 15. Karen has been diagnosed with aplastic
anemia. The nurse monitors for changes in
Answer: C . Because of its widespread which of the following physiologic functions?
vasodilating effects, nitroglycerin
often produces side effects such as headache, A. Bowel function
hypotension and dizziness. B. Peripheral sensation
C. Bleeding tendencies
12. The following are lipid abnormalities. D. Intake and out put
Which of the following is a risk factor for the
development of atherosclerosis and PVD? Answer: C . Aplastic anemia decreases the
bone marrow production of RBC’s,
A. High levels of low density lipid (LDL) white blood cells, and platelets. The client is
cholesterol at risk for bruising and bleeding tendencies.
B. High levels of high density lipid (HDL)
cholesterol 16. Lydia is scheduled for elective
C. Low concentration triglycerides splenectomy. Before the clients goes to
D. Low levels of LDL cholesterol. surgery, the nurse in charge final assessment
would be:
Answer: A. An increased in LDL cholesterol
concentration has been documented at risk A. signed consent
factor for the development of atherosclerosis. B. vital signs
LDL cholesterol is not broken down into the C. name band
liver but is deposited into the wall of the D. empty bladder
blood vessels.
Answer: B . An elective procedure is
13. Which of the following represents a scheduled in advance so that all preparations
significant risk immediately after surgery for can be completed ahead of time. The vital
repair of aortic aneurysm? signs are the final check that must
be completed before the client leaves the
A. Potential wound infection room so that continuity of care
B. Potential ineffective coping and assessment is provided for.
C. Potential electrolyte balance
D. Potential alteration in renal perfusion 17. What is the peak age range in acquiring
acute lymphocytic leukemia (ALL)?
Answer: D . There is a potential alteration in
renal perfusion manifested by A. 4 to 12 years.
decreased urine output. The altered renal B. 20 to 30 years
perfusion may be related to renal C. 40 to 50 years
artery embolism, prolonged hypotension, or D. 60 60 70 years
prolonged aortic cross-clamping during the
surgery. Answer: A . The peak incidence of Acute
Lymphocytic Leukemia (ALL) is 4 years
14. Nurse Josie should instruct the client to of age. It is uncommon after 15 years of age.
eat which of the following foods to obtain the
best supply of Vitamin B12? 18. Marie with acute lymphocytic leukemia
suffers from nausea and headache. These
A. dairy products clinical manifestations may indicate all of the
B. vegetables following except
C. Grains
D. Broccoli A. effects of radiation
B. chemotherapy side effects
C. meningeal irritation
D. gastric distension
22. Karina a client with myasthenia gravis is
Answer: D . Acute Lymphocytic Leukemia to receive immunosuppressive therapy. The
(ALL) does not cause gastric distention. nurse understands that this therapy is effective
It does invade the central nervous system, because it:
and clients experience headaches and
vomiting from meningeal irritation. A. Promotes the removal of antibodies that
impair the transmission of impulses
19. A client has been diagnosed with B. Stimulates the production of acetylcholine
Disseminated Intravascular Coagulation at the neuromuscular junction.
(DIC). Which of the following is C. Decreases the production of
contraindicated with the client? autoantibodies that attack the
acetylcholine receptors.
A. Administering Heparin D. Inhibits the breakdown of acetylcholine at
B. Administering Coumadin the neuromuscular junction.
C. Treating the underlying cause
D. Replacing depleted blood products Answer: C . Steroids decrease the body’s
immune response thus decreasing
Answer: B . Disseminated Intravascular the production of antibodies that attack the
Coagulation (DIC) has not been found acetylcholine receptors at the neuromuscular
to respond to oral anticoagulants such as junction
Coumadin.
23. A female client is receiving IV Mannitol.
20. Which of the following findings is the best An assessment specific to safe administration
indication that fluid replacement for the client of the said drug is:
with hypovolemic shock is adequate?
A. Vital signs q4h
A. Urine output greater than 30ml/hr B. Weighing daily
B. Respiratory rate of 21 breaths/minute C. Urine output hourly
C. Diastolic blood pressure greater than 90 D. Level of consciousness q4h
mmhg
D. Systolic blood pressure greater than 110 Answer: C . The osmotic diuretic mannitol is
mmhg contraindicated in the presence of inadequate
renal function or heart failure because it
Answer: A . Urine output provides the most increases the intravascular volume that must
sensitive indication of the client’s response to be filtered and excreted by the kidney.
therapy for hypovolemic shock. Urine output
should be consistently greater than 30 to 35 24. Patricia a 20 year old college student with
mL/hr. diabetes mellitus requests additional
information about the advantages of using a
21. Which of the following signs and pen like insulin
symptoms would Nurse Maureen include in delivery devices. The nurse explains that the
teaching plan as an early manifestation of advantages of these devices over syringes
laryngeal cancer? includes:

A. Stomatitis A. Accurate dose delivery


B. Airway obstruction B. Shorter injection time
C. Hoarseness C. Lower cost with reusable insulin
D. Dysphagia cartridges
D. Use of smaller gauge needle.
Answer: C . Early warning signs of laryngeal
cancer can vary depending on tumor location. Answer: A . These devices are more accurate
Hoarseness lasting 2 weeks should be because they are easily to used and have
evaluated because it is one of the most improved adherence in insulin regimens by
common warning signs.
young people because the medication can be A. Palms of the hands and axillary regions
administered discreetly. B. Palms of the hand
C. Axillary regions
25. A male client’s left tibia is fractures in an D. Feet, which are set apart
automobile accident, and a cast is applied. To
assess for damage to major blood vessels from Answer: B . The palms should bear the
the fracture tibia, the nurse in charge should client’s weight to avoid damage to the nerves
monitor the client for: in the axilla.

A. Swelling of the left thigh 29. Mang Jose with rheumatoid arthritis states,
B. Increased skin temperature of the foot “the only time I am without pain is when I lie
C. Prolonged reperfusion of the toes after in bed perfectly still”. During the
blanching convalescent stage, the nurse in charge with
D. Increased blood pressure Mang Jose should encourage:

Answer: C . Damage to blood vessels may A. Active joint flexion and extension
decrease the circulatory perfusion of the toes, B. Continued immobility until pain subsides
this would indicate the lack of blood supply C. Range of motion exercises twice daily
to the extremity. D. Flexion exercises three times daily

26. After a long leg cast is removed, the male Answer: A . Active exercises, alternating
client should: extension, flexion, abduction, and
adduction, mobilize exudates in the joints
A. Cleanse the leg by scrubbing with a brisk relieves stiffness and pain.
motion
B. Put leg through full range of motion twice 30. A male client has undergone spinal
daily surgery, the nurse should:
C. Report any discomfort or stiffness to the
physician A. Observe the client’s bowel movement and
D. Elevate the leg when sitting for long voiding patterns
periods of time. B. Log-roll the client to prone position
C. Assess the client’s feet for sensation
Answer: D . Elevation will help control the and circulation
edema that usually occurs. D. Encourage client to drink plenty of fluids

27. While performing a physical assessment Answer: C . Alteration in sensation and


of a male client with gout of the great toe, circulation indicates damage to the spinal
NurseVivian should assess for additional cord, if these occurs notify physician
tophi (urate deposits) on the: immediately.

A. Buttocks 31. Marina with acute renal failure moves into


B. Ears the diuretic phase after one week of therapy.
C. Face During this phase the client must be assessed
D. Abdomen for signs of developing:

Answer: B . Uric acid has a low solubility, it A. Hypovolemia


tends to precipitate and form deposits B. renal failure
at various sites where blood flow is least C. metabolic acidosis
active, including cartilaginous tissue such as D. Hyperkalemia
the ears.
Answer: A . In the diuretic phase fluid
28. Nurse Katrina would recognize that the retained during the oliguric phase is
demonstration of crutch walking with tripod excreted and may reach 3 to 5 liters daily,
gait was understood when the client places hypovolemia may occur and fluids should
weight on the: be replaced.
A. “Practice using the mechanical aids that
32. Nurse Judith obtains a specimen of clear you will need when future disabilities
nasal drainage from a client with a head injury. arise”.
Which of the following tests differentiates B. “Follow good health habits to change the
mucus from cerebrospinal fluid (CSF)? course of the disease”.
C. “Keep active, use stress reduction
A. Protein strategies, and avoid fatigue.
B. Specific gravity D. “You will need to accept the necessity for
C. Glucose a quiet and inactive lifestyle”.
D. Microorganism
Answer: C . The nurse most positive
Answer: C . The constituents of CSF are approach is to encourage the client with
similar to those of blood plasma. multiple sclerosis to stay active, use stress
An examination for glucose content is done reduction techniques and avoid
to determine whether a body fluid is a mucus fatigue because it is important to support the
or a CSF. A CSF normally contains glucose. immune system while remaining active.

33. A 22 year old client suffered from his first 36. The nurse is aware the early indicator of
tonic-clonic seizure. Upon awakening the hypoxia in the unconscious client is:
client asks the nurse, “What caused me to
have a seizure? Which of the following would A. Cyanosis
the nurse include in the primary cause of tonic B. Increased respirations
clonic seizures in adults more the 20 years? C. Hypertension
D. Restlessness
A. Electrolyte imbalance
B. Head trauma Answer: D . Restlessness is an early indicator
C. Epilepsy of hypoxia. The nurse should
D. Congenital defect suspect hypoxia in unconscious client who
suddenly becomes restless.
Answer: B . Trauma is one of the primary
cause of brain damage and seizure activity 37. A client is experiencing spinal shock.
in adults. Other common causes of seizure Nurse Myrna should expect the function of
activity in adults include the bladder to be which of the following?
neoplasms, withdrawal from drugs and
alcohol, and vascular disease. A. Normal
B. Atonic
34. What is the priority nursing assessment in C. Spastic
the first 24 hours after admission of the client D. Uncontrolled
with thrombotic CVA?
Answer: B . In spinal shock, the bladder
A. Pupil size and papillary response becomes completely atonic and will
B. cholesterol level continue to fill unless the client is
C. Echocardiogram catheterized.
D. Bowel sounds
38. Which of the following stage the
Answer: A . It is crucial to monitor the pupil carcinogen is irreversible?
size and papillary response to
indicate changes around the cranial nerves. A. Progression stage
B. Initiation stage
35. Nurse Linda is preparing a client with C. Regression stage
multiple sclerosis for discharge from the D. Promotion stage
hospital to home. Which of the following
instruction is most appropriate? Answer: A . Progression stage is the change
of tumor from the preneoplastic state or low
degree of malignancy to a fast growing tumor
that cannot be reversed. 43. Nurse Jon assesses vital signs on a client
undergone epidural anesthesia.
39. Among the following components Which of the following would the nurse
thorough pain assessment, which is the most assess next?
significant?
A. Headache
A. Effect B. Bladder distension
B. Cause C. Dizziness
C. Causing factors D. Ability to move legs
D. Intensity
Answer: B . The last area to return sensation
Answer: D . Intensity is the major indicative is in the perineal area, and the nurse
of severity of pain and it is important for in charge should monitor the client for
the evaluation of the treatment. distended bladder.

40. A 65 year old female is experiencing flare 44. Nurse Katrina should anticipate that all of
up of pruritus. Which of the client’s action the following drugs may be used in the
could aggravate the cause of flare ups? attempt to control the symptoms of Meniere’s
disease except:
A. Sleeping in cool and humidified
environment A. Antiemetics
B. Daily baths with fragrant soap B. Diuretics
C. Using clothes made from 100% cotton C. Antihistamines
D. Increasing fluid intake D. Glucocorticoids

Answer: B . The use of fragrant soap is very Answer: D . Glucocorticoids play no


drying to skin hence causing the pruritus. significant role in disease treatment.

41. Atropine sulfate (Atropine) is 45. Which of the following complications


contraindicated in all but one of the following associated with tracheostomy tube?
client?
A. Increased cardiac output
A. A client with high blood B. Acute respiratory distress syndrome
B. A client with bowel obstruction (ARDS)
C. A client with glaucoma C. Increased blood pressure
D. A client with U.T.I D. Damage to laryngeal nerves

Answer: C . Atropine sulfate is Answer: D . Tracheostomy tube has several


contraindicated with glaucoma patients potential complications including
because it increases intraocular pressure. bleeding, infection and laryngeal nerve
damage.
42. Among the following clients, which
among them is high risk for potential hazards 46. Nurse Faith should recognize that fluid
from the surgical experience? shift in an client with burn injury results from
increase in the:
A. 67-year-old client
B. 49-year-old client A. Total volume of circulating whole blood
C. 33-year-old client B. Total volume of intravascular plasma
D. 15-year-old client C. Permeability of capillary walls
D. Permeability of kidney tubules
Answer: A . A 67 year old client is greater
risk because the older adult client is Answer: C . In burn, the capillaries and small
more likely to have a less-effective immune vessels dilate, and cell damage cause
system. the release of a histamine-like substance. The
substance causes the capillary walls to C. Provide emotional support
become more permeable and significant D. Promote means of communication
quantities of fluid are lost.
Answer: A . Patent airway is the most
47. An 83-year-old woman has several priority; therefore removal of secretions
ecchymotic areas on her right arm. The is necessary.
bruises are probably caused by:

A. increased capillary fragility and


PNLE IV for
permeability
B. increased blood supply to the skin Psychiatric Nursing
C. self inflicted injury
D. elder abuse
1. Marco approached Nurse Trish asking for advice
on how to deal with his alcohol addiction. Nurse
Answer: A . Aging process involves
Trish should tell the client that the only effective
increased capillary fragility and permeability. treatment for alcoholism is:
Older adults have a decreased amount of
subcutaneous fat and cause an A. Psychotherapy
increased incidence of bruise like lesions B. Alcoholics anonymous (A.A.)
caused by collection of extravascular blood C. Total abstinence
inloosely structured dermis. D. Aversion Therapy

48. Nurse Anna is aware that early adaptation Answer: C . Total abstinence is the only effective
treatment for alcoholism
of client with renal carcinoma is:
2. Nurse Hazel is caring for a male client who
A. Nausea and vomiting experience false sensory perceptions with no basis
B. flank pain in reality. This perception is known as:
C. weight gain
D. intermittent hematuria A. Hallucinations
B. Delusions
Answer: D . Intermittent pain is the classic C. Loose associations
sign of renal carcinoma. It is primarily due D. Neologisms
to capillary erosion by the cancerous growth. Answer: A . Hallucinations are visual, auditory,
gustatory, tactile or olfactory perceptions that have
49. A male client with tuberculosis asks Nurse no basis in reality.
Brian how long the chemotherapy must be
continued. Nurse Brian’s accurate reply would 3. Nurse Monet is caring for a female client who
be: has suicidal tendency. When accompanying the
client to the restroom, Nurse Monet should…
A. 1 to 3 weeks
A. Give her privacy
B. 6 to 12 months
B. Allow her to urinate
C. 3 to 5 months C. Open the window and allow her to get some
D. 3 years and more fresh air
D. Observe her
Answer: B . Tubercle bacillus is a drug
resistant organism and takes a long time to Answer: D . The Nurse has a responsibility to
be eradicated. Usually a combination of three observe continuously the acutely suicidal client.
drugs is used for minimum of 6 months and The Nurse should watch for clues, such as
at least six months beyond culture conversion. communicating suicidal thoughts, and messages;
hoarding medications and talking about death.
50. A client has undergone laryngectomy. The 4. Nurse Maureen is developing a plan of care for a
immediate nursing priority would be: female client with anorexia nervosa. Which action
should the nurse include in the plan?
A. Keep trachea free of secretions
B. Monitor for signs of infection A. Provide privacy during meals
B. Set-up a strict eating plan for the client
C. Encourage client to exercise to reduce anxiety 9. Nurse Claire is caring for a client diagnosed with
D. Restrict visits with the family bulimia. The most appropriate initial goal for a
client diagnosed with bulimia is?
Answer: B . Establishing a consistent eating plan
and monitoring client’s weight are important to A. Encourage to avoid foods
this disorder. B. Identify anxiety causing situations
C. Eat only three meals a day
5. A client is experiencing anxiety attack. The most D. Avoid shopping plenty of groceries
appropriate nursing intervention should include?
Answer: B . Bulimia disorder generally is a
A. Turning on the television maladaptive coping response to stress
B. Leaving the client alone and underlying issues. The client should identify
C. Staying with the client and speaking in short anxiety causing situation that stimulate the bulimic
sentences behavior and then learn new ways of coping with
D. Ask the client to play with other clients the anxiety.

Answer: C . Appropriate nursing interventions for 10. Nurse Tony was caring for a 41 year old female
an anxiety attack include using short sentences, client. Which behavior by the client indicates adult
staying with the client, decreasing stimuli, cognitive development?
remaining calm and medicating as needed.
A. Generates new levels of awareness
6. A female client is admitted with a diagnosis of B. Assumes responsibility for her actions
delusions of GRANDEUR. This diagnosis reflects a C. Has maximum ability to solve problems and
belief that one is: learn new skills
D. Her perception are based on reality
A. Being Killed
B. Highly famous and important Answer: A . An adult age 31 to 45 generates new
C. Responsible for evil world level of awareness.
D. Connected to client unrelated to oneself
11.A neuromuscular blocking agent is administered
Answer: B . Delusion of grandeur is a false belief to a client before ECT therapy. The Nurse should
that one is highly famous and important. carefully observe the client for?

7. A 20 year old client was diagnosed with A. Respiratory difficulties


dependent personality disorder. Which behavior is B. Nausea and vomiting
not likely to be evidence of ineffective individual C. Dizziness
coping? D. Seizures

A. Recurrent self-destructive behavior Answer: A . Neuromuscular Blocker, such as


B. Avoiding relationship SUCCINYLCHOLINE (Anectine) produces
C. Showing interest in solitary activities respiratory depression because it inhibits
D. Inability to make choices and decision contractions of respiratory muscles.
without advise
12.A 75 year old client is admitted to the hospital
Answer: D . Individual with dependent personality with the diagnosis of dementia of the Alzheimer’s
disorder typically shows indecisiveness type and depression. The symptom that is unrelated
submissiveness and clinging behavior so that to depression would be?
others will make decisions with them.
A. Apathetic response to the environment
8. A male client is diagnosed with schizotypal B. “I don’t know” answer to questions
personality disorder. Which signs would this client C. Shallow of labile effect
exhibit during social situation? D. Neglect of personal hygiene

A. Paranoid thoughts Answer: C . With depression, there is little or no


B. Emotional affect emotional involvement therefore little alteration in
C. Independence need affect.
D. Aggressive behavior
13.Nurse Trish is working in a mental health facility;
Answer: A . Clients with schizotypal personality the nurse priority nursing intervention for a newly
disorder experience excessive social anxiety that admitted client with bulimia nervosa would be to?
can lead to paranoid thoughts
A. Teach client to measure I & O Answer: D . The nurse needs to set limits in the
B. Involve client in planning daily meal client’s manipulative behavior to help the client
C. Observe client during meals control dysfunctional behavior. A consistent
D. Monitor client continuously approach by the staff is necessary to decrease
manipulation.
Answer: D . These clients often hide food or force
vomiting; therefore they must be carefully 18.Conney with borderline personality disorder who
monitored. is to be discharge soon threatens to “do something”
to herself if discharged. Which of the
14.Nurse Patricia is aware that the major health following actions by the nurse would be most
complication associated with intractable anorexia important?
nervosa would be?
A. Ask a family member to stay with the client at
A. Cardiac dysrhythmias resulting to cardiac home temporarily
arrest B. Discuss the meaning of the client’s statement
B. Glucose intolerance resulting in protracted with her
hypoglycemia C. Request an immediate extension for the client
C. Endocrine imbalance causing cold amenorrhea D. Ignore the clients statement because it’s a sign
D. Decreased metabolism causing cold intolerance of manipulation

Answer: A . These clients have severely depleted Answer: B . Any suicidal statement must be
levels of sodium and potassium because of their assessed by the nurse. The nurse should discuss the
starvation diet and energy expenditure, these client’s statement with her to determine its meaning
electrolytes are necessary for cardiac functioning. in terms of suicide

15.Nurse Anna can minimize agitation in a 19.Joey a client with antisocial personality disorder
disturbed client by? belches loudly. A staff member asks Joey, “Do you
know why people find you repulsive?”
A. Increasing stimulation this statement most likely would elicit which of the
B. limiting unnecessary interaction following client reaction?
C. increasing appropriate sensory perception
D. ensuring constant client and staff contact A. Depensiveness
B. Embarrassment
Anaswer: B . Limiting unnecessary interaction will C. Shame
decrease stimulation and agitation. D. Remorsefulness

16.A 39 year old mother with obsessive-compulsive A . When the staff member ask the client if he
disorder has become immobilized by her elaborate wonders why others find him repulsive, the client
hand washing and walking rituals. Nurse Trish is likely to feel defensive because the question
recognizes that the basis of O.C. disorder is often: is belittling. The natural tendency is to
counterattack the threat to self image.
A. Problems with being too conscientious
B. Problems with anger and remorse 20.Which of the following approaches would be
C. Feelings of guilt and inadequacy most appropriate to use with a client suffering from
D. Feeling of unworthiness and hopelessness narcissistic personality disorder when
discrepancies exist between what the client states
Answer: C . Ritualistic behavior seen in this and what actually exist?
disorder is aimed at controlling guilt
and inadequacy by maintaining an absolute set A. Rationalization
pattern of behavior. B. Supportive confrontation
C. Limit setting
17.Mario is complaining to other clients about not D. Consistency
being allowed by staff to keep food in his room.
Which of the following interventions would be Answer: B . The nurse would specifically use
most appropriate? supportive confrontation with the client to point
out discrepancies between what the client states
A. Allowing a snack to be kept in his room and what actually exists to increase responsibility
B. Reprimanding the client for self.
C. Ignoring the clients behavior
D. Setting limits on the behavior 21.Cely is experiencing alcohol withdrawal exhibits
tremors, diaphoresis and hyperactivity. Blood
pressure is 190/87 mmhg and pulse is 92 bpm.
Which of the medications would the nurse expect to A. Manipulate the environment to bring about
administer? positive changes in behavior
B. Allow the client’s freedom to determine
A. Naloxone (Narcan) whether or not they will be involved in
B. Benzlropine (Cogentin) activities
C. Lorazepam (Ativan) C. Role play life events to meet individual needs
D. Haloperidol (Haldol) D. Use natural remedies rather than drugs to
control behavior
Answer: C . The nurse would most likely
administer benzodiazepine, such as Answer: A . Environmental (MILIEU) therapy
lorazepan (ativan) to the client who is experiencing aims at having everything in the
symptom: The client’s experiences symptoms of client’s surrounding area toward helping the client.
withdrawal because of the rebound phenomenon
when the sedation of the CNS from alcohol begins 26.Nurse Trish would expect a child with a
to decrease. diagnosis of reactive attachment disorder to:

22.Which of the following foods would the nurse A. Have more positive relation with the father
Trish eliminate from the diet of a client in alcohol than the mother
withdrawal? B. Cling to mother & cry on separation
C. Be able to develop only superficial relation
A. Milk with the others
B. Orange Juice D. Have been physically abuse
C. Soda
D. Regular Coffee Answer: C . Children who have experienced
attachment difficulties with primary caregiver are
Answer: D . Regular coffee contains caffeine not able to trust others and therefore relate
which acts as psychomotor stimulants and leads to superficially
feelings of anxiety and agitation. Serving coffee
top the client may add to tremors or wakefulness. 27.When teaching parents about childhood
depression Nurse Trina should say?
23.Which of the following would Nurse Hazel
expect to assess for a client who is exhibiting late A. It may appear acting out behavior
signs of heroin withdrawal? B. Does not respond to conventional treatment
C. Is short in duration & resolves easily
A. Yawning & diaphoresis D. Looks almost identical to adult depression
B. Restlessness & Irritability
C. Constipation & steatorrhea Answer: A . Children have difficulty verbally
D. Vomiting and Diarrhea expressing their feelings, acting out behavior, such
as temper tantrums, may indicate underlying
Answer: D . Vomiting and diarrhea are usually the depression.
late signs of heroin withdrawal, along with muscle
spasm, fever, nausea, repetitive, abdominal cramps 28.Nurse Perry is aware that language development
and backache. in autistic child resembles:

24.To establish open and trusting relationship with a A. Scanning speech


female client who has been hospitalized with severe B. Speech lag
anxiety, the nurse in charge should? C. Shuttering
D. Echolalia
A. Encourage the staff to have frequent interaction Answer: D . The autistic child repeat sounds or
with the client words spoken by others.
B. Share an activity with the client
C. Give client feedback about behavior 29.A 60 year old female client who lives alone tells
D. Respect client’s need for personal space the nurse at the community health center “I really
don’t need anyone to talk to”. The TV is my best
Answer: D . Moving to a client’s personal space friend. The nurse recognizes that the client is using
increases the feeling of threat, which increases the defense mechanism known as?
anxiety.
A. Displacement
25. Nurse Monette recognizes that the focus of B. Projection
environmental (MILIEU) therapy is to: C. Sublimation
D. Denial
Answer: D . The client statement is an example of
the use of denial, a defense that blocks problem by 34.Nurse Joey is aware that the signs & symptoms
unconscious refusing to admit they exist that would be most specific for diagnosis anorexia
are?
30.When working with a male client suffering
phobia about black cats, Nurse Trish should A. Excessive weight loss, amenorrhea &
anticipate that a problem for this client would be? abdominal distension
B. Slow pulse, 10% weight loss & alopecia
A. Anxiety when discussing phobia C. Compulsive behavior, excessive fears &
B. Anger toward the feared object nausea
C. Denying that the phobia exist D. Excessive activity, memory lapses & an
D. Distortion of reality when completing daily increased pulse
routines
Answer: A . These are the major signs of anorexia
Answer: A . Discussion of the feared object nervosa. Weight loss is excessive (15% of
triggers an emotional response to the object. expected weight)

31.Linda is pacing the floor and appears extremely 35.A characteristic that would suggest to Nurse
anxious. The duty nurse approaches in an attempt to Anne that an adolescent may have bulimia would be:
alleviate Linda’s anxiety. The most therapeutic
question by the nurse would be? A. Frequent regurgitation & re-swallowing of
food
A. Would you like to watch TV? B. Previous history of gastritis
B. Would you like me to talk with you? C. Badly stained teeth
C. Are you feeling upset now? D. Positive body image
D. Ignore the client
Answer: C . Dental enamel erosion occurs from
Answer: B . The nurse presence may provide the repeated self-induced vomiting.
client with support & feeling of control.
36.Nurse Monette is aware that extremely depressed
32.Nurse Penny is aware that the symptoms that clients seem to do best in settings where they have:
distinguish post traumatic stress disorder from other
anxiety disorder would be: A. Multiple stimuli
B. Routine Activities
A. Avoidance of situation & certain activities that C. Minimal decision making
resemble the stress D. Varied Activities
B. Depression and a blunted affect when
discussing the traumatic situation Answer: B . Depression usually is both emotional
C. Lack of interest in family & others & physical. A simple daily routine is the best, least
D. Re-experiencing the trauma in dreams or stressful and least anxiety producing.
flashback
37.To further assess a client’s suicidal potential.
Answer: D . Experiencing the actual trauma in Nurse Katrina should be especially alert to the client
dreams or flashback is the major symptom that expression of:
distinguishes post traumatic stress disorder from
other anxiety disorder. A. Frustration & fear of death
B. Anger & resentment
33.Nurse Benjie is communicating with a male C. Anxiety & loneliness
client with substance-induced persisting dementia; D. Helplessness & hopelessness
the client cannot remember facts and fills in the
gaps with imaginary information. Nurse Benjie is Answer: D . The expression of these feeling may
aware that this is typical of? indicate that this client is unable to continue the
struggle of life.
A. Flight of ideas
B. Associative looseness 38.A nursing care plan for a male client with bipolar
C. Confabulation I disorder should include:
D. Concretism
A. Providing a structured environment
Answer: C . Confabulation or the filling in of B. Designing activities that will require the client
memory gaps with imaginary facts is a defense to maintain contact with reality
mechanism used by people experiencing memory C. Engaging the client in conversing about current
deficits. affairs
D. Touching the client provide assurance Answer: C . Helping the client to develop feeling
of self worth would reduce the client’s need to use
Answer: A . Structure tends to decrease agitation pathologic defenses.
and anxiety and to increase the client’s feeling of
security. 43.A male client who is experiencing disordered
thinking about food being poisoned is admitted to
39.When planning care for a female client using the mental health unit. The nurse uses
ritualistic behavior, Nurse Gina must recognize that which communication technique to encourage the
the ritual: client to eat dinner?

A. Helps the client focus on the inability to deal A. Focusing on self-disclosure of own food
with reality preference
B. Helps the client control the anxiety B. Using open ended question and silence
C. Is under the client’s conscious control C. Offering opinion about the need to eat
D. Is used by the client primarily for secondary D. Verbalizing reasons that the client may not
gains choose to eat

Answer: B . The rituals used by a client with Answer: B . Open ended questions and silence are
obsessive compulsive disorder help control the strategies used to encourage clients to discuss their
anxiety level by maintaining a set pattern of action. problem in descriptive manner.

40.A 32 year old male graduate student, who has 44.Nurse Nina is assigned to care for a client
become increasingly withdrawn and neglectful of diagnosed with Catatonic Stupor. When Nurse Nina
his work and personal hygiene, is brought to the enters the client’s room, the client is found lying on
psychiatric hospital by his parents. After detailed the bed with a body pulled into a fetal position.
assessment, a diagnosis of schizophrenia is made. It Nurse Nina should?
is unlikely that the client will demonstrate:
A. Ask the client direct questions to encourage
A. Low self esteem talking
B. Concrete thinking B. Rake the client into the dayroom to be with
C. Effective self boundaries other clients
D. Weak ego C. Sit beside the client in silence and
occasionally ask open-ended question
Answer:C . A person with this disorder would not D. Leave the client alone and continue with
have adequate self-boundaries providing care to the other clients

41.A 23 year old client has been admitted with a Answer: C . Clients who are withdrawn may be
diagnosis of schizophrenia says to the nurse “Yes, immobile and mute, and require consistent,
its march, March is little woman”. That’s literal repeated interventions. Communication with
you know”. These statement illustrate: withdrawn clients requires much patience from the
nurse. The nurse facilitates communication with
A. Neologisms the client by sitting in silence, asking open-ended
B. Echolalia question and pausing to provide opportunities for
C. Flight of ideas the client to respond.
D. Loosening of association
45.Nurse Tina is caring for a client with delirium
Answer: D . Loose associations are thoughts that and states that “look at the spiders on the wall”.
are presented without the logical connections What should the nurse respond to the client?
usually necessary for the listening to interpret the
message. A. “You’re having hallucination, there are no
spiders in this room at all”
42.A long term goal for a paranoid male client who B. “I can see the spiders on the wall, but they are
has unjustifiably accused his wife of having many not going to hurt you”
extramarital affairs would be to help the C. “Would you like me to kill the spiders”
client develop: D. “I know you are frightened, but I do not see
spiders on the wall”
A. Insight into his behavior
B. Better self control Answer: D . When hallucination is present, the
C. Feeling of self worth nurse should reinforce reality with the client.
D. Faith in his wife
46.Nurse Jonel is providing information to a
community group about violence in the family.
Which statement by a group member would indicate Answer: D . Electroconvulsive therapy is an
a need to provide additional information? effective treatment for depression that has not
responded to medication
A. “Abuse occurs more in low-income families”
B. “Abuser Are often jealous or self-centered” 50.Mario is admitted to the emergency room with
C. “Abuser use fear and intimidation” drug-included anxiety related to over ingestion of
D. “Abuser usually have poor self-esteem” prescribed antipsychotic medication. The
most important piece of information the nurse in
Answer: A . Personal characteristics of abuser charge should obtain initially is the:
include low self-esteem, immaturity, dependence,
insecurity and jealousy A. Length of time on the med.
B. Name of the ingested medication & the
47.During electroconvulsive therapy (ECT) the amount ingested
client receives oxygen by mask via positive C. Reason for the suicide attempt
pressure ventilation. The nurse assisting with D. Name of the nearest relative & their phone
this procedure knows that positive pressure number
ventilation is necessary because?
Answer: B . In an emergency, lives saving facts are
A. Anesthesia is administered during the obtained first. The name and the amount of
procedure medication ingested are of outmost important in
B. Decrease oxygen to the brain increases treating this potentially life threatening situation.
confusion and disorientation
C. Grand mal seizure activity depresses
respirations
D. Muscle relaxations given to prevent injury FOUNDATION OF NURSING
during seizure activity depress respirations.

Answer: D . A short acting skeletal muscle


relaxant such as succinylcholine (Anectine) is
PNLE: FON Practice
administered during this procedure to prevent
injuries during seizure. Exam for History ,
48.When planning the discharge of a client with
chronic anxiety, Nurse Chris evaluates achievement
of the discharge maintenance goals. Which
Concepts and
goal would be most appropriately having been
included in the plan of care requiring evaluation? Theories
1. The four major concepts in nursing theory
A. The client eliminates all anxiety from daily
situations
are the
B. The client ignores feelings of anxiety
C. The client identifies anxiety producing A. Person, Environment, Nurse, Health
situations B. Nurse, Person, Environment, Cure
D. The client maintains contact with a crisis C. Promotive, Preventive, Curative,
counselor Rehabilitative
D. Person, Environment, Nursing, Health
Answer: C . Recognizing situations that produce
anxiety allows the client to prepare to cope with
Answer: D. Person, Environment, Nursing,
anxiety or avoid specific stimulus.
Health. This is an actual board exam question
49.Nurse Tina is caring for a client with depression and is a common board question. Theorist
who has not responded to antidepressant medication. always describes The nursing profession by
The nurse anticipates that what treatment procedure first defining what is NURSING, followed by
may be prescribed. the PERSON, ENVIRONMENT and
HEALTH CONCEPT. The most popular
A. Neuroleptic medication theory was perhaps Nightingale’s. She
B. Short term seclusion defined nursing as the utilization of the
C. Psychosurgery
D. Electroconvulsive therapy
persons environment to assist him towards
recovery. She defined the person as
somebody who has a reparative capabilities
mediated and enhanced by factors in his
environment. She describes the environment
as something that would facilitate the Answer: B. Orem. In self care deficit theory,
person’s reparative process and identified Nursing is defined as A helping or assistive
different factors like sanitation, noise, etc. profession to person who are wholly or partly
that affects a person’s reparative state. dependent or when people who are to give
care to them are no longer available. Self
2. The act of utilizing the environment of the care, are the activities that a person do for
patient to assist him in his recovery is himself to maintain health, life and well
theorized by being.

A. Nightingale 5. Nursing is a unique profession, Concerned


B. Benner with all the variables affecting an individual’s
C. Swanson response to stressors, which are intra, inter
D. King and extra personal in nature.

Answer: A. Nightingale. Florence A. Neuman


nightingale do not believe in the germ theory, B. Johnson
and perhaps this was her biggest mistake. Yet, C. Watson
her theory was the first in nursing. She D. Parse
believed that manipulation of environment
that includes appropriate noise, nutrition, Answer: A. Neuman. Neuman divided
hygiene, light, comfort, sanitation etc. could stressors as either intra, inter and extra
provide the client’s body the nurturance it personal in nature. She said that NURSING
needs for repair and recovery. is concerned with eliminating these stressors
to obtain a maximum level of wellness. The
3. For her, Nursing is a theoretical system of nurse helps the client through PRIMARY,
knowledge that prescribes a process of SECONDARY AND TERTIARY prevention
analysis and action related to care of the ill modes. Please do not confuse this with
person LEAVELL and CLARK’S level of
prevention.
A. King
B. Henderson 6. The unique function of the nurse is to assist
C. Roy the individual, sick or well, in the
D. Leininger performance of those activities contributing to
health that he would perform unaided if he has
Answer: C. Roy. Remember the word the necessary strength, will and knowledge,
“ THEOROYTICAL “ For Callista Roy, and do this in such a way as to help him gain
Nursing is a theoretical body of knowledge independence as rapidly as possible.
that prescribes analysis and action to care for
an ill person. She introduced the A. Henderson
ADAPTATION MODEL and viewed person B. Abdellah
as a BIOSPSYCHOSOCIAL BEING. She C. Levin
believed that by adaptation, Man can D. Peplau
maintain homeostasis.
Answer: A. Henderson. This was an actual
4. According to her, Nursing is a helping or board question. Remember this definition
assistive profession to persons who are wholly and associate it with Virginia Henderson.
or partly dependent or when those who are Henderson also describes the NATURE OF
supposedly caring for them are no longer able NURSING theory. She identified 14 basic
to give care. needs of the client. She describes nursing
roles as SUBSTITUTIVE : Doing everything
A. Henderson for the client, SUPPLEMENTARY : Helping
B. Orem the client and COMPLEMENTARY :
C. Swanson Working with the client. Breathing normally,
D. Neuman Eliminating waste, Eating and drinking
adquately, Worship and Play are some of the D. Swanson
basic needs according to her.
Answer: B. Watson. The deepest and
7. Caring is the essence and central unifying, a spiritual definition of Caring came from Jean
dominant domain that distinguishes nursing watson. For her, Caring expands the limits of
from other health disciplines. Care is an openess and allows access to higher human
essential human need. spirit.

A. Benner 10. Caring means that person, events, projects


B. Watson and things matter to people. It reveals stress
C. Leininger and coping options. Caring creates
D. Swanson responsibility. It is an inherent feature of
nursing practice. It helps the nurse assist
Answer:C. Leininger. There are many clients to recover in the face of the illness.
theorist that describes nursing as CARE. The
most popular was JEAN WATSON’S A. Benner
Human Caring Model. But this question B. Watson
pertains to Leininger’s definition of caring. C. Leininger
CUD I LIE IN GER? [ Could I Lie In There ] D. Swanson
Is the Mnemonics I am using not to get
confused. C stands for CENTRAL , U stands Answer: A. Benner. I think of CARE BEAR
for UNIFYING, D stands for DOMINANT to facilitate retainment of BENNER. As in,
DOMAIN. I emphasize on this matter due to Care Benner. For her, Caring means being
feedback on the last June 2006 batch about a CONNECTED or making things matter to
question about CARING. people. Caring according to Benner give
meaning to illness and re establish
8. Caring involves 5 processes, KNOWING, connection.
BEING WITH, DOING FOR, ENABLING
and MAINTAINING BELIEF. 11. Which of the following is NOT TRUE
about profession according to Marie Jahoda?
A. Benner
B. Watson A. A profession is an organization of an
C. Leininger occupational group based on the
D. Swanson application of special knowledge
B. It serves specific interest of a group
Answer:D. Swanson . Caring according to C. It is altruistic
Swanson involves 5 processes. Knowing D. Quality of work is of greater importance
means understanding the client. Being with than the rewards
emphasizes the Physical presence of the
nurse for the patient. Doing for means doing Answer: B. It serves specific interest of a
things for the patient when he is incapable of group.Believe it or not, you should know the
doing it for himself. Enabling means helping definition of profession according to Jahoda
client transcend maturational and because it is asked in the Local boards. A
developmental stressors in life while profession should serve the WHOLE
Maintaining belief is the ability of the Nurse COMMUNITY and not just a specific intrest
to inculcate meaning to these events. of a group. Everything else, are correct.

9. Caring is healing, it is communicated 12. Which of the following is NOT an


through the consciousness of the nurse to the attribute of a professional?
individual being cared for. It allows access to
higher human spirit. A. Concerned with quantity
B. Self directed
A. Benner C. Committed to spirit of inquiry
B. Watson D. Independent
C. Leininger
Answer: A. Concerned with quantity. A 16. As a nurse manager, which of the
professional is concerned with QUALITY following best describes this function?
and not QUANTITY. In nursing, We have
methods of quality assurance and control to A. Initiate modification on client’s lifestyle
evaluate the effectiveness of nursing care. B. Protect client’s right
Nurses, are never concerned with C. Coordinates the activities of other
QUANTITY of care provided. members of the health team in managing
patient care
13. The most unique characteristic of nursing D. Provide in service education programs,
as a profession is Use accurate nursing audit, formulate
philosophy and vision of the institution
A. Education
B. Theory Answer:Provide in service education
C. Caring programs, Use accurate nursing audit,
D. Autonomy formulate philosophy and vision of the
institution
Answer: C. Caring. Caring and caring alone,
is the most unique quality of the Nursing 17. What best describes nurses as a care
Profession. It is the one the delineate Nursing provider?
from other professions.
A. Determine client’s need
14. This is the distinctive individual qualities B. Provide direct nursing care
that differentiate a person to another C. Help client recognize and cope with
stressful psychological situation
A. Philosophy D. Works in combined effort with all those
B. Personality involved in patient’s care
C. Charm
D. Character Answer: A. Determine client’s need.You can
never provide nursing care if you don’t know
Answer: B. Personality. Personality are what are the needs of the client. How can you
qualities that make us different from each provide an effective postural drainage if you
other. These are impressions that we made, do not know where is the bulk of the client’s
or the footprints that we leave behind. This is secretion. Therefore, the best description of a
the result of the integration of one’s talents, care provider is the accurate and prompt
behavior, appearance, mood, character, determination of the client’s need to be able
morals and impulses into one harmonious to render an appropriate nursing care.
whole. Philosophy is the basic truth that fuel
our soul and give our life a purpose, it shapes 18. The nurse questions a doctors order of
the facets of a person’s character. Charm is to Morphine sulfate 50 mg, IM for a client with
attract other people to be a change agent. pancreatitis. Which role best fit that statement?
Character is our moral values and belief that
guides our actions in life. A. Change agent
B. Client advocate
15. Refers to the moral values and beliefs that C. Case manager
are used as guides to personal behavior and D. Collaborator
actions
Answer: B. Client advocate. As a client’s
A. Philosophy advocate, Nurses are to protect the client’s
B. Personality right and promotes what is best for the client.
C. Charm Knowing that Morphine causes spasm of the
D. Character sphincter of Oddi and will lead to further
increase in the client’s pain, The nurse knew
Answer:D. Character.Rationale: Refer to that the best treatment option for the client
number 14 was not provided and intervene to provide
the best possible care.
19. These are nursing intervention that 22. This patient care model works best when
requires knowledge, skills and expertise of there are plenty of patient but few nurses
multiple health professionals.
A. Functional nursing
A. Dependent B. Team nursing
B. Independent C. Primary nursing
C. Interdependent D. Total patient care
D. Intradependent
ANSWER:A. Functional nursing. Functional
Answer: C. Interdependent. Interdependent nursing is task oriented, One nurse is
functions are those that needs expertise and assigned on a particular task leading to task
skills of multiple health professionals. expertise and efficiency. The nurse will work
Example is when A child was diagnosed with fast because the procedures are repetitive
nephrotic syndrome and the doctor ordered a leading to task mastery. This care is not
high protein diet, Budek then work together recommended as this leads fragmented
with the dietician about the age appropriate nursing care.
high protein foods that can be given to the
child, Including the preparation to entice the 23. RN assumes 24 hour responsibility for the
child into eating the food. NOTE : It is still client to maintain continuity of care across
debated if the diet in NS is low, moderate or shifts, days or visits.
high protein, In the U.S, Protein is never
restricted and can be taken in moderate A. Functional nursing
amount. As far as the local examination is B. Team nursing
concerned, answer LOW PROTEIN HIGH C. Primary nursing
CALORIC DIET. D. Total patient care

20. What type of patient care model is the Answer: C. Primary nursing. Your keyword
most common for student nurses and private in Primary nursing is the 24 hours. This does
duty nurses? not necessarily means the nurse is awake for
24 hours, She can have a SECONDARY
A. Total patient care NURSES that will take care of the patient in
B. Team nursing shifts where she is not arround.
C. Primary Nursing
D. Case management
24. Who developed the first theory of nursing?
Answer: A. Total patient care. This is also
known as case nursing. It is a method of A. Hammurabi
nursing care wherein, one nurse is assigned B. Alexander
to one patient for the delivery of total care. C. Fabiola
These are the method use by Nursing D. Nightingale
students, Private duty nurses and those in
critical or isolation units. Answer: D. Nightingale . Refer to question #
2. Hammurabi is the king of babylon that
21. This is the best patient care model when introduces the LEX TALIONES law, If you
there are many nurses but few patients. kill me, you should be killed… If you rob me,
You should be robbed, An eye for an eye and
A. Functional nursing a tooth for a tooth. Alexander the great was
B. Team nursing the son of King Philip II and is from
C. Primary nursing macedonia but he ruled Greece including
D. Total patient care Persia and Egypt. He is known to use a
hammer to pierce a dying soldier’s medulla
Answer: D. Total patient care .Total patient towards speedy death when he thinks that the
care works best if there are many nurses but soldier will die anyway, just to relieve their
few patients. suffering. Fabiola was a beautiful roman
matron who converted her house into a The nurse can help the client maintain
hospital. stability against these stressors.

25. She introduces the NATURE OF 28. Conceptualized the BEHAVIORAL


NURSING MODEL. SYSTEM MODEL

A. Henderson A. Orem
B. Nightingale B. Johnson
C. Parse C. Henderson
D. Orlando D. Parse

Answer: A. Henderson. Refer to question # 6. Answer: B. Johnson. According to Dorothy


Johnson, Each person is a behavioral system
26. She described the four conservation that is composed of 7 subsystems. Man adjust
principle. or adapt to stressors by a using a LEARNED
PATTERN OF RESPONSE. Man uses his
A. Levin behavior to meet the demands of the
B. Leininger environment, and is able to modified his
C. Orlando behavior to support these demands
D. Parse
29. Developed the CLINICAL NURSING – A
Answer: A. Levin. Myra Levin described the HELPING ART MODEL
4 Conservation principles which are
concerned with the Unity and Integrity of an A. Swanson
individual. These are ENERGY : Our output B. Hall
to facilitate meeting of our needs. C. Weidenbach
STRUCTURAL INTEGRITY : We mus D. Zderad
maintain the integrity of our organs, tissues
and systems to be able to function and prevent Asnwer: C. Weidenbach.Just remember
harmful agents entering our body. ERNESTINE WEIDENBACHLINICAL.
PERSONAL INTEGRITY : These refers to
our self esteem, self worth, self concept, 30. Developed the ROLE MODELING and
identify and personality. SOCIAL MODELING theory
INTEGRITY : Reflects our societal roles to
our society, community, family, friends and A. Erickson,Tomlin,Swain
fellow individuals B. Neuman
C. Newman
27. Proposed the HEALTH CARE SYSTEM D. Benner and Wrubel
MODEL.
Answer: A. Erickson,Tomlin,Swain
A. Henderson
B. Orem 31. Proposed the GRAND THEORY OF
C. Parse NURSING AS CARING
D. Neuman
A. Erickson, Tomlin, Swain
Answer: D. Neuman . Betty Neuman asserted B. Peterson,Zderad
that nursing is a unique profession and is C. Bnner,Wrubel
concerned with all the variables affecting the D. Boykin,Schoenhofer
individual’s response to stressors. These are
INTRA or within ourselves, EXTRA or Answer: D. Boykin,Schoenhofer . This
outside the individual, INTER means theory was called GRAND THEORY
between two or more people. She proposed because boykin and schoenofer thinks that
the HEALTH CARE SYSTEM MODEL ALL MAN ARE CARING, And that nursing
which states that by PRIMARY, is a response to this unique call. According to
SECONDARY and TERTIARY prevention, them, CARING IS A MORAL
IMPERATIVE, meaning, ALL PEOPLE will
tend to help a man who fell down the stairs individual still follows the rules but can make
even if he is not trained to do so. a rule or bend part of these rules according to
his own MORALITY. He can change the
32. Postulated the INTERPERSONAL rules if he thinks that it is needed to be
ASPECT OF NURSING changed. Example is that, A nurse still
continue resuscitating the client even if the
A. Travelbee client has a DNR order because he believes
B. Swanson that the client can still recover and his mission
C. Zderad is to save lives, not watch patients die
D. Peplau 34. He proposed the theory of morality based
on PRINCIPLES
Answer: A. Travelbee. Travelbee’s theory
was referred to as INTERPERSONAL theory A. Freud
because she postulated that NURSING is to B. Erikson
assist the individual and all people that C. Kohlberg
affects this individual to cope with illness, D. Peters
recover and FIND MEANING to this
experience. For her, Nursing is a HUMAN Answer: D. Peters . Remember PETERS for
TO HUMAN relationship that is formed PRINCIPLES. P is to P. He believes that
during illness. To her, an individual is a morality has 3 components : EMOTION or
UNIQUE and irreplaceable being in how one feels, JUDGEMENT or how one
continuous process of becoming, evolving reason and BEHAVIOR or how one actuates
and changing. PLEASE do remember, that it his EMOTION and JUDGEMENT. He
is PARSE who postulated the theory of believes that MORALITY evolves with the
HUMAN BECOMING and not development of PRINCPLES or the person’s
TRAVELBEE, for I read books that say it vitrue and traits. He also believes in
was TRAVELBEE and not PARSE. AUTOMATICITY of virtues or he calls
HABIT, like kindness, charity, honesty,
33. He proposed the theory of morality that is sincerity and thirft which are innate to a
based on MUTUAL TRUST person and therfore, will be performed
automatically.
A. Freud
B. Erikson 35. Freud postulated that child adopts parental
C. Kohlberg standards and traits through
D. Peters
A. Imitation
Answer: C. Kohlberg. Kohlber states that B. Introjection
relationships are based on mutual trust. He C. Identification
postulated the levels of morality development. D. Regression
At the first stage called the PREMORAL or
preconventional, A child do things and label Answer:C. Identification. A child, according
them as BAD or GOOD depending on the to Freud adopts parental standards, traits,
PUNISHMENT or REWARD they get. They habits and norms through identication. A
have no concept of justice, fairness and equity, good example is the corned beef commercial ”
for them, If I punch this kid and mom gets WALK LIKE A MAN, TALK LIKE A
mad, thats WRONG. But if I dance and sing, MAN ” Where the child identifies with his
mama smiles and give me a new toy, then I father by wearing the same clothes and doing
am doing something good. In the the same thing.
Conventional level, The individual actuates
his act based on the response of the people 36. According to them, Morality is measured
around him. He will follow the rules, of how people treat human being and that a
regulations, laws and morality the society moral child strives to be kind and just
upholds. If the law states that I should not
resuscitate this man with a DNR order, then I A. Zderad and Peterson
would not. However, in the Post conventional B. Benner and Wrubel
level or the AUTONOMOUS level, the C. Fowler and Westerhoff
D. Schulman and Mekler
40. Anastacia Giron-Tupas was the first
Answer: D. Schulman and Filipino nurse to occupy the position of chief
Mekler . According to Schulman and Mekler, nurse in this hospital.
there are 2 components that makes an action
MORAL : The intention should be good and A. St. Paul Hospital
the Act must be just. A good example is B. Iloilo Mission Hospital
ROBIN HOOD, His intention is GOOD but C. Philippine General Hospital
the act is UNJUST, which makes his action D. St. Luke’s Hospital
IMMORAL.
Answer:C. Philippine General Hospital
37. Postulated that FAITH is the way of
behaving. He developed four theories of faith 41. She was the daughter of Hungarian kings,
and development based on his experience. who feed 300-900 people everyday in their
gate, builds hospitals, and care of the poor and
A. Giligan sick herself.
B. Westerhoff
C. Fowler A. Elizabeth
D. Freud B. Catherine
C. Nightingale
Answer: B. Westerhoff. There are only 2 D. Sairey Gamp
theorist of FAITH that might be asked in the
board examinations. Fowler and Westerhoff. Answer:A. Elizabeth.Saint Elizabeth of
What differs them is that, FAITH of fowler is Hungary was a daughter of a King and is the
defined abstractly, Fowler defines faith as a patron saint of nurses. She build hospitals
FORCE that gives a meaning to a person’s and feed hungry people everyday using the
life while Westerhoff defines faith as a kingdom’s money. She is a princess, but
behavior that continuously develops through devoted her life in feeding the hungry and
time. serving the sick.

38. He described the development of faith. He 42. She dies of yellow fever in her search for
suggested that faith is a spiritual dimension truth to prove that yellow fever is carried by a
that gives meaning to a persons life. Faith mosquitoes.
according to him, is a relational phenomenon. A. Clara louise Maas
A. Giligan B. Pearl Tucker
B. Westerhoff C. Isabel Hampton Robb
C. Fowler D. Caroline Hampton Robb
D. Freud
Answer:A. Clara louise Maas. Clara Louise
Answer:C. Fowler. Rationale: Refer to # 37 Maas sacrificed her life in research of
YELLOW FEVER. People during her time
39. Established in 1906 by the Baptist foreign do not believe that yellow fever was brought
mission society of America. Miss rose nicolet, by mosquitoes. To prove that they are wrong,
was it’s first superintendent. She allowed herself to be bitten by the vector
and after days, She died.
A. St. Paul Hospital School of nursing
B. Iloilo Mission Hospital School of 43. He was called the father of sanitation.
nursing
C. Philippine General Hospital School of A. Abraham
nursing B. Hippocrates
D. St. Luke’s Hospital School of nursing C. Moses
D. Willam Halstead
Answer:B. Iloilo Mission Hospital School of
nursing Answer: C. Moses
44. The country where SHUSHURUTU wounded soldiers. But almost all of them are
originated influenced by the christian faith to serve and
nurse the sick. When Fliedner build the first
A. China formal school for nurses, It marked the end
B. Egypt of the APPRENTICESHIP period.
C. India
D. Babylonia 48. Period of nursing where religious
Christian orders emerged to take care of the
Answer: india
sick
45. They put girls clothes on male infants to A. Apprentice period
drive evil forces away B. Dark period
C. Contemporary period
A. Chinese D. Educative period
B. Egyptian
C. Indian Answer: A. Apprentice period. Apprentice
D. Babylonian period is marked by the emergence of
religious orders the are devoted to religious
Answer: A. Chinese. Chinese believes that life and the practice of nursing.
male newborns are demon magnets. To fool
those demons, they put female clothes to 49. Founded the second order of St. Francis of
their male newborn.
Assisi
46. In what period of nursing does people A. St. Catherine
believe in TREPHINING to drive evil forces B. St. Anne
away? C. St. Clare
D. St. Elizabeth
A. Dark period
B. Intuitive period Answer:C. St. Clare. The poor clares, is the
C. Contemporary period second order of St. Francis of assisi. The first
D. Educative period order was founded by St. Francis himself. St.
Catherine of Siena was the first lady with the
Answer: B. Intuitive period.Egyptians lamp. St. Anne is the mother of mama mary.
believe that a sick person is someone with an St. Elizabeth is the patron saint of Nursing.
evil force or demon that is inside their heads.
To release these evil spirits, They would tend 50. This period marked the religious upheaval
to drill holes on the patient’s skull and it is of Luther, Who questions the Christian faith.
called TREPHINING.
A. Apprentice period
47. This period ended when Pastor Fliedner, B. Dark period
build Kaiserwerth institute for the training of C. Contemporary period
Deaconesses D. Educative period

A. Apprentice period Answer:B. Dark period. Protestantism


B. Dark period emerged with Martin Luther questions the
C. Contemporary period Pope and Christianity. This started the Dark
D. Educative period period of nursing when the christian faith
was smeared by controversies. These leads to
Answer: A. Apprentice period.What closure of some hospital and schools run by
dilineates apprentice period among others is the church. Nursing became the work of
that, it ENDED when formal schools were prostitutes, slaves, mother and least desirable
established. During the apprentice period, of women.
There is no formal educational institution for
nurses. Most of them receive training inside 51. According to the Biopsychosocial and
the convent or church. Some of them are spiritual theory of Sister Callista Roy, Man,
trained just for the purpose of nursing the As a SOCIAL being is
A. Like all other men Answer: B. Rogers. The wholistic theory by
B. Like some other men Martha Rogers states that MAN is greater
C. Like no other men than the sum of all its parts and that his
D. Like men dignity and worth will not be lessen even if
one of this part is missing. A good example
Answer:B. Like some other men.According is ANNE BOLEYN, The mother of Queen
to ROY, Man as a social being is like some Elizabeth and the wife of King Henry VIII.
other man. As a spiritual being and Biologic She was beheaded because Henry wants to
being, Man are all alike. As a psychologic mary another wife and that his divorce was
being, No man thinks alike. This basically not approved by the pope. Outraged, He
summarized her BIOPSYHOSOCIAL theory insisted on the separation of the Church and
which is included in our licensure exam State and divorce Anne himself by making
coverage. everyone believe that Anne is having an
affair to another man. Anne was beheaded
52. She conceptualized that man, as an Open while her lips is still saying a prayer. Even
system is in constant interaction and without her head, People still gave respect to
transaction with a changing environment. her diseased body and a separate head. She
was still remembered as Anne boleyn,
A. Roy Mother of Elizabeth who lead england to
B. Levin their GOLDEN AGE.
C. Neuman
D. Newman 55. She theorized that man is composed of sub
and supra systems. Subsystems are cells,
Answer:A. Roy. OPEN system theory is tissues, organs and systems while the
ROY. As an open system, man continuously
suprasystems are family, society and
allows input from the environment. Example
is when you tell me Im good looking, I will community.
be happy the entire day, Because I am an A. Roy
open system and continuously interact and B. Rogers
transact with my environment. A close C. Henderson
system is best exemplified by a CANDLE. D. Johnson
When you cover the candle with a glass, it
will die because it will eventually use all the Answer: B. Rogers. According to Martha
oxygen it needs inside the glass for Rogers, Man is composed of 2 systems :
combustion. A closed system do not allow SUB which includes cells, tissues, organs
inputs and output in its environment. and system and SUPRA which includes our
famly, community and society. She stated
53. In a CLOSED system, which of the that when any of these systems are affected,
following is true? it will affect the entire individual.
A. Affected by matter
56. Which of the following is not true about
B. A sole island in vast ocean
C. Allows input the human needs?
D. Constantly affected by matter, energy, A. Certain needs are common to all people
information B. Needs should be followed exactly in
accordance with their hierarchy
Answer: B. A sole island in vast ocean C. Needs are stimulated by internal factors
D. Needs are stimulated by external factors
54. Who postulated the WHOLISTIC concept
that the totality is greater than sum of its parts? Answer: B. Needs should be followed
exactly in accordance with their
A. Roy
hierarchy.Needs can be deferred. I can
B. Rogers
urinate later as not to miss the part of the
C. Henderson
movie’s climax. I can save my money that
D. Johnson
are supposedly for my lunch to watch my
idols in concert. The physiologic needs can
be meet later for some other needs and need Answer: B. Self actualization. The peak of
not be strictly followed according to their maslow’s hierarchy is the essence of mental
hierarchy. health

57. Which of the following is TRUE about the 61. Florence nightingale is born in
human needs?
A. Germany
A. May not be deferred B. Britain
B. Are not interrelated C. France
C. Met in exact and rigid way D. Italy
D. Priorities are alterable
Answer: D. Italy. Florence Nightingale was
Answer: D. Priorities are alterable. Refer to born in Florence, Italy, May 12, 1820.
question # 56. Studied in Germany and Practiced in
England.
58. According to Maslow, which of the
following is NOT TRUE about a self 62. Which is unlikely of Florence Nightingale?
actualized person? A. Born May 12, 1840
A. Understands poetry, music, philosophy, B. Built St. Thomas school of nursing when
science etc. she was 40 years old
B. Desires privacy, autonomous C. Notes in nursing
C. Follows the decision of the majority, D. Notes in hospital
uphold justice and truth
D. Problem centered 1. Answer: A. Born May 12, 1840

Answer: C. Follows the decision of the 63. What country did Florence Nightingale
majority, uphold justice and truth. A,B and D train in nursing?
are all qualities of a self actualized person. A A. Belgium
self actualized person do not follow the
B. US
decision of majority but is self directed and
C. Germany
can make decisions contrary to a popular
D. England
opinion.
Answer: germany
59. According to Maslow, which of the
following is TRUE about a self actualized 64. Which of the following is recognized for
person? developing the concept of HIGH LEVEL
A. Makes decision contrary to public WELLNESS?
opinion A. Erikson
B. Do not predict events
B. Madaw
C. Self centered C. Peplau
D. Maximum degree of self conflict
D. Dunn
Answer: A. Makes decision contrary to Answer:D. Dunn. According to Dunn, High
public opinion. Refer to question # 58.
level wellness is the ability of an individual
to maximize his full potential with the
60. This is the essence of mental health limitations imposed by his environment.
A. Self awareness According to him, An individual can be
B. Self actualization healthy or ill in both favorable and
C. Self esteem unfavorable environment.
D. Self worth
65. One of the expectations is for nurses to
join professional association primarily
because of
A. Promotes advancement and 69. Which of the following does not govern
professional growth among its nursing practice?
members
B. Works for raising funds for nurse’s A. RA 7164
benefit B. RA 9173
C. Facilitate and establishes acquaintances C. BON Res. Code Of Ethics
D. Assist them and securing jobs abroad D. BON Res. Scope of Nursing Practice

Answer: A. Promotes advancement and Answer: A. RA 7164. 7164 is an old law.


professional growth among its members This is the 1991 Nursing Law which was
repealed by the newer 9173.
66. Founder of the PNA
70. A nurse who is maintaining a private
A. Julita Sotejo clinic in the community renders service on
B. Anastacia Giron Tupas maternal and child health among the
C. Eufemia Octaviano neighborhood for a fee is:
D. Anesia Dionisio
A. Primary care nurse
Answer: Anastacia Giron Tupas B. Independent nurse practitioner
C. Nurse-Midwife
67. Which of the following provides that D. Nurse specialist
nurses must be a member of a national nurse
organization? Answer: B. Independent nurse practitioner

A. R.A 877 71. When was the PNA founded?


B. 1981 Code of ethics approved by the
house of delegates and the PNA A. September 22, 1922
C. Board resolution No. 1955 Promulgated B. September 02, 1920
by the BON C. October 21, 1922
D. RA 7164 D. September 02, 1922

Answer: C. Board resolution No. 1955 Answer: D. September 02, 1922. According
Promulgated by the BON. This is an old to the official PNA website, they are founded
board resolution. The new Board resolution September 02, 1922.
is No. 220 series of 2004 also known as the
Nursing Code Of ethics which states that 72. Who was the first president of the PNA ?
[ SECTION 17, A ] A nurse should be a A. Anastacia Giron-Tupas
member of an accredited professional B. Loreto Tupas
organization which is the PNA. C. Rosario Montenegro
D. Ricarda Mendoza
68. Which of the following best describes the
action of a nurse who documents her nursing Answer: C. Rosario Montenegro. Anastacia
diagnosis? Giron Tupas founded the FNA, the former
A. She documents it and charts it whenever name of the PNA but the first President was
necessary Rosario Montenegro.
B. She can be accused of malpractice
C. She does it regularly as an important 73. Defines health as the ability to maintain
responsibility internal milieu. Illness according to
D. She charts it only when the patient is him/her/them is the failure to maintain
acutely ill internal environment.
A. Cannon
Answer: C. She does it regularly as an
B. Bernard
important responsibility C. Leddy and Pepper
D. Roy
Answer: B. Bernard. According to Bernard, C. Rogers
Health is the ability to maintain and Internal D. King
Milieu and Illness is the failure to maintain
the internal environment. Answer: D. King .Emogene King states that
health is a state in the life cycle and Illness is
74. Postulated that health is a state and any interference on this cycle. I enjoyed the
process of being and becoming an integrated Movie LION KING and like what Mufasa
and whole person. said that they are all part of the CIRCLE OF
LIFE, or the Life cycle.
A. Cannon
B. Bernard 78. She defined health as the soundness and
C. Dunn wholness of developed human structure and
D. Roy bodily mental functioning.
Answer: D. Roy. According to ROY, Health A. Orem
is a state and process of becoming a WHOLE B. Henderson
AND INTEGRATED Person. C. Neuman
D. Clark
75. What regulates HOMEOSTASIS
according to the theory of Walter Cannon? Answer: A. Orem. Orem defined health as
the SOUNDNESS and WHOLENESS of
A. Positive feedback developed human structure and of bodily and
B. Negative feedback mental functioning.
C. Buffer system
D. Various mechanisms 79. According to her, Wellness is a condition
in which all parts and subparts of an
Answer: B. Negative feedback. The theory of
individual are in harmony with the whole
Health as the ability to maintain homeostasis
was postulated by Walter Cannon. According system.
to him, There are certain FEEDBACK A. Orem
Mechanism that regulates our Homeostasis. B. Henderson
A good example is that when we overuse our C. Neuman
arm, it will produce pain. PAIN is a negative D. Johnson
feedback that signals us that our arm needs a
rest. Answer: C. Neuman. Neuman believe that
man is composed of subparts and when this
76. Stated that health is WELLNESS. A subparts are in harmony with the whole
termed define by the culture or an individual. system, Wellness results. Please do not
confuse this with the SUB and SUPRA
A. Roy
systems of martha rogers.
B. Henderson
C. Rogers
80. Postulated that health is reflected by the
D. King
organization, interaction, interdependence and
Answer: C. Rogers. Martha Rogers states integration of the subsystem of the behavioral
that HEALTH is synonymous with system.
WELLNESS and that HEALTH and A. Orem
WELLNESS is subjective depending on the B. Henderson
definition of one’s culture. C. Neuman
D. Johnson
77. Defined health as a dynamic state in the
life cycle, and Illness as interference in the Answer: D. Johnson . Once you see the
life cycle. phrase BEHAVIORAL SYSTEM, answer
A. Roy Dorothy Johnson.
B. Henderson
81. According to them, Well being is a sanitation and over crowding. Heridity
subjective perception of BALANCE, includes congenital and diseases acquired
HARMONY and VITALITY through the genes. There are no social
precursors according to DUNN.
A. Leavell and Clark
B. Peterson and Zderad 85. According to DUNN, Overcrowding is
C. Benner and Wruber what type of illness precursor?
D. Leddy and Pepper
A. Heredity
Answer: D. Leddy and Pepper .According to B. Social
Leddy and Pepper, Wellness is subjective C. Behavioral
and depends on an individuals perception of D. Environmental
balance, harmony and vitality. Leavell and
Clark postulared the ecologic model of health Answer: Environmental
and illness or the AGENT-HOST-
ENVIRONMENT model. Peterson and 86. Health belief model was formulated in
Zderad developed the HUMANISTIC 1975 by who?
NURSING PRACTICE theory while Benner
and Wruber postulate the PRIMACY OF A. Becker
CARING MODEL. B. Smith
C. Dunn
82. He describes the WELLNESS-ILLNESS D. Leavell and Clark
Continuum as interaction of the environment
Answer:A. Becker. According to Becker,
with well being and illness. The belief of an individual greatly affects his
A. Cannon behavior. If a man believes that he is
B. Bernard susceptible to an illness, He will alter his
C. Dunn behavior in order to prevent its occurence.
D. Clark For example, If a man thinks that diabetes is
acquired through high intake of sugar and
Answer: Dunn simple carbohydrates, then he will limit the
intake of foods rich in these components.
83. An integrated method of functioning that
is oriented towards maximizing one’s 87. In health belief model, Individual
potential within the limitation of the perception matters. Which of the following is
environment. highly UNLIKELY to influence preventive
behavior?
A. Well being
B. Health A. Perceived susceptibility to an illness
C. Low level Wellness B. Perceived seriousness of an illness
D. High level Wellness C. Perceived threat of an illness
D. Perceived curability of an illness
Answer: High level Wellness
Answer: D. Perceived curability of an
84. What kind of illness precursor, according illness . If a man think he is susceptibe to a
to DUNN is cigarette smoking? certain disease, thinks that the disease is
serious and it is a threat to his life and
A. Heredity functions, he will use preventive behaviors to
B. Social avoid the occurence of this threat.
C. Behavioral
D. Environmental 88. Which of the following is not a
PERCEIVED BARRIER in preventive action?
Answer: C. Behavioral. Behavioral
precursors includes smoking, alcoholism, A. Difficulty adhering to the lifestyle
high fat intake and other lifestyle choices. B. Economic factors
Environmental factors involved poor C. Accessibility of health care facilities
D. Increase adherence to medical therapies 91. Knowledge about the disease and prior
contact with it is what type of VARIABLE
Answer:A. Difficulty adhering to the lifestyle according to the health belief model?
and B. Economic factors. Perceived barriers
are those factors that affects the individual’s A. Demographic
health preventive actions. Both A and B can B. Sociopsychologic
affect the individual’s ability to prevent the C. Structural
occurence of diseases. C and D are called D. Cues to action
Preventive Health Behaviors which enhances
the individual’s preventive capabilities. Answer: C. Structural. Modifying variables in
Becker’s health belief model includes
89. Conceptualizes that health is a condition DEMOGRAPHIC : Age, sex, race etc.
of actualization or realization of person’s SOCIOPSYCHOLOGIC : Social and Peer
potential. Avers that the highest aspiration of influence. STRUCTURAL : Knowledge about
the disease and prior contact with it and
people is fulfillment and complete
CUES TO ACTION : Which are the sign and
development actualization.
symptoms of the disease or advice from
A. Clinical Model friends, mass media and others that forces or
B. Role performance Model makes the individual seek help
C. Adaptive Model
D. Eudaemonistic Model 92. It includes internal and external factors
that leads the individual to seek help
Answer: D. Eudaemonistic Model . Smith
A. Demographic
formulated 5 models of health. Clinical
B. Sociopsychologic
model simply states that when people
C. Structural
experience sign and symptoms, they would
D. Cues to action
think that they are unhealthy therefore,
Health is the absence of clinical sign and
Answer: D. Cues to action . Refer to question
symptoms of a disease. Role performance
# 91.
model states that when a person does his role
and activities without deficits, he is healthy
93. Influence from peers and social pressure is
and the inability to perform usual roles
means that the person is ill. Adaptive Model included in what variable of HBM?
states that if a person adapts well with his A. Demographic
environment, he is healthy and maladaptation B. Sociopsychologic
equates illness. Eudaemonistic Model of C. Structural
health according to smith is the actualization D. Cues to action
of a person’s fullest potential. If a person
functions optimally and develop self Answer: B. Sociopsychologic. Refer to
actualization, then, no doubt that person is question # 91.
healthy.
94. Age, Sex, Race etc. is included in what
90. Views people as physiologic system and variable of HBM?
Absence of sign and symptoms equates health.
A. Demographic
A. Clinical Model B. Sociopsychologic
B. Role performance Model C. Structural
C. Adaptive Model D. Cues to action
D. Eudaemonistic Model
Answer: A. Demographic. Refer to question
Answer:A. Clinical Model. Rationale: Refer # 91.
to question # 89.
95. According to Leavell and Clark’s ecologic
model, All of this are factors that affects
health and illness except
A. Reservoir
B. Agent Answer: B. Disease. Disease are alteration in
C. Environment body functions resulting in reduction of
D. Host capabilities or shortening of life span.

Answer: A. Reservoir. According to L&C’s 99. Personal state in which a person feels
Ecologic model, there are 3 factors that affect unhealthy
health and illness. These are the AGENT or
the factor the leads to illness, either a bacteria A. Illness
or an event in life. HOST are persons that B. Disease
may or may not be affected by these agents. C. Health
ENVIRONMENT are factors external to the D. Wellness
host that may or may not predispose him to
the AGENT. Answer:A. Illness. Illness is something
PERSONAL. Unlike disease, Illness are
96. Is a multi dimensional model developed personal state in which person feels
by PENDER that describes the nature of unhealthy. An old person might think he is
ILL but in fact, he is not due, to diminishing
persons as they interact within the
functions and capabilities, people might think
environment to pursue health
they are ILL. Disease however, is something
A. Ecologic Model with tangible basis like lab results, X ray
B. Health Belief Model films or clinical sign and symptoms.
C. Health Promotion Model
D. Health Prevention Model 100. According to her, Caring is defined as a
nurturant way of responding to a valued client
Answer: C. Health Promotion Model. Pender towards whom the nurse feels a sense of
developed the concept of HEALTH commitment and responsibility.
PROMOTION MODEL which postulated
that an individual engages in health A. Benner
promotion activities to increase well being B. Watson
and attain self actualization. These includes C. Leininger
exercise, immunization, healthy lifestyle, D. Swanson
good food, self responsibility and all other
factors that minimize if not totally eradicate
risks and threats of health.
Answer: B. Watson. This is Jean Watson’s
97. Defined by Pender as all activities directed definition of Nursing as caring. This was
toward increasing the level of well being and asked word per word last June 06′ NLE.
Benner defines caring as something that
self actualization.
matters to people. She postulated the
A. Health prevention responsibility created by Caring in nursing.
B. Health promotion She was also responsible for the PRIMACY
C. Health teaching OF CARING MODEL. Leininger defind the
D. Self actualization 4 conservation principle while Swanson
introduced the 5 processes of caring.
Answer:B. Health promotion. Refer to
question # 96.

98. Defined as an alteration in normal PNLE: FON Practice


function resulting in reduction of capacities
and shortening of life span. Exam for Infection,
A. Illness
B. Disease Asepsis, Basic
C. Health
D. Wellness
C. Clients BP will increase, there will be
concept of Stress and vasodilation
D. There will be increase glycogenolysis,
Pancrease will decrease insulin secretion
Illness
1. When the General adaptation Answer: C. Clients BP will increase, there
syndrome is activated, FLIGHT OR FIGHT will be vasodilation. If vasodilation will
response sets in. Sympathetic nervous system occur, The BP will not increase but decrease.
releases norepinephrine while the adrenal It is true that Blood pressure increases during
medulla secretes epinephrine. Which of the SNS Stimulation due to the fact that we need
following is true with regards to that more BLOOD to circulate during the FIGHT
statement? or FLIGHT Response because the oxygen
demand has increased, but this is facilitated
A. Pupils will constrict by vasoconstriction and not vasodilation. A,B
B. Client will be lethargic and D are all correct. The liver will increase
C. Lungs will bronchodilate glycogenolysis or glycogen store utilization
D. Gastric motility will increase due to a heightened demand for energy.
Pancrease will decrease insulin secretion
Answer: C. Lungs will bronchodilate. To because almost every aspect of digestion that
better understand the concept : The is controlled by Parasympathetic nervous
autonomic nervous system is composed of system is inhibited when the SNS dominates.
SYMPATHETIC and
PARASYMPATHETIC Nervous system. It 3. State in which a person’s physical,
is called AUTONOMIC Because it is emotional, intellectual and social development
Involuntary and stimuli based. You cannot or spiritual functioning is diminished or
tell your heart to kindly beat for 60 per impaired compared with a previous
minute, Nor, Tell your blood vessels, Please experience.
constrict, because you need to wear skirt
today and your varicosities are bulging. A. Illness
Sympathetic Nervous system is the FIGHT B. Disease
or FLIGHT mechanism. When people C. Health
FIGHT or RUN, we tend to stimulate the D. Wellness
ANS and dominate over SNS. Just Imagine a
person FIGHTING and RUNNING to get the Answer: A. Illness. Disease is a PROVEN
idea on the signs of SNS Domination. FACT based on a medical theory, standards,
Imagine a resting and digesting person to get diagnosis and clinical feature while ILLNESS
a picture of PNS Domination. A person Is a subjective state of not feeling well based
RUNNING or FIGHTING Needs to on subjective appraisal, previous experience,
bronchodilate, because the oxygen need is peer advice etc
increased due to higher demand of the body.
Pupils will DILATE to be able to see the 4. This is the first stage of illness wherein, the
enemy clearly. Client will be fully alert to person starts to believe that something is
dodge attacks and leap through obstacles wrong. Also known as the transition phase
during running. The client’s gastric motility from wellness to illness.
will DECREASE Because you cannot afford
to urinate or defecate during fighting nor A. Symptom Experience
running. B. Assumption of sick role
C. Medical care contact
2. Which of the following response is not D. Dependent patient role
expected to a person whose GAS is activated
and the FIGHT OR FLIGHT response sets in? Answer: A. Symptom Experience. A favorite
board question are Stages of Illness. When a
A. The client will not urinate due to person starts to believe something is wrong,
relaxation of the detrusor muscle that person is experiencing signs and
B. The client will be restless and alert symptoms of an illness. The patient will then
ASSUME that he is sick. This is called D. Dependent patient role
assumption of the sick role where the patient
accepts he is Ill and try to give up some Answer: C. Medical care contact. At this
activities. Since the client only ASSUMES stage, The patient seeks for validation of his
his illness, he will try to ask someone to symptom experience. He wants to find out if
validate if what he is experiencing is a what he feels are normal or not normal. He
disease, This is now called as MEDICAL wants someone to explain why is he feeling
CARE CONTACT. The client seeks these signs and symptoms and wants to know
professional advice for validation, the probable outcome of this experience.
reassurance, clarification and explanation of
the symptoms he is experiencing. client will 8. The following are true with regards to
then start his dependent patient role of aspect of the sick role except
receiving care from the health care providers.
The last stage of Illness is the RECOVERY A. One should be held responsible for his
stage where the patient gives up the sick role condition
and assumes the previous normal gunctions. B. One is excused from his societal role
C. One is obliged to get well as soon as
5. In this stage of illness, the person accepts or possible
rejects a professionals suggestion. The person D. One is obliged to seek competent help
also becomes passive and may regress to an
earlier stage. Answer: A. One should be held responsible
for his condition. The nurse should not judge
A. Symptom Experience the patient and not view the patient as the
B. Assumption of sick role cause or someone responsible for his illness.
C. Medical care contact A sick client is excused from his societal roles,
D. Dependent patient role Oblige to get well as soon as possible and
Obliged to seek competent help.
Answer: D. Dependent patient role. In the
dependent patient role stage, Client needs 9. Refers to conditions that increases
professionals for help. They have a choice vulnerability of individual or group to illness
either to accept or reject the professional’s or accident
decisions but patients are usually passive and
accepting. Regression tends to occur more in A. Predisposing factor
this period. B. Etiology
C. Risk factor
6. In this stage of illness, The person learns to D. Modifiable Risks
accept the illness.
Answer: C. Risk factor
A. Symptom Experience
B. Assumption of sick role 10. Refers to the degree of resistance the
C. Medical care contact potential host has against a certain pathogen
D. Dependent patient role
A. Susceptibility
Answer:B. Assumption of sick B. Immunity
role. Acceptance of illness occurs in the C. Virulence
Assumption of sick role phase of illness. D. Etiology

7. In this stage, the person tries to find Answer: A. Susceptibility. Immunity is the
answers for his illness. He wants his illness to ABSOLUTE Resistance to a pathogen
be validated, his symptoms explained and the considering that person has an INTACT
outcome reassured or predicted IMMUNITY while susceptibility is the
DEGREE of resistance. Degree of resistance
A. Symptom Experience means how well would the individual combat
B. Assumption of sick role the pathogens and repel infection or invasion
C. Medical care contact of these disease causing organisms. A
susceptible person is someone who has a 13. The classification of CANCER according
very low degree of resistance to combat to its etiology Is best described as:
pathogens. An Immune person is someone
that can easily repel specific pathogens. 1. Nosocomial
However, Remember that even if a person is 2. Idiopathic
IMMUNE [ Vaccination ] Immunity can 3. Neoplastic
always be impaired in cases of chemotherapy, 4. Traumatic
HIV, Burns, etc. 5. Congenital
6. Degenrative
11. A group of symptoms that sums up or A. 5 and 2
constitute a disease B. 2 and 3
C. 3 and 4
A. Syndrome D. 3 and 5
B. Symptoms
C. Signs Answer: B. 2 and 3. Aside from being
D. Etiology NEOPLASTIC, Cancer is considered as
IDIOPATHIC because the cause is
Answer: A. Syndrome. Symptoms are UNKNOWN.
individual manifestation of a certain disease.
For example, In Tourette syndrome, patient 14. Term to describe the reactiviation and
will manifest TICS, but this alone is not recurrence of pronounced symptoms of a
enough to diagnose the patient as other disease
diseases has the same tic manifestation.
Syndrome means COLLECTION of these A. Remission
symptoms that occurs together to B. Emission
characterize a certain disease. Tics with C. Exacerbation
coprolalia, echolalia, palilalia, choreas or D. Sub acute
other movement disorders are characteristics
of TOURETTE SYNDROME. Answer:C. Exacerbation

12. A woman undergoing radiation therapy 15. A type of illness characterized by periods
developed redness and burning of the skin of remission and exacerbation
around the best. This is best classified as what
type of disease? A. Chronic
B. Acute
A. Neoplastic C. Sub acute
B. Traumatic D. Sub chronic
C. Nosocomial
D. Iatrogenic Answer: A. Chronic. A good example is
Multiple sclerosis that characterized by
Answer: D. Iatrogenic. Iatrogenic diseases periods of remissions and exacerbation and it
refers to those that resulted from treatment of is a CHRONIC Disease. An acute and sub
a certain disease. For example, A child acute diseases occurs too short to manifest
frequently exposed to the X-RAY Machine remissions. Chronic diseases persists longer
develops redness and partial thickness burns than 6 months that is why remissions and
over the chest area. Neoplastic are malignant exacerbation are observable.
diseases cause by proliferation of abnormally
growing cells. Traumatic are brought about 16. Diseases that results from changes in the
by injuries like Motor vehicular accidents. normal structure, from recognizable
Nosocomial are infections that acquired anatomical changes in an organ or body tissue
INSIDE the hospital. Example is UTI is termed as
Because of catheterization, This is commonly
caused by E.Coli. A. Functional
B. Occupational
C. Inorganic
D. Organic D. Geography

Answer:D. Organic. As the word implies, Answer: A. Epidemiology. Refer to number


ORGANIC Diseases are those that causes a 17.
CHANGE in the structure of the organs and
systems. Inorganic diseases is synonymous 19. Refers to diseases that produced no
with FUNCTIONAL diseases wherein, There anatomic changes but as a result from
is no evident structural, anatomical or abnormal response to a stimuli.
physical change in the structure of the organ
or system but function is altered due to other A. Functional
causes, which is usually due to abnormal B. Occupational
response of the organ to stressors. Therefore, C. Inorganic
ORGANIC BRAIN SYNDROME are D. Organic
anatomic and physiologic change in the
BRAIN that is NON PROGRESSIVE BUT Answer: C. Inorganic. Refer to number 16.
IRREVERSIBLE caused by alteration in
structure of the brain and it’s supporting 20. In what level of prevention according to
structure which manifests different sign and Leavell and Clark does the nurse support the
symptoms of neurological, physiologic and client in obtaining OPTIMAL HEALTH
psychologic alterations. Mental disorders STATUS after a disease or injury?
manifesting symptoms of psychoses without
any evident organic or structural damage are A. Primary
termed as INORGANIC PSYCHOSES while B. Secondary
alteration in the organ structures that causes C. Tertiary
symptoms of bizaare pyschotic behavior is D. None of the above
termed as ORGANIC PSYCHOSES.
Answer: C. Tertiary. Perhaps one of the
17. It is the science of organism as affected by easiest concept but asked frequently in the
factors in their environment. It deals with the NLE. Primary refers to preventions that aims
relationship between disease and geographical in preventing the disease. Examples are
environment. healthy lifestyle, good nutrition, knowledge
seeking behaviors etc. Secondary prevention
A. Epidemiology are those that deals with early diagnostics,
B. Ecology case finding and treatments. Examples are
C. Statistics monthly breast self exam, Chest X-RAY,
D. Geography Antibiotic treatment to cure infection, Iron
therapy to treat anemia etc. Tertiary
Answer: B. Ecology. Ecology is the science prevention aims on maintaining optimum
that deals with the ECOSYSTEM and its level of functioning during or after the
effects on living things in the biosphere. It impact of a disease that threatens to alter the
deals with diseases in relationship with the normal body functioning. Examples are
environment. Epidimiology is simply the prosthetis fitting for an amputated leg after
Study of diseases and its occurence and an accident, Self monitoring of glucose
distribution in man for the purpose of among diabetics, TPA Therapy after stroke
controlling and preventing diseases. This was etc. The confusing part is between the
asked during the previous boards. treatment in secondary and treatment in
tertiary. To best differentiate the two, A
18. This is the study of the patterns of health client with ANEMIA that is being treated
and disease. Its occurrence and distribution in with ferrous sulfate is considered being in the
man, for the purpose of control and prevention SECONDARY PREVENTION because
of disease. ANEMIA once treated, will move the client
on PRE ILLNESS STATE again. However,
A. Epidemiology In cases of ASPIRING Therapy in cases of
B. Ecology stroke, ASPIRING no longer cure the patient
C. Statistics or PUT HIM IN THE PRE ILLNESS
STATE. ASA therapy is done in order to
prevent coagulation of the blood that can lead 24. BCG in community health nursing is what
to thrombus formation and a another possible type of prevention?
stroke. You might wonder why I spelled
ASPIRIN as ASPIRING, Its side effect is A. Primary
OTOTOXICITY [ CN VIII ] that leads to B. Secondary
TINNITUS or ringing of the ears. C. Tertiary
D. None of the above
21. In what level of prevention does the nurse
encourage optimal health and increases Answer: primary
person’s susceptibility to illness?
25. A regular pap smear for woman every 3
A. Primary years after establishing normal pap smear for
B. Secondary 3 consecutive years Is advocated. What level
C. Tertiary of prevention does this belongs?
D. None of the above
A. Primary
Answer: D. None of the above. The nurse B. Secondary
never increases the person’s susceptibility to C. Tertiary
illness but rather, LESSEN the person’s D. None of the above
susceptibility to illness.
Answer: secondary
22. Also known as HEALTH
MAINTENANCE prevention. 26. Self monitoring of blood glucose for
diabetic clients is on what level of prevention?
A. Primary
B. Secondary A. Primary
C. Tertiary B. Secondary
D. None of the above C. Tertiary
D. None of the above
Answer: B. Secondary. Secondary prevention
is also known as HEALTH Answer: tertiary
MAINTENANCE Prevention. Here, The
person feels signs and symptoms and seeks 27. Which is the best way to disseminate
Diagnosis and treatment in order to prevent information to the public?
deblitating complications. Even if the person
feels healthy, We are required to A. Newspaper
MAINTAIN our health by monthly check B. School bulletins
ups, Physical examinations, Diagnostics etc. C. Community bill boards
D. Radio and Television
23. PPD In occupational health nursing is
what type of prevention? Answer: D. Radio and Television. An actual
board question, The best way to disseminate
A. Primary information to the public is by TELEVISION
B. Secondary followed by RADIO. This is how the DOH
C. Tertiary establish its IEC Programs other than
D. None of the above publising posters, leaflets and brochures. An
emerging new way to disseminate is through
Answer: A. Primary. PPD or PERSONAL the internet.
PROTECTIVE DEVICES are worn by the
workes in a hazardous environment to protect 28. Who conceptualized health as integration
them from injuries and hazards. This is of parts and subparts of an individual?
considered as a PRIMARY prevention
because the nurse prevents occurence of A. Newman
diseases and injuries. B. Neuman
C. Watson 31. Excessive alcohol intake is what type of
D. Rogers risk factor?

Answer: B. Neuman. The supra and A. Genetics


subsystems are theories of Martha Rogers but B. Age
the parts and subparts are Betty Neuman’s. C. Environment
She stated that HEALTH is a state where in D. Lifestyle
all parts and subparts of an individual are in
harmony with the whole system. Margarex Answer: lifestyle
Newman defined health as an EXPANDING
CONSCIOUSNESS. Her name is Margaret 32. Osteoporosis and degenerative diseases
not Margarex, I just used that to help you like Osteoarthritis belongs to what type of risk
remember her theory of health. factor?

29. The following are concept of health: A. Genetics


B. Age
1. Health is a state of complete physical, C. Environment
mental and social wellbeing and not D. Lifestyle
merely an absence of disease or infirmity.
2. Health is the ability to maintain balance Answer: age
3. Health is the ability to maintain internal
milieu 33. Also known as STERILE TECHNIQUE
4. Health is integration of all parts and
subparts of an individual A. Surgical Asepsis
A. 1,2,3 B. Medical Asepsis
B. 1,3,4 C. Sepsis
C. 2,3,4 D. Asepsis
D. 1,2,3,4
Answer: A. Surgical Asepsis. Surgical
Answer: D. 1,2,3,4. All of the following are Asepsis is also known as STERILE
correct statement about health. The first one TECHNIQUE while Medical Asepsis is
is the definition by WHO, The second one is synonymous with CLEAN TECHNIQUE.
from Walter Cannon’s homeostasis theory.
Third one is from Claude Bernard’s concept 34. This is a person or animal, who is without
of Health as Internal Milieu and the last one signs of illness but harbors pathogen within
is Neuman’s Theory. his body and can be transferred to another

30. The theorist the advocated that health is A. Host


the ability to maintain dynamic equilibrium is B. Agent
C. Environment
A. Bernard D. Carrier
B. Selye
C. Cannon Answer: carrier
D. Rogers
35. Refers to a person or animal, known or
Answer: C. Cannon. Walter Cannon believed to have been exposed to a disease.
advocated health as HOMEOSTASIS or the
ability to maintain dynamic equilibrium. A. Carrier
Hans Selye postulated Concepts about Stress B. Contact
and Adaptation. Bernard defined health as C. Agent
the ability to maintain internal milieu and D. Host
Rogers defined Health as Wellness that is
influenced by individual’s culture. Answer: contact
36. A substance usually intended for use on characterized by the appearance of specific
inanimate objects, that destroys pathogens but signs and symptoms or refer tp as time with
not the spores. the greatest symptom experience. Acme is
the PEAK of illness intensity while the
A. Sterilization convalescent period is characterized by the
B. Disinfectant abatement of the disease process or it’s
C. Antiseptic gradual disappearance.
D. Autoclave
39. A child with measles developed fever and
Answer: B. Disinfectant. Disinfectants are general weakness after being exposed to
used on inanimate objects while Antiseptics another child with rubella. In what stage of
are intended for use on persons and other infectious process does this child belongs?
living things. Both can kill and inhibit
growth of microorganism but cannot kill A. Incubation period
their spores. That is when autoclaving or B. Prodromal period
steam under pressure gets in, Autoclaving C. Illness period
can kill almost ALL type of microoganism D. Convalescent period
including their spores.
Answer: B. Prodromal period. To be able to
37. This is a process of removing pathogens categorize MEASLES in the Illness period,
but not their spores the specific signs of Fever, Koplik’s Spot and
Rashes must appear. In the situation above,
A. Sterilization Only general signs and symptoms appeared
B. Auto claving and the Specific signs and symptoms is yet to
C. Disinfection appear, therefore, the illness is still in the
D. Medical asepsis Prodromal period. Signs and symptoms of
measles during the prodromal phase are
Answer: C. Disinfection. Both A and B are Fever, fatigue, runny nose, cough and
capable on killing spores. Autoclaving is a conjunctivitis. Koplik’s spot heralds the
form of Sterilization. Medical Asepsis is a Illness period and cough is the last symptom
PRACTICE designed to minimize or reduce to disappear. All of this processes take place
the transfer of pathogens, also known as your in 10 days that is why, Measles is also known
CLEAN TECHNIQUE. Disinfection is the as 10 day measles.
PROCESS of removing pathogens but not
their spores. 40. A 50 year old mailman carried a mail with
anthrax powder in it. A minute after exposure,
38. The third period of infectious processes he still hasn’t developed any signs and
characterized by development of specific symptoms of anthrax. In what stage of
signs and symptoms infectious process does this man belongs?

A. Incubation period A. Incubation period


B. Prodromal period B. Prodromal period
C. Illness period C. Illness period
D. Convalescent period D. Convalescent period

Answer: C. Illness period. In incubation Answer: A. Incubation period. Anthrax can


period, The disease has been introduced to have an incubation period of hours to 7 days
the body but no sign and symptom appear with an average of 48 hours. Since the
because the pathogen is not yet strong question stated exposure, we can now assume
enough to cause it and may still need to that the mailman is in the incubation period.
multiply. The second period is called
prodromal period. This is when the 41. Considered as the WEAKEST LINK in
appearance of non specific signs and the chain of infection that nurses can
symptoms sets in, This is when the sign and manipulate to prevent spread of infection and
symptoms starts to appear. Illness period is diseases
A. Etiologic/Infectious agent B. Vehicle borne transmission
B. Portal of Entry C. Air borne transmission
C. Susceptible host D. Vector borne transmission
D. Mode of transmission
Answer: D. Vector borne transmission. Lyme
Answer: D. Mode of transmission. Mode of disease is caused by Borrelia Burdorferi and
transmission is the weakest link in the chain is transmitted by a TICK BITE.
of infection. It is easily manipulated by the
Nurses using the tiers of prevention, either by 44. The ability of the infectious agent to cause
instituting transmission based precautions, a disease primarily depends on all of the
Universal precaution or Isolation techniques. following except

42. Which of the following is the exact order A. Pathogenicity


of the infection chain? B. Virulence
C. Invasiveness
1. Susceptible host D. Non Specificity
2. Portal of entry
3. Portal of exit Answer: D. Non Specificity. To be able to
4. Etiologic agent cause a disease, A pathogen should have a
5. Reservoir TARGET ORGAN/S. The pathogen should
6. Mode of transmission be specific to these organs to cause an
A. 1,2,3,4,5,6 infection. Mycobacterium Avium is NON
B. 5,4,2,3,6,1 SPECIFIC to human organs and therefore,
C. 4,5,3,6,2,1 not infective to humans but deadly to birds.
D. 6,5,4,3,2,1 An immunocompromised individual,
specially AIDS Patient, could be infected
Answer: C. 4,5,3,6,2,1. Chain of infection with these NON SPECIFIC diseases due to
starts with the SOURCE : The etiologic agent impaired immune system.
itself. It will first proliferate on a
RESERVOIR and will need a PORTAL OF 45. Contact transmission of infectious
EXIT to be able to TRANSMIT irslef using a organism in the hospital is usually cause by
PORTAL OF ENTRY to a SUSCEPTIBLE
HOST. A simple way to understand the A. Urinary catheterization
process is by looking at the lives of a young B. Spread from patient to patient
queen ant that is starting to build her colony. C. Spread by cross contamination via
Imagine the QUEEN ANT as a SOURCE or hands of caregiver
the ETIOLOGIC AGENT. She first need to D. Cause by unclean instruments used by
build a COLONY, OR the RESERVOIR doctors and nurses
where she will start to lay the first eggs to be
able to produce her worker ants and soldier Answer: C. Spread by cross contamination
ants to be able to defend and sustain the new via hands of caregiver. The hands of the
colony. They need to EXIT [PORTAL OF caregiver like nurses, is the main cause of
EXIT] their colony and crawl [MODE OF cross contamination in hospital setting. That
TRANSMISSION] in search of foods by is why HANDWASHING is the single most
ENTERING / INVADING [PORTAL OF important procedure to prevent the occurence
ENTRY] our HOUSE [SUSCEPTIBLE of cross contamination and nosocomial
HOST]. By imagining the Ant’s life cycle, infection. D refers to Nosocomial infection
we can easily arrange the chain of infection. and UTI is the most common noscomial
infection in the hospital caused by urinary
43. Markee, A 15 year old high school student catheterization. E.Coli seems to be the major
asked you. What is the mode of transmission cause of this incident. B best fits Cross
of Lyme disease. You correctly answered him Contamination, It is the spread of
that Lyme disease is transmitted via microogranisms from patient o patient.

A. Direct contact transmission


46. Transmission occurs when an infected Answer: C. Artificial active immunity. TT1 ti
person sneezes, coughs or laugh that is usually TT2 are considered the primary dose, while
projected at a distance of 3 feet. TT3 to TT5 are the booster dose. A woman
with completed immunization of DPT need
A. Droplet transmission not receive TT1 and TT2. Tetanus toxoid is
B. Airborne transmission the actual toxin produce by clostridium tetani
C. Vehicle transmission but on its WEAK and INACTIVATED form.
D. Vector borne transmission It is Artificial because it did not occur in the
course of actual illness or infection, it is
Answer: Droplet transmission Active because what has been passed is an
actual toxin and not a ready made
47. Considered as the first line of defense of immunoglobulin.
the body against infection
50. Agatha, was hacked and slashed by a
A. Skin psychotic man while she was crossing the
B. WBC railway. She suffered multiple injuries and
C. Leukocytes was injected Tetanus toxoid Immunoglobulin.
D. Immunization Agatha asked you, What immunity does TTIg
provides? You best answered her by saying
Answer: A. Skin. Remember that intact skin TTIg provides
and mucus membrane is our first line of
defense against infection. A. Natural active immunity
B. Natural passive immunity
48. All of the following contributes to host C. Artificial active immunity
susceptibility except D. Artificial passive immunity

A. Creed Answer: D. Artificial passive immunity. In


B. Immunization this scenario, Agatha was already wounded
C. Current medication being taken and has injuries. Giving the toxin [TT
D. Color of the skin Vaccine] itself would not help Agatha
because it will take time before the immune
Answer: A. Creed. Creed, Faith or religious system produce antitoxin. What agatha needs
belief do not affect person’s susceptibility to now is a ready made anti toxin in the form of
illness. Medication like corticosteroids could ATS or TTIg. This is artificial, because the
supress a person’s immune system that will body of agatha did not produce it. It is
lead to increase susceptibility. Color of the passive because her immune system is not
skin could affect person’s susceptibility to stimulated but rather, a ready made Immune
certain skin diseases. A dark skinned person globulin is given to immediately supress the
has lower risk of skin cancer than a fair invasion.
skinned person. Fair skinned person also has
a higher risk for cholecystitis and 51. This is the single most important
cholelithiasis. procedure that prevents cross contamination
and infection
49. Graciel has been injected TT5, her last
dosed for tetanus toxoid immunization. A. Cleaning
Graciel asked you, what type of immunity is B. Disinfecting
TT Injections? You correctly answer her by C. Sterilizing
saying Tetanus toxoid immunization is a/an D. Handwashing

A. Natural active immunity Answer: D. Handwashing. When you see the


B. Natural passive immunity word HANDWASHING as one of the
C. Artificial active immunity options, 90% Chance it is the correct answer
D. Artificial passive immunity in the local board. Or should I say, 100%
because I have yet to see question from 1988
to 2005 board questions that has option
HANDWASHING on it but is not the correct handwashing to effectively remove transient
answer. microorganisms.

52. This is considered as the most important 55. The minimum time in washing each hand
aspect of handwashing should never be below

A. Time A. 5 seconds
B. Friction B. 10 seconds
C. Water C. 15 seconds
D. Soap D. 30 seconds

Answer: B. Friction. The most important Answer:B. 10 seconds. According to Kozier,


aspect of handwashing is FRICTION. The rest, The minimum time required for watching
will just enhance friction. The use of soap each hands is 10 seconds and should not be
lowers the surface tension thereby increasing lower than that. The recommended time,
the effectiveness of friction. Water helps again, is 15 to 30 seconds.
remove transient bacteria by working with
soap to create the lather that reduces surface 56. How many ml of liquid soap is
tension. Time is of essence but friction is the recommended for handwashing procedure?
most essential aspect of handwashing
A. 1-2 ml
53. In handwashing by medical asepsis, Hands B. 2-3 ml
are held …. C. 2-4 ml
D. 5-10 ml
A. Above the elbow, The hands must always
be above the waist Answer: C. 2-4 ml. If a liquid soap is to be
B. Above the elbow, The hands are cleaner used, 1 tsp [ 5ml ] of liquid soap is
than the elbow recommended for handwashing procedure.
C. Below the elbow, Medical asepsis do not
require hands to be above the waist 57. Which of the following is not true about
D. Below the elbow, Hands are dirtier sterilization, cleaning and disinfection?
than the lower arms
A. Equipment with small lumen are easier
Answer: D. Below the elbow, Hands are to clean
dirtier than the lower arms. Hands are held B. Sterilization is the complete destruction of
BELOW the elbow in medical asepsis in all viable microorganism including spores
contrast with surgical asepsis, wherein, C. Some organism are easily destroyed,
nurses are required to keep the hands above while other, with coagulated protein
the waist. The rationale is because in medical requires longer time
asepsis, Hands are considered dirtier than the D. The number of organism is directly
elbow and therefore, to limit contamination proportional to the length of time required
of the lower arm, The hands should always for sterilization
be below the elbow.
Answer: A. Equipment with small lumen are
54. The suggested time per hand on easier to clean. Equipments with LARGE
handwashing using the time method is LUMEN are easier to clean than those with
small lumen. B C and D are all correct.
A. 5 to 10 seconds each hand
B. 10 to 15 seconds each hand 58. Karlita asked you, How long should she
C. 15 to 30 seconds each hand boil her glass baby bottle in water? You
D. 30 to 60 seconds each hand correctly answered her by saying

Answer: C. 15 to 30 seconds each hand. Each A. The minimum time for boiling articles is
hands requires atleast 15 to 30 seconds of 5 minutes
B. Boil the glass baby bottler and other ongoing efforts implented during the client’s
articles for atleast 10 minutes stay to remove or limit pathogens in his
C. For boiling to be effective, a minimum supplies, belongings, immediate environment
of 15 minutes is required in order to control the spread of the disease.
D. It doesn’t matter how long you boil the An example is cleaning the bedside
articles, as long as the water reached 100 commode of a client with radium implant on
degree Celsius her cervix with a bleach disinfectant after
each voiding.
Answer: C. For boiling to be effective, a
minimum of 15 minutes is required. Boiling 61. Which of the following is not true in
is the most common and least expensive implementing medical asepsis
method of sterilization used in home. For it
to be effective, you should boil articles for A. Wash hand before and after patient
atleast 15 minutes. contact
B. Keep soiled linens from touching the
59. This type of disinfection is best done in clothings
sterilizing drugs, foods and other things that C. Shake the linens to remove dust
are required to be sterilized before taken in by D. Practice good hygiene
the human body
Answer: C. Shake the linens to remove
A. Boiling Water dust. NEVER shake the linens. Once soiled,
B. Gas sterilization fold it inwards clean surface out. Shaking the
C. Steam under pressure linen will further spread pathogens that has
D. Radiation been harbored by the fabric.

Answer: D. Radiation. Imagine foods and 62. Which of the following is true about
drugs that are being sterilized by a boiling autoclaving or steam under pressure?
water, ethylene oxide gas and autoclave or
steam under pressure, They will be A. All kinds of microorganism and their
inactivated by these methods. Ethylene oxide spores are destroyed by autoclave
gas used in gas sterlization is TOXIC to machine
humans. Boiling the food will alter its B. The autoclaved instruments can be used
consistency and nutrients. Autoclaving the for 1 month considering the bags are still
food is never performed. Radiation using intact
microwave oven or Ionizing radiation C. The instruments are put into unlocked
penetrates to foods and drugs thus, sterilizing position, on their hinge, during the
them. autoclave
D. Autoclaving different kinds of metals at
60. A TB patient was discharged in the one time is advisable
hospital. A UV Lamp was placed in the room
where he stayed for a week. What type of Answer: C. The instruments are put into
disinfection is this? unlocked position, on their hinge, during the
autoclave. Only C is correct. Metals with
A. Concurrent disinfection locks, like clamps and scissors should be
B. Terminal disinfection UNLOCKED in order to minimize stiffening
C. Regular disinfection caused by autoclave to the hinges of these
D. Routine disinfection metals. NOT ALL microorganism are
destroyed by autoclaving. There are recently
Answer: B. Terminal disinfection. Terminal discovered microorganism that is
disinfection refers to practices to remove invulnarable to extreme heat. Autoclaved
pathogens that stayed in the belongings or instruments are to be used within 2 weeks.
immediate environemnt of an infected client Only the same type of metals should be
who has been discharged. An example would autoclaved as this will alteration in plating of
be Killing airborne TB Bacilli using UV these metals.
Light. Concurrent disinfection refers to
63. Which of the following is true about supposedly, it should be in her cervix. What
masks? should be your initial action?

A. Mask should only cover the nose A. Using a long forceps, Push it back
B. Mask functions better if they are wet with towards the cervix then call the physician
alcohol B. Wear gloves, remove it gently and place it
C. Masks can provide durable protection on a lead container
even when worn for a long time and after C. Using a long forceps, Remove it and
each and every patient care place it on a lead container
D. N95 Mask or particulate masks can D. Call the physician, You are not allowed to
filter organism as mall as 1 touch, re insert or remove it
micromillimeter
Answer: C. Using a long forceps, Remove it
Answer: D. N95 Mask or particulate masks and place it on a lead container. A dislodged
can filter organism as mall as 1 radioactive cervical implant in brachytherapy
micromillimeter. Mask should cover both are to be picked by a LONG FORCEP and
nose and mouth. Masks will not function stored in a LEAD CONTAINER in order to
optimally when wet. Masks should be worn prevent damage on the client’s normal tissue.
not greater than 4 hours, as it will lose Calling the physician is the second most
effectiveness after 4 hours. N95 mask or appropriate action among the choices. A
particulate mask can filter organism as small nurse should never attempt to put it back nor,
as 1 micromillimeter. touch it with her bare hands.

64. Where should you put a wet adult diaper? 67. After leech therapy, Where should you put
the leeches?
A. Green trashcan
B. Black trashcan A. In specially marked BIO HAZARD
C. Orange trashcan Containers
D. Yellow trashcan B. Yellow trashcan
C. Black trashcan
Answer: D. Yellow trashcan. Infectious D. Leeches are brought back to the culture
waste like blood and blood products, wet room, they are not thrown away for they
diapers and dressings are thrown in yellow are reusable
trashcans.
Answer: A. In specially marked BIO
65. Needles, scalpels, broken glass and lancets HAZARD Containers. Leeches, in leech
are considered as injurious wastes. As a nurse, therapy or LEECH PHLEBOTOMY are to
it is correct to put them at disposal via a/an be disposed on a BIO HAZARD container.
They are never re used as this could cause
A. Puncture proof container transfer of infection. These leeches are
B. Reused PET Bottles hospital grown and not the usual leeches
C. Black trashcan found in swamps.
D. Yellow trashcan with a tag “INJURIOUS
WASTES” 68. Which of the following should the nurse
AVOID doing in preventing spread of
Answer: A. Puncture proof infection?
container. Needles, scalpels and other sharps
are to be disposed in a puncture proof A. Recapping the needle before disposal to
container. prevent injuries
B. Never pointing a needle towards a body
66. Miranda Priestly, An executive of RAMP part
magazine, was diagnosed with cancer of the C. Using only Standard precaution to AIDS
cervix. You noticed that the radioactive Patients
internal implant protrudes to her vagina where
D. Do not give fresh and uncooked fruits and 71. A client has been diagnosed with
vegetables to Mr. Gatchie, with MEASLES. What precaution is used for this
Neutropenia patient?

Answer: A. Recapping the needle before A. Standard precaution


disposal to prevent injuries. Never recap B. Airborne precaution
needles. They are directly disposed in a C. Droplet precaution
puncture proof container after used. D. Contact precaution
Recapping the needles could cause injury to
the nurse and spread of infection. B C and D Answer: B. Airborne precaution. Measles is
are all appropriate. Standard precaution is highly communicable and more contagious
sufficient for an HIV patient. A client with than Rubella, It requires airborne precaution
neutropenia are not given fresh and uncooked as it is spread by small particle droplets that
fruits and vegetables for even the non remains suspended in air and disperesed by
infective organisms found in these foods air movements.
could cause severe infection on an
immunocompromised patients. 72. A client has been diagnosed with
IMPETIGO. What precaution is used for this
69. Where should you put Mr. Alejar, with patient?
Category II TB?
A. Standard precaution
A. In a room with positive air pressure and B. Airborne precaution
atleast 3 air exchanges an hour C. Droplet precaution
B. In a room with positive air pressure and D. Contact precaution
atleast 6 air exchanges an hour
C. In a room with negative air pressure and Answer:D. Contact precaution. Impetigo
atleast 3 air exchanges an hour causes blisters or sores in the skin. It is
D. In a room with negative air pressure generally caused by GABS or Staph Aureaus.
and atleast 6 air exchanges an hour It is spread by skin to skin contact or by
scratching the lesions and touching another
Answer: D. In a room with negative air person’s skin.
pressure and atleast 6 air exchanges an
hour. TB patients should have a private room 73. The nurse is to insert an NG Tube when
with negative air pressure and atleast 6 to 12 suddenly, she accidentally dip the end of the
air exhanges per hour. Negative pressure tube in the client’s glass containing distilled
room will prevent air inside the room from drinking water which is definitely not sterile.
escaping. Air exchanges are necessary since As a nurse, what should you do?
the client’s room do not allow air to get out
of the room. A. Don’t mind the incident, continue to
insert the NG Tube
70. A client has been diagnosed with B. Obtain a new NG Tube for the client
RUBELLA. What precaution is used for this C. Disinfect the NG Tube before reinserting
patient? it again
D. Ask your senior nurse what to do
A. Standard precaution
B. Airborne precaution Answer: A. Don’t mind the incident,
C. Droplet precaution continue to insert the NG Tube. The
D. Contact precaution digestive tract is not sterile, and therefore,
simple errors like this would not cause harm
Answer: C. Droplet precaution. Droplet to the patient. NGT tube need not be sterile,
precaution is sufficient on client’s with and so is colostomy and rectal tubes. Clean
RUBELLA or german measles. technique is sufficient during NGT and
colostomy care.
74. All of the following are principle of Answer: B. The non dominant hand. Gloves
SURGICAL ASEPSIS except are put on the non dominant hands first and
then, the dominant hand. The rationale is
A. Microorganism travels to moist surfaces simply because humans tend to use the
faster than with dry surfaces dominant hand first before the non dominant
B. When in doubt about the sterility of an hand. Out of 10 humans that will put on their
object, consider it not sterile sterile gloves, 8 of them will put the gloves
C. Once the skin has been sterilized, on their non dominant hands first.
considered it sterile
D. If you can reach the object by 77. As the scrubbed nurse, when should you
overreaching, just move around the sterile apply the goggles, shoe cap and mask prior to
field to pick it rather than reaching for it the operation?

Answer: C. Once the skin has been sterilized, A. Immediately after entering the sterile field
considered it sterile. Human skin is B. After surgical hand scrub
impossible to be sterilized. It contains normal C. Before surgical hand scrub
flora of microorganism. A B and D are all D. Before entering the sterile field
correct.
Answer: C. Before surgical hand scrub. The
75. Which of the following is true in nurse should put his goggles, cap and mask
SURGICAL ASEPSIS? prior to washing the hands. If he wash his
hands prior to putting all these equipments,
A. Autoclaved linens and gowns are he must wash his hands again as these
considered sterile for about 4 months as equipments are said to be UNSTERILE.
long as the bagging is intact
B. Surgical technique is a sole effort of each 78. Which of the following should the nurse
nurse do when applying gloves prior to a surgical
C. Sterile conscience, is the best method to procedure?
enhance sterile technique
D. If a scrubbed person leaves the area of the A. Slipping gloved hand with all fingers
sterile field, He/she must do handwashing when picking up the second glove
and gloving again, but the gown need not B. Grasping the first glove by inserting four
be changed fingers, with thumbs up underneath the
cuff
Answer:C. Sterile conscience, is the best C. Putting the gloves into the dominant hand
method to enhance sterile technique. Sterile first
conscience, or the moral imperative of a nurse D. Adjust only the fitting of the gloves
to be honest in practicing sterile technique, is after both gloves are on
the best method to enhance sterile technique.
Autoclaved linens are considered sterile only Answer: D. Adjust only the fitting of the
within 2 weeks even if the bagging is intact. gloves after both gloves are on. The nurse
Surgical technique is a team effort of each should only adjust fitting of the gloves when
nurse. If a scrubbed person leave the sterile they are both on the hands. Not doing so will
field and area, he must do the process all over break the sterile technique. Only 4 gingers
again. are slipped when picking up the second
76. In putting sterile gloves, Which should be gloves. You cannot slip all of your fingers as
gloved first? the cuff is limited and the thumb would not
be able to enter the cuff. The first glove is
A. The dominant hand grasp by simply picking it up with the first 2
B. The non dominant hand fingers and a thumb in a pinching motion.
C. The left hand Gloves are put on the non dominant hands
D. No specific order, Its up to the nurse for first.
her own convenience
79. Which gloves should you remove first?
A. The glove of the non dominant hand Answer: A. 4,3,5,1,2. When the nurse is
B. The glove of the dominant hand about to remove his protective devices, The
C. The glove of the left hand nurse will remove the GLOVES first
D. Order in removing the gloves Is followed by the MASK and GOWN then,
unnecessary other devices like cap, shoe cover, etc. This
is to prevent contamination of hair, neck and
Answer: A. The glove of the non dominant face area.
hand. Gloves are worn in the non dominant
hand first, and is removed also from the non 82. In pouring a plain NSS into a receptacle
dominant hand first. Rationale is simply located in a sterile field, how high should the
because in 10 people removing gloves, 8 of nurse hold the bottle above the receptacle?
them will use the dominant hand first and
remove the gloves of the non dominant hand. A. 1 inch
B. 3 inches
80. Before a surgical procedure, Give the C. 6 inches
sequence on applying the protective items D. 10 inches
listed below
Answer: C. 6 inches. Even if you do not
1. Eye wear or goggles know the answer to this question, you can
2. Cap answer it correctly by imagining. If you pour
3. Mask the NSS into a receptacle 1 to 3 inch above it,
4. Gloves Chances are, The mouth of the NSS bottle
5. Gown would dip into the receptacle as you fill it,
A. 3,2,1,5,4 making it contaminated. If you pour the NSS
B. 3,2,1,4,5 bottle into a receptacle 10 inches above it,
C. 2,3,1,5,4 that is too high, chances are, as you pour the
D. 2,3,1,4,5 NSS, most will spill out because the force
will be too much for the buoyant force to
Answer: D. 2,3,1,4,5. The nurse should handle. It will also be difficult to pour
use CaMEy Hand and Body Lotion in something precisely into a receptacle as the
moisturizing his hand before surgical height increases between the receptacle and
procedure and after handwashing. Ca stands the bottle. 6 inches is the correct answer. It is
for CAP, Mstands for MASK, Ey stands for not to low nor too high.
eye goggles. The nurse will do handwashing
and then [HAND], Don the gloves first and 83. The tip of the sterile forceps is considered
wear the Gown [BODY]. I created this sterile. It is used to manipulate the objects in
mnemonic and I advise you use it because the sterile field using the non sterile hands.
you can never forget Camey hand and body How should the nurse hold a sterile forceps?
lotion. [ Yes, I know it is spelled as
CAMAY ]] A. The tip should always be lower than the
handle
81. In removing protective devices, which B. The tip should always be above the
should be the exact sequence? handle
C. The handle and the tip should be at the
1. Eye wear or goggles same level
2. Cap D. The handle should point downward and
3. Mask the tip, always upward
4. Gloves
5. Gown Answer: A. The tip should always be lower
A. 4,3,5,1,2 than the handle. A sterile forcep is usually
B. 2,3,1,5,4 dipped into a disinfectant or germicidal
C. 5,4,3,2,1 solution. Imagine, if the tip is HIGHER than
D. 1,2,3,4,5’ the handle, the solution will go into the
handle and into your hands and as you use
the forcep, you will eventually lower its tip
making the solution in your hand go BACK Answer: C. 1,2,3. Cap, mask and shoe cover
into the tip thus contaminating the sterile area are worn BEFORE scrubbing.
of the forcep. To prevent this, the tip should
always be lower than the handle. In situation 86. When removing gloves, which of the
questions like this, IMAGINATION is very following is an inappropriate nursing action?
important.
A. Wash gloved hand first
84. The nurse enters the room of the client on B. Peel off gloves inside out
airborne precaution due to tuberculosis. C. Use glove to glove skin to skin technique
Which of the following are appropriate D. Remove mask and gown before
actions by the nurse? removing gloves

1. She wears mask, covering the nose and Answer: D. Remove mask and gown before
mouth removing gloves. Gloves are the dirtiest
2. She washes her hands before and after protective item nurses are wearing and
removing gloves, after suctioning the therefore, the first to be removed to prevent
client’s secretion spread of microorganism as you remove the
3. She removes gloves and hands before mask and gown.
leaving the client’s room
4. She discards contaminated suction 87. Which of the following is TRUE in the
catheter tip in trashcan found in the concept of stress?
clients room
A. 1,2 A. Stress is not always present in diseases
B. 1,2,3 and illnesses
C. 1,2,3,4 B. Stress are only psychological and
D. 1,3 manifests psychological symptoms
C. All stressors evoke common adaptive
Answer: C. 1,2,3,4. All soiled equipments response
use in an infectious client are disposed D. Hemostasis refers to the dynamic state of
INSIDE the client’s room to prevent equilibrium
contamination outside the client’s room. The
nurse is correct in using Mask the covers Answer: C. All stressors evoke common
both nose and mouth. Hands are washed adaptive response. All stressors evoke
before and after removing the gloves and common adaptive response. A psychologic
before and after you enter the client’s room. fear like nightmare and a real fear or real
Gloves and contaminated suction tip are perceive threat evokes common
thrown in trashcan found in the clients room. manifestation like tachycardia, tachypnea,
sweating, increase muscle tension etc. ALL
85. When performing surgical hand scrub, diseases and illness causes stress. Stress can
which of the following nursing action is be both REAL or IMAGINARY. Hemostasis
required to prevent contamination? refers to the ARREST of blood flowing
abnormally through a damage vessel.
1. Keep fingernail short, clean and with nail Homeostasis is the one that refers to dynamic
polish state of equilibrium according to Walter
2. Open faucet with knee or foot control Cannon.
3. Keep hands above the elbow when
washing and rinsing 88. According to this theorist, in his modern
4. Wear cap, mask, shoe cover after you stress theory, Stress is the non specific
scrubbed response of the body to any demand made
A. 1,2 upon it.
B. 2,3
C. 1,2,3 A. Hans Selye
D. 2,3,4 B. Walter Cannon
C. Claude Bernard
D. Martha Rogers
when more stressors are encountered by the
answer: A. Hans Selye. Hans Selye is the body. Man can encounter stress even while
only theorist who proposed an intriguing asleep, example is nightmare. Disease are
theory about stress that has been widely used multifactorial, No diseases are caused by a
and accepted by professionals today. He single stressors. Stress are sometimes
conceptualized two types of human response favorable and are not always a cause for
to stress, The GAS or general adaptation distress. An example of favorable stress is
syndrome which is characterized by stages of when a carpenter meets the demand and
ALARM, RESISTANCE and stress of everyday work. He then develops
EXHAUSTION. The Local adaptation calluses on the hand to lessen the pressure of
syndrome controls stress through a particular the hammer against the tissues of his hand.
body part. Example is when you have been He also develop larger muscle and more
wounded in your finger, it will produce dense bones in the arm, thus, a stress will
PAIN to let you know that you should protect lead to adaptations to decrease that particular
that particular damaged area, it will also stress.
produce inflammation to limit and control the
spread of injury and facilitate healing process. 91. Which of the following is TRUE in the
Another example is when you are frequently stage of alarm of general adaptation syndrome?
lifting heavy objects, eventually, you arm,
back and leg muscles hypertorphies to adapt A. Results from the prolonged exposure to
to the stress of heavy lifting. stress
B. Levels or resistance is increased
89. Which of the following is NOT TRUE C. Characterized by adaptation
with regards to the concept of Modern Stress D. Death can ensue
Theory?
Answer: D. Death can ensue. Death can
A. Stress is not a nervous energy ensue as early as the stage of alarm.
B. Man, whenever he encounters stresses, Exhaustion results to a prolonged exposure to
always adapts to it stress. Resistance is when the levels of
C. Stress is not always something to be resistance increases and characterized by
avoided being able to adapt.
D. Stress does not always lead to distress
92. The stage of GAS where the adaptation
Answer: B. Man, whenever he encounters mechanism begins
stresses, always adapts to it. Man, do not
always adapt to stress. Sometimes, stress can A. Stage of Alarm
lead to exhaustion and eventually, death. A,C B. Stage of Resistance
and D are all correct. C. Stage of Homeostasis
D. Stage of Exhaustion
90. Which of the following is TRUE with
regards to the concept of Modern Stress Answer: A. Stage of Alarm. Adaptation
Theory? mechanisms begin in the stage of alarm. This
is when the adaptive mechanism are
A. Stress is essential mobilized. When someone shouts SUNOG!!!
B. Man does not encounter stress if he is your heart will begin to beat faster, you
asleep vessels constricted and bp increased.
C. A single stress can cause a disease
D. Stress always leads to distress 93. Stage of GAS Characterized by adaptation

Answer:A. Stress is essential. Stress is A. Stage of Alarm


ESSENTIAL. No man can live normally B. Stage of Resistance
without stress. It is essential because it is C. Stage of Homeostasis
evoked by the body’s normal pattern of D. Stage of Exhaustion
response and leads to a favorable adaptive
mechanism that are utilized in the future Answer: Stage of Resistance
the hospital. Which of the following mode of
94. Stage of GAS wherein, the Level of adaptation is Andy experiencing?
resistance are decreased
A. Biologic/Physiologic adaptive mode
A. Stage of Alarm B. Psychologic adaptive mode
B. Stage of Resistance C. Sociocultural adaptive mode
C. Stage of Homeostasis D. Technological adaptive mode
D. Stage of Exhaustion
Answer: Technological adaptive mode
Answer: A. Stage of Alarm. Resistance are
decreased in the stage of alarm. Resistance is 99. Andy is not yet fluent in French, but he
absent in the stage of exhaustion. Resistance works in Quebec where majority speaks
is increased in the stage of resistance. French. He is starting to learn the language of
the people. What type of adaptation is Andy
95. Where in stages of GAS does a person experiencing?
moves back into HOMEOSTASIS?
A. Biologic/Physiologic adaptive mode
A. Stage of Alarm B. Psychologic adaptive mode
B. Stage of Resistance C. Sociocultural adaptive mode
C. Stage of Homeostasis D. Technological adaptive mode
D. Stage of Exhaustion
Answer: C. Sociocultural adaptive
Answer: Stage of Resistance mode. Sociocultural adaptive modes include
language, communication, dressing, acting
96. Stage of GAS that results from prolonged and socializing in line with the social and
exposure to stress. Here, death will ensue cultural standard of the people around the
unless extra adaptive mechanisms are utilized adapting individual.

A. Stage of Alarm 100. Andy made an error and his senior nurse
B. Stage of Resistance issued a written warning. Andy arrived in his
C. Stage of Homeostasis house mad and kicked the door hard to shut it
D. Stage of Exhaustion off. What adaptation mode is this?

Answer:Stage of Exhaustion A. Biologic/Physiologic adaptive mode


B. Psychologic adaptive mode
97. All but one is a characteristic of adaptive C. Sociocultural adaptive mode
response D. Technological adaptive mode

A. This is an attempt to maintain Answer: Psychologic adaptive mode


homeostasis
B. There is a totality of response PNLE: FON Practice Exam for Stress,
C. Adaptive response is immediately Crisis, Crisis Intervention,
mobilized, doesn’t require time
D. Response varies from person to person
Communication, Recording, Learning
and Documentation
Answer: C. Adaptive response is 1. The coronary vessels, unlike any other
immediately mobilized, doesn’t require blood vessels in the body, respond to
time. Aside from having limits that leads to sympathetic stimulation by
exhaustion. Adaptive response requires time
for it to act. It requires energy, physical and A. Vasoconstriction
psychological taxes that needs time for our B. Vasodilatation
body to mobilize and utilize. C. Decreases force of contractility
D. Decreases cardiac output
98. Andy, a newly hired nurse, starts to learn
the new technology and electronic devices at Answer: Vasodilatation
C. Purulent
2. What stress response can you expect from a D. Sanguinous
patient with blood sugar of 50 mg / dl?
Answer: Purulent
A. Body will try to decrease the glucose
level 7. The first manifestation of inflammation is
B. There will be a halt in release of sex
hormones A. Redness on the affected area
C. Client will appear restless B. Swelling of the affected area
D. Blood pressure will increase C. Pain, which causes guarding of the area
D. Increase heat due to transient vasodilation
Answer: Blood pressure will increase
Answer: Redness on the affected area
3. All of the following are purpose of
inflammation except
8. The client has a chronic tissue injury. Upon
A. Increase heat, thereby produce examining the client’s antibody for a
abatement of phagocytosis particular cellular response, Which of the
B. Localized tissue injury by increasing following WBC component is responsible for
capillary permeability phagocytosis in chronic tissue injury?
C. Protect the issue from injury by producing
pain A. Neutrophils
D. Prepare for tissue repair B. Basophils
C. Eosinophils
Answer: A. Increase heat, thereby produce D. Monocytes
abatement of phagocytosis
Answer: Monocytes
4. The initial response of tissue after injury is
9. Which of the following WBC component
A. Immediate Vasodilation proliferates in cases of Anaphylaxis?
B. Transient Vasoconstriction
C. Immediate Vasoconstriction A. Neutrophils
D. Transient Vasodilation B. Basophils
C. Eosinophil
Answer: C. Immediate Vasoconstriction D. Monocytes

5. The last expected process in the stages of Answer: Eosinophil


inflammation is characterized by
10. Icheanne, ask you, her Nurse, about WBC
A. There will be sudden redness of the Components. She got an injury yesterday after
affected part she twisted her ankle accidentally at her
B. Heat will increase on the affected part gymnastic class. She asked you, which WBC
C. The affected part will loss its normal Component is responsible for proliferation at
function the injured site immediately following an
D. Exudates will flow from the injured site injury. You answer:

Answer: . The affected part will loss its A. Neutrophils


normal function B. Basophils
C. Eosinophils
6. What kind of exudates is expected when D. Monocytes
there is an antibody-antigen reaction as a
result of microorganism infection? Answer: Neutrophils

A. Serous
B. Serosanguinous
11. Icheanne then asked you, what is the first B. Second intention
process that occurs in the inflammatory C. Third intention
response after injury, You tell her: D. Fourth intention

A. Phagocytosis Answer: Second intention


B. Emigration
C. Pavementation 16. Imelda is in the recovery stage after the
D. Chemotaxis incident. As a nurse, you know that the diet
that will be prescribed to Miss Imelda is
Answer: Pavementation
A. Low calorie, High protein with Vitamin A
12. Icheanne asked you again, What is that and C rich foods
term that describes the magnetic attraction of B. High protein, High calorie with
injured tissue to bring phagocytes to the site Vitamin A and C rich foods
of injury? C. High calorie, Low protein with Vitamin A
and C rich foods
A. Icheanne, you better sleep now, you asked D. Low calorie, Low protein with Vitamin A
a lot of questions and C rich foods
B. It is Diapedesis
C. We call that Emigration B. High protein, High calorie with Vitamin A
D. I don’t know the answer, perhaps I can and C rich foods
tell you after I find it out later
17. Miss Imelda asked you, What is WET TO
Answer: D. I don’t know the answer, perhaps DRY Dressing method? Your best response is
I can tell you after I find it out later
A. It is a type of mechanical debridement
13. This type of healing occurs when there is a using Wet dressing that is applied and
delayed surgical closure of infected wound left to dry to remove dead tissues
B. It is a type of surgical debridement with
A. First intention the use of Wet dressing to remove the
B. Second intention necrotic tissues
C. Third intention C. It is a type of dressing where in, The
D. Fourth intention wound is covered with Wet or Dry
dressing to prevent contamination
Answer: Third intention D. It is a type of dressing where in, A
cellophane or plastic is placed on the
14. Type of healing when scars are minimal wound over a wet dressing to stimulate
due to careful surgical incision and good healing of the wound in a wet medium
healing
Answer: A. It is a type of mechanical
A. First intention debridement using Wet dressing that is
B. Second intention applied and left to dry to remove dead tissues
C. Third intention
D. Fourth intention 18. The primary cause of pain in inflammation
is
Answer: First intention
A. Release of pain mediators
15. Imelda, was slashed and hacked by an B. Injury to the nerve endings
unknown suspects. She suffered massive C. Compression of the local nerve endings
tissue loss and laceration on her arms and by the edema fluids
elbow in an attempt to evade the criminal. As D. Circulation is lessen, Supply of oxygen is
a nurse, you know that the type of healing that insufficient
will most likely occur to Miss Imelda is
Answer: C. Compression of the local nerve
A. First intention endings by the edema fluids
Answer:A. Directed towards helping an
19. The client is in stress because he was told individual both physically and emotionally
by the physician he needs to undergo surgery
for removal of tumor in his bladder. Which of 23. According to her, A nurse patient
the following are effects of sympatho-adreno- relationship is composed of 4 stages :
medullary response by the client? Orientation, Identification, Exploitation and
Resolution
1. Constipation
2. Urinary frequency A. Roy
3. Hyperglycemia B. Peplau
4. Increased blood pressure C. Rogers
A. 3,4 D. Travelbee
B. 1,3,4
C. 1,2,4 Answer: Peplau
D. 1,4
24. In what phase of Nurse patient
Answer: B. 1,3,4 relationship does a nurse review the client’s
medical records thereby learning as much as
20. The client is on NPO post midnight. possible about the client?
Which of the following, if done by the client,
is sufficient to cancel the operation in the A. Pre Orientation
morning? B. Orientation
C. Working
A. Eat a full meal at 10:00 P.M D. Termination
B. Drink fluids at 11:50 P.M
C. Brush his teeth the morning before Answer: Pre Orientation
operation
D. Smoke cigarette around 3:00 A.M 25. Nurse Aida has seen her patient, Roger for
the first time. She establish a contract about
Answer: D. Smoke cigarette around 3:00 the frequency of meeting and introduce to
A.M Roger the expected termination. She started
taking baseline assessment and set
21. The client place on NPO for preparation interventions and outcomes. On what phase of
of the blood test. Adreno-cortical response is NPR Does Nurse Aida and Roger belong?
activated and which of the following below is
an expected response? A. Pre Orientation
B. Orientation
A. Low BP C. Working
B. Decrease Urine output D. Termination
C. Warm, flushed, dry skin
D. Low serum sodium levels Answer: orientation

Answer: Decrease Urine output 26. Roger has been seen agitated, shouting
and running. As Nurse Aida approaches, he
22. Which of the following is true about shouts and swear, calling Aida names. Nurse
therapeutic relationship? Aida told Roger “That is an unacceptable
behavior Roger, Stop and go to your room
A. Directed towards helping an individual now.” The situation is most likely in what
both physically and emotionally phase of NPR?
B. Bases on friendship and mutual trust
C. Goals are set by the solely nurse A. Pre Orientation
D. Maintained even after the client doesn’t B. Orientation
need anymore of the Nurse’s help C. Working
D. Termination
Answer: working A. It has physiologic component
B. It has psychologic component
27. Nurse Aida, in spite of the incident, still C. The source of dread or uneasiness is from
consider Roger as worthwhile simply because an unrecognized entity
he is a human being. What major ingredient of D. The source of dread or uneasiness is
a therapeutic communication is Nurse Aida from a recognized entity
using?
Answer: D. The source of dread or
A. Empathy uneasiness is from a recognized entity
B. Positive regard
C. Comfortable sense of self 32. Lorraine, a 27 year old executive was
D. Self awareness brought to the ER for an unknown reason. She
is starting to speak but her speech is
Answer: Positive regard disorganized and cannot be understood. On
what level of anxiety does this features
28. Nurse Irma saw Roger and told Nurse belongs?
Aida “ Oh look at that psychotic patient
“ Nurse Aida should intervene and correct A. Mild
Nurse Irma because her statement shows that B. Moderate
she is lacking? C. Severe
D. Panic
A. Empathy
B. Positive regard Answer: Panic
C. Comfortable sense of self
D. Self awareness 33. Elton, 21 year old nursing student is
taking the board examination. She is sweating
Answer: Positive regard profusely, has decreased awareness of his
environment and is purely focused on the
29. Which of the following statement is not exam questions characterized by his selective
true about stress? attentiveness. What anxiety level is Elton
exemplifying?
A. It is a nervous energy
B. It is an essential aspect of existence A. Mild
C. It has been always a part of human B. Moderate
experience C. Severe
D. It is something each person has to cope D. Panic

Answer: A. It is a nervous energy Answer: Moderate

30. Martina, a Tennis champ was devastated 34. You noticed the patient chart : ANXIETY
after many new competitors outpaced her in +3 What will you expect to see in this client?
the Wimbledon event. She became depressed
and always seen crying. Martina is clearly on A. An optimal time for learning, Hearing and
what kind of situation? perception is greatly increased
B. Dilated pupils
A. Martina is just stressed out C. Unable to communicate
B. Martina is Anxious D. Palliative Coping Mechanism
C. Martina is in the exhaustion stage of GAS
D. Martina is in Crisis Answer: B. Dilated pupils

Answer: D. Martina is in Crisis 35. When should the nurse starts giving
XANAX?
31. Which of the following statement is not
true with regards to anxiety? A. When anxiety is +1
B. When the client starts to have a narrow D. Exercise combined with meditation to
perceptual field and selective foster relaxation and mental alacrity
inattentiveness
C. When problem solving is not possible Answer: B. Concentrating on breathing
D. When the client is immobile and without tensing the muscle, Letting go and
disorganized repeating a word or sound after each
exhalation
Answer: B. When the client starts to have a
narrow perceptual field and selective 40. What type of relaxation technique does
inattentiveness Lyza uses if a machine is showing her pulse
rate, temperature and muscle tension which
36. Which of the following behavior is not a she can visualize and assess?
sign or a symptom of Anxiety?
A. Biofeedback
A. Frequent hand movement B. Massage
B. Somatization C. Autogenic training
C. The client asks a question D. Visualization and Imagery
D. The client is acting out
Answer: biofeedback
Answer: C. The client asks a question
41. This is also known as Self-suggestion or
37. Which of the following intervention is Self-hypnosis
inappropriate for client’s with anxiety?
A. Biofeedback
A. Offer choices B. Meditation
B. Provide a quiet and calm environment C. Autogenic training
C. Provide detailed explanation on each and D. Visualization and Imagery
every procedures and equipments
D. Bring anxiety down to a controllable level Answer:Autogenic training

Answer:Offer choices 42. Which among these drugs is NOT an


anxiolytic?
38. Which of the following statement, if made
by the nurse, is considered not therapeutic? A. Valium
B. Ativan
A. “How did you deal with your anxiety C. Milltown
before?” D. Luvox
B. “It must be awful to feel anxious.”
C. “How does it feel to be anxious?” Answer: Luvox
D. “What makes you feel anxious?”
43. Kenneth, 25 year old diagnosed with HIV
Answer: D. “What makes you feel anxious?” felt that he had not lived up with God’s
expectation. He fears that in the course of his
39. Marissa Salva, Uses Benson’s relaxation. illness, God will be punitive and not be
How is it done? supportive. What kind of spiritual crisis is
Kenneth experiencing?
A. Systematically tensing muscle groups
from top to bottom for 5 seconds, and 1. Spiritual Pain
then releasing them 2. Spiritual Anxiety
B. Concentrating on breathing without 3. Spiritual Guilt
tensing the muscle, Letting go and 4. Spiritual Despair
repeating a word or sound after each A. 1,2
exhalation B. 2,3
C. Using a strong positive, feeling-rich C. 3,4
statement about a desired change D. 1,4
you difficulty sleeping?” Which therapeutic
Answer: technique is used in this situation?
44. Grace, believes that her relationship with
God is broken. She tried to go to church to ask A. Reflecting
forgiveness everyday to remedy her feelings. B. Restating
What kind of spiritual distress is Grace C. Exploring
experiencing? D. Seeking clarification
50. Myra told Budek “I cannot sleep, I stay
A. Spiritual Pan away all night” Budek told her “You have
B. Spiritual Alienation difficulty sleeping” This is what type of
C. Spiritual Guilt therapeutic communication technique?
D. Spiritual Despair
45. Remedios felt “EMPTY” She felt that she A. Reflecting
has already lost God’s favor and love because B. Restating
of her sins. This is a type of what spiritual C. Exploring
crisis? D. Seeking clarification
51. Myra said “I saw my dead grandmother
A. Spiritual Anger here at my bedside a while ago” Budek
B. Spiritual Loss responded “Really? That is hard to believe,
C. Spiritual Despair How do you feel about it?” What technique
D. Spiritual Anxiety did Budek used?
46. Budek is working with a schizophrenic
patient. He noticed that the client is agitated, A. Disproving
pacing back and forth, restless and B. Disagreeing
experiencing Anxiety +3. Budek said “You C. Voicing Doubt
appear restless” What therapeutic technique D. Presenting Reality
did Budek used? 52. Which of the following is a therapeutic
communication in response to “I am a GOD,
A. Offering general leads bow before me Or ill summon the dreaded
B. Seeking clarification thunder to burn you and purge you to pieces!”
C. Making observation
D. Encouraging description of perception A. “You are not a GOD, you are Professor
47. Rommel told Budek “ I SEE DEAD Tadle and you are a PE Teacher, not a
PEOPLE “ Budek responded “You see dead Nurse. I am Glen, Your nurse.”
people?” This Is an example of therapeutic B. “Oh hail GOD Tadle, everyone bow or
communication technique? face his wrath!”
C. “Hello Mr. Tadle, You are here in the
A. Reflecting hospital, I am your nurse and you are a
B. Restating patient here”
C. Exploring D. “How can you be a GOD Mr. Tadle? Can
D. Seeking clarification you tell me more about it?”
48. Rommel told Budek, “Do you think Im 53. Erik John Senna, Told Nurse Budek “ I
crazy?” Budek responded, “Do you think your don’t want to that, I don’t want that thing..
crazy?” Budek uses what example of that’s too painful!” Which of the following
therapeutic communication? response is NON THERAPEUTIC

A. Reflecting A. “ This must be difficult for you, But I


B. Restating need to inject you this for your own good”
C. Exploring B. “ You sound afraid”
D. Seeking clarification C. “Are you telling me you don’t want this
49. Myra, 21 year old nursing student has injection?”
difficulty sleeping. She told Nurse Budek “I D. “Why are you so anxious? Please tell
really think a lot about my x boyfriend me more about your feelings Erik”
recently” Budek told Myra “And that causes 54. Legrande De Salvaje Y Cobrador La
Jueteng, was caught by the bacolod police
because of his illegal activities. When he got lives. The people affected by the Tsunami are
home after paying for the bail, He shouted at saddened and do not know how to start all
his son. What defense mechanism did Mr. La over again. What type of crisis is this?
Jueteng used?
A. Situational
A. Restitution B. Maturational
B. Projection C. Social
C. Displacement D. Phenomenal
D. Undoing 61. Which of the following is the BEST goal
55. Later that day, he bought his son ice for crisis intervention?
cream and food. What defense mechanism is
Legrande unconsciously doing? A. Bring back the client in the pre crisis state
B. Make sure that the client becomes better
A. Restitution C. Achieve independence
B. Conversion D. Provide alternate coping mechanism
C. Redoing 62. What is the best intervention when the
D. Reaction formation client has just experienced the crisis and still
56. Crisis is a sudden event in ones life that at the first phase of the crisis?
disturbs a person’s homeostasis. Which of the
following is NOT TRUE in crisis? A. Behavior therapy
B. Gestalt therapy
A. The person experiences heightened C. Cognitive therapy
feeling of stress D. Milieu Therapy
B. Inability to function in the usual 63. Therapeutic nurse client relationship is
organized manner describes as follows
C. Lasts for 4 months
D. Indicates unpleasant emotional feelings 1. Based on friendship and mutual interest
57. Which of the following is a characteristic 2. It is a professional relationship
of crisis? 3. It is focused on helping the patient solve
problems and achieve health-related goals
A. Lasts for an unlimited period of time 4. Maintained only as long as the patient
B. There is a triggering event requires professional helpA. 1,2,3
C. Situation is not dangerous to the person B. 1,2,4
D. Person totality is not involved C. 2,3,4
58. Levito Devin, The Italian prime minister, D. 1,3,4
is due to retire next week. He feels depressed 64. The client is scheduled to have surgical
due to the enormous loss of influence, power, removal of the tumor on her left breast. Which
fame and fortune. What type of crisis is Devin of the following manifestation indicates that
experiencing? she is experiencing Mild Anxiety?

A. Situational A. She has increased awareness of her


B. Maturational environmental details
C. Social B. She focused on selected aspect of her
D. Phenomenal illness
59. Estrada, The Philippine president, has C. She experiences incongruence of action,
been unexpectedly impeached and was out of thoughts and feelings
office before the end of his term. He is in D. She experiences random motor activities
what type of crisis? 65. Which of the following nursing
intervention would least likely be effective
A. Situational when dealing with a client with aggressive
B. Maturational behavior?
C. Social
D. Phenomenal A. Approach him in a calm manner
60. The tsunami in Thailand and Indonesia B. Provide opportunities to express
took thousands of people and change million feelings
C. Maintain eye contact with the client A. POMR
D. Isolate the client from others B. POR
66. Whitney, a patient of nurse Budek, C. Traditional
verbalizes… “I have nothing, nothing… D. Resource oriented
nothing! Don’t make me close one more door, 72. Type of recording that integrates all data
I don’t wanna hurt anymore!” Which of the about the problem, gathered by members of
following is the most appropriate response by the health team.
Budek?
A. POMR
A. Why are you singing? B. Traditional
B. What makes you say that? C. Resource oriented
C. Ofcourse you are everything! D. Source oriented
D. What is that you said? 73. These are data that are monitored by using
67. Whitney verbalizes that she is anxious that graphic charts or graphs that indicated the
the diagnostic test might reveal laryngeal progression or fluctuation of client’s
cancer. Which of the following is the most Temperature and Blood pressure.
appropriate nursing intervention?
A. Progress notes
A. Tell the client not to worry until the B. Kardex
results are in C. Flow chart
B. Ask the client to express feelings and D. Flow sheet
concern 74. Provides a concise method of organizing
C. Reassure the client everything will be and recording data about the client. It is a
alright series of flip cards kept in portable file used in
D. Advice the client to divert his attention by change of shift reports.
watching television and reading
newspapers A. Kardex
68. Considered as the most accurate B. Progress Notes
expression of person’s thought and feelings C. SOAPIE
D. Change of shift report
A. Verbal communication 75. You are about to write an information on
B. Non verbal communication the Kardex. There are 4 available writing
C. Written communication instruments to use. Which of the following
D. Oral communication should you use?
69. Represents inner feeling that a person do
not like talking about. A. Mongol #2
B. Permanent Ink
A. Overt communication C. A felt or fountain pen
B. Covert communication D. Pilot Pentel Pen marker
C. Verbal communication 76. The client has an allergy to Iodine based
D. Non verbal communication dye. Where should you put this vital
70. Which of the following is NOT a information in the client’s chart?
characteristic of an effective Nurse-Client
relationship? A. In the first page of the client’s chart
B. At the last page of the client’s chart
A. Focused on the patient C. At the front metal plate of the chart
B. Based on mutual trust D. In the Kardex
C. Conveys acceptance 77. Which of the following is NOT TRUE
D. Discourages emotional bond about the Kardex
71. A type of record wherein , each person or
department makes notation in separate records. A. It provides readily available information
A nurse will use the nursing notes, The doctor B. It is a tool of end of shift reports
will use the Physician’s order sheet etc. Data C. The primary basis of endorsement
is arranged according to information source. D. Where Allergies information are written
78. Which of the following, if seen on the C. Marsha said “ I understand “ after you a
Nurses notes, violates characteristic of good health teaching about family planning
recording? D. John rated 100% on your given quiz about
smoking and alcoholism
A. The client has a blood pressure of
120/80, Temperature of 36.6 C Pulse Answer: A. Matuts starts exercising every
rate of 120 and Respiratory rate of 22 morning and eating a balance diet after you
B. Ate 50% of food served taught her mag HL tayo program
C. Refused administration of betaxolol
D. Visited and seen By Dr. Santiago 85. In his theory of learning as a
79. The physician ordered : Mannerix a.c , BEHAVIORISM, he stated that transfer of
what does a.c means? knowledge occurs if a new situation closely
resembles an old one.
A. As desired
B. Before meals A. Bloom
C. After meals B. Lewin
D. Before bed time C. Thorndike
80. The physician ordered, Maalox, 2 hours D. Skinner
p.c, what does p.c means? 86. Which of the following is TRUE with
regards to learning?
A. As desired
B. Before meals A. Start from complex to simple
C. After meals B. Goals should be hard to achieve so patient
D. Before bed time can strive to attain unrealistic goals
81. The physician ordered, Maxitrol, Od. C. Visual learning is the best for every
What does Od means? individual
D. Do not teach a client when he is in pain
A. Left eye 87. According to Bloom, there are 3 domains
B. Right eye in learning. Which of these domains is
C. Both eye responsible for the ability of Donya Delilah to
D. Once a day inject insulin?
82. The physician orderd, Magnesium
Hydroxide cc Aluminum Hydroxide. What A. Cognitive
does cc means? B. Affective
C. Psychomotor
A. without D. Motivative
B. with 88. Which domains of learning is responsible
C. one half for making John and Marsha understand the
D. With one half dose different kinds of family planning methods?
83. Physician ordered, Paracetamol tablet ss.
What does ss means? A. Cognitive
B. Affective
A. without C. Psychomotor
B. with D. Motivative
C. one half 89. Which of the following statement clearly
D. With one half dose defines therapeutic communication?
84. Which of the following indicates that
learning has been achieved? A. Therapeutic communication is an
interaction process which is primarily
A. Matuts starts exercising every morning directed by the nurse
and eating a balance diet after you B. It conveys feeling of warmth, acceptance
taught her mag HL tayo program and empathy from the nurse to a patient in
B. Donya Delilah has been able to repeat the relaxed atmosphere
steps of insulin administration after you C. Therapeutic communication is a
taught it to her reciprocal interaction based on trust
and aimed at identifying patient needs A. Detailed explanation
and developing mutual goals B. Demonstration
D. Therapeutic communication is an C. Use of pamphlets
assessment component of the nursing D. Film showing
process 95. What is the most important characteristic
of a nurse patient relationship?
Answer:C. Therapeutic communication is a
reciprocal interaction based on trust and A. It is growth facilitating
aimed at identifying patient needs and B. Based on mutual understanding
developing mutual goals C. Fosters hope and confidence
D. Involves primarily emotional bond
90. Which of the following concept is most 96. Which of the following nursing
important in establishing a therapeutic nurse intervention is needed before teaching a client
patient relationship? post spleenectomy deep breathing and
coughing exercises?
A. The nurse must fully understand the
patient’s feelings, perception and A. Tell the patient that deep breathing
reactions before goals can be established and coughing exercises is needed to
B. The nurse must be a role model for health promote good breathing, circulation
fostering behavior and prevent complication
C. The nurse must recognize that the patient B. Tell the client that deep breathing and
may manifest maladaptive behavior after coughing exercises is needed to prevent
illness Thrombophlebitis, hydrostatic pneumonia
D. The nurse should understand that and atelectasis
patients might test her before trust is C. Medicate client for pain
established D. Tell client that cooperation is vital to
91. Which of the following communication improve recovery
skill is most effective in dealing with covert 97. The client has an allergy with penicillin.
communication? What is the best way to communicate this
information?
A. Validation
B. Listening A. Place an allergy alert in the Kardex
C. Evaluation B. Notify the attending physician
D. Clarification C. Write it on the patient’s chart
92. Which of the following are qualities of a D. Take note when giving medications
good recording? 98. An adult client is on extreme pain. He is
moaning and grimacing. What is the best way
1. Brevity to assess the client’s pain?
2. Completeness and chronology
3. Appropriateness A. Perform physical assessment
4. Accuracy B. Have the client rate his pain on the smiley
A. 1,2 pain rating scale
B. 3,4 C. Active listening on what the patient
C. 1,2,3 says
D. 1,2,3,4 D. Observe the client’s behavior
93. All of the following chart entries are 99. Therapeutic communication begins with?
correct except
A. Knowing your client
A. V/S 36.8 C,80,16,120/80 B. Knowing yourself
B. Complained of chest pain C. Showing empathy
C. Seems agitated D. Encoding
D. Able to ambulate without assistance 100. The PCS gave new guidelines including
94. Which of the following teaching method is leaflets to educate cancer patients. As a nurse,
effective in client who needs to be educated When using materials like this, what is your
about self injection of insulin? responsibility?
A. Read it for the patient B. Humanistic
B. Give it for the patient to read himself C. Efficient
C. Let the family member read the material D. Effective
for the patient 6. A characteristic of the nursing process that
D. Read it yourself then, Have the client is essential to promote client satisfaction and
read the material progress. The care should also be relevant
with the client’s needs.
PNLE: FON Practice Exam for A. Organized and Systematic
Nursing Process, Physical and Health B. Humanistic
Assessment and Routine Procedures C. Efficient
1. She is the first one to coin the term D. Effective
“NURSING PROCESS” She introduced 3 7. Rhina, who has Menieres disease, said that
steps of nursing process which are her environment is moving. Which of the
Observation, Ministration and Validation. following is a valid assessment?

A. Nightingale 1. Rhina is giving an objective data


B. Johnson 2. Rhina is giving a subjective data
C. Rogers 3. The source of the data is primary
D. Hall 4. The source of the data is secondary
2. The American Nurses association A. 1,3
formulated an innovation of the Nursing B. 2,3
process. Today, how many distinct steps are C. 2.4
there in the nursing process? D. 1,4
8. Nurse Angela, observe Joel who is very
A. APIE – 4 apprehensive over the impending operation.
B. ADPIE – 5 The client is experiencing dyspnea,
C. ADOPIE – 6 diaphoresis and asks lots of questions. Angela
D. ADOPIER – 7 made a diagnosis of ANXIETY R/T
3. They are the first one to suggest a 4 step INTRUSIVE PROCEDURE. This is what
nursing process which are : APIE , or type of Nursing Diagnosis?
assessment, planning, implementation and
evaluation. A. Actual
B. Probable
1. Yura C. Possible
2. Walsh D. Risk
3. Roy 9. Nurse Angela diagnosed Mrs. Delgado,
4. Knowles who have undergone a BKA. Her diagnosis is
A. 1,2 SELF ESTEEM DISTURBANCE R/T
B. 1,3 CHANGE IN BODY IMAGE. Although the
C. 3,4 client has not yet seen her lost leg, Angela
D. 2,3 already anticipated the diagnosis. This is what
4. Which characteristic of nursing process is type of Diagnosis?
responsible for proper utilization of human
resources, time and cost resources? A. Actual
B. Probable
A. Organized and Systematic C. Possible
B. Humanistic D. Risk
C. Efficient 10. Nurse Angela is about to make a diagnosis
D. Effective but very unsure because the S/S the client is
5. Which characteristic of nursing process experiencing is not specific with her diagnosis
addresses the INDIVIDUALIZED care a of POWERLESSNESS R/T DIFFICULTY
client must receive? ACCEPTING LOSS OF LOVED ONE. She
then focus on gathering data to refute or prove
A. Organized and Systematic her diagnosis but her plans and interventions
are already ongoing for the diagnosis. Which D. Example is : After discharge planning,
type of Diagnosis is this? Client demonstrated the proper
psychomotor skills for insulin injection.
A. Actual 16. Which of the following is a NOT a correct
B. Probable statement of an Outcome criteria?
C. Possible
D. Risk A. Ambulates 30 feet with a cane before
11. Nurse Angela knew that Stephen Lee Mu discharge
Chin, has just undergone an operation with an B. Discusses fears and concerns regarding
incision near the diaphragm. She knew that the surgical procedure
this will contribute to some complications C. Demonstrates proper coughing and
later on. She then should develop what type of breathing technique after a teaching
Nursing diagnosis? session
D. Reestablishes a normal pattern of
A. Actual elimination
B. Probable 17. Which of the following is a OBJECTIVE
C. Possible data?
D. Risk
12. Which of the following Nursing diagnosis A. Dizziness
is INCORRECT? B. Chest pain
C. Anxiety
A. Fluid volume deficit R/T Diarrhea D. Blue nails
B. High risk for injury R/T Absence of 18. A patient’s chart is what type of data
side rails source?
C. Possible ineffective coping R/T Loss of
loved one A. Primary
D. Self esteem disturbance R/T Effects of B. Secondary
surgical removal of the leg C. Tertiary
13. Among the following statements, which D. Can be A and B
should be given the HIGHEST priority? 19. All of the following are characteristic of
the Nursing process except
A. Client is in extreme pain
B. Client’s blood pressure is 60/40 A. Dynamic
C. Client’s temperature is 40 deg. Centigrade B. Cyclical
D. Client is cyanotic C. Universal
14. Which of the following need is given a D. Intrapersonal
higher priority among others? 20. Which of the following is true about the
NURSING CARE PLAN?
A. The client has attempted suicide and
safety precaution is needed A. It is nursing centered
B. The client has disturbance in his body B. Rationales are supported by interventions
image because of the recent operation C. Verbal
C. The client is depressed because her D. Atleast 2 goals are needed for every
boyfriend left her all alone nursing diagnosis
D. The client is thirsty and dehydrated 21. A framework for health assessment that
15. Which of the following is TRUE with evaluates the effects of stressors to the mind,
regards to Client Goals? body and environment in relation with the
ability of the client to perform ADL.
A. They are specific, measurable, attainable
and time bounded A. Functional health framework
B. They are general and broadly stated B. Head to toe framework
C. They should answer for WHO, WHAT C. Body system framework
ACTIONS, WHAT CIRCUMSTANCES, D. Cephalocaudal framework
HOW WELL and WHEN.
22. Client has undergone Upper GI and Lower 28. John has a fever of 38.5 Deg. Celsius. It
GI series. Which type of health assessment surges at around 40 Degrees and go back to
framework is used in this situation? 38.5 degrees 6 times today in a typical pattern.
What kind of fever is John having?
A. Functional health framework
B. Head to toe framework A. Relapsing
C. Body system framework B. Intermittent
D. Cephalocaudal framework C. Remittent
23. Which of the following statement is true D. Constant
regarding temperature? 29. John has a fever of 39.5 degrees 2 days
ago, But yesterday, he has a normal
A. Oral temperature is more accurate than temperature of 36.5 degrees. Today, his
rectal temperature temperature surges to 40 degrees. What type
B. The bulb used in Rectal temperature of fever is John having?
reading is pear shaped or round
C. The older the person, the higher his BMR A. Relapsing
D. When the client is swimming, BMR B. Intermittent
Decreases C. Remittent
24. A type of heat loss that occurs when the D. Constant
heat is dissipated by air current 30. John’s temperature 10 hours ago is a
normal 36.5 degrees. 4 hours ago, He has a
A. Convection fever with a temperature of 38.9 Degrees.
B. Conduction Right now, his temperature is back to normal.
C. Radiation Which of the following best describe the fever
D. Evaporation john is having?
25. Which of the following is TRUE about
temperature? A. Relapsing
B. Intermittent
A. The highest temperature usually occurs C. Remittent
later in a day, around 8 P.M to 12 M.N D. Constant
B. The lowest temperature is usually in the 31. The characteristic fever in Dengue Virus
Afternoon, Around 12 P.M is characterized as:
C. Thyroxin decreases body temperature
D. Elderly people are risk for hyperthermia A. Tricyclic
due to the absence of fats, Decreased B. Bicyclic
thermoregulatory control and sedentary C. Biphasic
lifestyle. D. Triphasic
26. Hyperpyrexia is a condition in which the 32. When John has been given paracetamol,
temperature is greater than his fever was brought down dramatically from
40 degrees Celsius to 36.7 degrees in a matter
A. 40 degree Celsius of 10 minutes. The nurse would assess this
B. 39 degree Celsius event as:
C. 100 degree Fahrenheit
D. 105.8 degree Fahrenheit A. The goal of reducing john’s fever has
27. Tympanic temperature is taken from John, been met with full satisfaction of the
A client who was brought recently into the ER outcome criteria
due to frequent barking cough. The B. The desired goal has been partially met
temperature reads 37.9 Degrees Celsius. As a C. The goal is not completely met
nurse, you conclude that this temperature is D. The goal has been met but not with the
desired outcome criteria
A. High 33. What can you expect from Marianne, who
B. Low is currently at the ONSET stage of fever?
C. At the low end of the normal range
D. At the high end of the normal range A. Hot, flushed skin
B. Increase thirst
C. Convulsion 41. How long should the thermometer stay in
D. Pale,cold skin the Client’s Axilla?
34. Marianne is now at the Defervescence
stage of the fever, which of the following is A. 3 minutes
expected? B. 4 minutes
C. 7 minutes
A. Delirium D. 10 minutes
B. Goose flesh 42. Which of the following statement is
C. Cyanotic nail beds TRUE about pulse?
D. Sweating
35. Considered as the most accessible and A. Young person have higher pulse than
convenient method for temperature taking older persons
B. Males have higher pulse rate than females
A. Oral after puberty
B. Rectal C. Digitalis has a positive chronotropic
C. Tympanic effect
D. Axillary D. In lying position, Pulse rate is higher
36. Considered as Safest and most non 43. The following are correct actions when
invasive method of temperature taking taking radial pulse except:

A. Oral A. Put the palms downward


B. Rectal B. Use the thumb to palpate the artery
C. Tympanic C. Use two or three fingers to palpate the
D. Axillary pulse at the inner wrist
37. Which of the following is NOT a D. Assess the pulse rate, rhythm, volume and
contraindication in taking ORAL temperature? bilateral quality
44. The difference between the systolic and
A. Quadriplegic diastolic pressure is termed as
B. Presence of NGT
C. Dyspnea A. Apical rate
D. Nausea and Vomitting B. Cardiac rate
38. Which of the following is a C. Pulse deficit
contraindication in taking RECTAL D. Pulse pressure
temperature? 45. Which of the following completely
describes PULSUS PARADOXICUS?
A. Unconscious
B. Neutropenic A. A greater-than-normal increase in systolic
C. NPO blood pressure with inspiration
D. Very young children B. A greater-than-normal decrease in
39. How long should the Rectal Thermometer systolic blood pressure with inspiration
be inserted to the clients anus? C. Pulse is paradoxically low when client is
in standing position and high when supine.
A. 1 to 2 inches D. Pulse is paradoxically high when client is
B. 5 to 1.5 inches in standing position and low when supine.
C. 3 to 5 inches 46. Which of the following is TRUE about
D. 2 to 3 inches respiration?
40. In cleaning the thermometer after use, The
direction of the cleaning to follow Medical A. I:E 2:1
Asepsis is : B. I:E : 4:3
C. I:E 1:1
A. From bulb to stem D. I:E 1:2
B. From stem to bulb 47. Contains the pneumotaxic and the
C. From stem to stem apneutic centers
D. From bulb to bulb
A. Medulla oblongata
B. Pons 55. Refers to the pressure when the ventricles
C. Carotid bodies are at rest
D. Aortic bodies
48. Which of the following is responsible for A. Diastole
deep and prolonged inspiration B. Systole
C. Preload
A. Medulla oblongata D. Pulse pressure
B. Pons 56. Which of the following is TRUE about the
C. Carotid bodies blood pressure determinants?
D. Aortic bodies
49. Which of the following is responsible for A. Hypervolemia lowers BP
the rhythm and quality of breathing? B. Hypervolemia increases GFR
C. HCT of 70% might decrease or increase
A. Medulla oblongata BP
B. Pons D. Epinephrine decreases BP
C. Carotid bodies 57. Which of the following do not correctly
D. Aortic bodies correlates the increase BP of Ms. Aida, a 70
50. The primary respiratory center year old diabetic?

A. Medulla oblongata A. Females, after the age 65 tends to have


B. Pons lower BP than males
C. Carotid bodies B. Disease process like Diabetes increase BP
D. Aortic bodies C. BP is highest in the morning, and lowest
51. Which of the following is TRUE about the during the night
mechanism of action of the Aortic and Carotid D. Africans, have a greater risk of
bodies? hypertension than Caucasian and Asians.
58. How many minutes are allowed to pass if
A. If the BP is elevated, the RR increases the client had engaged in strenuous activities,
B. If the BP is elevated, the RR decreases smoked or ingested caffeine before taking
C. Elevated BP leads to Metabolic alkalosis his/her BP?
D. Low BP leads to Metabolic acidosis
52. All of the following factors correctly A. 5
influence respiration except one. Which of the B. 10
following is incorrect? C. 15
D. 30
A. Hydrocodone decreases RR 59. Too narrow cuff will cause what change in
B. Stress increases RR the Client’s BP?
C. Increase temperature of the
environment, Increase RR A. True high reading
D. Increase altitude, Increase RR B. True low reading
53. When does the heart receives blood from C. False high reading
the coronary artery? D. False low reading
60. Which is a preferable arm for BP taking?
A. Systole
B. Diastole A. An arm with the most contraptions
C. When the valves opens B. The left arm of the client with a CVA
D. When the valves closes affecting the right brain
54. Which of the following is more life C. The right arm
threatening? D. The left arm
61. Which of the following is INCORRECT
A. BP = 180/100 in assessing client’s BP?
B. BP = 160/120
C. BP = 90/60 A. Read the mercury at the upper
D. BP = 80/50 meniscus, preferably at the eye level to
prevent error of parallax
B. Inflate and deflate slowly, 2-3 mmHg at a C. RUQ,RLQ,LLQ,LUQ
time D. RLQ,RUQ,LUQ,LLQ
C. The sound heard during taking BP is 67. In inspecting the abdomen, which of the
known as KOROTKOFF sound following is NOT DONE?
D. If the BP is taken on the left leg using the
popliteal artery pressure, a BP of 160/80 A. Ask the client to void first
is normal. B. Knees and legs are straighten to relax
62. Which of the following is the correct the abdomen
interpretation of the ERROR OF PARALLAX C. The best position in assessing the
abdomen is Dorsal recumbent
A. If the eye level is higher than the level of D. The knees and legs are externally rotated
the meniscus, it will cause a false high 68. Dr. Fabian De Las Santas, is about to
reading conduct an ophthalmoscope examination.
B. If the eye level is higher than the level Which of the following, if done by a nurse, is
of the meniscus, it will cause a false low a Correct preparation before the procedure?
reading
C. If the eye level is lower than the level of A. Provide the necessary draping to ensure
the meniscus, it will cause a false low privacy
reading B. Open the windows, curtains and light to
D. If the eye level is equal to that of the level allow better illumination
of the upper meniscus, the reading is C. Pour warm water over the
accurate ophthalmoscope to ensure comfort
63. How many minute/s is/are allowed to pass D. Darken the room to provide better
before making a re-reading after the first one? illumination
69. If the client is female, and the doctor is a
A. 1 male and the patient is about to undergo a
B. 5 vaginal and cervical examination, why is it
C. 15 necessary to have a female nurse in
D. 30 attendance?
64. Which of the following is TRUE about the
auscultation of blood pressure? A. To ensure that the doctor performs the
procedure safely
A. Pulse + 4 is considered as FULL B. To assist the doctor
B. The bell of the stethoscope is use in C. To assess the client’s response to
auscultating BP examination
C. Sound produced by BP is considered as D. To ensure that the procedure is done in
HIGH frequency sound an ethical manner
D. Pulse +1 is considered as NORMAL 70. In palpating the client’s breast, Which of
65. In assessing the abdomen, Which of the the following position is necessary for the
following is the correct sequence of the patient to assume before the start of the
physical assessment? procedure?

A. Inspection, Auscultation, Percussion, A. Supine


Palpation B. Dorsal recumbent
B. Palpation, Auscultation, Percussion, C. Sitting
Inspection D. Lithotomy
C. Inspection, Palpation, Auscultation, 71. When is the best time to collect urine
Percussion specimen for routine urinalysis and C/S?
D. Inspection, Auscultation, Palpation,
Percussion A. Early morning
66. The sequence in examining the quadrants B. Later afternoon
of the abdomen is: C. Midnight
D. Before breakfast
A. RUQ,RLQ,LUQ,LLQ
B. RLQ,RUQ,LLQ,LUQ
72. Which of the following is among an ideal D. Clamp below the port for 5 to 10 minutes
way of collecting a urine specimen for culture before drawing the urine from the port
and sensitivity? 77. A community health nurse should be
resourceful and meet the needs of the client. A
A. Use a clean container villager ask him, Can you test my urine for
B. Discard the first flow of urine to ensure glucose? Which of the following technique
that the urine is not contaminated allows the nurse to test a client’s urine for
C. Collect around 30-50 ml of urine glucose without the need for intricate
D. Add preservatives, refrigerate the instruments.
specimen or add ice according to the
agency’s protocol A. Acetic Acid test
73. In a 24 hour urine specimen started Friday, B. Nitrazine paper test
9:00 A.M, which of the following if done by a C. Benedict’s test
Nurse indicate a NEED for further procedural D. Litmus paper test
debriefing? 78. A community health nurse is assessing
client’s urine using the Acetic Acid solution.
A. The nurse ask the client to urinate at Which of the following, if done by a nurse,
9:00 A.M, Friday and she included the indicates lack of correct knowledge with the
urine in the 24 hour urine specimen procedure?
B. The nurse discards the Friday 9:00 A M
urine of the client A. The nurse added the Urine as the 2/3 part
C. The nurse included the Saturday 9:00 of the solution
A.M urine of the client to the specimen B. The nurse heats the test tube after
collection adding 1/3 part acetic acid
D. The nurse added preservatives as per C. The nurse heats the test tube after
protocol and refrigerates the specimen adding 2/3 part of Urine
74. This specimen is required to assess D. The nurse determines abnormal result if
glucose levels and for the presence of albumin she noticed that the test tube becomes
the the urine cloudy
79. Which of the following is incorrect with
A. Midstream clean catch urine regards to proper urine testing using
B. 24 hours urine collection Benedict’s Solution?
C. Postprandial urine collection
D. Second voided urine A. Heat around 5ml of Benedict’s solution
75. When should the client test his blood together with the urine in a test tube
sugar levels for greater accuracy? B. Add 8 to 10 drops of urine
C. Heat the Benedict’s solution without the
A. During meals urine to check if the solution is
B. In between meals contaminated
C. Before meals D. If the color remains BLUE, the result is
D. 2 Hours after meals POSITIVE
76. In collecting a urine from a catheterized 80. +++ Positive result after Benedicts test is
patient, Which of the following statement depicted by what color?
indicates an accurate performance of the
procedure? A. Blue
B. Green
A. Clamp above the port for 30 to 60 C. Yellow
minutes before drawing the urine from the D. Orange
port 81. Clinitest is used in testing the urine of a
B. Clamp below the port for 30 to 60 client for glucose. Which of the following, If
minutes before drawing the urine from committed by a nurse indicates error?
the port
C. Clamp above the port for 5 to 10 minutes A. Specimen is collected after meals
before drawing the urine from the port B. The nurse puts 1 clinitest tablet into a test
tube
C. She added 5 drops of urine and 10 drops B. NPO for 12 hours pre procedure
of water C. Ask the client to drink 1 glass of water 1
D. If the color becomes orange or red, It is hour prior to the procedure
considered postitive D. Tell the client that the normal serum
82. Which of the following nursing lipase level is 50 to 140 U/L
intervention is important for a client 87. The primary factor responsible for body
scheduled to have a Guaiac Test? heat production is the

A. Avoid turnips, radish and horseradish A. Metabolism


3 days before procedure B. Release of thyroxin
B. Continue iron preparation to prevent C. Muscle activity
further loss of Iron D. Stress
C. Do not eat read meat 12 hours before 88. The heat regulating center is found in the
procedure
D. Encourage caffeine and dark colored A. Medulla oblongata
foods to produce accurate results B. Thalamus
83. In collecting a routine specimen for C. Hypothalamus
fecalysis, Which of the following, if done by a D. Pons
nurse, indicates inadequate knowledge and 89. A process of heat loss which involves the
skills about the procedure? transfer of heat from one surface to another is

A. The nurse scoop the specimen specifically A. Radiation


at the site with blood and mucus B. Conduction
B. She took around 1 inch of specimen or a C. Convection
teaspoonful D. Evaporation
C. Ask the client to call her for the 90. Which of the following is a primary factor
specimen after the client wiped off his that affects the BP?
anus with a tissue
D. Ask the client to defecate in a bedpan, A. Obesity
Secure a sterile container B. Age
84. In a routine sputum analysis, Which of the C. Stress
following indicates proper nursing action D. Gender
before sputum collection? 91. The following are social data about the
client except
A. Secure a clean container
B. Discard the container if the outside A. Patient’s lifestyle
becomes contaminated with the sputum B. Religious practices
C. Rinse the client’s mouth with Listerine C. Family home situation
after collection D. Usual health status
D. Tell the client that 4 tablespoon of sputum 92. The best position for any procedure that
is needed for each specimen for a routine involves vaginal and cervical examination is
sputum analysis
85. Who collects Blood specimen? A. Dorsal recumbent
B. Side lying
A. The nurse C. Supine
B. Medical technologist D. Lithotomy
C. Physician 93. Measure the leg circumference of a client
D. Physical therapist with bipedal edema is best done in what
86. David, 68 year old male client is position?
scheduled for Serum Lipid analysis. Which of
the following health teaching is important to A. Dorsal recumbent
ensure accurate reading? B. Sitting
C. Standing
A. Tell the patient to eat fatty meals 3 days D. Supine
prior to the procedure
94. In palpating the client’s abdomen, Which 100. In assessing the client’s chest, which
of the following is the best position for the position best show chest expansion as well as
client to assume? its movements?

A. Dorsal recumbent A. Sitting


B. Side lying B. Prone
C. Supine C. Sidelying
D. Lithotomy D. Supine
95. Rectal examination is done with a client in
what position?

A. Dorsal recumbent PNLE: FON Practice


B. Sims position
C. Supine
D. Lithotomy
Exam for
96. Which of the following is a correct
nursing action when collecting urine specimen Oxygenation and
from a client with an Indwelling catheter?

A. Collect urine specimen from the drainage


Nutrition
bag
B. Detach catheter from the connecting tube
1. Which one of the following is NOT a
and draw the specimen from the port
function of the Upper airway?
C. Use sterile syringe to aspirate urine
specimen from the drainage port For clearance mechanism such as
D. Insert the syringe straight to the port to
coughing
allow self sealing of the port Transport gases to the lower airways
97. Which of the following is inappropriate in Warming, Filtration and Humidification of
collecting mid stream clean catch urine inspired air
specimen for urine analysis? Protect the lower airway from foreign mater
2. It is the hair the lines the vestibule which
A. Collect early in the morning, First voided
function as a filtering mechanism for foreign
specimen
objects
B. Do perineal care before specimen
collection Cilia
C. Collect 5 to 10 ml for urine Nares
D. Discard the first flow of the urine Carina
98. When palpating the client’s neck for
Vibrissae
lymphadenopathy, where should the nurse 3. This is the paranasal sinus found between
position himself? the eyes and the nose that extends backward
into the skull
A. At the client’s back
B. At the client’s right side
Ehtmoid
C. At the client’s left side Sphenoid
D. In front of a sitting client Maxillary
99. Which of the following is the best position Frontal
for the client to assume if the back is to be 4. Which paranasal sinus is found over the
examined by the nurse? eyebrow?
A. Standing Ehtmoid
B. Sitting Sphenoid
C. Side lying Maxillary
D. Prone Frontal
5. Gene De Vonne Katrouchuacheulujiki
wants to change her surname to something
shorter, The court denied her request which Residual volume
depresses her and find herself binge eating. 12. This is the amount of air remained in the
She accidentally aspirate a large piece of nut lungs after a forceful expiration
and it passes the carina. Probabilty wise,
Where will the nut go? Inspiratory reserve volume
Expiratory reserve volume
Right main stem bronchus Functional residual capacity
Left main stem bronchus Residual volume
Be dislodged in between the carina 13. Casssandra, A 22 year old grade Agnostic,
Be blocked by the closed epiglottis Asked you, how many spikes of bones are
6. Which cell secretes mucus that help protect there in my ribs? Your best response is which
the lungs by trapping debris in the respiratory of the following?
tract?
We have 13 pairs of ribs Cassandra
Type I pneumocytes We have 12 pairs of ribs Cassandra
Type II pneumocytes Humans have 16 pairs of ribs, and that was
Goblet cells noted by Vesalius in 1543
Adipose cells Humans have 8 pairs of ribs. 4 of which are
7. How many lobes are there in the RIGHT floating
LUNG? 14. Which of the following is considered as
the main muscle of respiration?
One
Two Lungs
Three Intercostal Muscles
Four Diaphragm
8. The presence of the liver causes which Pectoralis major
anatomical difference of the Kidneys and the 15. Cassandra asked you : How many air is
Lungs? there in the oxygen and how many does
human requires? Which of the following is the
Left kidney slightly lower, Left lung best response :
slightly shorter
Left kidney slightly higher, Left lung slightly God is good, Man requires 21% of oxygen
shorter and we have 21% available in our air
Right kidney lower, Right lung shorter Man requires 16% of oxygen and we have 35%
Right kidney higher, Right lung shorter available in our air
9. Surfactant is produced by what cells in the Man requires 10% of oxygen and we have 50%
alveoli? available in our air
Human requires 21% of oxygen and we
Type I pneumocytes have 21% available in our air
Type II pneumocytes 16. Which of the following is TRUE about
Goblet cells Expiration?
Adipose cells
10. The normal L:S Ratio to consider the A passive process
newborn baby viable is The length of which is half of the length of
Inspiration
1:2 Stridor is commonly heard during expiration
2:1 Requires energy to be carried out
3:1 17. Which of the following is TRUE in
1:3 postural drainage?
11. Refers to the extra air that can be inhaled
beyond the normal tidal volume Patient assumes position for 10 to 15
minutes
Inspiratory reserve volume Should last only for 60 minutes
Expiratory reserve volume Done best P.C
Functional residual capacity An independent nursing action
10-15 mmHg
18. All but one of the following is a purpose 15-25 mmHg
of steam inhalation 24. There are four catheter sizes available for
use, which one of these should you use for Mr.
Mucolytic Hamilton?
Warm and humidify air
Administer medications Fr. 18
Promote bronchoconstriction Fr. 12
Fr. 10
19. Which of the following is NOT TRUE in Fr, 5
steam inhalation? 25. Which of the following, if done by the
nurse, indicates incompetence during
It is a dependent nursing action suctioning an unconscious client?
Spout is put 12-18 inches away from the nose
Render steam inhalation for atleast 60 Measure the length of the suction catheter
minutes to be inserted by measuring from the
Cover the client’s eye with wash cloth to tip of the nose, to the earlobe, to the
prevent irritation xiphoid process
20. When should a nurse suction a client? Use KY Jelly if suctioning nasopharyngeal
secretion
As desired The maximum time of suctioning should not
As needed exceed 15 seconds
Every 1 hour Allow 30 seconds interval between
Every 4 hours suctioning
21. Ernest Arnold Hamilton, a 60 year old
American client was mobbed by teen 26. Which of the following is the initial sign
gangsters near New york, Cubao. He was of hypoxemia in an adult client?
rushed to John John Hopio Medical Center
and was Unconscious. You are his nurse and Tachypnea
you are to suction his secretions. In which Tachycardia
position should you place Mr. Hamilton? Cyanosis
Pallor
High fowlers Irritability
Semi fowlers Flaring of NaresA. 1,2
Prone B. 2,5
Side lying C. 2,6
22. You are about to set the suction pressure D. 3,4
to be used to Mr. Hamilton. You are using a 27. Which method of oxygenation least likely
Wall unit suction machine. How much produces anxiety and apprehension?
pressure should you set the valve before
suctioning Mr. Hamilton? Nasal Cannula
Simple Face mask
50-95 mmHg Non Rebreather mask
200-350 mmHg Partial Rebreather mask
100-120 mmHg
10-15 mmHg 28. Which of the following oxygen delivery
23. The wall unit is not functioning; You then method can deliver 100% Oxygen at 15 LPM?
try to use the portable suction equipment
available. How much pressure of suction Nasal Cannula
equipment is needed to prevent trauma to Simple Face mask
mucus membrane and air ways in case of Non Rebreather mask
portable suction units? Partial Rebreather mask
29. Which of the following is not true about
2-5 mmHg OXYGEN?
5-10 mmHg
Oxygen is odorless, tasteless and colorless 35. Where will the CTT be inserted if we are
gas. to drain fluids accumulated in Mang dagul’s
Oxygen can irritate mucus membrane pleura?
Oxygen supports combustion
Excessive oxygen administration results in 2nd ICS
respiratory acidosis 4th ICS
5th ICS
30. Roberto San Andres, A new nurse in the 8th ICS
hospital is about to administer oxygen on 36. There is a continuous bubbling in the
patient with Respiratory distress. As his senior water sealed drainage system with suction.
nurse, you should intervene if Roberto will: And oscillation is observed. As a nurse, what
should you do?
Uses venture mask in oxygen administration
Put a non rebreather mask in the patient Consider this as normal findings
before opening the oxygen source Notify the physician
Use a partial rebreather mask to deliver Check for tube leak
oxygen Prepare a petrolatum gauze dressing
Check for the doctor’s order for Oxygen 37. Which of the following is true about
administration nutrition?
31. Which of the following will alert the nurse
as an early sign of hypoxia? It is the process in which food are broken
down, for the body to use in growth and
Client is tired and dyspneic development
The client is coughing out blood It is a process in which digested proteins, fats,
The client’s heart rate is 50 BPM minerals, vitamins and carbohydrates are
Client is frequently turning from side to transported into the circulation
side It is a chemical process that occurs in the cell
32. Miguelito de balboa, An OFW presents at that allows for energy production, energy
the admission with an A:P Diameter ratio of use, growth and tissue repair
2:1, Which of the following associated finding It is the study of nutrients and the process
should the nurse expect? in which they are use by the body
38. The majority of the digestion processes
Pancytopenia take place in the
Anemia
Fingers are Club-like Mouth
Hematocrit of client is decreased Small intestine
33. The best method of oxygen administration Large intestine
for client with COPD uses: Stomach
39. All of the following is true about digestion
Cannula that occurs in the Mouth except
Simple Face mask
Non rebreather mask It is where the digestion process starts
Venturi mask Mechanical digestion is brought about by
34. Mang dagul, a 50 year old chronic smoker mastication
was brought to the E.R because of difficulty The action of ptyalin or the salivary
in breathing. Pleural effusion was the tyrpsin breaks down starches into
diagnosis and CTT was ordered. What does maltose
C.T.T Stands for? Deglutition occurs after food is broken down
into small pieces and well mixed with
Chest tube thoracotomy saliva
Chest tube thoracostomy 40. Which of the following foods lowers the
Closed tube thoracotomy cardiac sphincter pressure?
Closed tube thoracostmy
Roast beef, Steamed cauliflower and Rice
Orange juice, Non fat milk, Dry crackers
Decaffeinated coffee, Sky flakes crackers, Trypsin
Suman Enterokinase
Coffee with coffee mate, Bacon and Egg Enterogastrone
41. Where does the digestion of carbohydrates Amylase
start? 48. The end product of protein digestion or
the “Building blocks of Protein” is what we
Mouth call
Esophagus
Small intestine Nucleotides
Stomach Fatty acids
42. Protein and Fat digestion begins where? Glucose
Amino Acids
Mouth 49. Enzyme secreted by the small intestine
Esophagus after it detects a bolus of fatty food. This will
Small intestine contract the gallbladder to secrete bile and
Stomach relax the sphincter of Oddi to aid in the
43. All but one is true about digestion that emulsification of fats and its digestion.
occurs in the Stomach
Lipase
Carbohydrates are the fastest to be digested, Amylase
in about an hour Cholecystokinin
Fat is the slowest to be digested, in about 5 Pancreozymin
hours 50. Which of the following is not true about
HCl inhibits absorption of Calcium in the the Large Intestine?
gastric mucosa
HCl converts pepsinogen to pepsin, which It absorbs around 1 L of water making the
starts the complex process of protein feces around 75% water and 25% solid
digestion The stool formed in the transverse colon is
44. Which of the following is NOT an enzyme not yet well formed
secreted by the small intestine? It is a sterile body cavity
It is called large intestine because it is longer
Sucrase than the small intestine
Enterokinase 51. This is the amount of heat required to raise
Amylase the temperature of 1 kg water to 1 degree
Enterokinase Celsius
45. The hormone secreted by the Small
intestine that stimulates the production of Calorie
pancreatic juice which primarily aids in Joules
buffering the acidic bolus passed by the Metabolism
Stomach Basal metabolic rate
52. Assuming a cup of rice provides 50 grams
Enterogastrone of carbohydrates. How many calories are
Cholecystokinin there in that cup of rice?
Pancreozymin
Enterokinase 150 calories
46. When the duodenal enzyme sucrase acts 200 calories
on SUCROSE, which 2 monosaccharides are 250 calories
formed? 400 calories
53. An average adult filipino requires how
Galactose + Galactose many calories in a day?
Glucose + Fructose
Glucose + Galactose 1,000 calories
Fructose + Fructose 1,500 calories
47. This is the enzyme secreted by the 2,000 calories
pancrease that completes the protein digestion 2,500 calories
54. Which of the following is true about an Vitamin B2
individual’s caloric needs? Vitamin B3
Vitamin C
All individual have the same caloric needs 61. Which of the following is the best source
Females in general have higher BMR and of Vitamin E?
therefore, require more calories
During cold weather, people need more Green leafy vegetables
calories due to increase BMR Vegetable oil
Dinner should be the heaviest meal of the Fortified Milk
day Fish liver oil
55. Among the following people, who
requires the greatest caloric intake? 62. Among the following foods, which food
should you emphasize giving on an Alcoholic
An individual in a long state of client?
gluconeogenesis
An individual in a long state of Pork liver and organ meats, Pork
glycogenolysis Red meat, Eggs and Dairy products
A pregnant individual Green leafy vegetables, Yellow vegetables,
An adolescent with a BMI of 25 Cantaloupe and Dairy products
56. Which nutrient deficiency is associated Chicken, Peanuts, Bananas, Wheat germs
with the development of Pellagra, Dermatitis and yeasts
and Diarrhea? 63. Which food group should you emphasize
giving on a pregnant mother in first trimester
Vitamin B1 to prevent neural tube defects?
Vitamin B2
Vitamin B3 Broccoli, Guava, Citrus fruits, Tomatoes
Vitamin B6 Butter, Sardines, Tuna, Salmon, Egg yolk
57. Which Vitamin is not given in conjunction Wheat germ, Vegetable Oil, soybeans, corn,
with the intake of LEVODOPA in cases of peanuts
Parkinson’s Disease due to the fact that Organ meats, Green leafy vegetables,
levodopa increases its level in the body? Liver, Eggs
64. A client taking Coumadin is to be
Vitamin B1 educated on his diet. As a nurse, which of the
Vitamin B2 following food should you instruct the client
Vitamin B3 to avoid?
Vitamin B6
58. A vitamin taken in conjunction with Spinach, Green leafy vegetables, Cabbage,
ISONIAZID to prevent peripheral neuritis Liver
Salmon, Sardines, Tuna
Vitamin B1 Butter, Egg yolk, breakfast cereals
Vitamin B2 Banana, Yeast, Wheat germ, Chicken
Vitamin B3
Vitamin B6 65. Vitamin E plus this mineral works as one
59. The inflammation of the Lips, Palate and of the best anti oxidant in the body according
Tongue is associated in the deficiency of this to the latest research. They are combined with
vitamin 5 Alpha reductase inhibitor to reduce the risk
of acquiring prostate cancer
Vitamin B1
Vitamin B2 Zinc
Vitamin B3 Iron
Vitamin B6 Selenium
60. Beri beri is caused by the deficiency of Vanadium
which Vitamin? 66. Incident of prostate cancer is found to
have been reduced on a population exposed in
Vitamin B1
tolerable amount of sunlight. Which vitamin 73. Jose Miguel, a 50 year old business man is
is associated with this phenomenon? 6’0 Tall and weights 179 lbs. As a nurse, you
know that Jose Miguel is :
Vitamin A
Vitamin B Overweight
Vitamin C Underweight
Vitamin D Normal
67. Micronutrients are those nutrients needed Obese
by the body in a very minute amount. Which 74. Jose Miguel is a little bit nauseous.
of the following vitamin is considered as a Among the following beverages, Which could
MICRONUTRIENT help relieve JM’s nausea?

Phosphorous Coke
Iron Sprite
Calcium Mirinda
Sodium Orange Juice or Lemon Juice
68. Deficiency of this mineral results in tetany, 75. Which of the following is the first sign of
osteomalacia, osteoporosis and rickets. dehydration?

Vitamin D Tachycardia
Iron Restlessness
Calcium Thirst
Sodium Poor skin turgor
69. Among the following foods, which has the 76. What Specific gravity lab result is
highest amount of potassium per area of their compatible with a dehydrated client?
meat?
1.007
Cantaloupe 1.020
Avocado 1.039
Raisin 1.029
Banana 77. Which hematocrit value is expected in a
70. A client has HEMOSIDEROSIS. Which dehydrated male client?
of the following drug would you expect to be
given to the client? 67%
50%
Acetazolamide 36%
Deferoxamine 45%
Calcium EDTA 78. Which of the following statement by a
Activated charcoal client with prolonged vomiting indicates the
71. Which of the following provides the initial onset of hypokalemia?
richest source of Iron per area of their meat?
My arm feels so weak
Pork meat I felt my heart beat just right now
Lean read meat My face muscle is twitching
Pork liver Nurse, help! My legs are cramping
Green mongo 79. Which of the following is not an anti-
emetic?
72. Which of the following is considered the
best indicator of nutritional status of an Marinol
individual? Dramamine
Benadryl
Height Alevaire
Weight
Arm muscle circumference 80. Which is not a clear liquid diet?
BMI
Hard candy ordered by the doctor. You then instill 60 cc
Gelatin of water to clear the lumen and the tube. How
Coffee with Coffee mate much will you put in the client’s chart as input?
Bouillon
81. Which of the following is included in a 250 cc
full liquid diet? 290 cc
350 cc
Popsicles 310 cc
Pureed vegetable meat 87. Which of the following if done by a nurse
Pineapple juice with pulps indicates deviation from the standards of NGT
Mashed potato feeding?
82. Which food is included in a BLAND
DIET? Do not give the feeding and notify the doctor
of residual of the last feeding is greater
Steamed broccoli than or equal to 50 ml
Creamed potato Height of the feeding should be 12 inches
Spinach in garlic about the tube point of insertion to allow
Sweet potato slow introduction of feeding
83. Which of the following if done by the Ask the client to position in supine position
nurse, is correct during NGT Insertion? immediately after feeding to prevent
dumping syndrome
Use an oil based lubricant Clamp the NGT before all of the water is
Measure the amount of the tube to be instilled to prevent air entry in the
inserted from the Tip of the nose, to the stomach
earlobe, to the xiphoid process 88. What is the most common problem in
Soak the NGT in a basin of ice water to TUBE FEEDING?
facilitate easy insertion
Check the placement of the tube by Diarrhea
introducing 10 cc of sterile water and Infection
auscultating for bubbling sound Hyperglycemia
84. Which of the following is the BEST Vomiting
method in assessing for the correct placement 89. Which of the following is TRUE in
of the NGT? colostomy feeding?

X-Ray Hold the syringe 18 inches above the stoma


Immerse tip of the tube in water to check for and administer the feeding slowly
bubbles produced Pour 30 ml of water before and after feeding
Aspirating gastric content to check if the administration
content is acidic Insert the ostomy feeding tube 1 inch towards
Instilling air in the NGT and listening for a the stoma
gurgling sound at the epigastric area A Pink stoma means that circulation towards
the stoma is all well
85. A terminally ill cancer patient is scheduled 90. A client with TPN suddenly develops
for an NGT feeding today. How should you tremors, dizziness, weakness and diaphoresis.
position the patient? The client said “I feel weak” You saw that his
TPN is already empty and another TPN is
Semi fowlers in bed scheduled to replace the previous one but its
Bring the client into a chair provision is already 3 hours late. Which of the
Slightly elevated right side lying position following is the probable complication being
Supine in bed experienced by the client?
86. A client is scheduled for NGT Feeding.
Checking the residual volume, you Hyperglycemia
determined that he has 40 cc residual from the Hypoglycemia
last feeding. You reinstill the 40 cc of residual Infection
volume and added the 250 cc of feeding Fluid overload
91. To assess the adequacy of food intake, 19
which of the following assessment parameters 15
is best used? 25
99. Which finding is consistent with
Food likes and dislikes PERNICIOUS ANEMIA?
Regularity of meal times
3 day diet recall Strawberry tongue
Eating style and habits Currant Jelly stool
Beefy red tongue
92. The vomiting center is found in the Pale [ HYPOCHROMIC ] RBC
100. The nurse is browsing the chart of the
Medulla Oblongata patient and notes a normal serum lipase level.
Pons Which of the following is a normal serum
Hypothalamus lipase value?
Cerebellum
93. The most threatening complication of 10 U/L
vomiting in client’s with stroke is 100 U/L
200 U/L
Aspiration 350 U/L
Dehydration
Fluid and electrolyte imbalance
Malnutrition
94. Which among this food is the richest
PNLE Fundamentals
source of Iron?
in Nursing Exam 1
Ampalaya 1. Jake is complaining of shortness of breath.
Broccoli The nurse assesses his respiratory rate to be
Mongo 30 breaths per minute and documents that
Malunggay leaves Jake is tachypneic. The nurse understands that
95. Which of the following is a good source of tachypnea means:
Vitamin A?
Pulse rate greater than 100 beats per minute
Egg yolk Blood pressure of 140/90
Liver Respiratory rate greater than 20 breaths
Fish
per minute
Peanuts Frequent bowel sounds
96. The most important nursing action before
gastrostomy feeding is (C) Respiratory rate greater than 20 breaths
per minute. A respiratory rate of greater than
Check V/S
20 breaths per minute is tachypnea. A blood
Assess for patency of the tube
pressure of 140/90 is considered hypertension.
Measure residual feeding
Pulse greater than 100 beats per minute is
Check the placement of the tube
tachycardia. Frequent bowel sounds refer to
97. The primary advantage of gastrostomy
hyper-active bowel sounds.
feeding is
2. The nurse listens to Mrs. Sullen’s lungs and
Ensures adequate nutrition
notes a hissing sound or musical sound. The
It prevents aspiration
nurse documents this as:
Maintains Gastro esophageal sphincter
integrity
Wheezes
Minimizes fluid-electrolyte imbalance
Rhonchi
98. What is the BMI Of Budek, weighing 120
Gurgles
lbs and has a height of 5 feet 7 inches.
Vesicular
20
(A) Wheezes. Wheezes are indicated by
continuous, lengthy, musical; heard 6. During the planning phase of the nursing
during inspiration or expiration. Rhonchi process, which of the following is the
are usually coarse breath sounds. Gurgles outcome?
are loud gurgling, bubbling sound.
Vesicular breath sounds are low pitch, Nursing history
soft intensity on expiration Nursing notes
Nursing care plan
3. The nurse in charge measures a patient’s Nursing diagnosis
temperature at 101 degrees F. What is the
equivalent centigrade temperature? (C) Nursing care plan. The outcome, or the
product of the planning phase of the
36.3 degrees C nursing process is a Nursing care plan.
37.95 degrees C
40.03 degrees C 7. What is an example of a subjective data?
38.01 degrees C
Heart rate of 68 beats per minute
(B) 37.95 degrees C. To convert °F to °C use Yellowish sputum
this formula, ( °F – 32 ) (0.55). While Client verbalized, “I feel pain when
when converting °C to °F use this formula, urinating.”
( °C x 1.8) + 32. Note that 0.55 is 5/9 and Noisy breathing
1.8 is 9/5.
(C) Client verbalized, “I feel pain when
4. Which approach to problem solving tests urinating.”. Subjective data are those that
any number of solutions until one is found can be described only by the person
that works for that particular problem? experiencing it. Therefore, only the
patient can describe or verify whether he
Intuition is experiencing pain or not.
Routine
Scientific method 8. Which expected outcome is correctly
Trial and error written?

(D) Trial and error. The trial and error “The patient will feel less nauseated in 24
method of problem solving isn’t hours.”
systematic (as in the scientific method of “The patient will eat the right amount of food
problem solving) routine, or based on daily.”
inner prompting (as in the intuitive “The patient will identify all the high-salt
method of problem solving). food from a prepared list by discharge.”

5. What is the order of the nursing process? (C) “The patient will identify all the high-salt
food from a prepared list by
Assessing, diagnosing, implementing, discharge.”. Expected outcomes are
evaluating, planning specific, measurable, realistic statements
Diagnosing, assessing, planning, of goal attainment. The phrases “right
implementing, evaluating amount”, “less nauseated” and “enough
Assessing, diagnosing, planning, sleep” are vague and not measurable.
implementing, evaluating
Planning, evaluating, diagnosing, assessing, “The patient will have enough sleep.”
implementing 9. Which of the following behaviors by Nurse
Jane Robles demonstrates that she
(C) Assessing, diagnosing, planning, understands well th elements of effecting
implementing, evaluating. The correct charting?
order of the nursing process is assessing,
diagnosing, planning, implementing, She writes in the chart using a no. 2 pencil.
evaluating. She noted: appetite is good this afternoon.
She signs on the medication sheet after Patient and relatives
administering the medication. Nurse and patient
She signs her charting as follow: J.R Doctor and family
Nurse and doctor
(C) She signs on the medication sheet after
administering the medication.A nurse (B) Nurse and patient. Although diagnosing
should record a nursing intervention (ex. is basically the nurse’s responsibility,
Giving medications) after performing the input from the patient is essential to
nursing intervention (not before). formulate the correct nursing diagnosis.
Recording should also be done using a
pen, be complete, and signed with the 13. Mrs. Caperlac has been diagnosed to have
nurse’s full name and title. hypertension since 10 years ago. Since then,
she had maintained low sodium, low fat diet,
10. What is the disadvantage of computerized to control her blood pressure. This practice is
documentation of the nursing process? viewed as:

Accuracy Cultural belief


Legibility Personal belief
Concern for privacy Health belief
Rapid communication Superstitious belief

(C) Concern for privacy. A patient’s privacy (C) Health belief. Health belief of an
may be violated if security measures individual influences his/her preventive
aren’t used properly or if policies and health behavior.
procedures aren’t in place that determines
what type of information can be retrieved, 14. Becky is on NPO since midnight as
by whom, and for what purpose. preparation for blood test. Adreno-cortical
response is activated. Which of the following
11. The theorist who believes that adaptation is an expected response?
and manipulation of stressors are related to
foster change is: Low blood pressure
Warm, dry skin
Dorothea Orem Decreased serum sodium levels
Sister Callista Roy Decreased urine output
Imogene King
Virginia Henderson (D) Decreased urine output. Adreno-cortical
response involves release of aldosterone
(B) Sister Callista Roy. Sister Roy’s theory is that leads to retention of sodium and
called the adaptation theory and she water. This results to decreased urine
viewed each person as a unified output.
biophysical system in constant interaction
with a changing environment. Orem’s 15. What nursing action is appropriate when
theory is called self-care deficit theory obtaining a sterile urine specimen from an
and is based on the belief that individual indwelling catheter to prevent infection?
has a need for self-care actions. King’s
theory is the Goal attainment theory and Use sterile gloves when obtaining urine.
described nursing as a helping profession Open the drainage bag and pour out the urine.
that assists individuals and groups in Disconnect the catheter from the tubing and
society to attain, maintain, and restore get urine.
health. Henderson introduced the nature Aspirate urine from the tubing port using
of nursing model and identified the 14 a sterile syringe.
basic needs.
(D) Aspirate urine from the tubing port using
12. Formulating a nursing diagnosis is a joint a sterile syringe. The nurse should
function of: aspirate the urine from the port using a
sterile syringe to obtain a urine specimen. point of insertion, bot 20 inches. If the
Opening a closed drainage system height of feeding is too high, this results
increase the risk of urinary tract infection. to very rapid introduction of feeding. This
may trigger nausea and vomiting.
16. A client is receiving 115 ml/hr of
continuous IVF. The nurse notices that the 19. A female patient is being discharged after
venipuncture site is red and swollen. Which of thyroidectomy. After providing the
the following interventions would the nurse medication teaching. The nurse asks the
perform first? patient to repeat the instructions. The nurse is
performing which professional role?
Stop the infusion
Call the attending physician Manager
Slow that infusion to 20 ml/hr Caregiver
Place a clod towel on the site Patient advocate
Educator
(A) Stop the infusion. The sign and
symptoms indicate extravasation so the (D) Educator. When teaching a patient about
IVF should be stopped immediately and medications before discharge, the nurse is
put warm not cold towel on the affected acting as an educator. A caregiver
site. provides direct care to the patient. The
nurse acts as s patient advocate when
17. The nurse enters the room to give a making the patient’s wishes known to the
prescribed medication but the patient is inside doctor.
the bathroom. What should the nurse do?
20. Which data would be of greatest concern
Leave the medication at the bedside and to the nurse when completing the nursing
leave the room. assessment of a 68-year-old woman
After few minutes, return to that patient’s hospitalized due to Pneumonia?
room and do not leave until the patient
takes the medication. Oriented to date, time and place
Instruct the patient to take the medication and Clear breath sounds
leave it at the bedside. Capillary refill greater than 3 seconds and
Wait for the patient to return to bed and just buccal cyanosis
leave the medication at the bedside. Hemoglobin of 13 g/dl

(B) After few minutes, return to that patient’s (C) Capillary refill greater than 3 seconds
room and do not leave until the patient and buccal cyanosis. Capillary refill
takes the medication. This is to verify or greater than 3 seconds and buccal
to make sure that the medication was cyanosis indicate decreased oxygen to the
taken by the patient as directed. tissues which requires immediate
attention/intervention. Oriented to date,
18. Which of the following is inappropriate time and place, hemoglobin of 13 g/dl are
nursing action when administering NGT normal data.
feeding?
21. During a change-of-shift report, it would
Place the feeding 20 inches above the pint be important for the nurse relinquishing
if insertion of NGT. responsibility for care of the patient to
Introduce the feeding slowly. communicate. Which of the following facts to
Instill 60ml of water into the NGT after the nurse assuming responsibility for care of
feeding. the patient?
Assist the patient in fowler’s position.
That the patient verbalized, “My headache is
(A) Place the feeding 20 inches above the gone.”
pint if insertion of NGT. The height of the That the patient’s barium enema performed 3
feeding is above 12 inches above the days ago was negative
Patient’s NGT was removed 2 hours ago Anxiety related to impending surgery, as
Patient’s family came for a visit this morning. evidenced by insomnia.
Risk of injury related to autoimmune
(C) Patient’s NGT was removed 2 hours dysfunction
ago. The change-of-shift report should Impaired verbal communication related to
indicate significant recent changes in the tracheostomy, as evidenced by inability to
patient’s condition that the nurse speak.
assuming responsibility for care of the
patient will need to monitor. The other (A) Ineffective breathing pattern related to
options are not critical enough to include pain, as evidenced by shortness of
in the report. breath.. Physiologic needs (ex. Oxygen,
fluids, nutrition) must be met before
22. Which statement is the most appropriate lower needs (such as safety and security,
goal for a nursing diagnosis of diarrhea? love and belongingness, self-esteem and
self-actualization) can be met. Therefore,
“The patient will experience decreased physiologic needs have the highest
frequency of bowel elimination.” priority.
“The patient will take anti-diarrheal
medication.” 25. When performing an abdominal
“The patient will give a stool specimen for examination, the patient should be in a supine
laboratory examinations.” position with the head of the bed at what
“The patient will save urine for inspection by position?
the nurse.
30 degrees
(A) “The patient will experience decreased 90 degrees
frequency of bowel elimination.” The 45 degrees
goal is the opposite, healthy response of 0 degree
the problem statement of the nursing
diagnosis. In this situation, the problem (D) 0 degree. The patient should be positioned
statement is diarrhea. with the head of the bed completely flattened
to perform an abdominal examination. If the
23. Which of the following is the most head of the bed is elevated, the abdominal
important purpose of planning care with this muscles and organs can be bunched up,
patient? altering the findings
Development of a standardized NCP.
Expansion of the current taxonomy of
nursing diagnosis
PNLE Fundamentals
Making of individualized patient care
Incorporation of both nursing and medical in Nursing Exam 2
diagnoses in patient care

(C) Making of individualized patient care. To 1. A patient is wearing a soft wrist-safety


be effective, the nursing care plan device. Which of the following nursing
developed in the planning phase of the assessment is considered abnormal?
nursing process must reflect the
individualized needs of the patient. Palpable radial pulse
Palpable ulnar pulse
24. Using Maslow’s hierarchy of basic human Capillary refill within 3 seconds
needs, which of the following nursing Bluish fingernails, cool and pale fingers
diagnoses has the highest priority?
(D) Bluish fingernails, cool and pale
Ineffective breathing pattern related to fingers. A safety device on the wrist may
pain, as evidenced by shortness of impair blood circulation. Therefore, the
breath. nurse should assess the patient for signs
of impaired circulation such as bluish stagnant
fingernails, cool and pale fingers. inflexible
Palpable radial and ulnar pulses, capillary asystematic
refill within 3 seconds are all normal goal-oriented
findings.
(D) goal-oriented. The nursing process is
2. Pia’s serum sodium level is 150 mEq/L. goal-oriented. It is also systematic,
Which of the following food items does the patient-centered, and dynamic.
nurse instruct Pia to avoid?
6. A skin lesion which is fluid-filled, less than
broccoli 1 cm in size is called:
sardines
cabbage papule
Tomatoes vesicle
bulla
(B) sardines. The normal serum sodium level Macule
is 135 to 145 mEq/L, the client is having
hypernatremia. Pia should avoid food (B) vesicle. Vesicle is a circumscribed
high in sodium like processed food. circulation containing serous fluid or
Broccoli, cabbage and tomatoes are good blood and less than 1 cm (ex. Blister,
source of Vitamin C. chicken pox).

3. Jason, 3 years old vomited. His mom stated, 7. During application of medication into the
“He vomited 6 ounces of his formula this ear, which of the following is inappropriate
morning.” This statement is an example of: nursing action?

objective data from a secondary source In an adult, pull the pinna upward.
objective data from a primary source Instill the medication directly into the
subjective data from a primary source tympanic membrane.
subjective data from a secondary source Warm the medication at room or body
temperature.
(A) objective data from a secondary Press the tragus of the ear a few times to
source. Jason is the primary source; his assist flow of medication into the ear
mother is a secondary source. The data is canal.
objective because it can be perceived by
the senses, verified by another person (B) Instill the medication directly into the
observing the same patient, and tested tympanic membrane. During the
against accepted standards or norms. application of medication it is
inappropriate to instill the medication
4. Which of the following is a nursing directly into the tympanic membrane. The
diagnosis? right thing to do is instill the medication
along the lateral wall of the auditory canal.
Hypethermia
Diabetes Mellitus 8. Which of the following is appropriate
Angina nursing intervention for a client who is
Chronic Renal Failure grieving over the death of her child?

A) Hypethermia. Hyperthermia is a Tell her not to cry and it will be better.


NANDA-approved nursing diagnosis. Provide opportunity to the client to tell
Diabetes Mellitus, Angina and Chronic their story.
Renal Failure are medical diagnoses. Encourage her to accept or to replace the lost
person.
5. What is the characteristic of the nursing Discourage the client in expressing her
process? emotions.
(B) Provide opportunity to the client to tell scurvy
their story. Providing a grieving person an pellagra
opportunity to tell their story allows the megaloblastic anemia
person to express feelings. This is pernicious anemia
therapeutic in assisting the client resolve
grief. (C) megaloblastic anemia. Prolonged
Vitamin B9 deficiency will lead to
9. It is the gradual decrease of the body’s megaloblastic anemia while pernicious
temperature after death. anemia results in deficiency in Vitamin
B12. Prolonged deficiency of Vitamin C
livor mortis leads to scurvy and Pellagra results in
rigor mortis deficiency in Vitamin B3.
algor mortis
none of the above 13. Nurse Cherry is teaching a 72 year old
patient about a newly prescribed medication.
(C) algor mortis. Algor mortis is the decrease What could cause a geriatric patient to have
of the body’s temperature after death. difficulty retaining knowledge about the
Livor mortis is the discoloration of the newly prescribed medication?
skin after death. Rigor mortis is the
stiffening of the body that occurs about 2- Absence of family support
4 hours after death. Decreased sensory functions
Patient has no interest on learning
10. When performing an admission Decreased plasma drug levels
assessment on a newly admitted patient, the
nurse percusses resonance. The nurse knows (B) Decreased sensory functions. Decreased
that resonance heard on percussion is most in sensory functions could cause a
commonly heard over which organ? geriatric patient to have difficulty
retaining knowledge about the newly
thigh prescribed medications. Absence of
liver family support and no interest on learning
intestine may affect compliance, not knowledge
Lung retention. Decreased plasma levels do not
alter patient’s knowledge about the drug.
(D) lung. Resonance is loud, low-pitched and
long duration that’s heard most 14. When assessing a patient’s level of
commonly over an air-filled tissue such as consciousness, which type of nursing
a normal lung. intervention is the nurse performing?

11. The nurse is aware that Bell’s palsy Independent


affects which cranial nerve? Dependent
Collaborative
2nd CN (Optic) Professional
3rd CN (Occulomotor)
4th CN (Trochlear) (A) Independent. Independent nursing
7th CN (Facial) interventions involve actions that nurses
initiate based on their own knowledge and
(D) 7th CN (Facial). Bells’ palsy is the skills without the direction or supervision
paralysis of the motor component of the of another member of the health care team.
7th caranial nerve, resulting in facial sag,
inability to close the eyelid or the mouth, 15. Claire is admitted with a diagnosis of
drooling, flat nasolabial fold and loss of chronic shoulder pain. By definition, the nurse
taste on the affected side of the face. understands that the patient has had pain for
more than:
12. Prolonged deficiency of Vitamin B9 leads
to: 3 months
6 months indicate the adequacy of food intake of
9 months the client.
1 year
19. Van Fajardo is a 55 year old who was
(B) 6 months. Chronic pain s usually defined admitted to the hospital with newly diagnosed
as pain lasting longer than 6 months. hepatitis. The nurse is doing a patient teaching
with Mr. Fajardo. What kind of role does the
16. Which of the following statements nurse assume?
regarding the nursing process is true?
talker
It is useful on outpatient settings. teacher
It progresses in separate, unrelated steps. thinker
It focuses on the patient, not the nurse. Doer
It provides the solution to all patient health
problems. (B) teacher. The nurse will assume the role of
a teacher in this therapeutic relationship.
(C) It focuses on the patient, not the The other roles are inappropriate in this
nurse. The nursing process is patient- situation.
centered, not nurse-centered. It can be use
in any setting, and the steps are related. 20. When providing a continuous enteral
The nursing process can’t solve all patient feeding, which of the following action is
health problems. essential for the nurse to do?

17. Which of the following is considered Place the client on the left side of the bed.
significant enough to require immediate Attach the feeding bag to the current tubing.
communication to another member of the Elevate the head of the bed.
health care team? Cold the formula before administering it

Weight loss of 3 lbs in a 120 lb female (C) Elevate the head of the bed. Elevating the
patient. head of the bed during an enteral feeding
Diminished breath sounds in patient with prevents aspiration. The patient may be
previously normal breath sounds placed on the right side to prevent
Patient stated, “I feel less nauseated.” aspiration. Enteral feedings are given at
Change of heart rate from 70 to 83 beats per room temperature to lessen GI distress.
minute. The enteral tubing should be changed
every 24 hours to limit microbial growth.
(B) Diminished breath sounds in patient with .
previously normal breath 21. Kussmaul’s breathing is;
sounds. Diminished breath sound is a life
threatening problem therefore it is highly Shallow breaths interrupted by apnea.
priority because they pose the greatest Prolonged gasping inspiration followed by a
threat to the patient’s well-being. very short, usually inefficient expiration.
Marked rhythmic waxing and waning of
18. To assess the adequacy of food intake, respirations from very deep to very
which of the following assessment parameters shallow breathing and temporary apnea.
is best used? Increased rate and depth of respiration.

food preferences (D) Increased rate and depth of


regularity of meal times respiration. Kussmaul breathing is also
3-day diet recall called as hyperventilation. Seen in
eating style and habits metabolic acidosis and renal failure.
Option A refers to Biot’s breathing.
(C) 3-day diet recall. 3-day diet recall is an Option B is apneustic breathing and
example of dietary history. This is used to option C is the Cheyne-stokes breathing.
22. Presty has terminal cancer and she refuses .
to believe that loss is happening ans she
assumes artificial cheerfulness. What stage of
grieving is she in?
PNLE Fundamentals
depression in Nursing Exam 3
bargaining 1. Nurse Brenda is teaching a patient about a
denial newly prescribed drug. What could cause a
Acceptance geriatric patient to have difficulty retaining
knowledge about prescribed medications?
(C) denial. The client is in denial stage
because she is unready to face the reality Decreased plasma drug levels
that loss is happening and she assumes Sensory deficits
artificial cheerfulness. Lack of family support
History of Tourette syndrome
23. Immunization for healthy babies and
preschool children is an example of what level (B) Sensory deficits. Sensory deficits could
of preventive health care? cause a geriatric patient to have difficulty
retaining knowledge about prescribed
Primary medications. Decreased plasma drug
Secondary levels do not alter the patient’s knowledge
Tertiary about the drug. A lack of family support
Curative may affect compliance, not knowledge
retention. Toilette syndrome is unrelated
(A) Primary. The primary level focuses on to knowledge retention.
health promotion. Secondary level
focuses on health maintenance. Tertiary 2. When examining a patient with abdominal
focuses on rehabilitation. There is n pain the nurse in charge should assess:
Curative level of preventive health care
problems. Any quadrant first
The symptomatic quadrant first
24. Which is an example of a subjective data? The symptomatic quadrant last
The symptomatic quadrant either second or
Temperature of 38 0C third
Vomiting for 3 days
Productive cough (C) The symptomatic quadrant last. The
Patient stated, “My arms still hurt.” nurse should systematically assess all
areas of the abdomen, if time and the
(D) Patient stated, “My arms still patient’s condition permit, concluding
hurt.”. Subjective data are apparent only with the symptomatic area. Otherwise, the
to the person affected and can or verified nurse may elicit pain in the symptomatic
only by that person. area, causing the muscles in other areas to
tighten. This would interfere with further
25. The nurse is assessing the endocrine assessment.
system. Which organ is part of the endocrine
system? 3. The nurse is assessing a postoperative adult
patient. Which of the following should the
Heart nurse document as subjective data?
Sinus
Thyroid Vital signs
Thymus Laboratory test result
Patient’s description of pain
(C) Thyroid. The thyroid is part of the Electrocardiographic (ECG) waveforms
endocrine system. Heart, sinus and
thymus are not.
(C) Patient’s description of pain. Subjective (A) Shock dismay. Shock and dismay are
data come directly from the patient and early signs of denial-the first stage of
usually are recorded as direct quotations grief. The other options are associated
that reflect the patient’s opinions or with depression—a later stage of grief.
feelings about a situation. Vital signs,
laboratory test result, and ECG 7. The nurse in charge is transferring a patient
waveforms are examples of objective data. from the bed to a chair. Which action does the
nurse take during this patient transfer?
4. A male patient has a soft wrist-safety
device. Which assessment finding should the Position the head of the bed flat
nurse consider abnormal? Helps the patient dangle the legs
Stands behind the patient
A palpable radial pulse Places the chair facing away from the bed
A palpable ulnar pulse
Cool, pale fingers (B) Helps the patient dangle the legs. After
Pink nail beds placing the patient in high Fowler’s
position and moving the patient to the
(C) Cool, pale fingers. A safety device on the side of the bed, the nurse helps the patient
wrist may impair circulation and restrict sit on the edge of the bed and dangle the
blood supply to body tissues. Therefore, legs; the nurse then faces the patient and
the nurse should assess the patient for places the chair next to and facing the
signs of impaired circulation, such as cool, head of the bed.
pale fingers. A palpable radial or lunar
pulse and pink nail beds are normal 8. A female patient who speaks a little English
findings. has emergency gallbladder surgery, during
discharge preparation, which nursing action
5. Which of the following planes divides the would best help this patient understand wound
body longitudinally into anterior and posterior care instruction?
regions?
Asking frequently if the patient understands
Frontal plane the instruction
Sagittal plane Asking an interpreter to replay the
Midsagittal plane instructions to the patient.
Transverse plane Writing out the instructions and having a
family member read them to the patient
(A) Frontal plane. Frontal or coronal plane Demonstrating the procedure and having
runs longitudinally at a right angle to a the patient return the demonstration
sagittal plane dividing the body in
anterior and posterior regions. A sagittal (D) Demonstrating the procedure and having
plane runs longitudinally dividing the the patient return the
body into right and left regions; if exactly demonstration. Demonstrating by the
midline, it is called a midsagittal plane. A nurse with a return demonstration by the
transverse plane runs horizontally at a patient ensures that the patient can
right angle to the vertical axis, dividing perform wound care correctly. Patients
the structure into superior and inferior may claim to understand discharge
regions. instruction when they do not. An
interpreter of family member may
6. A female patient with a terminal illness is communicate verbal or written
in denial. Indicators of denial include: instructions inaccurately.

Shock dismay 9. Before administering the evening dose of a


Numbness prescribed medication, the nurse on the
Stoicism evening shift finds an unlabeled, filled syringe
Preparatory grief in the patient’s medication drawer. What
should the nurse in charge do?
Discard the syringe to avoid a medication giving role when providing direct care,
error including bathing patients and
Obtain a label for the syringe from the administering medications and prescribed
pharmacy treatments. The nurse acts as a patient
Use the syringe because it looks like it advocate when making the patient’s
contains the same medication the nurse wishes known to the doctor.
was prepared to give
Call the day nurse to verify the contents of 12. A female patient exhibits signs of
the syringe heightened anxiety. Which response by the
nurse is most likely to reduce the patient’s
(A) Discard the syringe to avoid a medication anxiety?
error. As a safety precaution, the nurse
should discard an unlabeled syringe that “Everything will be fine. Don’t worry.”
contains medication. The other options “Read this manual and then ask me any
are considered unsafe because they questions you may have.”
promote error. “Why don’t you listen to the radio?”
“Let’s talk about what’s bothering you.”
10. When administering drug therapy to a
male geriatric patient, the nurse must stay (D) “Let’s talk about what’s bothering
especially alert for adverse effects. Which you.” Anxiety may result from feeling of
factor makes geriatric patients to adverse drug helplessness, isolation, or insecurity. This
effects? response helps reduce anxiety by
encouraging the patient to express
Faster drug clearance feelings. The nurse should be supportive
Aging-related physiological changes and develop goals together with the
Increased amount of neurons patient to give the patient some control
Enhanced blood flow to the GI tract over an anxiety-inducing situation.
Because the other options ignore the
(B) Aging-related physiological patient’s feeling and block
changes. Aging-related physiological communication, they would not reduce
changes account for the increased anxiety.
frequency of adverse drug reactions in
geriatric patients. Renal and hepatic 13. A scrub nurse in the operating room has
changes cause drugs to clear more slowly which responsibility?
in these patients. With increasing age,
neurons are lost and blood flow to the GI Positioning the patient
tract decreases. Assisting with gowning and gloving
Handling surgical instruments to the
11. A female patient is being discharged after surgeon
cataract surgery. After providing medication Applying surgical drapes
teaching, the nurse asks the patient to repeat
the instructions. The nurse is performing (C) Handling surgical instruments to the
which professional role? surgeon. The scrub nurse assist the
surgeon by providing appropriate surgical
Manager instruments and supplies, maintaining
Educator strict surgical asepsis and, with the
Caregiver circulating nurse, accounting for all gauze,
Patient advocate sponges, needles, and instruments. The
circulating nurse assists the surgeon and
(B) Educator. When teaching a patient about scrub nurse, positions the patient, applies
medications before discharge, the nurse is appropriate equipment and surgical
acting as an educator. The nurse acts as a drapes, assists with gowning and gloving,
manager when performing such activities and provides the surgeon and scrub nurse
as scheduling and making patient care with supplies.
assignments. The nurse performs the care
14. A patient is in the bathroom when the 17. To evaluate a patient for hypoxia, the
nurse enters to give a prescribed medication. physician is most likely to order which
What should the nurse in charge do? laboratory test?

Leave the medication at the patient’s bedside Red blood cell count
Tell the patient to be sure to take the Sputum culture
medication. And then leave it at the Total hemoglobin
bedside Arterial blood gas (ABG) analysis
Return shortly to the patient’s room and
remain there until the patient takes the (D) Arterial blood gas (ABG) analysis. All of
medication these test help evaluate a patient with
Wait for the patient to return to bed, and then respiratory problems. However, ABG
leave the medication at the bedside analysis is the only test evaluates gas
exchange in the lungs, providing
(C) Return shortly to the patient’s room and information about patient’s oxygenation
remain there until the patient takes the status.
medication. The nurse should return
shortly to the patient’s room and remain 18. The nurse uses a stethoscope to auscultate
there until the patient takes the a male patient’s chest. Which statement about
medication to verify that it was taken as a stethoscope with a bell and diaphragm is
directed. The nurse should never leave true?
medication at the patient’s bedside unless
specifically requested to do so. The bell detects high-pitched sounds best
The diaphragm detects high-pitched
15. The physician orders heparin, 7,500 units, sounds best
to be administered subcutaneously every 6 The bell detects thrills best
hours. The vial reads 10,000 units per The diaphragm detects low-pitched sounds
millilitre. The nurse should anticipate giving best
how much heparin for each dose?
(B) The diaphragm detects high-pitched
¼ ml sounds best. The diaphragm of a
½ ml stethoscope detects high-pitched sound
¾ ml best; the bell detects low pitched sounds
1 ¼ ml best. Palpation detects thrills best.

(C) ¾ ml. The nurse solves the problem as 19. A male patient is to be discharged with a
follows: 10,000 units/7,500 units = 1 prescription for an analgesic that is a
ml/X 10,000 X = 7,500 X= 7,500/10,000 controlled substance. During discharge
or ¾ ml teaching, the nurse should explain that the
patient must fill this prescription how soon
16. The nurse in charge measures a patient’s after the date on which it was written?
temperature at 102 degrees F. what is the
equivalent Centigrade temperature? Within 1 month
Within 3 months
39 degrees C Within 6 months
47 degrees C Within 12 months
38.9 degrees C
40.1 degrees C (C) Within 6 months. In most cases, an
outpatient must fill a prescription for a
(C) 38.9 degrees C. To convert Fahrenheit controlled substance within 6 months of
degrees to centigrade, use this formula: C the date on which the prescription was
degrees = (F degrees – 32) x 5/9 C written.
degrees = (102 – 32) 5/9 + 70 x 5/9 38.9
degrees C
20. Which human element considered by the 22. All of the following parts of the syringe
nurse in charge during assessment can affect are sterile except the:
drug administration?
Barrel
The patient’s ability to recover Inside of the plunger
The patient’s occupational hazards Needle tip
The patient’s socioeconomic status Barrel tip
The patient’s cognitive abilities
(A) Barrel. All syringes have three parts: a
(D) The patient’s cognitive abilities. The tip, which connects the needle to the
nurse must consider the patient’s syringe; a barrel, the outer part on which
cognitive abilities to understand drug the measurement scales are printed; and a
instructions. If not, the nurse must find a plunger, which fits inside the barrel to
family member or significant other to take expel the medication. The external part of
on the responsibility of administering the barrel and the plunger and (flange)
medications in the home setting. The must be handled during the preparation
patient’s ability to recover, occupational and administration of the injection.
hazards, and socioeconomic status do not However, the inside and trip of the barrel,
affect drug administration. the inside (shaft) of the plunger, and the
needle tip must remain sterile until after
21. When explaining the initiation of I.V. the injection.
therapy to a 2-year-old child, the nurse should:
23. The best way to instill eye drops is to:
Ask the child, “Do you want me to start the
I.V. now?” Instruct the patient to lock upward, and
Give simple directions shortly before the drop the medication into the center of
I.V. therapy is to start the lower lid
Tell the child, “This treatment is for your Instruct the patient to look ahead, and drop
own good” the medication into the center of the lower
Inform the child that the needle will be in lid
place for 10 days Drop the medication into the inner canthus
regardless of eye position
(B) Give simple directions shortly before the Drop the medication into the center of the
I.V. therapy is to start. Because a 2-year- canthus regardless of eye position
old child has limited understanding, the
nurse should give simple directions and (A) Instruct the patient to lock upward, and
explanations of what will occur shortly drop the medication into the center of the
before the procedure. She should try to lower lid. Having the patient look upward
avoid frightening the child with the reduces blinking and protects the cornea.
explanation and allow the child to make Instilling drops in the center of the lower
simple choices, such as choosing the I.V. lid promotes absorption because the drops
insertion site, if possible. However, she are less likely to run into the nasolacrimal
shouldn’t ask the child if he wants the duct or out of the eye.
therapy, because the answer may be “No!”
Telling the child that the treatment is for
his own good is ineffective because a 2- 24. The difference between an 18G needle
year-old perceives pain as a negative and a 25G needle is the needle’s:
sensation and cannot understand that a Length
painful procedure can have position Bevel angle
results. Telling the child how long the Thickness
therapy will last is ineffective because the Sharpness
2-year-old doesn’t have a good
understanding of time. (C) Thickness. Gauge is a measure of the
needle’s thickness: The higher the number
the thinner the shaft. Therefore, an 18G
needle is considerably thicker than a 25G
needle. (D) 18th week of pregnancy. On the 8th week of
pregnancy, the uterus is still within the pelvic area.
On the 10th week, the uterus is still within the
25. A patient receiving an anticoagulant pelvic area. On the 12th week, the uterus and
should be assessed for signs of: placenta have grown, expanding into the abdominal
cavity. On the 18th week, the uterus has already
risen out of the pelvis and is expanding into the
Hypotension abdominal area.
Hypertension
An elevated hemoglobin count 4. Which of the following urinary symptoms does the
An increased number of erythrocytes pregnant woman most frequently experience during the
first trimester:

Maternal and Child Nursing a) frequency


b) dysuria
c) incontinence
d) Burning

PNLE Maternal and (A) frequency. Pressure and irritation of the bladder by
the growing uterus during the first trimester is
Child Health Nursing responsible for causing urinary frequency. Dysuria,
incontinence and burning are symptoms associated
with urinary tract infection.
Exam 1
1. A client asks the nurse what a third degree laceration 5. Mrs. Jimenez went to the health center for pre-natal
is. She was informed that she had one. The nurse check-up. the student nurse took her weight and revealed
explains that this is: 142 lbs. She asked the student nurse how much should
she gain weight in her pregnancy.
a) that extended their anal sphincter
b) through the skin and into the muscles a. 20-30 lbs
c) that involves anterior rectal wall b. 25-35 lbs
d) that extends through the perineal muscle. c. 30- 40 lbs
d. 10-15 lbs
Answer. (A) that extended their anal sphincter. Third
degree laceration involves all in the second degree (B) 25-35 lbs. A weight gain of 11. 2 to 15.9 kg (25 to
laceration and the external sphincter of the rectum. 35 lbs) is currently recommended as an average
Options B, C and D are under the second degree weight gain in pregnancy. This weight gain consists
laceration. of the following: fetus- 7.5 lb; placenta- 1.5 lb;
amniotic fluid- 2 lb; uterus- 2.5 lb; breasts- 1.5 to 3
lb; blood volume- 4 lb; body fat- 7 lb; body fluid- 7
2. Betina 30 weeks AOG discharged with a diagnosis of lb.
placenta previa. The nurse knows that the client
understands her care at home when she says: 6. The nurse is preparing Mrs. Jordan for cesarean
delivery. Which of the following key concept should the
A. I am happy to note that we can have sex occasionally nurse consider when implementing nursing care?
when I have no bleeding.
B. I am afraid I might have an operation when my due a) Explain the surgery, expected outcome and kind of
comes anesthetics.
C. I will have to remain in bed until my due date b) Modify preoperative teaching to meet the needs
comes of either a planned or emergency cesarean birth.
D/ I may go back to work since I stay only at the office. c) Arrange for a staff member of the anesthesia
department to explain what to expect post-
Answer. (C) I will have to remain in bed until my due operatively.
date comes. Placenta previa means that the placenta is d) Instruct the mother’s support person to remain in
the presenting part. On the first and second trimester the family lounge until after the delivery.
there is spotting. On the third trimester there is bleeding
that is sudden, profuse and painless (B) Modify preoperative teaching to meet the needs of
either a planned or emergency cesarean birth. A
3. The uterus has already risen out of the pelvis and is key point to consider when preparing the client for
experiencing farther into the abdominal area at about the: a cesarean delivery is to modify the preoperative
teaching to meet the needs of either planned or
a. 8th week of pregnancy emergency cesarean birth, the depth and breadth of
b. 10th week of pregnancy instruction will depend on circumstances and time
c. 12th week of pregnancy available.
d. 18th week of pregnancy
7. Bettine Gonzales is hospitalized for the treatment of indicate increases ICP or sunken, which may
severe preecplampsia. Which of the following represents indicate hydration.
an unusual finding for this condition?

a. generalized edema 11. Mrs. Quijones gave birth by spontaneous delivery to


b. proteinuria 4+ a full term baby boy. After a minute after birth, he is
c. blood pressure of 160/110 crying and moving actively. His birth weight is 6.8 lbs.
d. Convulsions What do you expect baby Quijones to weigh at 6 months?

(D) convulsions. Options A, B and C are findings of a. 13 -14 lbs


severe preeclampsia. Convulsions is a finding of b. 16 -17 lbs
eclampsia—an obstetrical emergency. c. 22 -23 lbs
d. 27 -28 lbs

8. Nurse Geli explains to the client who is 33 weeks A) 13 -14 lbs. The birth weight of an infant is doubled at
pregnant and is experiencing vaginal bleeding that coitus: 6 months and is tripled at 12 months

a) Need to be modified in any way by either partner 12. During the first hours following delivery, the post
b) Is permitted if penile penetration is not deep. partum client is given IVF with oxytocin added to them.
c) Should be restricted because it may stimulate The nurse understands the primary reason for this is:
uterine activity.
d) Is safe as long as she is in side-lying position. a) To facilitate elimination
(C) Should be restricted because it may stimulate b) To promote uterine contraction
uterine activity.. Coitus is restricted when there is c) To promote analgesia
watery discharge, uterine contraction and vaginal d) To prevent infection
bleeding. Also those women with a history of B To promote uterine contraction. Oxytocin is a
spontaneous miscarriage may be advised to avoid hormone produced by the pituitary gland that produces
coitus during the time of pregnancy when a intermittent uterine contractions, helping to promote
previous miscarriage occurred. uterine involution.

9. Mrs. Precilla Abuel, a 32 year old mulripara is 13. Nurse Luis is assessing the newborn’s heart rate.
admitted to labor and delivery. Her last 3 pregnancies in Which of the following would be considered normal if
short stage one of labor. The nurses decide to observe the newborn is sleeping?
her closely. The physician determines that Mrs. Abuel’s
cervix is dilated to 6 cm. Mrs. Abuel states that she is a. 80 beats per minute
extremely uncomfortable. To lessen Mrs. Abuel’s b. 100 beats per minute
discomfort, the nurse can advise her to: c. 120 beats per minute
d. 140 beats per minute
a. lie face down
b. not drink fluids (B) 100 beats per minute. The normal heart rate for a
c. practice holding breaths between contractions newborn that is sleeping is approximately 100 beats
d. assume Sim’s position per minute. If the newborn was awake, the normal
heart rate would range from 120 to 160 beats per
(D) assume Sim’s position. When the woman is in minute.
Sim’s position, this puts the weight of the fetus on
bed, not on the woman and allows good circulation 14. The infant with Down Syndrome should go through
in the lower extremities. which of the Erikson’s developmental stages first?

10. Which is true regarding the fontanels of the newborn? a) Initiative vs. Self doubt
b) Industry vs. Inferiority
a) The anterior is large in shape when compared to c) Autonomy vs. Shame and doubt
the posterior fontanel. d) Trust vs. Mistrust
b) The anterior is triangular shaped; the posterior is
diamond shaped. D) Trust vs. Mistrust. The child with Down syndrome
c) The anterior is bulging; the posterior appears will go through the same first stage, trust vs.
sunken. mistrust, only at a slow rate. Therefore, the nurse
d) The posterior closes at 18 months; the anterior should concentrate on developing on bond between
closes at 8 to 12 months. the primary caregiver and the child.

(A) The anterior is large in shape when compared to the 15. The child with phenylketonuria (PKU) must
posterior fontanel.. The anterior fontanel is larger in maintain a low phenylalanine diet to prevent which of
size than the posterior fontanel. Additionally, the the following complications?
anterior fontanel, which is diamond shaped closes
at 18 month, whereas the posterior fontanel, which a. Irreversible brain damage
is triangular in shape closes at 8 to 12 weeks. b. Kidney failure
Neither fontanel should appear bulging, which may c. Blindness
d. Neutropenia
observe that the child does not show any relief from the
(A) Irreversible brain damage. The child with PKU must treatment given. Upon assessment, you noticed that both
maintain a strict low phenylalanine diet to prevent the heart and respiratory rate are still elevated and the
central nervous system damage, seizures and eventual child shows difficulty of exhaling. You suspect:
dea
a. Bronchiectasis
16. Which age group is with imaginative minds and b. Atelectasis
creates imaginary friends? c. Epiglotitis
d. Status Asthmaticus
a) Toddler (D) Status Asthmaticus. Status asthmaticus leads to
b) Preschool respiratory distress and bronchospasm despite of
c) School treatment and interventions. Mechanical ventilation
d) Adolescence maybe needed due to respiratory failure.

(B) Preschool. During preschool, this is the time when 21. Nurse Jonas assesses a 2 year old boy with a
children do imitative play, imaginative play—the tentative diagnosis of nephroblastoma. Symptoms the
occurrence of imaginative playmates, dramatic play nurse observes that suggest this problem include:
where children like to act, dance and sing.
a) Lymphedema and nerve palsy
17. Which of the following situations would alert you to b) Hearing loss and ataxia
a potentially developmental problem with a child? c) Headaches and vomiting
d) Abdominal mass and weakness
a) Pointing to body parts at 15 months of age.
b) Using gesture to communicate at 18 months. (D)Abdominal mass and weakness. Nephroblastoma or
c) Cooing at 3 months. Wilm’s tumor is caused by chromosomal abnormalities,
d) Saying “mama” or “dada” for the first time at 18 most common kidney cancer among children
months of age. characterized by abdominal mass, hematuria,
hypertension and fever
(D) Saying “mama” or “dada” for the first time at 18
months of age.. A child should say “mama” or 22. Which of the following danger sings should be
“dada” during 10 to 12 months of age. Options A, reported immediately during the antepartum period?
B and C are all normal assessments of language
development of a child. a. blurred vision
b. nasal stuffiness
c. breast tenderness
18. Isabelle, a 2 year old girl loves to move around and d. Constipation
oftentimes manifests negativism and temper tantrums. (A) blurred vision. Danger signs that require prompt
What is the best way to deal with her behavior? reporting are leaking of amniotic fluid, blurred
vision, vaginal bleeding, rapid weight gain and
a. Tell her that she would not be loved by others is she elevated blood pressure. Nasal stuffiness, breast
behaves that way.. tenderness, and constipation are common
b. Withholding giving her toys until she behaves discomforts associated with pregnancy.
properly.
c. Ignore her behavior as long as she does not hurt 23. Nurse Jacob is assessing a 15 month old child with
herself and others. acute otitis media. Which of the following symptoms
d. Ask her what she wants and give it to pacify her. would the nurse anticipate finding?
(B) Ignore her behavior as long as she does not hurt
herself and others.. If a child is trying to get attention or a) periorbital edema, absent light reflex and
trying to get something through tantrums—ignore translucent tympanic membrane
his/her behavio b) irritability, purulent drainage in middle ear,
nasal congestion and cough
19. Baby boy Villanueva, 4 months old, was seen at the c) diarrhea, retracted tympanic membrane and
pediatric clinic for his scheduled check-up. By this enlarged parotid gland
period, baby Villanueva has already increased his height d) Vomiting, pulling at ears and pearly white tympanic
by how many inches? membrane
(B) irritability, purulent drainage in middle ear, nasal
a) 3 inches congestion and cough. Irritability, purulent
b) 4 inches drainage in middle ear, nasal congestion and cough,
c) 5 inches fever, loss of appetite, vomiting and diarrhea are
d) 6 inches clinical manifestations of otitis media. Acute otitis
(B) 4 inches. From birth to 6 months, the infant grows 1 media is common in children 6 months to 3 years
inch (2.5 cm) per month. From 6 to 12 months, the old and 8 years old and above. Breast fed infants
infant grows ½ inch (1.25 cm) per month. have higher resistance due to protection of
Eustachian tubes and middle ear from breast milk.
20. Alice, 10 years old was brought to the ER because of
Asthma. She was immediately put under aerosol
administration of Terbutaline. After sometime, you
24. Which of the following is the most appropriate b. Fetal movement
intervention to reduce stress in a preterm infant at 33 c. Maternal blood pressure
weeks gestation? d. Maternal uterine contractions

a. Sensory stimulation including several senses at a (B) Fetal movement. Non-stress test measures response
time of the FHR to the fetal movement. With fetal
b. tactile stimulation until signs of over stimulation movement, FHR increase by 15 beats and remain
develop for 15 seconds then decrease to average rate. No
c. An attitude of extension when prone or side lying increase means poor oxygenation perfusion to fetus.
d. Kangaroo care
2. During a 2 hour childbirth focusing on labor and
(D) Kangaroo care. Kangaroo care is the use of skin-to- delivery process for primigravida. The nurse describes
skin contact to maintain body heat. This method of the second maneuver that the fetus goes through during
care not only supplies heat but also encourages labor progress when the head is the presenting part as
parent-child interaction. which of the following:

25. The parent of a client with albinism would need to be a. Flexion


taught which preventive healthcare measure by the nurse: b. Internal rotation
c. Descent
a) Ulcerative colitis diet d. External rotation
b) Use of a high-SPF sunblock (A) Flexion. The 6 cardinal movements of labor are
c) Hair loss monitoring descent, flexion, internal rotation, extension,
d) Monitor for growth retardation external rotation and expulsion.
(B) Use of a high-SPF sunblock. Without melanin
production, the child with albinism is at risk for 3. Mrs. Jovel Diaz went to the hospital to have her serum
severe sunburns. Maximum sun protection should blood test for alpha-fetoprotein. The nurse informed her
be taken, including use of hats, long sleeves, about the result of the elevation of serum AFP. The
minimal time in the sun and high-SPF sunblock, to patient asked her what was the test for:
prevent any problems.
a) Congenital Adrenal Hyperplasia
b) PKU
c) Down Syndrome
d) Neural tube defects

(D)Neural tube defects. Alpha-fetoprotein is a substance


produces by the fetal liver that is present in amniotic
fluid and maternal serum. The level is abnormally high
in the maternal serum if the fetus has an open spinal or
abdominal defect because the open defect allows more
AFP to appear

4. Fetal heart rate can be auscultated with a fetoscope as


early as:

a. 5 weeks of gestation
b. 10 weeks of gestation
c. 15 weeks of gestation
d. 20 weeks of gestation

(D) 20 weeks of gestation. The FHR can be auscultated


with a fetoscope at about 20 weeks of gestation.
FHR is usually auscultated at the midline
suprapubic region with Doppler ultrasound at 10 to
12 weeks of gestation. FHR cannot be heard any
earlier than 10 weeks of gestation.
PNLE Maternal and Child 5. Mrs. Bendivin states that she is experiencing aching
swollen, leg veins. The nurse would explain that this is
Health Nursing Exam 2 most probably the result of which of the following:

a) Thrombophlebitis
b) PIH
1. Nurse Bella explains to a 28 year old pregnant woman c) Pressure on blood vessels from the enlarging
undergoing a non-stress test that the test is a way of uterus
evaluating the condition of the fetus by comparing the d) The force of gravity pulling down on the uterus
fetal heart rate with:
(C) Pressure on blood vessels from the enlarging
a. Fetal lie uterus. Pressure of the growing fetus on blood
vessels results in an increase risk for venous stasis c. Rubella titer less than 1:8
in the lower extremities. Subsequently, edema and d. One hour glucose challenge test 110 g/dL
varicose vein formation may occur.
C Rubella titer less than 1:8. A rubella titer should be 1:8
6. Mrs. Ella Santoros is a 25 year old primigravida who or greater. Thus, a finding of a titer less than 1:8 is
has Rheumatic heart disease lesion. Her pregnancy has significant, indicating that the client may not possess
just been diagnosed. Her heart disease has not caused her immunity to rubella. A hematocrit of 33.5%, WBC of
to limit physical activity in the past. Her cardiac disease 8,000/mm3, and a 1 hour glucose challenge test of 110
and functional capacity classification is: g/dL are within normal parameters.

a. Class I
b. Class II 11. Aling Patricia is a patient with preeclampsia. You
c. Class III advise her about her condition, which would tell you that
d. class IV she has not really understood your instructions?

(A) Class I. Clients under class I has no physical activity a) “I will restrict my fat in my diet.”
limitation. There is a slight limitation of physical activity b) “I will limit my activities and rest more
in class II, ordinary activity causes fatigue, palpitation, frequently throughout the day.”
dyspnea or angina. Class III is moderate limitation of c) “I will avoid salty foods in my diet.”
physical activity; less than ordinary activity causes d) “I will come more regularly for check-up.”
fatigue. Unable to carry on any activity without
experiencing discomfort is under class IV (B) “I will limit my activities and rest more frequently
throughout the day.”Pregnant woman with
7. The client asks the nurse, “When will this soft spot at preeclampsia should be in a complete bed rest.
the top of the head of my baby will close?” The nurse When body is in recumbent position, sodium tends
should instruct the mother that the neonate’s anterior to be excreted at a faster rate. It is the best method
fontanel will normally close by age: of aiding increased excretion of sodium and
encouraging diuresis. Rest should always be in a
a) 2-3 months lateral recumbent position to avoid uterine pressure
b) 6-8 months on the vena cava and prevent supine hypotension.
c) 10-12 months
d) 12-18 months 12. Mrs. Grace Evangelista is admitted with severe
preeclampsia. What type of room should the nurse select
(D) 12-18 months. Anterior fontanel closes at 12-18 this patient?
months while posterior fontanel closes at birth until
2 months. a. A room next to the elevator.
b. The room farthest from the nursing station.
8. When a mother bleeds and the uterus is relaxed, soft c. The quietest room on the floor.
and non-tender, you can account the cause to: d. The labor suite.
(C) The quietest room on the floor.A loud noise such as
a. Atony of the uterus a crying baby, or a dropped tray of equipment may
b. Presence of uterine scar be sufficient to trigger a seizure initiating eclampsia,
c. Laceration of the birth canal a woman with severe preeclampsia should be
d. Presence of retained placenta fragments admiotted to a private room so she can rest as
undisturbed as possible. Darken the room if
A) Atony of the uterus. Uterine atony, or relaxation of possible because bright light can trigger seizures.
the uterus is the most frequent cause of postpartal
hemorrhage. It is the inability to maintain the uterus in 13. During a prenatal check-up, the nurse explains to a
contracted state. client who is Rh negative that RhoGAM will be given:

9. Mrs. Pichie Gonzales’s LMP began April 4, 2010. Her a) Weekly during the 8th month because this is her
EDD should be which of the following: third pregnancy.
b) During the second trimester, if amniocentesis
a) February 11, 2011 indicates a problem.
b) January 11, 20111 c) To her infant immediately after delivery if the
c) December 12, 2010 Coomb’s test is positive.
d) Nowember 14, 2010 d) Within 72 hours after delivery if infant is found
to be Rh positive.
(B) January 11, 20111. Using the Nagel’s rule, he use
this formula ( -3 calendar months + 7 days). (D) Within 72 hours after delivery if infant is found to
be Rh positive. RhoGAM is given to Rh-negative
mothers within 72 hours after birth of Rh-positive
10. Which of the following prenatal laboratory test baby to prevent development of antibodies in the
values would the nurse consider as significant? maternal blood stream, which will be fata to
succeeding Rh-positive offspring.
a. Hematocrit 33.5%
b. WBC 8,000/mm3
14. A baby boy was born at 8:50pm. At 8:55pm, the b) an excess of WBC
heart rate was 99 bpm. She has a weak cry, irregular c) a deficiency of clotting factor VIII
respiration. She was moving all extremities and only her d) a deficiency of clotting factor IX
hands and feet were still slightly blue. The nurse should
enter the APGAR score as: C) a deficiency of clotting factor VIII. Hemophillia A
(classic hemophilia) is a deficiency in factor VIII
a. 5 (an alpha globulin that stabilizes fibrin clots).
b. 6
c. 7
d. 8 19. Celine, a mother of a 2 year old tells the nurse that
her child “cries and has a fit when I have to leave him
(B) 6. Heart rate of 99 bpm-1; weak cry-1; irregular with a sitter or someone else.” Which of the following
respiration-1; moving all extremities-2; extremities statements would be the nurse’s most accurate analysis
are slightly blue-1; with a total score of 6. of the mother’s comment?

15. Billy is a 4 year old boy who has an IQ of 140 which a. The child has not experienced limit-setting or
means: structure.
b. The child is expressing a physical need, such as
a. average normal hunger.
b. very superior c. The mother has nurtured overdependence in the
c. above average child.
d. Genius d. The mother is describing her child’s separation
anxiety.
(D) genius. IQ= mental age/chronological age x 100. (D) The mother is describing her child’s separation
Mental age refers to the typical intelligence level anxiety. Before coming to any conclusion, the
found for people at a give chronological age. OQ of nurse should ask the mother focused questions;
140 and above is considered genius. however, based on initial information, the analysis
of separation anxiety would be most valid.
Separation anxiety is a normal toddler response.
16. A newborn is brought to the nursery. Upon When the child senses he is being sent away from
assessment, the nurse finds that the child has short those who most provide him with love and security.
palpebral fissures, thinned upper lip. Based on this data, Crying is one way a child expresses a physical need;
the nurse suspects that the newborn is MOST likely however, the nurse would be hasty in drawing this
showing the effects of: as first conclusion based on what the mother has
said. Nurturing overdependence or not providing
a) Chronic toxoplasmosis structure for the toddler are inaccurate conclusions
b) Lead poisoning based on the information provided.
c) Congenital anomalies
d) Fetal alcohol syndrome 20. Mylene Lopez, a 16 year old girl with scoliosis has
recently received an invitation to a pool party. She asks
(D) Fetal alcohol syndrome. The newborn with fetal the nurse how she can disguise her impairment when
alcohol syndrome has a number of possible dressed in a bathing suit. Which nursing diagnosis can
problems at birth. Characteristics that mark the be justified by Mylene’s statement?
syndrome include pre and postnatal growth
retardation; CNS involvement such as cognitive a) Anxiety
challenge, microcephally and cerebral palsy; and a b) Body image disturbance
distinctive facial feature of a short palpebral fissure c) Ineffective individual coping
and thin upper lip. d) Social isolation

17. A priority nursing intervention for the infant with (B) Body image disturbance. Mylene is experiencing
cleft lip is which of the following: uneasiness about the curvative of her spine, which
will be more evident when she wears a bathing suit.
a. Monitoring for adequate nutritional intake This data suggests a body image disturbance. There
b. Teaching high-risk newborn care is no evidence of anxiety or ineffective coping. The
c. Assessing for respiratory distress fact that Mylene is planning to attend a pool party
d. Preventing injury dispels a diagnosis of social isolation.

(A) Monitoring for adequate nutritional intake. The 21. The foul-smelling, frothy characteristic of the stool
infant with cleft lip is unable to create an adequate in cystic fibrosis results from the presence of large
seal for sucking. The child is at risk for inadequate amounts of which of the following:
nutritional intake as well as aspiration.
a. sodium and chloride
18. Nurse Jacob is assessing a 12 year old who has b. undigested fat
hemophilia A. Which of the following assessment c. semi-digested carbohydrates
findings would the nurse anticipate? d. lipase, trypsin and amylase

a) an excess of RBC
(B) undigested fat. The client with cystic fibrosis crisis. During a crisis such as that seen in sickle cell
absorbs fat poorly because of the think secretions anemia, aldosterone release is stimulated. In what way
blocking the pancreatic duct. The lack of natural might this influence Catherine’s fluid and electrolyte
pancreatic enzyme leads to poor absorption of balance?
predominantly fats in the duodenum. Foul-smelling,
frothy stool is termed steatorrhea. a. sodium loss, water loss and potassium retention
b. sodium loss, water los and potassium loss
22. Which of the following would be a disadvantage of c. sodium retention, water loss and potassium
breast feeding? retention
d. sodium retention, water retention and potassium
a) involution occurs rapidly loss
b) the incidence of allergies increases due to maternal
antibodies (D) sodium retention, water retention and potassium
c) the father may resent the infant’s demands on loss. Stress stimulates the adrenal cortex to increase the
the mother’s body release of aldosterone. Aldosterone promotes the
d) there is a greater chance of error during preparation resorption of sodium, the retention of water and the loss
of potassium
(C) the father may resent the infant’s demands on the
mother’s body. With breast feeding, the father’s
body is not capable of providing the milk for the
newborn, which may interfere with feeding the
newborn, providing fewer chances for bonding, or
he may be jealous of the infant’s demands on his PNLE Maternal and
wife time and body. Breast feeding is advantageous
because uterine involution occurs more rapidly,
thus minimizing blood loss. The presence of
maternal antibodies in breast milk helps decrease
Child Health Nursing
the incidence of allergies in the newborn. A greater
chance for error is associated with bottle feeding.
No preparation required for breast feeding.
Exam 3
1. A pregnant woman who is at term is admitted to
23. A client is noted to have lymphedema, webbed neck
and low posterior hairline. Which of the following the birthing unit in active labor. The client has only
diagnoses is most appropriate? progressed from 2cm to 3 cm in 8 hours. She is
diagnosed with hypotonic dystocia and the
a. Turner’s syndrome physician ordered Oxytocin (Pitocin) to augment
b. Down’s syndrome her contractions. Which of the following is the most
c. Marfan’s syndrome important aspect of nursing intervention at this time?
d. Klinefelter’s syndrome
a) Timing and recording length of contractions.
(A) Turner’s syndrome. Lymphedema, webbed neck b) Monitoring.
and low posterior hairline, these are the 3 key c) Preparing for an emergency cesarean birth.
assessment features in Turner’s syndrome. If the d) Checking the perineum for bulging.
child is diagnosed early in age, proper treatment
can be offered to the family. All newborns should
be screened for possible congenital defects. A. The oxytocic effect of Pitocin increases the
intensity and durations of contractions;
prolonged contractions will jeopardize the
24. A 4 year old boy most likely perceives death in safetyof the fetus and necessitate discontinuing
which way: the drug.

a) An insignificant event unless taught otherwise 2. A client who hallucinates is not in touch with
b) Punishment for something the individual did reality. It is important for the nurse to:
c) Something that just happens to older people
d) Temporary separation from the loved one. a. Isolate the client from other patients.
b. Maintain a safe environment.
(D) Temporary separation from the loved one. The
c. Orient the client to time, place, and person.
predominant perception of death by preschool age
children is that death is temporary separation. d. Establish a trusting relationship.
Because that child is losing someone significant
and will not see that person again, it’s inaccurate to B. It is of paramount importance to prevent the
infer death is insignificant, regardless of the child’s client from hurting himself or herself or others
response.
3. The nurse is caring to a child client who has had
25. Catherine Diaz is a 14 year old patient on a a tonsillectomy. The child complains of having
hematology unit who is being treated for sickle cell
dryness of the throat. Which of the following would c. Decreased light and near-vision
the nurse give to the child? accommodation due to miotic effects of
pilocarpine.
a) Cola with ice d. The painful and insidious progression of this
b) Yellow noncitrus Jello type of glaucoma.
c) Cool cherry Kool-Aid
d) A glass of milk C. The most frequent cause of noncompliance to
the treatment of chronic, or open-angle
B. After tonsillectomy, clear, cool liquids should glaucoma is the miotic effects of pilocarpine.
be given. Citrus, carbonated, and hot or cold Pupillary constriction impedes normal
liquids should be avoided because they may accommodation, making night driving difficult
irritate the throat. Red liquids should be and hazardous, reducing the client’s ability to
avoided because they give the appearance of read for extended periods and making
blood if the child vomits. Milk and milk participation in games with fast-moving objects
products including pudding are avoided impossible.
because they coat the throat, cause the child to
clear the throat, and increase the risk of 7. In the morning shift, the nurse is making rounds
bleeding. in the nursing care units. The nurse enters in a
client’s room and notes that the client’s tube has
4. The physician ordered Phenylephrine (Neo- become disconnected from the Pleurovac. What
Synephrine) nasal spray to a 13-year-old client. The would be the initial nursing action?
nurse caring to the client provides instructions that
the nasal spray must be used exactly as directed to a) Apply pressure directly over the incision site.
prevent the development of: b) Clamp the chest tube near the incision site.
c) Clamp the chest tube closer to the drainage
a. Increased nasal congestion. system.
b. Nasal polyps. d) Reconnect the chest tube to the Pleurovac.
c. Bleeding tendencies.
d. Tinnitus and diplopia. B. This stops the sucking of air through the tube
and prevents the entry of contaminants. In
A. Phenylephrine, with frequent and continued use, addition, clamping near the chest wall provides
can cause rebound congestion of mucous for some stability and may prevent the clamp
membranes. from pulling on the chest tube.

5. A client with tuberculosis is to be admitted in the 8. Which of the following complications during a
hospital. The nurse who will be assigned to care for breech birth the nurse needs to be alarmed?
the client must institute appropriate precautions.
The nurse should: a. Abruption placenta.
b. Caput succedaneum.
a) Place the client in a private room. c. Pathological hyperbilirubinemia.
b) Wear an N 95 respirator when caring for the d. Umbilical cord prolapse.
client.
c) Put on a gown every time when entering the D. Because umbilical cord’s insertion site is born
room. before the fetal head, the cord may be
d) Don a surgical mask with a face shield when compressed by the after-coming head in a
entering the room. breech birth.

B. The N 95 respirator is a high-particulate


filtration mask that meets the CDC 9. The nurse is caring to a client diagnosed with
performance criteria for a tuberculosis severe depression. Which of the following nursing
respirator. approach is important in depression?

6. Which of the following is the most frequent cause a) Protect the client against harm to others.
of noncompliance to the medical treatment of open- b) Provide the client with motor outlets for
angle glaucoma? aggressive, hostile feelings.
c) Reduce interpersonal contacts.
a. The frequent nausea and vomiting d) Deemphasizing preoccupation with elimination,
accompanying use of miotic drug. nourishment, and sleep.
b. Loss of mobility due to severe driving
restrictions. B. It is important to externalize the anger away
from self.
b. Report the incident to the nursing supervisor
10. A 3-month-old client is in the pediatric unit. and request to be floated.
During assessment, the nurse is suspecting that the c. Report the nursing assessment of the client
baby may have hypothyroidism when mother states in transitional labor to the nurse manager
that her baby does not: and discuss misgivings about the new
assignment.
a. Sit up. Accept the new assignment and provide the best
b. Pick up and hold a rattle. care.
c. Roll over.
d. Hold the head up. C. The nurse is obligated to inform the nurse
manager about changes in the condition of the
D. Development normally proceeds client, which may change the decision made by
cephalocaudally; so the first major the nurse manager.
developmental milestone that the infant
achieves is the ability to hold the head up 13. A newborn infant with Down syndrome is to be
within the first 8-12 weeks of life. In discharged today. The nurse is preparing to give the
hypothyroidism, the infant’s muscle tone discharge teaching regarding the proper care at
would be poor and the infant would not be able home. The nurse would anticipate that the mother is
to achieve this milestone. probably at the:

a) 40 years of age.
11. The physician calls the nursing unit to leave an b) 20 years of age.
order. The senior nurse had conversation with the c) 35 years of age.
other staff. The newly hired nurse answers the d) 20 years of age.
phone so that the senior nurses may continue their
conversation. The new nurse does not knowthe A. Perinatal risk factors for the development of
physician or the client to whom the order pertains. Down syndrome include advanced maternal
The nurse should: age, especially with the first pregnancy.

a) Ask the physician to call back after the nurse 14. The emergency department has shortage of staff.
has read the hospital policy manual. The nurse manager informs the staff nurse in the
b) Take the telephone order. critical care unit that she has to float to the
c) Refuse to take the telephone order. emergency department. What should the staff nurse
d) Ask the charge nurse or one of the other expect under these conditions?
senior staff nurses to take the telephone
order. a. The float staff nurse will be informed of the
D. Get a senior nurse who know s the policies, the situation before the shift begins.
client, and the doctor. Generally speaking, a b. The staff nurse will be able to negotiate the
nurse should not accept telephone orders. assignments in the emergency department.
However, if it is necessary to take one, follow c. Cross training will be available for the staff
the hospital’s policy regarding telephone orders. nurse.
Failure to followhospital policy could be d. Client assignments will be equally divided
considered negligence. In this case, the nurse among the nurses.
was new and did not know the hospital’s policy B. Assignments should be based on scope of
concerning telephone orders. The nurse was practice and expertise.
also unfamiliar with the doctor and the client.
Therefore the nurse should not take the order 15. The nurse is assigned to care for a child client
unless a) no one else is available and b) it is an admitted in the pediatrics unit. The client is
emergency situation. receiving digoxin. Which of the following questions
will be asked by the nurse to the parents of the child
12. The staff nurse on the labor and delivery unit is in order to assess the client’s risk for digoxin
assigned to care to a primigravida in transition toxicity?
complicated by hypertension. A new pregnant
woman in active labor is admitted in the same unit. a) “Has he been exposed to any childhood
The nurse manager assigned the same nurse to the communicable diseases in the past 2-3 weeks?”
second client. The nurse feels that the client with b) “Has he been taking diuretics at home?”
hypertension requires one-to-one care. What would c) “Do any of his brothers and sisters have history
be the initial actionof the nurse? of cardiac problems?”
d) “Has he been going to school regularly?”
a. Accept the new assignment and complete an
incident report describing a shortage of nursing B. The child who is concurrently taking digoxin
staff. and diuretics is at increased risk for digoxin
toxicity due to the loss of potassium. The child
and parents should be taught what foods are 19. Before the administration of digoxin, the nurse
high in potassium, and the child should be completes an assessment to a toddler client for signs
encouraged to eat a high-potassium diet. In and symptoms of digoxin toxicity. Which of the
addition, the child’s serum potassium level following is the earliest and most significant sign of
should be carefully monitored. digoxin toxicity?

16. The nurse noticed that the signed consent form a) Tinnitus
has an error. The form states, “Amputation of the b) Nausea and vomiting
right leg” instead of the left leg that is to be c) Vision problem
amputated. The nurse has administered already the d) Slowing in the heart rate
preoperative medications. What should the nurse do?
D. One of the earliest signs of digoxin toxicity is
a. Call the physician to reschedule the surgery. Bradycardia. For a toddler, any heart rate that
b. Call the nearest relative to come in to sign a falls below the norm of about 100-120 bpm
new form. would indicate Bradycardia and would
c. Cross out the error and initial the form. necessitate holding the medication and
d. Have the client sign another form. notifying the physician.

A. The responsible for an accurate informed consent 20. Which of the following treatment modality is
is the physician. An exception to this answer would appropriate for a client with paranoid tendency?
be a life-threatening emergency, but there are no
data to support another response a. Activity therapy.
b. Individual therapy.
17. The nurse in the nursing care unit checks the c. Group therapy.
fluctuation in the water-seal compartment of a d. Family therapy.
closed chest drainage system. The fluctuation has
stopped, the nurse would: B. This option is least threatening.

a) Vigorously strip the tube to dislodge a clot. 21. The client with rheumatoid arthritis is for
b) Raise the apparatus above the chest to move discharge. In preparing the client for discharge on
fluid. prednisone therapy, the nurse should advise the
c) Increase wall suction above 20 cm H2O client to:
pressure.
d) Ask the client to cough and take a deep a) Wear sunglasses if exposed to bright light for
breath. an extended period of time.
b) Take oral preparations of prednisone before
D. Asking the client to cough and take a deep breath meals.
will help determine if the chest tube is kinked or if c) Have periodic complete blood counts while on
the lungs has reexpanded the medication.
d) Never stop or change the amount of the
18. The pediatric nurse in the neonatal unit was medication without medical advice.
informed that the baby that is brought to the mother
in the hospital room is wrong. The nurse determines D. In preparing the client for discharge that is
that two babies were placed in the wrong cribs. The receiving prednisone, the nurse should caution the
most appropriate nursing action would be to: client to (a) take oral preparations after meals; (b)
remember that routine checks of vital signs, weight,
a. Determine who is responsible for the mistake and lab studies are critical; (c) NEVER STOP OR
and terminate his or her employment. CHANGE THE AMOUNT OF MEDICATION
b. Record the event in an incident/variance WITHOUT MEDICAL ADVICE; (d) store the
report and notify the nursing supervisor. medication in a light-resistant container.
c. Reassure both mothers, report to the charge
nurse, and do not record. 22. A pregnant client tells the nurse that she is
d. Record detailed notes of the event on the worried about having urinary frequency. What will
mother’s medical record. be the most appropriate nursing response?

B. Every event that exposes a client to harm should a. “Try using Kegel (perineal) exercises and
be recorded in an incident report, as well as limiting fluids before bedtime. If you have
reported to the appropriate supervisors in order frequency associated with fever, pain on
to resolve the current problems and permit the voiding, or blood in the urine, call your
institution to prevent the problem from doctor/nurse-midwife.
happening again.
b. “Placental progesterone causes irritability of b) Check the phenytoin solution to be sure it is
the bladder sphincter. Your symptoms will go clear or light yellow in color, never cloudy.
away after the baby comes.” c) Plan to give phenytoin over 30-60 minutes,
c. “Pregnant women urinate frequently to get rid using an in-line filter.
of fetal wastes. Limit fluids to 1L/daily.” d) Flush the IV tubing with normal saline before
d. “Frequency is due to bladder irritation from starting phenytoin.
concentrate urine and is normal in pregnancy.
Increase your daily fluid intake to 3L.” A. Phenytoin should be infused or injected into
larger veins to avoid the discoloration know as
A. Progesterone also reduces smooth muscle purple glove syndrome; infusing into a smaller
motility in the urinary tract and predisposes the vein is not appropriate.
pregnant woman to urinary tract infections.
Women should contact their doctors if they
exhibit signs of infection. Kegel exercise will 26. The pregnant woman visits the clinic for check
help strengthen the perineal muscles; limiting –up. Which assessment findings will help the nurse
fluids at bedtime reduces the possibility of determine that the client is in 8-week gestation?
being awakened by the necessity of voiding.
a. Leopold maneuvers.
23. Which of the following will help the nurse b. Fundal height.
determine that the expression of hostility is useful? c. Positive radioimmunoassay test (RIA test).
d. Auscultation of fetal heart tones.
a) Expression of anger dissipates the energy.
b) Energy from anger is used to accomplish C. Serum radioimmunoassay (RIA) is accurate
what needs to be done. within 7days of conception. This test is specific
c) Expression intimidates others. for HCG, and accuracy is not compromised by
d) Degree of hostility is less than the provocation. confusion with LH.

B. This is the proper use of anger.


27. Which of the following nursing intervention is
essential for the client who had pneumonectomy?
24. The nurse is providing an orientation regarding
case management to the nursing students. Which a) Medicate for pain only when needed.
characteristics should the nurse include in the b) Connect the chest tube to water-seal drainage.
discussion in understanding case management? c) Notify the physician if the chest drainage
exceeds 100mL/hr.
a. Main objective is a written plan that combines d) Encourage deep breathing and coughing.
discipline-specific processes used to measure
outcomes of care. D. Surgery and anesthesia can increase mucus
b. Main purpose is to identify expected client, production. Deep breathing and coughing are
family and staff performance against the essential to prevent atelectasis and pneumonia
timeline for clients with the same diagnosis. in the client’s only remaining lung.
c. Main focus is comprehensive coordination of
client care, avoid unnecessary duplication of 28. The nurse is providing a health teaching to a
services, improve resource utilization and group of parents regarding Chlamydia trachomatis.
decrease cost. The nurse is correct in the statement, “Chlamydia
d. Primary goal is to understand why predicted trachomatis is not only an intracellular bacterium
outcomes have not been met and the correction that causes neonatal conjunctivitis, but it also can
of identified problems. cause:

C. There are several models of case management, a. Discoloration of baby and adult teeth.
but the commonality is comprehensive b. Pneumonia in the newborn.
coordination of care to better predict needs of c. Snuffles and rhagades in the newborn.
high-risk clients, decrease exacerbations and d. Central hearing defects in infancy.
continually monitor progress overtime.
B. Newborns can get pneumonia (tachypnea, mild
25. The physician orders a dose of IV phenytoin to hypoxia, cough, eosinophilia) and
a child client. In preparing in the administration of conjunctivitis from Chlamydia.
the drug, which nursing action is not correct?
29. The nurse is assigned to care to a 17-year-old
a) Infuse the phenytoin into a smaller vein to male client with a history of substance abuse. The
prevent purple glove syndrome. client asks the nurse, “Have you ever tried or used
drugs?” The most correct response of the nurse B. In case management, the nurse assumes total
would be: responsibility for meeting the needs of the client
during the entire time on duty.
a) “Yes, once I tried grass.”
b) “No, I don’t think so.”
c) “Why do you want to know that?” 33. The ambulance team calls the emergency
d) “How will my answer help you?” department that they are going to bring a client who
sustained burns in a house fire. While waiting for
D. The client may perceive this as avoidance, but it the ambulance, the nurse will anticipate emergency
is more important to redirect back to the client, care to include assessment for:
especially in light of the manipulative behavior
of drug abusers and adolescents. a. Gas exchange impairment.
b. Hypoglycemia.
c. Hyperthermia.
30. Which of the following describes a health care d. Fluid volume excess.
team with the principles of participative leadership?
A. Smoke inhalation affects gas exchange.
a. Each member of the team can independently
make decisions regarding the client’s care
without necessarily consulting the other 34. Most couples are using “natural” family
members. planning methods. Most accidental pregnancies in
b. The physician makes most of the decisions couples preferred to use this method have been
regarding the client’s care. related to unprotected intercourse before ovulation.
c. The team uses the expertise of its members Which of the following factor explains why
to influence the decisions regarding the pregnancy may be achieved by unprotected
client’s care. intercourse during the preovulatory period?
d. Nurses decide nursing care; physicians decide
medical and other treatment for the client. a) Ovum viability.
b) Tubal motility.
C. It describes a democratic process in which all c) Spermatozoal viability.
members have input in the client’s care. d) Secretory endometrium.

C. Sperm deposited during intercourse may remain


31. A nurse is giving a health teaching to a woman viable for about 3 days. If ovulation occurs
who wants to breastfeed her newborn baby. Which during this period, conception may result.
hormone, normally secreted during the postpartum
period, influences both the milk ejection reflex and 35. An older adult client wakes up at 2 o’clock in
uterine involution? the morning and comes to the nurse’s station saying,
“I am having difficulty in sleeping.” What is the
a) Oxytocin. best nursing response to the client?
b) Estrogen.
c) Progesterone. a. “I’ll give you a sleeping pill to help you get
d) Relaxin. more sleep now.”
b. “Perhaps you’d like to sit here at the nurse’s
A. Contraction of the milk ducts and let-down station for a while.”
reflex occur under the stimulation of oxytocin c. “Would you like me to show you where the
released by the posterior pituitary gland. bathroom is?”
d. “What woke you up?”
32. One staff nurse is assigned to a group of 5
patients for the 12-hour shift. The nurse is B. This option shows acceptance (key concept) of
responsible for the overall planning, giving and this age-typical sleep pattern (that of waking in
evaluating care during the entire shift. After the the early morning).
shift, same responsibility will be endorsed to the
next nurse in charge. This describes nursing care 36. The nurse is taking care of a multipara who is at
delivered via the: 42 weeks of gestation and in active labor, her
membranes ruptured spontaneously 2 hours ago.
a) Primary nursing method. While auscultating for the point of maximum
b) Case method. intensity of fetal heart tones before applying an
c) Functional method. external fetal monitor, the nurse counts 100 beats
d) Team method. per minute. The immediate nursing action is to:

a) Start oxygen by mask to reduce fetal distress.


b) Examine the woman for signs of a prolapsed decisions of the holder, even if opposed by the
cord. spouse, are enforced.
c) Turn the woman on her left side to increase
placental perfusion. 40. A client diagnosed with schizophrenia is
d) Take the woman’s radial pulse while still shouting and banging on the door leading to the
auscultating the FHR. outside, saying, “I need to go to an appointment.”
What is the appropriate nursing intervention?
D. Taking the mother’s pulse while listening to the
FHR will differentiate between the maternal a) Tell the client that he cannot bang on the door.
and fetal heart rates and rule out fetal b) Ignore this behavior.
Bradycardia. c) Escort the client going back into the room.
d) Ask the client to move away from the door.

37. The nurse must instruct a client with glaucoma C. Gentle but firm guidance and nonverbal
to avoid taking over-the-counter medications like: direction is needed to intervene when a client
with schizophrenic symptoms is being
a. Antihistamines. disruptive.
b. NSAIDs.
c. Antacids. 41. Which of the following action is an accurate
d. Salicylates. tracheal suctioning technique?

A. Antihistamines cause pupil dilation and should a. 25 seconds of continuous suction during
be avoided with glaucoma. catheter insertion.
b. 20 seconds of continuous suction during
catheter insertion.
38. A male client is brought to the emergency c. 10 seconds of intermittent suction during
department due to motor vehicle accident. While catheter withdrawal.
monitoring the client, the nurse suspects increasing d. 15 seconds of intermittent suction during
intracranial pressure when: catheter withdrawal.
C. Suctioning is only done for 10 seconds,
a) Client is oriented when aroused from sleep, intermittently, as the catheter is being
and goes back to sleep immediately. withdrawn.
b) Blood pressure is decreased from 160/90 to
110/70. 42. The client’s jaw and cheekbone is sutured and
c) Client refuses dinner because of anorexia. wired. The nurse anticipates that the most important
d) Pulse is increased from 88-96 with occasional thing that must be ready at the bedside is:
skipped beat.
a) Suture set.
A. This suggests that the level of consciousness is b) Tracheostomy set.
decreasing. c) Suction equipment.
d) Wire cutters.
39. The nurse is conducting a lecture to a class of
nursing students about advance directives to D. The priority for this client is being able to
preoperative clients. Which of the following establish an airway.
statement by the nurse js correct?
43. A mother is in the third stage of labor. Which of
a. “The spouse, but not the rest of the family, may the following signs will help the nurse determine
override the advance directive.” the signs of placental separation?
b. “An advance directive is required for a “do not
resuscitate” order.” a. The uterus becomes globular.
c. “A durable power of attorney, a form of b. The umbilical cord is shortened.
advance directive, may only be held by a blood c. The fundus appears at the introitus.
relative.” d. Mucoid discharge is increased.
d. “The advance directive may be enforced
even in the face of opposition by the spouse.” A. Signs of placental separation include a change
in the shape of the uterus from ovoid to
D. An advance directive is a form of informed globular.
consent, and only a competent adult or the
holder of a durable power of attorney has the 44. After therapy with the thrombolytic alteplase (t-
right to consent or refuse treatment. If the PA), what observation will the nurse report to the
spouse does not hold the power of attorney, the physician?
a) 3+ peripheral pulses. D. After surgery for a ruptured appendix, the client
b) Change in level of consciousness and should be placed in a semi-Fowler’s position to
headache. promote drainage and to prevent possible
c) Occasional dysrhythmias. complications.
d) Heart rate of 100/bpm.

B. This could indicate intracranial bleeding. 48. Which of the following will best describe a
Alteplase is a thrombolytic enzyme that lyses management function?
thrombi and emboli. Bleeding is an adverse
effect. Monitor clotting times and signs of any a) Writing a letter to the editor of a nursing
gastrointestinal or internal bleeding. journal.
b) Negotiating labor contracts.
45. A client who undergone left nephrectomy has a c) Directing and evaluating nursing staff
large flank incision. Which of the following nursing members.
action will facilitate deep breathing and coughing? d) Explaining medication side effects to a client.

a. Push fluid administration to loosen respiratory C. Directing and evaluation of staff is a major
secretions. responsibility of a nursing manager
b. Have the client lie on the unaffected side.
c. Maintain the client in high Fowler’s position. 49. The parents of an infant client ask the nurse to
d. Coordinate breathing and coughing exercise teach them how to administer Cortisporin eye drops.
with administration of analgesics. The nurse is correct in advising the parents to place
the drops:
D. Because flank incision in nephrectomy is
directly below the diaphragm, deep breathing is a. In the middle of the lower conjunctival sac of
painful. Additionally, there is a greater the infant’s eye.
incisional pull each time the person moves than b. Directly onto the infant’s sclera.
there is with abdominal surgery. Incisional pain c. In the outer canthus of the infant’s eye.
following nephrectomy generally requires d. In the inner canthus of the infant’s eye.
analgesics administration every 3-4 hours for
24-48 hours after surgery. Therefore, turning, A. The recommended procedure for administering
coughing and deep-breathing exercises should eyedrops to any client calls for the drops to be
be planned to maximize the analgesic effects. placed in the middle of the lower conjunctival
sac.
46. The community nurse is teaching the group of
mothers about the cervical mucus method of natural
family planning. Which characteristics are typical 50. The nurse is assessing on the client who is
of the cervical mucus during the “fertile” period of admitted due to vehicle accident. Which of the
the menstrual cycle? following findings will help the nurse that there is
internal bleeding?
a) Absence of ferning.
b) Thin, clear, good spinnbarkeit. a) Frank blood on the clothing.
c) Thick, cloudy. b) Thirst and restlessness.
d) Yellow and sticky. c) Abdominal pain.
d) Confusion and altered of consciousness.
B. Under high estrogen levels, during the period
surrounding ovulation, the cervical mucus B. Thirst and restlessness indicate hypovolemia
becomes thin, clear, and elastic (spinnbarkeit), and hypoxemia. Internal bleeding is difficult to
facilitating sperm passage. recognized and evaluate because it is not
apparent.
47. A client with ruptured appendix had surgery an
hour ago and is transferred to the nursing care unit.
The nurse placed the client in a semi-Fowler’s 51. The nurse is completing an assessment to a
position primarily to: newborn baby boy. The nurse observes that the skin
of the newborn is dry and flaking and there are
a. Facilitate movement and reduce complications several areas of an apparent macular rash. The nurse
from immobility. charts this as:
b. Fully aerate the lungs.
c. Splint the wound. a. Icterus neonatorum
d. Promote drainage and prevent b. Multiple hemangiomas
subdiaphragmatic abscesses. c. Erythema toxicum
d. Milia
C. Erythema toxicum is the normal, C. The stoma drainage bag is applied in the
nonpathological macular newborn rash operating room. Drainage from the ileostomy
contains secretions that are rich in digestive
52. The client is brought to the emergency enzymes and highly irritating to the skin.
department because of serious vehicle accident. Protection of the skin from the effects of these
After an hour, the client has been declared brain enzymes is begun at once. Skin exposed to
dead. The nurse who has been with the client must these enzymes even for a short time becomes
now talk to the family about organ donation. Which reddened, painful and excoriated.
of the following consideration is necessary?
56. A female client who has a 28-day menstrual
a) Include as many family members as possible. cycle asks the community health nurse when she get
b) Take the family to the chapel. pregnant during her cycle. What will be the best
c) Discuss life support systems. nursing response?
d) Clarify the family’s understanding of brain
death. a) It is impossible to determine the fertile period
reliably. So it is best to assume that a woman is
D. The family needs to understand what brain always fertile.
death is before talking about organ donation. b) In a 28-day cycle, ovulation occurs at or
They need time to accept the death of their about day 14. The egg lives for about 24
family member. An environment conducive to hours and the sperm live for about 72 hours.
discussing an emotional issue is needed. The fertile period would be approximately
between day 11 and day 15.
53. The nurse is teaching exercises that are good for c) In a 28- day cycle, ovulation occurs at or about
pregnant women increasing tone and fitness and day 14. The egg lives for about 72 hours and
decreasing lower backache. Which of the following the sperm live for about 24 hours. The fertile
should the nurse exclude in the exercise program? period would be approximately between day 13
and 17.
a. Stand with legs apart and touch hands to d) In a 28-day cycle, ovulation occurs 8 days
floor three times per day. before the next period or at about day 20. The
b. Ten minutes of walking per day with an fertile period is between day 20 and the
emphasis on good posture. beginning of the next period.
c. Ten minutes of swimming or leg kicking in
pool per day. B. It is the most accurate statement of
d. Pelvic rock exercise and squats three times a physiological facts for a 28-day menstrual
day. cycle: ovulation at day 14, egg life span 24
hours, sperm life span of 72 hours. Fertilization
A. Bending from the waist in pregnancy tends to could occur from sperm deposited before
make backache worse. ovulation.

54. A client with obsessive-compulsive behavior is 57. Which of the following statement describes the
admitted in the psychiatric unit. The nurse taking role of a nurse as a client advocate?
care of the client knows that the primary treatment
goal is to: a. A nurse may override clients’ wishes for their
own good.
a) Provide distraction. b. A nurse has the moral obligation to prevent
b) Support but limit the behavior. harm and do well for clients.
c) Prohibit the behavior. c. A nurse helps clients gain greater
d) Point out the behavior. independence and self-determination.
d. A nurse measures the risk and benefits of
B. Support and limit setting decrease anxiety and various health situations while factoring in cost.
provide external control.
C. An advocate role encourage freedom of choice,
55. After ileostomy, the nurse expects that the includes speaking out for the client, and
drainage appliance will be applied to the stoma: supports the client’s best interests.

a. When the client is able to begin self-care 58. A community health nurse is providing a health
procedures. teaching to a woman infected with herpes simplex 2.
b. 24 hours later, when the swelling subsided. Which of the following health teaching must the
c. In the operating room after the ileostomy nurse include to reduce the chances of transmission
procedure. of herpes simplex 2?
d. After the ileostomy begins to function.
a) “Abstain from intercourse until lesions heal.” c. A confused elderly woman who needs
b) “Therapy is curative.” assistance with eating.
c) “Penicillin is the drug of choice for treatment.” d. Routine temperature check that must be done
d) “The organism is associated with later for a client at end of shift.
development of hydatidiform mole.
B. The registered nurse cannot delegate the
A. Abstinence will eliminate any unnecessary pain responsibility for assessment and evaluation of
during intercourse and will reduce the clients. The status of the client in restraint
possibility of transmitting infection to one’s requires further assessment to determine if
sexual partner. there are additional causes for the behavior.

59. The nurse in the psychiatric ward informed the


male client that he will be attending the 9:00 AM
group therapy sessions. The client tells the nurse 62. In the admission care unit, which of the
that he must wash his hands from 9:00 to 9:30 AM following client would the nurse give immediate
each day and therefore he cannot attend. Which attention?
concept does the nursing staff need to keep in mind
in planning nursing intervention for this client? a) A client who is 3 days postoperative with left
calf pain.
a. Depression underlines ritualistic behavior. b) A client who is postoperative hip pinning who
b. Fear and tensions are often expressed in is complaining of pain.
disguised form through symbolic processes. c) New admitted client with chest pain.
c. Ritualistic behavior makes others d) A client with diabetes who has a glucoscan
uncomfortable. reading of 180.
d. Unmet needs are discharged through ritualistic
behavior. C. The client with chest pain may be having a
myocardial infarction, and immediate
B. Anxiety is generated by group therapy at 9:00 assessment and intervention is a priority.
AM. The ritualistic behavioral defense of hand
washing decreases anxiety by avoiding group 63. A couple seeks medical advice in the
therapy. community health care unit. A couple has been
unable to conceive; the man is being evaluated for
60. The nurse assesses the health condition of the possible problems. The physician ordered semen
female client. The client tells the nurse that she analysis. Which of the following instructions is
discovered a lump in the breast last year and correct regarding collection of a sperm specimen?
hesitated to seek medical advice. The nurse
understands that, women who tend to delay seeking a. Collect a specimen at the clinic, place in iced
medical advice after discovering the disease are container, and give to laboratory personnel
displaying what common defense mechanism? immediately.
b. Collect specimen after 48-72 hours of
a) Intellectualization. abstinence and bring to clinic within 2 hours.
b) Suppression. c. Collect specimen in the morning after 24 hours
c) Repression. of abstinence and bring to clinic immediately.
d) Denial. d. Collect specimen at night, refrigerate, and bring
to clinic the next morning.
D. Denial is a very strong defense mechanism used
to allay the emotional effects of discovering a B. Is correct because semen analysis requires that a
potential threat. Although denial has been freshly masturbated specimen be obtained after
found to be an effective mechanism for a rest (abstinence) period of 48-72 hours.
survival in some instances, such as during
natural disasters, it may in greater pathology in 64. The physician ordered Betamethasone to a
a woman with potential breast carcinoma. pregnant woman at 34 weeks of gestation with sign
of preterm labor. The nurse expects that the drug
61. Which of the following situations cannot be will:
delegated by the registered nurse to the nursing
assistant? a) Treat infection.
b) Suppress labor contraction.
a. A postoperative client who is stable needs to c) Stimulate the production of surfactant.
ambulate. d) Reduce the risk of hypertension.
b. Client in soft restraint who is very agitated
and crying. C. Betamethasone, a form of cortisone, acts on the
fetal lungs to produce surfactant.
d) Logically point out that the client is jumping to
65. A tracheostomy cuff is to be deflated, which of conclusions.
the following nursing intervention should be
implemented before starting the procedures? C. Listening is probably the most effective
response of the four choices
a. Suction the trachea and mouth.
b. Have the obdurator available. 69. After a cystectomy and formation of an ileal
c. Encourage deep breathing and coughing. conduit, the nurse provides instruction regarding
d. Do a pulse oximetry reading. prevention of leakage of the pouch and backflow of
the urine. The nurse is correct to include in the
A. Secretions may have pooled above the instruction to empty the urine pouch:
tracheostomy cuff. If these are not suctioned
before deflation, the secretions may be a. Every 3-4 hours.
aspirated. b. Every hour.
c. Twice a day.
66. A client is diagnosed with Tuberculosis and d. Once before bedtime.
respiratory isolation is initiated. This means that:
A. Urine flow is continuous. The pouch has an
a) Gloves are worn when handling the client’s outlet valve for easy drainage every 3-4 hours.
tissue, excretions, and linen. (the pouch should be changed every 3-5 days,
b) Both client and attending nurse must wear or sooner if the adhesive is loose).
masks at all times.
c) Nurse and visitors must wear masks until 70. Which telephone call from a student’s mother
chemotherapy is begun. Client is instructed should the school nurse take care of at once?
in cough and tissue techniques.
d) Full isolation; that is, caps and gowns are a) A telephone call notifying the school nurse
required during the period of contagion. that the child’ pediatrician has informed the
mother that the child will need cardiac
C. Proper handling of sputum is essential to allay repair surgery within the next few weeks.
droplet transference of bacilli in the air. Clients b) A telephone call notifying the school nurse that
need to be taught to cover their nose and mouth the child’s pediatrician has informed the mother
with tissues when sneezing or coughing. that the child has head lice.
Chemotherapy generally renders the client c) A telephone call notifying the school nurse that
noninfectious within days to a few weeks, a child has a temperature of 102ºF and a rash
usually before cultures for tubercle bacilli are covering the trunk and upper extremities of the
negative. Until chemical isolation is body.
established, many institutions require the client d) A telephone call notifying the school nurse that
to wear a mask when visitors are in the room or a child underwent an emergency appendectomy
when the nurse is in attendance. Client should during the previous night.
be in a well-ventilated room, without air
recirculation, to prevent air contamination. C. A high fever accompanied by a body rash could
indicate that the child has a communicable
67. A client with lung cancer is admitted in the disease and would have exposed other students
nursing care unit. The husband wants to know the to the infection. The school nurse would want
condition of his wife. How should the nurse respond to investigate this telephone call immediately
to the husband? so that plans could be instituted to control the
spread of such infection.
a. Find out what information he already has.
b. Suggest that he discuss it with his wife. 71. Which of the following signs and symptoms that
c. Refer him to the doctor. require immediate attention and may indicate most
d. Refer him to the nurse in charge. serious complications during pregnancy?

A. It is best to establish baseline information first. a. Severe abdominal pain or fluid discharge
from the vagina.
68. A hospitalized client cannot find his b. Excessive saliva, “bumps around the areolae,
handkerchief and accuses other cient in the room and increased vaginal mucus.
and the nurse of stealing them. Which is the most c. Fatigue, nausea, and urinary frequency at any
therapeutic approach to this client? time during pregnancy.
d. Ankle edema, enlarging varicosities, and
a) Divert the client’s attention. heartburn.
b) Listen without reinforcing the client’s belief.
c) Inject humor to defuse the intensity.
A. Severe abdominal pain may indicate 75. A female client with cancer has radium implants.
complications of pregnancy such as abortion, The nurse wants to maintain the implants in the
ectopic pregnancy, or abruption placenta; fluid correct position. The nurse should position the
discharge from the vagina may indicate client:
premature rupture of the membrane.
a. Flat in bed.
72. The nurse is assessing the newborn boy. Apgar b. On the side only.
scores are 7 and 9. The newborn becomes slightly c. With the foot of the bed elevated.
cyanotic. What is the initial nursing action? d. With the head elevated 45-degrees (semi-
Fowler’s).
a) Elevate his head to promote gravity drainage of
secretions. A. Clients with radioactive implants should be
b) Wrap him in another blanket, to reduce heat positioned flat in bed to prevent dislodgement
loss. of the vaginal packing. The client may roll to
c) Stimulate him to cry,, to increase oxygenation. the side for meals but the upper body should
d) Aspirate his mouth and nose with bulb not be raised more than 20 degrees.
syringe.
76. The nurse wants to know if the mother of a
D. Gentle aspiration of mucus helps maintain a toddler understands the instructions regarding the
patent airway, required for effective gas administration of syrup of ipecac. Which of the
exchange. following statement will help the nurse to know that
the mother needs additional teaching?
73. The nurse is formulating a plan of care to a
client with a somatoform disorder. The nurse needs a) “I’ll give the medicine if my child gets into
to have knowledge of which psychodynamic some toilet bowl cleaner.”
principle? b) “I’ll give the medicine if my child gets into
some aspirin.”
a. The symptoms of a somatoform disorder are c) “I’ll give the medicine if my child gets into
an attempt to adjust to painful life situations some plant bulbs.”
or to cope with conflicting sexual, aggressive, d) “I’ll give the medicine if my child gets into
or dependent feelings. some vitamin pills.”
b. The major fundamental mechanism is
regression. A. Syrup of ipecac is not administered when the
c. The client’s symptoms are imaginary and the ingested substances is corrosive in nature.
suffering is faked. Toilet bowl cleaners, as a collective whole, are
d. An extensive, prolonged study of the symptoms highly corrosive substances. If the ingested
will be reassuring to the client, who seeks substance “burned” the esophagus going down,
sympathy, attention and love. it will “burn” the esophagus coming back up
when the child begins to vomit after
A. Somatoform disorders provide a way of coping administration of syrup of ipecac.
with conflicts.
77. To assess if the cranial nerve VII of the client
74. An infant is brought to the health care clinic for was damaged, which changes would not be
three immunizations at the same time. The nurse expected?
knows that hepatitis B, DPT, and Haemophilus
influenzae type B immunizations should: a. Drooling and drooping of the mouth.
b. Inability to open eyelids on operative side.
a) Be drawn in the same syringe and given in one c. Sagging of the face on the operative side.
injection. d. Inability to close eyelid on operative side.
b) Be mixed and inject in the same sites.
c) Not be mixed and the nurse must give three B. Inability to open eyelids on operative side is
injections in three sites. seen with cranial nerve III damage.
d) Be mixed and the nurse must give the injection
in three sites. 78. The community health nurse makes a home visit
to a family. During the visit, the nurse observes that
C. Immunization should never be mixed together the mother is beating her child. What is the priority
in a syringe, thus necessitating three separate nursing intervention in this situation?
injections in three sites. Note: some
manufacturers make a premixed combination a. Assess the child’s injuries.
of immunization that is safe and effective. b. Report the incident to protective agencies.
c. Refer the family to appropriate support group.
d. Assist the family to identify stressors and use of d) “Have sex as often as you can, especially
other coping mechanisms to prevent further around the time of ovulation, to increase your
incidents. chances of pregnancy.”

A. Assessment of physical injuries (like bruises, A. Infertility is not diagnosed until atleast
lacerations, bleeding and fractures) is the first 12months of unprotected intercourse has failed
priority. to produce a pregnancy. Older couples will
experience a longer time to get pregnant.
79. The nurse in the neonatal care unit is
supervising the actions of a certified nursing 82. The nurse is caring for a cient who Is a retired
assistant in giving care to the newborns. The nurse. A 24-hour urine collection for Creatinine
nursing assistant mistakenly gives a formula feeding clearance is to be done. The client tells the nurse, “I
to a newborn that is on water feeding only. The can’t remember what this test is for.” The best
nurse is responsible for the mistake of the nursing response by the nurse is:
assistant:
a. “It provides a way to see if you are passing any
a) Always, as a representative of the institution. protein in your urine.”
b) Always, because nurses who supervise less- b. “It tells how well the kidneys filter wastes
trained individuals are responsible for their from the blood.”
mistakes. c. “It tells if your renal insufficiency has affected
c) If the nurse failed to determine whether the your heart.”
nursing assistant was competent to take care d. “The test measures the number of particles the
of the client. kidney filters.”
d) Only if the nurse agreed that the newborn could
be fed formula. B. Determining how well the kidneys filter wastes
states the purpose of a Creatinine clearance test.
C. The nurse who is supervising others has a legal
obligation to determine that they are competent
to perform the assignment, as well as legal 83. The nurse observes the female client in the
obligation to provide adequate supervision. psychiatric ward that she is having a hard time
sleeping at night. The nurse asks the client about it
and the client says, “I can’t sleep at night because of
80. The nurse is assigned to care for a client with fear of dying.” What is the best initial nursing
urinary calculi. Fluid intake of 2L/day is response?
encouraged to the client. the primary reason for this
is to: a) “It must be frightening for you to feel that
way. Tell me more about it.”
a. Reduce the size of existing stones. b) “Don’t worry, you won’t die. You are just here
b. Prevent crystalline irritation to the ureter. for some test.”
c. Reduce the size of existing stones c) “Why are you afraid of dying?”
d. Increase the hydrostatic pressure in the d) “Try to sleep. You need the rest before
urinary tract. tomorrow’s test.”

D. Increasing hydrostatic pressure in the urinary A. Acknowledging a feeling tone is the most
tract will facilitate passage of the calculi. therapeutic response and provides a broad
opening for the client to elaborate feelings.
81. The nurse is counseling a couple in their mid
30’s who have been unable to conceive for about 6 84. In the hospital lobby, the registered nurse
months. They are concerned that one or both of overhears a two staff members discussing about the
them may be infertile. What is the best advice the health condition of her client. What would be the
nurse could give to the couple? appropriate action for the registered nurse to take?

a) “it is no unusual to take 6-12 months to get a. Join in the conversation, giving her input about
pregnant, especially when the partners are the case.
in their mid-30s. Eat well, exercise, and b. Ignore them, because they have the right to
avoid stress.” discuss anything they want to.
b) “Start planning adoption. Many couples get c. Tell them it is not appropriate to discuss
pregnant when they are trying to adopt.” such things.
c) “Consult a fertility specialist and start testing d. Report this incident to the nursing supervisor.
before you get any older.”
C. The behavior should be stopped. The first is to c. Discuss the situation with friends who are also
remind the staff that confidentiality maybe nurses to get ideas .
violated. d. Approach the partner of this medical staff
member with these concerns.
85. The client has had a right-sided cerebrovascular
accident. In transferring the client from the B. The submission of reports about incidents that
wheelchair to bed, in what position should a client expose clients to harm does not remove the
be placed to facilitate safe transfer? obligation to report ongoing behavior as long
as the risk to the client continues.
a) Weakened (L) side of the cient next to bed.
b) Weakened (R) side of the client next to bed. 89. The physician ordered tetracycline PO qid to a
c) Weakened (L) side of the client away from child client who weights 20kg. The recommended
bed. PO tetracycline dose is 25-50 mg/kg/day. What is
d) Weakened (R) side of the cient away from bed. the maximum single dose that can be safely
administered to this child?
C. With a right-sided cerebrovascular accident the
client would have left-sided hemiplegia or a) 1g
weakness. The client’s good side should be b) 500 mg
closest to the bed to facilitate the transfer. c) 250 mg
d) 125 mg
86. The child client has undergone hip surgery and
is in a spica cast. Which of the following toy should C. The recommended dosage of tetracycline is 25-
be avoided to be in the child’s bed? 50mg/kg/day. If the child weighs 20kg and the
maximum dose is 50mg/kg, this would indicate
a. A toy gun. a total daily dose of 1000mg of tetracycline. In
b. A stuffed animal. this case, the child is being given this
c. A ball. medication four times a day. Therefore the
d. Legos. maximum single dose that can be given is
250mg (1000 mg of tetracycline divided by
D. Legos are small plastic building blocks that four doses.)
could easily slip under the child’s cast and lead
to a break in skin integrity and even infection. 90. The nurse is completing an obstetric history of a
Pencils, backscratchers, and marbles are some woman in labor. Which event in the obstetric
other narrow or small items that could easily history will help the nurse suspects dysfunctional
slip under the child’s cast and lead to a break in labor in the current pregnancy?
skin integrity and infection.
a. Total time of ruptured membranes was 24
87. The LPN/LVN asks the registered nurse why hours with the second birth.
oxytocin (Pitocin), 10 units (IV or IM) must be b. First labor lasting 24 hours.
given to a client after birth fo the fetus. The nurse is c. Uterine fibroid noted at time of cesarean
correct to explain that oxytocin: delivery.
d. Second birth by cesarean for face presentation.
a) Minimizes discomfort from “afterpains.”
b) Suppresses lactation. C. An abnormality in the uterine muscle could
c) Promotes lactation. reduce the effectiveness of uterine contractions
d) Maintains uterine tone. and lengthen the duration of subsequent labors.

D. Oxytocin (Pitocin) is used to maintain uterine 91. The nurse is planning to talk to the client with
tone. an antisocial personality disorder. What would be
the most therapeutic approach?
88. The nurse in the nursing care unit is aware that
one of the medical staff displays unlikely behaviors a) Provide external controls.
like confusion, agitation, lethargy and unkempt b) Reinforce the client’s self-concept.
appearance. This behavior has been reported to the c) Give the client opportunities to test reality.
nurse manager several times, but no changes d) Gratify the client’s inner needs.
observed. The nurse should:
A. Personality disorders stem from a weak
a. Continue to report observations of unusual superego, implying a lack of adequate controls.
behavior until the problem is resolved.
b. Consider that the obligation to protect the 92. The nurse is teaching a group of women about
patient from harm has been met by the prior fertility awareness, the nurse should emphasize that
reports and do nothing further. basal body temperature:
a. Can be done with a mercury thermometer but C. “I don’t see a bug in your bed, but you seem
no a digital one. afraid.”
b. The average temperature taken each morning. D. “You must be seeing things.”
c. Should be recorded each morning before any
activity. C. This response does not contradict the client’s
d. Has a lower degree of accuracy in predicting perception, is honest, and shows empathy.
ovulation than the cervical mucus test.
96. A pregnant client in late pregnancy is
C. The basal body temperature is the lowest body complaining of groin pain that seems worse on the
temperature of a healthy person that is taken right side. Which of the following is the most likely
immediately after waking and before getting cause of it?
out of bed. The BBT usually varies from 36.2
ºC to 36.3ºC during menses and for about 5-7 a. Beginning of labor.
days afterward. About the time of ovulation, a b. Bladder infection.
slight drop in temperature may be seen, after c. Constipation.
ovulation in concert with the increasing d. Tension on the round ligament.
progesterone levels of the early luteal phase,
the BBT rises 0.2-0.4 ºC. This elevation D. Tension on round ligament occurs because of
remains until 2-3 days before menstruation, or the erect human posture and pressure exerted
if pregnancy has occurred. by the growing fetus.
97. The nurse is conducting a lecture to a group of
93. The nursing applicant has given the chance to volunteer nurses. The nurse is correct in imparting
ask questions during a job interview at a local the idea that the Good Samaritan law protects the
hospital. What should be the most important nurse from a suit for malpractice when:
question to ask that can increase chances of
securing a job offer? a) The nurse stops to render emergency aid and
leaves before the ambulance arrives.
a) Begin with questions about client care b) The nurse acts in an emergency at his or her
assignments, advancement opportunities, place of employment.
and continuing education. c) The nurse refuses to stop for an emergency
b) Decline to ask questions, because that is the outside of the scope of employment.
responsibility of the interviewer. d) The nurse is grossly negligent at the scene of
c) Ask as many questions about the facility as an emergency.
possible.
d) Clarify information regarding salary, benefits, D. The Good Samaritan Law does not impose a
and working hours first, because this will help duty to stop at the scene of an emergency
in deciding whether or not to take the job. outside of the scope of employment, therefore
nurses who do not stop are not liable for suit.
A. This choice implies concern for client care and
self-improvement. 98. A woman is hospitalized with mild
preeclampsia. The nurse is formulating a plan of
94. The nurse advised the pregnant woman that care for this client, which nursing care is least likely
smoking and alcohol should be avoided during to be done?
pregnancy. The nurse takes into account that the
developing fetus is most vulnerable to environment a. Deep-tendon reflexes once per shift.
teratogens that cause malformation during: b. Vital signs and FHR and rhythm q4h while
awake.
a. The entire pregnancy. c. Absolute bed rest.
b. The third trimester. d. Daily weight.
c. The first trimester.
d. The second trimester. C. Although reducing environment stimuli and
activity is necessary for a woman with mild
C. The first trimester is the period of preeclampsia, she will most probably have
organogenesis, that is, cell differentiation into bathroom privileges.
the various organs, tissues, and structures.
99. While feeding a newborn with an unrepaired
95. A male client tells the nurse that there is a big cardiac defect, the nurse keeps on assessing the
bug in his bed. The most therapeutic nursing condition of the client. The nurse notes that the
response would be: newborn’s respiration is 72 breaths per minute.
What would be the initial nursing action?
A. Silence.
B. “Where’s the bug? I’ll kill it for you.” a) Burp the newborn.
b) Stop the feeding.
c) Continue the feeding.
d) Notify the physician. PNLE Pediatric
B. A normal respiratory rate for a newborn is 30-
40 breaths per minute.
Nursing Exam
Situation 1: Raphael, a 6 year’s old prep
100. A client who undergone appendectomy 3 days
pupil is seen at the school clinic for
ago is scheduled for discharge today. The nurse
notes that the client is restless, picking at bedclothes growth and development monitoring
and saying, “I am late on my appointment,” and (Questions 1-5)
calling the nurse by the wrong name. The nurse
suspects: 1. Which of the following is characterized the rate
of growth during this period?
a. Panic reaction.
b. Medication overdose. most rapid period of growth
c. Toxic reaction to an antibiotic. a decline in growth rate
d. Delirium tremens. growth spurt
slow uniform growth rate
D. The behavior described is likely to be
symptoms of delirium tremens, or alcohol B. a decline in growth rate. During the
withdrawal (often unsuspected on a surgical Preschooler stage growth is very minimal.
unit.) Weight gain is only 4.5lbs (2kgs) per year and
Height is 3.5in (6-8cm) per year.
Review:
Most rapid growth and development-
Infancy
Slow growth- Toddler hood and
Preschooler
Slower growth- School age
Rapid growth- Adolescence

2. In assessing Raphael’s growth and development,


the nurse is guided by principles of growth and
development. Which is not included?

All individuals follow cephalo-caudal and


proximo-distal
Different parts of the body grows at different rate
All individual follow standard growth rate
Rate and pattern of growth can be modified

D. Rate and pattern of growth can be


modified. Growth and development occurs in
cephalo-caudal meaning development occurs
through out the body’s axis. Example: the child
must be able to lift the head before he is able to
lift his chest. Proximo-distal is development
that progresses from center of the body to the
extremities. Example: a child first develops
arm movement before fine-finger movement.
Different parts of the body grows at different
range because some body tissue mature faster
than the other such as the neurologic tissues
peaks its growth during the first years of life
while the genital tissue doesn’t till puberty.
Also G&D is predictable in the sequence which
a child normally precedes such as motor skills
and behavior. Lastly G&D can never be
modified .
3. What type of play will be ideal for Raphael at this
period? 7. Growth and development in a child progresses in
the following ways EXCEPT
Make believe
Hide and seek From cognitive to psychosexual
Peek-a-boo From trunk to the tip of the extremities
Building blocks From head to toe
From general to specific
A. Make believe. Make believe is most appropriate
because it enhances the imitative play and A. From cognitive to psychosexual. Growth and
imagination of the preschooler. C and D are for development occurs in cephalo-caudal (head to
infants while letter A is B is recommended for toe), proximo-distal (trunk to tips of the
schoolers because it enhances competitive play. extremities and general to specific, but it
doesn’t occurs in cognitive to psychosexual
4. Which of the following information indicate that because they can develop at the same time.
Raphael is normal for his age?
8. As described by Erikson, the major psychosexual
Determine own sense self conflict of the above situation is
Develop sense of whether he can trust the world
Has the ability to try new things Autonomy vs. Shame and doubt
Learn basic skills within his culture Industry vs. Inferiority
Trust vs. mistrust
C. Has the ability to try new things. Erickson Initiation vs. guilt
defines the developmental task of a preschool
period is learning Initiative vs. Guilt. Children C. Trust vs. mistrust. According to Erikson,
can initiate motor activities of various sorts on children 0-18 months are under the
their own and no longer responds to or imitate developmental task of Trust vs. Mistrust.
the actions of other children or of their
parents. 9. Which of the following is true about Mongolian
Spots?
5. Based on Kohlberg’s theory, what is the stage of
moral development of Raphael? Disappears in about a year
Are linked to pathologic conditions
Punishment-obedience Are managed by tropical steroids
“good boy-Nice girl” Are indicative of parental abuse
naïve instrumental orientation
social contact A. Disappears in about a year. Mongolian spots are
stale grey or bluish patches of discoloration
C. naïve instrumental orientation. According to commonly seen across the sacrum or buttocks
Kohlber, a preschooler is under Pre- due to accumulation of melanocytes and they
conventional where a child learns about disappears in 1 year. They are not linked to
instrumental purpose and exchange, that is they steroid use and pathologic conditions.
will something do for another if that that
person does something with the child in return. 10. Signs of cold stress that the nurse must be alert
Letter A is applicable for Toddlers and letter B when caring for a Newborn is:
is for a School age child
Hypothermia
Situation 2 Baby boy Lacson delivered at Decreased activity level
36 weeks gestation weighs 3,400 gm and Shaking
Increased RR
height of 59 cm (6-10)
6. Baby boy Lacson’s height is D. Increased RR. Hypothermia is inaccurate cause
normally, temperature of a newborn drop, Also
a child under cold stress will kick and cry to
Long
Short increase the metabolic rate thereby increasing
Average heat so B isn’t a good choice. A newborn
doesn’t have the ability to shiver, so letter B
Too short
and C is wrong. A newborn will increase its
RR because the NB will need more oxygen
A. Long. The average length of full-term babies at
because of too much activity.
birth is 20 in. (51 cm), although the normal
range is 46 cm (18 in.) to 56 cm (22 in.).
Situation 3 Nursing care after delivery catabolize brownfat which will produce
has an important aspect in every stages of ketones which is an acid will cause metabolic
acidosis. Also a NB with severe hypothermia is
delivery in high risk for kernicterus (too much bilirubin
11. After the baby is delivered, the cord was cut in the brain) can lead to Cerebral palsy. There
between two clamps using a sterile scissors and is no connection in the increase of ICP with
blade, then the baby is placed at the: hypothermia. (NOTE: pathognomonic sign of
Kernicterus in adult- asterexis, or involuntary
flapping of the hand.)
Mother’s breast
Mother’s side
15. During the feto-placental circulation, the shunt
Give it to the grandmother
between two atria is called
Baby’s own mat or bed

A. Mother’s breast. Place it at the mother’s breast Ductus venosous


for latch-on. (Note: for NSD breast feed ASAP Foramen Magnum
while for CS delivery, breast feed after 4 hours) Ductus arteriosus
Foramen Ovale
12. The baby’s mother is RH(-). Which of the
following laboratory tests will probably be ordered D. Foramen Ovale. Foramen ovale is opening
for the newborn? between two atria, Ductus venosus is the shunt
from liver to the inferior vena cava, and your
Ductus Arteriosus is the shunt from the
Direct Coomb’s
pulmonary artery to the aorta.
Indirect Coomb’s
Blood culture
16. What would cause the closure of the Foramen
Platelet count
ovale after the baby had been delivered?
A. Direct Coomb’s. Coomb’s test is the test to
determine if RH antibodies are present. Indirect Decreased blood flow
Coomb’s is done to the mother and Direct Shifting of pressures from right side to the left
Coomb’s is the one don’t to the baby. Blood side of the heart
culture and Platelet count doesn’t help detect Increased PO2
RH antibodies. Increased in oxygen saturation

13. Hypothermia is common in newborn because of B. Shifting of pressures from right side to the left
their inability to control heat. The following would side of the heart. During feto-placental
be an appropriate nursing intervention to prevent circulation, the pressure in the heart is much
heat loss except: higher in the right side, but once
breathing/crying is established, the pressure
will shift from the R to the L side, and will
Place the crib beside the wall
facilitate the closure of Foramen Ovale. (Note:
Doing Kangaroo care
that is why you should position the NB in R
By using mechanical pressure
side lying position to increase pressure in the L
Drying and wrapping the baby
side of the heart.)
Review:
A. Place the crib beside the wall. Placing the crib
Increase PO2-> closure of ductus
beside the wall is inappropriate because it can
arteriosus
provide heat loss by radiation. Doing Kangaroo
Decreased bloodflow -> closure of the
care or hugging the baby, mechanical pressure
ductus venosus
or incubators and drying and wrapping the
Circulation in the lungs is initiated by ->
baby will help conserve heat.
lung expansion and pulmonary
ventilation
14. The following conditions are caused by cold
What will sustain 1st breath-> decreased
stress except
artery pressure
What will complete circulation-> cutting
Hypoglycemia of the cord
Increase ICP
Metabolic acidosis 17. Failure of the Foramen Ovale to close will cause
Cerebral palsy what Congenital Heart Disease?
B. Increase ICP. Hypoglycemia may occur due to
Total anomalous Pulmunary Artery
increase metabolic rate, and because of
Atrial Septal defect
newborns are born slightly acidic, and they
Transposition of great arteries
Pulmunary Stenosis
21. When assessing gross motor development in a 3
B. Atrial Septal defect. Foramen ovale is the year old, which of the following activities would the
opening between two Atria so, if its will not nurse expect to finds?
close Atrial Septal defect can occur.
Riding a tricycle
Situation 4 Children are vulnerable to Hopping on one foot
some minor health problems or injuries Catching a ball
Skipping on alternate foot.
hence the nurse should be able to teach
mothers to give appropriate home care. A. Riding a tricycle. Answer is A, riding a tricycle
is appropriate for a 3 y/o child. Hopping on one
18. A mother brought her child to the clinic with foot can be done by a 4 y/o child, as well as
nose bleeding. The nurse showed the mother the catching and throwing a ball over hand.
most appropriate position for the child which is: Skipping can be done by a 5 y/o.
Sitting up 22. When assessing the weight of a 5-month old,
With low back rest which of the following indicates healthy growth?
With moderate back rest
Lying semi flat Doubling of birth weight
Tripling of birth weight
A. Sitting up. The correct position is making the Quadrupling of birth weight
child having an upright sitting position with the Stabilizing of birth weight
head slightly tilted forward. This position will
minimize the amount of blood pressure in nasal A. Doubling of birth weight. During the first 6
vessels and keep blood moving forward not months of life the weight from birth will be
back into the nasopharynx, which will have the doubled and as soon as the baby reaches 1 year,
choking sensation and increase risk of its birth weight is tripled.
aspiration. Choices b, c, d, are inappropriate
cause they can cause blood to enter the 23. An appropriate toy for a 4 year old child is:
nasopharynx.
Push-pull toys
19. A common problem in children is the
Card games
inflammation of the middle ear. This is related to
Doctor and nurse kits
the malfunctioning of the:
Books and Crafts
Tympanic membrane C. Doctor and nurse kits. Letter C is appropriate
Eustachian tube because it will enhance the creativity and
Adenoid imagination of a pre-school child. Letter B and
Nasopharynx D are inappropriate because they are too
complex for a 4 y/o. Push-pull toys are
B. Eustachian tube. This is because children has recommended for infants.
short, horizontal Eustachian tubes. The
dysfunction in the Eustachian tube enables 24. Which of the following statements would the
bacterial invasion of the middle ear and nurse expects a 5-year old boy to say whose pet
obstructs drainage of secretions. gerbil just died
20. For acute otitis media, the treatment is prompt
“The boogieman got him”
antibiotic therapy. Delayed treatment may result in
“He’s just a bit dead”
complications of:
“Ill be good from now own so I wont die like my
gerbil”
Tonsillitis “Did you hear the joke about…”
Eardrum Problems
Brain damage B. “He’s just a bit dead”. A 5 y/o views death in
Diabetes mellitus “degrees”, so the child most likely will say that
“he is just a bit dead”. Personification of death
C. Brain damage. One of the complication of like boogeyman occurs in ages 7 to 9 as well as
recurring acute otitis media is risk for having denying death can if they will be good.
Meningitis, thereby causing possible brain Denying death using jokes and attributing life
damage. That is why patient must follow a qualities to death occurs during age 3-5.
complete treatment regimen and follow up care.
A,B and D are not complications of AOM.
25. When assessing the fluid and electrolyte balance 29. Which of the following is a possible indicator of
in an infant, which of the following would be Munchausen syndrome by proxy type of child abuse?
important to remember?
Bruises found at odd locations, with different
Infant can concentrate urine at an adult level stages of healing
The metabolic rate of an infant is slower than in STD’s and genital discharges
adults Unexplained symptoms of diarrhea, vomiting
Infants have more intracellular water that adult do and apnea with no organic basis
Infant have greater body surface area than Constant hunger and poor hygiene
adults
C. Unexplained symptoms of diarrhea, vomiting
D. Infant have greater body surface area than and apnea with no organic basis. Munchausen
adults. Infants have greater body surface area syndrome by Proxy is the fabrication or
than adult, increasing their risk to F&E inducement of an illness by one person to
imbalances. Also infants cant concentrate a another person, usually mother to child. It is
urine at an adult level and their metabolic rate, characterized by symptoms such as apnea and
also called water turnover, is 2 to 3 times siezures, which may be due to suffocation,
higher than adult. Plus more fluids of the drugs or poisoning, vomiting which can be
infants are at the ECF spaces not in the ICF induced with poisons and diarrhea with the use
spaces. of laxatives. Letter A can be seen in a Physical
abuse, Letter B for sexual abuse and Letter C is
26. When assessing a child with aspirin overdose, for Physical Neglect.
which of the following will be expected?
30. Which of the following is an inappropriate
Metabolic alkalosis interventions when caring for a child with HIV?
Respiratory alkalosis
Metabolic acidosis Teaching family about disease transmission
Respiratory acidosis Offering large amount of fresh fruits and
vegetables
C. Metabolic acidosis. Remember that Aspirin is Encouraging child to perform at optimal level
acid (Acetylsalicylic ACID). Teach proper hand washing technique

27. Which of the following is not a possible B. Offering large amount of fresh fruits and
systemic clinical manifestation of severe burns? vegetables. A child with HIV is
immunocompromised. Fresh fruits and
Growth retardation vegetables, which may be contaminated with
Hypermetabolism organisms and pesticides can be harmful, if not
Sepsis fatal to the child, therefore these items should
Blisters and edema be avoided.

D. Blisters and edema. The question was asking Situation 5 Agata, 2 years old is rushed to
for a SYSTEMIC clinical manifestation, the ER due to cyanosis precipitated by
Letters A,B and C are systemic manifestations
while Blisters and Edema weren’t. crying. Her mother observed that after
playing she gets tired. She was diagnosed
28. When assessing a family for potential child with Tetralogy of Fallot.
abuse risks, the nurse would observe for which of
the following? 31. The goal of nursing care fro Agata is to:

Periodic exposure to stress Prevent infection


Low socio-economic status Promote normal growth and development
High level of self esteem Decrease hypoxic spells
Problematic pregnancies Hydrate adequately

D. Problematic pregnancies. Typical factors that C. Decrease hypoxic spells. The correct answer is
may be risk for Child abuse are problematic letter C. Though letter B would be a good
pregnancies, chronic exposure to stress not answer too, this goal is too vague and not
periodic, low level of self esteem not high level. specific. Nursing interventions will not solely
Also child abuse can happen in all socio- promote normal G&D unless he will undergo
economic status not just on low socio- surgical repair. So decreasing Hypoxic Spells
economic status is more SMART. Letter A and D are
inappropriate.
32. The immediate nursing intervention for cyanosis 35. When Agata was brought to the OR, her parents
of Agata is: where crying. What would be the most appropriate
nursing diagnosis?
Call up the pediatrician
Place her in knee chest position Infective family coping r/t situational crisis
Administer oxygen inhalation Anxiety r/t powerlessness
Transfer her to the PICU Fear r/t uncertain prognosis
Anticipatory grieving r/t gravity of child’s
B. Place her in knee chest position. The immediate physical status
intervention would be to place her on knee-
chest or “squatting” position because it traps D. Anticipatory grieving r/t gravity of child’s
blood into the lower extremities. Though also physical status. In this item letter A and be are
letter C would be a good choice but the inappropriate response so remove them. The
question is asking for “Immediate” so letter B possible answers are C and D. Fear defined as
is more appropriate. Letter A and D are the perceived threat (real or imagined) that is
incorrect because its normal for a child who consciously recognized as danger (NANDA) is
have ToF to have hypoxic or “tets” spells so applicable in the situation but its defining
there is no need to transfer her to the NICU or characteristics are not applicable. Crying per se
to alert the Pediatrician. can not be a subjective cue to signify fear, and
most of the symptoms of fear in NANDA are
33. Agata was scheduled for a palliative surgery, physiological. Anticipatory grieving on the
which creates anastomosis of the subclavian artery other hand are intellectual and EMOTIONAL
to the pulmonary artery. This procedure is: responses based on a potential loss. And
remember that procedures like this cannot
Waterston-Cooley assure total recovery. So letter D is a more
Raskkind Procedure appropriate Nursing diagnosis
Coronary artery bypass 36. Which of the following respiratory condition is
Blalock-Taussig always considered a medical emergency?

D. Blalock-Taussig. Blalock-Taussig procedure its Laryngeotracheobronchitis (LTB)


just a temporary or palliative surgery which Epiglottitis
creates a shunt between the aorta and Asthma
pulmonary artery so that the blood can leave Cystic Fibrosis
the aorta and enter the pulmonary artery and
thus oxygenating the lungs and return to the B. Epiglottitis. Acute and sever inflammation of
left side of the heart, then to the aorta then to the epiglottis can cause life threatening airway
the body. This procedure also makes use of the obstruction, that is why its always treated as a
subclavian vein so pulse is not palpable at the medical emergency. NSG intervention :
right arm. The full repair for ToF is called the Prepare tracheostomy set at bed side. LTB, can
Brock procedure. Raskkind is a palliative also cause airway obstruction but its not an
surgery for TOGA. emergency. Asthma is also not an emergency.
CF is a chronic disease, so its not a medical
34. Which of the following is not an indicator that emergency
Agata experiences separation anxiety brought about
her hospitalization? 37. Which of the following statements by the family
of a child with asthma indicates a need for
Friendly with the nurse additional teaching?
Prolonged loud crying, consoled only by mother
Occasional temper tantrums and always says NO “We need to identify what things triggers his
Repeatedly verbalizes desire to go home attacks”
“He is to use bronchodilator inhaler before steroid
A. Friendly with the nurse. Because toddlers views inhaler”
hospitalization is abandonment, separation “We’ll make sure he avoids exercise to prevent
anxiety is common. Its has 3 phases: PDD asthma attacks”
(parang c puff daddy LOL) 1. Protest 2. despair “he should increase his fluid intake regularly to
3. detachment (or denial). Choices B, C, D are thin secretions”
usually seen in a child with separation anxiety
(usually in the protest stage). C. “We’ll make sure he avoids exercise to prevent
REVIEW: asthma attacks”. Asthmatic children don’t have
Separation anxiety begin at: 9 months to avoid exercise. They can participate on
Peaks: 18 months physical activities as tolerated. Using a
bronchodilator before administering steroids is Target setting
correct because steroids are just anti-
inflammatory and they don’t have effects on 42. What does herd immunity means?
the dilation
of the bronchioles. OF course letters A and B are Interruption of transmission
obviously correct. All to be vaccinated
Selected group for vaccination
38. Which of the following would require careful Shorter incubation\
monitoring in the child with ADHD who is
receiving Methylphenidate (Ritalin)? 43. Measles vaccine can be given simultaneously.
What is the combined vaccine to be given to
Dental health children starting at 15 months?
Mouth dryness
Height and weight MCG
Excessive appetite MMR
BCG
C. Height and weight. Dental problems are more BBR
likely to occur in children under going TCA
therapy. Mouth dryness is a expected side B. MMR. MMR or Measles, Mumps, Rubella is a
effects of Ritalin since it activates the SNS. vaccine furnished in one vial and is routinely
Also loss of appetite is more likely to happen, given in one injection (Sub-Q). It can be given
not increase in appetite. The correct answer is at 15 months but can also be given as early as
letter C, because Ritalin can affect the child’s 12th month.
G&D. Intervention: medication “holidays or
vacation”. (This means during weekends or Situation 7: Braguda brought her 5-
holidays or school vacations, where the child
wont be in school, the drug can be withheld.)
month old daughter in the nearest RHU
because her baby sleeps most of the time,
Situation 6 Laura is assigned as the Team with decreased appetite, has colds and
Leader during the immunization day at fever for more than a week. The physician
the RHU diagnosed pneumonia.
39. What program for the DOH is launched at 1976 44. Based on this data given by Braguda, you can
in cooperation with WHO and UNICEF to reduce classify Braguda’s daughter to have:
morbidity and mortality among infants caused by
immunizable disease? Pneumonia: cough and colds
Severe pneumonia
Patak day Very severe pneumonia
Immunization day on Wednesday Pneumonia moderate
Expanded program on immunization
Bakuna ng kabtaan B. Severe pneumonia. For a child aging 2months
up to 5 years old can be classified to have sever
C. Expanded program on immunization pneumonia when he have any of the following
danger signs:
40. One important principle of the immunization Not able to drink
program is based on? Convulsions
Abnormally sleepy or difficult to wake
Statistical occurrence Stridor in calm child or
Epidemiologic situation Severe under-nutrition
Cold chain management
Surveillance study 45. For a 3-month old child to be classified to have
Pneumonia (not severe), you would expect to find
B. Epidemiologic situation. Letters A, C and D are RR of:
not included in the principles of EPI.
60 bpm
41. The main element of immunization program is 40 bpm
one of the following? 70 bpm
50 pbm
Information, education and communication
Assessment and evaluation of the program D. 50 pbm. A child can be classified to have
Research studies Pneumonia (not severe) if:
the young infant is less than 2 months- 60 49. A newborn’s failure to pass meconium within
bpm or more 24 hours after birth may indicate which of the
if the child is 2 months up to less than 12 following?
months- 50 bpm or more
if the child is 12 months to 4 y/o- 40 bpm or Aganglionic Mega colon
more Celiac disease
Intussusception
46. You asked Braguda if her baby received all Abdominal wall defect
vaccines under EPI. What legal basis is used in
implementing the UN’s goal on Universal Child A. Aganglionic Mega colon. Failure to pass
Immunization? meconium of Newborn during the first 24
hours of life may indicate Hirschsprung disease
PD no. 996 or Congenital Aganglionic Megacolon, an
PD no. 6 anomaly resulting in mechanical obstruction
PD no. 46 due to inadequate motility in an intestinal
RA 9173 segment. B, C, and D are not associated in the
failure to pass meconium of the newborn
B. PD no. 6 Presidential Proclamation no. 6 (April 50. The nurse understands that a good snack for a 2
3, 1986) is the “Implementing a United Nations year old with a diagnosis of acute asthma would be:
goal on Universal Child Immunization by
1990”. PD 996 (September 16, 1976) is Grapes
“providing for compulsory basic immunization Apple slices
for infants and children below 8 years of age. A glass of milk
PD no. 46 (September 16, 1992) is the A glass of cola
“Reaffirming the commitment of the
Philippines to the universal Child and Mother B. Apple slices. Grapes is in appropriate because
goal of the World Health Assembly. RA 9173 of its “balat” that can cause choking. A glass of
is of course the “Nursing act of 2002” milk is not a good snack because it’s the most
common cause of Iron-deficiency anemia in
47. Braguda asks you about Vitamin A children (milk contains few iron), A glass of
supplementation. You responded that giving cola is also not appropriate cause it contains
Vitamin A starts when the infant reaches 6 months complex sugar. (walang kinalaman ang asthma
and the first dose is” dahil ala naman itong diatery restricted foods
na nasa choices.)
200,000 “IU”
100,000 “IU” 51. Which of the following immunizations would
500,000 “IU” the nurse expect to administer to a child who is HIV
10,000 “IU” (+) and severely immunocomromised?

B. 100,000 “IU”. An infant aging 6-11 months will Varicella


be given Vitamin supplementation of 100, 000 Rotavirus
IU and for Preschoolers ages 12-83 months MMR
200,000 “IU” will be given. IPV

48. As part of CARI program, assessment of the D. IPV. IPV or Inactivated polio vaccine does not
child is your main responsibility. You could ask the contain live micro organisms which can be
following question to the mother except: harmful to an immunocompromised child.
Unlike OPV, IPV is administered via IM route.
“How old is the child?”
“IS the child coughing? For how long?” 52. When assessing a newborn for developmental
“Did the child have chest indrawing?” dysplasia of the hip, the nurse would expect to
“Did the child have fever? For how long?” assess which of the following?

C. “Did the child have chest indrawing?”. The Symmetrical gluteal folds
CARI program of the DOH includes the “ASK” Trendelemburg sign
and “LOOK, LISTEN” as part of the Ortolani’s sign
assessment of the child who has suspected Characteristic limp
Pneumonia. Choices A, B and D are included
in the “ASK” assessment while Chest C. Ortolani’s sign. Correct answer is Ortolani’s
indrawings is included in the “LOOK, LISTEN” sign; it is the abnormal clicking sound when
and should not be asked to the mother. the hips are abducted. The sound is produced
when the femoral head enters the acetabulum.
Letter A is wrong because its should be Intellectual development
“asymmetrical gluteal fold”. Letter B and C are Body image
not applicable for newborns because they are
seen in older children. D. Body image. Because of edema, associated with
nephroitic syndrome, potential self concept and
53. While assessing a male neonate whose mother body image disturbance related to changes in
desires him to be circumcised, the nurse observes appearance and social isolation should be
that the neonate’s urinary meatus appears to be considered.
located on the ventral surface of the penis. The
physician is notified because the nurse would 57. An inborn error of metabolism that causes
suspect which of the following? premature destruction of RBC?

Phimosis G6PD
Hydrocele Hemocystinuria
Epispadias Phenylketonuria
Hypospadias Celiac Disease

D. Hypospadias. Hypospadias is a c condition in A. G6PD. G6PD is the premature destruction of


which the urethral opening is located below the RBC when the blood is exposed to antioxidants,
glans penis or anywhere along the ventral ASA (ano un? Aspirin), legumes and flava
surface of the penile shaft. Epispadias, the beans.
urethral meatus is located at the dorsal surface
of the penile shaft. (Para di ka malilto, I- 58. Which of the following would be a diagnostic
alphabetesize mo Dorsal, (Above) eh mauuna test for Phenylketonuria which uses fresh urine
sa Ventral (Below) , Epis mauuna sa Hypo.) mixed with ferric chloride?

54. When teaching a group of parents about seat Guthrie Test


belt use, when would the nurse state that the child Phenestix test
be safely restrained in a regular automobile seatbelt? Beutler’s test
Coomb’s test
30 lb and 30 in
35 lb and 3 y/o B. Phenestix test. Phenestix test is a diagnostic test
40 lb and 40 in which uses a fresh urine sample (diapers) and
60 lb and 6 y/o mixed with ferric chloride. If positive, there
will be a presence of green spots at the diapers.
C. 40 lb and 40 in. Basta tandaan ang rule of 4! 4 Guthrie test is another test for PKU and is the
years old, 40 lbs and 40 in. one that mostly used. The specimen used is the
blood and it tests if CHON is converted to
55. When assessing a newborn with cleft lip, the amino acid.
nurse would be alert which of the following will
most likely be compromised? 59. Dietary restriction in a child who has
Hemocystenuria will include which of the following
Sucking ability amino acid?
Respiratory status
Locomotion Lysine
GI function Methionine
Isolensine tryptophase
A. Sucking ability. Because of the defect, the child Valine
will be unable to form the mouth adequately
arounf the nipple thereby requiring special B. Methionine. Hemocystenuria is the elevated
devices to allow feeding and sucking excretion of the amino acid hemocystiene, and
gratification. Respiratory status may be there is inability to convert the amino acid
compromised when the child is fed improperly methionine or cystiene. So dietary restriction of
or during post op period this amino acids is advised. This disease can
lead to mental retardation.
56. For a child with recurring nephritic syndrome,
which of the following areas of potential 60. A milk formula that you can suggest for a child
disturbances should be a prime consideration when with Galactosemia:
planning ongoing nursing care? Lofenalac
Lactum
Muscle coordination Neutramigen
Sexual maturation Sustagen
C. Neutramigen. Neutramien is suggested for a The greater need is identified through community
child with Galactosemia. Lofenalac is diagnosis.
suggested for a child with PKU.
4. R.A. 1054 is also known as the Occupational
PNLE Community Health Act. Aside from number of employees, what
other factor must be considered in determining the
occupational health privileges to which the workers
Health Nursing Exam will be entitled?

A. Type of occupation: agricultural, commercial,


1 industrial
B. Location of the workplace in relation to
1. Which is the primary goal of community health health facilities
nursing? C. Classification of the business enterprise based
on net profit
A. To support and supplement the efforts of the D. Sex and age composition of employees
medical profession in the promotion of health
and prevention of illness Answer: (B) Location of the workplace in
B. To enhance the capacity of individuals, relation to health facilities. Based on R.A.
families and communities to cope with their 1054, an occupational nurse must be employed
health needs when there are 30 to 100 employees and the
C. To increase the productivity of the people by workplace is more than 1 km. away from the
providing them with services that will increase nearest health center
their level of health
D. To contribute to national development through 5. A business firm must employ an occupational
promotion of family welfare, focusing health nurse when it has at least how many
particularly on mothers and children. employees?

Answer: (B) To enhance the capacity of A. 21


individuals, families and communities to cope B. 101
with their health needs C. 201
D. 301
2. CHN is a community-based practice. Which best
explains this statement? Answer: (B) 101. Again, this is based on R.A.
1054
A. The service is provided in the natural
environment of people. 6. When the occupational health nurse employs
B. The nurse has to conduct community ergonomic principles, she is performing which of
diagnosis to determine nursing needs and her roles?
problems.
A. Health care provider
C. The services are based on the available B. Health educator
resources within the community. C. Health care coordinator
D. Priority setting is based on the magnitude of D. Environmental manager
the health problems identified.
Answer: (D) Environmental
Answer: (B) The nurse has to conduct manager. Ergonomics is improving efficiency of
community diagnosis to determine nursing workers by improving the worker’s environment
needs and problems through appropriately designed furniture, for
example
3. Population-focused nursing practice requires
which of the following processes? 7. A garment factory does not have an occupational
nurse. Who shall provide the occupational health
A. Community organizing needs of the factory workers?
B. Nursing process
C. Community diagnosis A. Occupational health nurse at the Provincial
D. Epidemiologic process Health Office
B. Physician employed by the factory
Answer: (C) Community diagnosis. Population- C. Public health nurse of the RHU of their
focused nursing care means providing care based on municipality
the greater need of the majority of the population. D. Rural sanitary inspector of the RHU of their
municipality
Answer: (C) Public health nurse of the RHU of D. Public health nursing focuses on preventive,
their municipality. You’re right! This question is not curative, services.
based on R.A.1054
Answer: (D) Public health nursing focuses on
8. “Public health services are given free of charge.” preventive, not curative, services.The catchment
Is this statement true or false? area in PHN consists of a residential community,
many of whom are well individuals who have
A. The statement is true; it is the responsibility of greater need for preventive rather than curative
government to provide basic services. services.
B. The statement is false; people pay indirectly
for public health services. 12. According to Margaret Shetland, the philosophy
C. The statement may be true or false, depending of public health nursing is based on which of the
on the specific service required. following?
D. The statement may be true or false, depending
on policies of the government concerned. A. Health and longevity as birthrights
B. The mandate of the state to protect the
Answer: (B) The statement is false; people pay birthrights of its citizens
indirectly for public health services. Community C. Public health nursing as a specialized field of
health services, including public health services, are nursing
pre-paid services, though taxation, for example. D. The worth and dignity of man

9. According to C.E.Winslow, which of the Answer: (D) The worth and dignity of man. This
following is the goal of Public Health? is a direct quote from Dr. Margaret Shetland’s
statements on Public Health Nursing
A. For people to attain their birthrights of
health and longevity 13. Which of the following is the mission of the
B. For promotion of health and prevention of Department of Health?
disease
C. For people to have access to basic health A. Health for all Filipinos
services B. Ensure the accessibility and quality of
D. For people to be organized in their health health care
efforts C. Improve the general health status of the
population
Answer: (A) For people to attain their D. Health in the hands of the Filipino people by
birthrights of health and longevity. According to the year 2020
Winslow, all public health efforts are for people to
realize their birthrights of health and longevity Answer: (B) Ensure the accessibility and quality
of health care
10. We say that a Filipino has attained longevity
when he is able to reach the average lifespan of 14. Region IV Hospital is classified as what level of
Filipinos. What other statistic may be used to facility?
determine attainment of longevity?
A. Primary
A. Age-specific mortality rate B. Secondary
B. Proportionate mortality rate C. Intermediate
C. Swaroop’s index D. Tertiary
D. Case fatality rate
Answer: (D) Tertiary. Regional hospitals are
Answer: (C) Swaroop’s index. Swaroop’s index is tertiary facilities because they serve as training
the percentage of the deaths aged 50 years or older. hospitals for the region.
Its inverse represents the percentage of untimely
deaths (those who died younger than 50 years 15. Which is true of primary facilities?

11. Which of the following is the most prominent A. They are usually government-run.
feature of public health nursing? B. Their services are provided on an out-
patient basis.
A. It involves providing home care to sick people C. They are training facilities for health
who are not confined in the hospital. professionals.
B. Services are provided free of charge to people D. A community hospital is an example of this
within the catchment area. level of health facilities.
C. The public health nurse functions as part of a
team providing a public health nursing services.
Answer: (B) Their services are provided on an Answer: (C) To empower the people and
out-patient basis. Primary facilities government promote their self-reliance. People
and non-government facilities that provide basic empowerment is the basic motivation behind
out-patient services devolution of basic services to LGU’s.

16. Which is an example of the school nurse’s 20. Who is the Chairman of the Municipal Health
health care provider functions? Board?

A. Requesting for BCG from the RHU for school A. Mayor


entrant immunization B. Municipal Health Officer
B. Conducting random classroom inspection C. Public Health Nurse
during a measles epidemic D. Any qualified physician
C. Taking remedial action on an accident hazard
in the school playground Answer: (A) Mayor. The local executive serves as
D. Observing places in the school where pupils the chairman of the Municipal Health Board.
spend their free time
21. Which level of health facility is the usual point
Answer: (B) Conducting random classroom of entry of a client into the health care delivery
inspection during a measles epidemic. Random system?
classroom inspection is assessment of
pupils/students and teachers for signs of a health A. Primary
problem prevalent in the community B. Secondary
C. Intermediate
17. When the nurse determines whether resources D. Tertiary
were maximized in implementing Ligtas Tigdas,
she is evaluating Answer: (A) Primary. The entry of a person into
the health care delivery system is usually through a
A. Effectiveness consultation in out-patient services.
B. Efficiency
C. Adequacy 22. The public health nurse is the supervisor of rural
D. Appropriateness health midwives. Which of the following is a
supervisory function of the public health nurse?
Answer: (B) Efficiency. Efficiency is determining
whether the goals were attained at the least possible A. Referring cases or patients to the midwife
cost. B. Providing technical guidance to the midwife
C. Providing nursing care to cases referred by the
18. You are a new B.S.N. graduate. You want to midwife
become a Public Health Nurse. Where will you D. Formulating and implementing training
apply? programs for midwives

A. Department of Health Answer: (B) Providing technical guidance to the


B. Provincial Health Office midwife. The nurse provides technical guidance to
C. Regional Health Office the midwife in the care of clients, particularly in the
D. Rural Health Unit implementation of management guidelines, as in
Integrated Management of Childhood Illness.
Answer: (D) Rural Health Unit. R.A. 7160
devolved basic health services to local government 23. One of the participants in a hilot training class
units (LGU’s ). The public health nurse is an asked you to whom she should refer a patient in
employee of the LGU. labor who develops a complication. You will
answer, to the
19. R.A. 7160 mandates devolution of basic
services from the national government to local A. Public Health Nurse
government units. Which of the following is the B. Rural Health Midwife
major goal of devolution? C. Municipal Health Officer
D. Any of these health professionals
A. To strengthen local government units
B. To allow greater autonomy to local Answer: (C) Municipal Health Officer. A public
government units health nurse and rural health midwife can provide
C. To empower the people and promote their care during normal childbirth. A physician should
self-reliance attend to a woman with a complication during labor.
D. To make basic services more accessible to the
people
24. You are the public health nurse in a health nurse develops the health capability of
municipality with a total population of about 20,000. people through health education and community
There are 3 rural health midwives among the RHU organizing activities
personnel. How many more midwife items will the
RHU need? 28. Which disease was declared through
Presidential Proclamation No. 4 as a target for
A. 1 eradication in the Philippines?
B. 2
C. 3 A. Poliomyelitis
D. The RHU does not need any more midwife B. Measles
item. C. Rabies
D. Neonatal tetanus
Answer: (A) 1. Each rural health midwife is given a
population assignment of about 5,000. Answer: (B) Measles. Presidential Proclamation No.
4 is on the Ligtas Tigdas Program.
25. If the RHU needs additional midwife items, you
will submit the request for additional midwife items 29. The public health nurse is responsible for
for approval to the presenting the municipal health statistics using
graphs and tables. To compare the frequency of the
A. Rural Health Unit leading causes of mortality in the municipality,
B. District Health Office which graph will you prepare?
C. Provincial Health Office
D. Municipal Health Board A. Line
B. Bar
Answer: (D) Municipal Health Board. As C. Pie
mandated by R.A. 7160, basic health services have D. Scatter diagram
been devolved from the national government to
local government units Answer: (B) Bar. A bar graph is used to present
comparison of values, a line graph for trends over
26. As an epidemiologist, the nurse is responsible time or age, a pie graph for population composition
for reporting cases of notifiable diseases. What law or distribution, and a scatter diagram for correlation
mandates reporting of cases of notifiable diseases? of two variables.

A. Act 3573 30. Which step in community organizing involves


B. R.A. 3753 training of potential leaders in the community?
C. R.A. 1054
D. R.A. 1082 A. Integration
B. Community organization
Answer: (A) Act 3573. Act 3573, the Law on C. Community study
Reporting of Communicable Diseases, enacted in D. Core group formation
1929, mandated the reporting of diseases listed in
the law to the nearest health station Answer: (D) Core group formation. In core group
formation, the nurse is able to transfer the
27. According to Freeman and Heinrich, community technology of community organizing to the
health nursing is a developmental service. Which of potential or informal community leaders through a
the following best illustrates this statement? training program.

A. The community health nurse continuously


develops himself personally and professionally.
PNLE Community
B. Health education and community organizing
are necessary in providing community
health services.
Health Nursing Exam
C. Community health nursing is intended
primarily for health promotion and prevention
and treatment of disease.
2
D. The goal of community health nursing is to 1. In which step are plans formulated for solving
provide nursing services to people in their own community problems?
places of residence.
A. Mobilization
Answer: (B) Health education and community B. Community organization
organizing are necessary in providing C. Follow-up/extension
community health services. The community D. Core group formation
the terminally ill (those in the terminal stage of a
Answer: (B) Community disease)
organization. Community organization is the step
when community assemblies take place. During 5. Isolation of a child with measles belongs to what
the community assembly, the people may opt to level of prevention?
formalize the community organization and make
plans for community action to resolve a A. Primary
community health problem. B. Secondary
C. Intermediate
2. The public health nurse takes an active role in D. Tertiary
community participation. What is the primary goal
of community organizing? Answer: (A) Primary. The purpose of isolating a
client with a communicable disease is to protect
A. To educate the people regarding community those who are not sick (specific disease
health problems prevention).
B. To mobilize the people to resolve community
health problems 6. On the other hand, Operation Timbang is _____
C. To maximize the community’s resources in prevention.
dealing with health problems
D. To maximize the community’s resources in A. Primary
dealing with health problems B. Secondary
C. Intermediate
Answer: (D) To maximize the community’s D. Tertiary
resources in dealing with health
problems. Community organizing is a Answer: (B) Secondary. Operation Timbang is
developmental service, with the goal of developing done to identify members of the susceptible
the people’s self-reliance in dealing with population who are malnourished. Its purpose is
community health problems. A, B and C are early diagnosis and, subsequently, prompt
objectives of contributory objectives to this goal. treatment.

3. An indicator of success in community organizing 7. Which type of family-nurse contact will provide
is when people are able to you with the best opportunity to observe family
dynamics?
A. Participate in community activities for the
solution of a community problem A. Clinic consultation
B. Implement activities for the solution of the B. Group conference
community problem C. Home visit
C. Plan activities for the solution of the D. Written communication
community problem
D. Identify the health problem as a common Answer: (C) Home visit. Dynamics of family
concern relationships can best be observed in the family’s
natural environment, which is the home.
Answer: (A) Participate in community
activities for the solution of a community 8. The typology of family nursing problems is used
problem. Participation in community activities in the statement of nursing diagnosis in the care of
in resolving a community problem may be in families. The youngest child of the de los Reyes
any of the processes mentioned in the other family has been diagnosed as mentally retarded.
choices This is classified as a:

A. Health threat
4. Tertiary prevention is needed in which stage of B. Health deficit
the natural history of disease? C. Foreseeable crisis
D. Stress point
A. Pre-pathogenesis
B. Pathogenesis Answer: (B) Health deficit. Failure of a family
C. Prodromal member to develop according to what is expected,
D. Terminal as in mental retardation, is a health deficit.

Answer: (D) Terminal. Tertiary prevention 9. The de los Reyes couple have a 6-year old child
involves rehabilitation, prevention of permanent entering school for the first time. The de los Reyes
disability and disability limitation appropriate for family has a:
convalescents, the disabled, complicated cases and
A. Health threat performed before and after handling a client in the
B. Health deficit home to prevent transmission of infection to and
C. Foreseeable crisis from the client.
D. Stress point
13. To maintain the cleanliness of the bag and its
Answer: (C) Foreseeable crisis. Entry of the 6- contents, which of the following must the nurse do?
year old into school is an anticipated period of
unusual demand on the family. A. Wash his/her hands before and after
providing nursing care to the family
10. Which of the following is an advantage of a members.
home visit? B. In the care of family members, as much as
possible, use only articles taken from the bag.
A. It allows the nurse to provide nursing care to a C. Put on an apron to protect her uniform and fold
greater number of people. it with the right side out before putting it back
B. It provides an opportunity to do first hand into the bag.
appraisal of the home situation. D. At the end of the visit, fold the lining on which
C. It allows sharing of experiences among people the bag was placed, ensuring that the
with similar health problems. contaminated side is on the outside.
D. It develops the family’s initiative in providing
for health needs of its members. Answer: (A) Wash his/her hands before and
after providing nursing care to the family
Answer: (B) It provides an opportunity to do members. Choice B goes against the idea of
first hand appraisal of the home utilizing the family’s resources, which is
situation.. Choice A is not correct since a home encouraged in CHN. Choices C and D goes against
visit requires that the nurse spend so much time the principle of asepsis of confining the
with the family. Choice C is an advantage of a contaminated surface of objects.
group conference, while choice D is true of a clinic
consultation. 14. The public health nurse conducts a study on the
factors contributing to the high mortality rate due to
11. Which is CONTRARY to the principles in heart disease in the municipality where she works.
planning a home visit? Which branch of epidemiology does the nurse
practice in this situation?
A. A home visit should have a purpose or
objective. A. Descriptive
B. The plan should revolve around family health B. Analytical
needs. C. Therapeutic
C. A home visit should be conducted in the D. Evaluation
manner prescribed by the RHU.
D. Planning of continuing care should involve a Answer: (B) Analytical. Analytical epidemiology
responsible family member. is the study of factors or determinants affecting the
patterns of occurrence and distribution of disease
Answer: (C) A home visit should be conducted in a community.
in the manner prescribed by the RHU.The home
visit plan should be flexible and practical, 15. Which of the following is a function of
depending on factors, such as the family’s needs epidemiology?
and the resources available to the nurse and the
family. A. Identifying the disease condition based on
manifestations presented by a client
12. The PHN bag is an important tool in providing B. Determining factors that contributed to the
nursing care during a home visit. The most occurrence of pneumonia in a 3 year old
important principle of bag technique states that it C. Determining the efficacy of the antibiotic used
in the treatment of the 3 year old client with
A. Should save time and effort. pneumonia
B. Should minimize if not totally prevent the D. Evaluating the effectiveness of the
spread of infection. implementation of the Integrated
C. Should not overshadow concern for the patient Management of Childhood Illness
and his family.
D. May be done in a variety of ways depending on Answer: (D) Evaluating the effectiveness of the
the home situation, etc. implementation of the Integrated Management
of Childhood Illness. Epidemiology is used in the
Answer: (B) Should minimize if not totally assessment of a community or evaluation of
prevent the spread of infection. Bag technique is interventions in community health practice.
B. Testing the hypothesis
16. Which of the following is an epidemiologic C. Formulation of the hypothesis
function of the nurse during an epidemic? D. Appraisal of facts

A. Conducting assessment of suspected cases to Answer: (A) Establishing the


detect the communicable disease epidemic. Establishing the epidemic is
B. Monitoring the condition of the cases affected determining whether there is an epidemic or not.
by the communicable disease This is done by comparing the present number of
C. Participating in the investigation to cases with the usual number of cases of the disease
determine the source of the epidemic at the same time of the year, as well as establishing
D. Teaching the community on preventive the relatedness of the cases of the disease.
measures against the disease
20. The number of cases of Dengue fever usually
Answer: (C) Participating in the investigation to increases towards the end of the rainy season. This
determine the source of the pattern of occurrence of Dengue fever is best
epidemic. Epidemiology is the study of patterns of described as
occurrence and distribution of disease in the
community, as well as the factors that affect A. Epidemic occurrence
disease patterns. The purpose of an epidemiologic B. Cyclical variation
investigation is to identify the source of an C. Sporadic occurrence
epidemic, i.e., what brought about the epidemic. D. Secular variation

17. The primary purpose of conducting an Answer: (B) Cyclical variation. A cyclical
epidemiologic investigation is to variation is a periodic fluctuation in the number of
cases of a disease in the community.
A. Delineate the etiology of the epidemic
B. Encourage cooperation and support of the 21. In the year 1980, the World Health Organization
community declared the Philippines, together with some other
C. Identify groups who are at risk of contracting countries in the Western Pacific Region, “free” of
the disease which disease?
D. Identify geographical location of cases of the
disease in the community A. Pneumonic plague
B. Poliomyelitis
Answer: (A) Delineate the etiology of the C. Small pox
epidemic. Delineating the etiology of an epidemic D. Anthrax
is identifying its source.
Answer: (C) Small pox. The last documented case
18. Which is a characteristic of person-to-person of Small pox was in 1977 at Somalia.
propagated epidemics?
22. In the census of the Philippines in 1995, there
A. There are more cases of the disease than were about 35,299,000 males and about 34,968,000
expected. females. What is the sex ratio?
B. The disease must necessarily be transmitted
through a vector. A. 99.06:100
C. The spread of the disease can be attributed to a B. 100.94:100
common vehicle. C. 50.23%
D. There is a gradual build up of cases before D. 49.76%
the epidemic becomes easily noticeable.
Answer: (B) 100.94:100. Sex ratio is the number
Answer: (D) There is a gradual build up of of males for every 100 females in the population
cases before the epidemic becomes easily
noticeable. A gradual or insidious onset of the 23. Primary health care is a total approach to
epidemic is usually observable in person-to-person community development. Which of the following is
propagated epidemics. an indicator of success in the use of the primary
health care approach?
19. In the investigation of an epidemic, you
compare the present frequency of the disease with A. Health services are provided free of charge to
the usual frequency at this time of the year in this individuals and families.
community. This is done during which stage of the B. Local officials are empowered as the major
investigation? decision makers in matters of health.
C. Health workers are able to provide care based
A. Establishing the epidemic on identified health needs of the people.
D. Health programs are sustained according to C. Presidential Decree No. 147
the level of development of the community. D. Presidential Decree 996

Answer: (D) Health programs are sustained Answer: (B) Letter of Instruction No. 949. Letter
according to the level of development of the of Instruction 949 was issued by then President
community. Primary health care is essential Ferdinand Marcos, directing the formerly called
health care that can be sustained in all stages of Ministry of Health, now the Department of Health,
development of the community. to utilize Primary Health Care approach in
planning and implementing health programs
24. Sputum examination is the major screening tool
for pulmonary tuberculosis. Clients would 29. Which of the following demonstrates
sometimes get false negative results in this exam. intersectoral linkages?
This means that the test is not perfect in terms of
which characteristic of a diagnostic examination? A. Two-way referral system
B. Team approach
A. Effectiveness C. Endorsement done by a midwife to another
B. Efficacy midwife
C. Specificity D. Cooperation between the PHN and public
D. Sensitivity school teacher
Answer: (D) Sensitivity. Sensitivity is the capacity
of a diagnostic examination to detect cases of the Answer: (D) Cooperation between the PHN
disease. If a test is 100% sensitive, all the cases and public school teacher. Intersectoral
tested will have a positive result, i.e., there will be linkages refer to working relationships between
no false negative results. the health sector and other sectors involved in
community development.
25. Use of appropriate technology requires
knowledge of indigenous technology. Which 30. The municipality assigned to you has a
medicinal herb is given for fever, headache and population of about 20,000. Estimate the number of
cough? 1-4 year old children who will be given Retinol
capsule 200,000 I.U. every 6 months.
A. Sambong
B. Tsaang gubat A. 1,500
C. Akapulko B. 1,800
D. Lagundi C. 2,000
D. 2,300
Answer: (D) Lagundi. Sambong is used as a
diuretic. Tsaang gubat is used to relieve diarrhea. Answer: (D) 2,300. Based on the Philippine
Akapulko is used for its antifungal property population composition, to estimate the number of
1-4 year old children, multiply total population by
26. What law created the Philippine Institute of 11.5%.
Traditional and Alternative Health Care?

A. R.A. 8423
B. R.A. 4823
C. R.A. 2483
D. R.A. 3482
27. In traditional Chinese medicine, the yielding, PNLE Community
negative and feminine force is termed

A. Yin
Health Nursing Exam
B. Yang
C.
D.
Qi
Chai
3
1. Estimate the number of pregnant women who
Answer: (A) Yin. Yang is the male dominating, will be given tetanus toxoid during an immunization
positive and masculine force. outreach activity in a barangay with a population of
about 1,500.
28. What is the legal basis for Primary Health Care
approach in the Philippines? A. 265
B. 300
A. Alma Ata Declaration on PHC C. 375
B. Letter of Instruction No. 949 D. 400
6. Which statistic can give the most accurate
Answer: (A) 265. To estimate the number of reflection of the health status of a community?
pregnant women, multiply the total population by
3.5%. A. 1-4 year old age-specific mortality rate
B. Infant mortality rate
2. To describe the sex composition of the C. Swaroop’s index
population, which demographic tool may be used? D. Crude death rate

A. Sex ratio Answer: (C) Swaroop’s index. Swaroop’s


B. Sex proportion index is the proportion of deaths aged 50 years
C. Population pyramid and above. The higher the Swaroop’s index of a
D. Any of these may be used. population, the greater the proportion of the
deaths who were able to reach the age of at least
Answer: (D) Any of these may be used. Sex ratio 50 years, i.e., more people grew old before they
and sex proportion are used to determine the sex died.
composition of a population. A population pyramid
is used to present the composition of a population 7. In the past year, Barangay A had an average
by age and sex. population of 1655. 46 babies were born in that year,
2 of whom died less than 4 weeks after they were
3. Which of the following is a natality rate? born. There were 4 recorded stillbirths. What is the
neonatal mortality rate?
A. Crude birth rate
B. Neonatal mortality rate A. 27.8/1,000
C. Infant mortality rate B. 43.5/1,000
D. General fertility rate C. 86.9/1,000
D. 130.4/1,000
Answer: (A) Crude birth rate. Natality means
birth. A natality rate is a birth rate. Answer: (B) 43.5/1,000. To compute for neonatal
mortality rate, divide the number of babies who
4. You are computing the crude death rate of your died before reaching the age of 28 days by the total
municipality, with a total population of about number of live births, then multiply by 1,000.
18,000, for last year. There were 94 deaths. Among
those who died, 20 died because of diseases of the 8. Which statistic best reflects the nutritional status
heart and 32 were aged 50 years or older. What is of a population?
the crude death rate?
A. 1-4 year old age-specific mortality rate
A. 4.2/1,000 B. Proportionate mortality rate
B. 5.2/1,000 C. Infant mortality rate
C. 6.3/1,000 D. Swaroop’s index
D. 7.3/1,000
Answer: (A) 1-4 year old age-specific mortality
Answer: (B) 5.2/1,000. To compute crude death rate. Since preschoolers are the most susceptible to
rate divide total number of deaths (94) by total the effects of malnutrition, a population with poor
population (18,000) and multiply by 1,000 nutritional status will most likely have a high 1-4
year old age-specific mortality rate, also known as
5. Knowing that malnutrition is a frequent child mortality rate.
community health problem, you decided to conduct
nutritional assessment. What population is 9. What numerator is used in computing general
particularly susceptible to protein energy fertility rate?
malnutrition (PEM)?
A. Estimated midyear population
A. Pregnant women and the elderly B. Number of registered live births
B. Under-5 year old children C. Number of pregnancies in the year
C. 1-4 year old children D. Number of females of reproductive age
D. School age children
Answer: (B) Number of registered live births. To
Answer: (C) 1-4 year old children. Preschoolers compute for general or total fertility rate, divide the
are the most susceptible to PEM because they have number of registered live births by the number of
generally been weaned. Also, this is the population females of reproductive age (15-45 years), then
who, unable to feed themselves, are often the multiply by 1,000.
victims of poor intrafamilial food distribution.
10. You will gather data for nutritional assessment A. P.D. 651
of a purok. You will gather information only from B. Act 3573
families with members who belong to the target C. R.A. 3753
population for PEM. What method of data gathering D. R.A. 3375
is best for this purpose? Answer: (A) P.D. 651. P.D. 651 amended R.A.
3753, requiring the registry of births within 30
A. Census days from their occurrence
B. Survey
C. Record review 15. Which of the following professionals can sign
D. Review of civil registry the birth certificate?

Answer: (B) Survey. A survey, also called sample A. Public health nurse
survey, is data gathering about a sample of the B. Rural health midwife
population C. Municipal health officer
D. Any of these health professionals
11. In the conduct of a census, the method of
population assignment based on the actual physical Answer: (D) Any of these health professionals. D.
location of the people is termed R.A. 3753 states that any birth attendant may sign
the certificate of live birth.
A. De jure
B. De locus 16. Which criterion in priority setting of health
C. De facto problems is used only in community health care?
D. De novo
A. Modifiability of the problem
Answer: (C) De facto. The other method of B. Nature of the problem presented
population assignment, de jure, is based on the C. Magnitude of the health problem
usual place of residence of the people. D. Preventive potential of the health problem

12. The Field Health Services and Information Answer: (C) Magnitude of the health
System (FHSIS) is the recording and reporting problem. Magnitude of the problem refers to the
system in public health care in the Philippines. The percentage of the population affected by a health
Monthly Field Health Service Activity Report is a problem. The other choices are criteria considered
form used in which of the components of the FHSIS? in both family and community health care

A. Tally report 17. The Sentrong Sigla Movement has been


B. Output report launched to improve health service delivery. Which
C. Target/client list of the following is/are true of this movement?
D. Individual health record
A. This is a project spearheaded by local
Answer: (A) Tally report. A tally report is government units.
prepared monthly or quarterly by the RHU B. It is a basis for increasing funding from local
personnel and transmitted to the Provincial Health government units.
Office C. It encourages health centers to focus on disease
prevention and control.
13. To monitor clients registered in long-term D. Its main strategy is certification of health
regimens, such as the Multi-Drug Therapy, which centers able to comply with standards.
component will be most useful?
Answer: (D) Its main strategy is certification
A. Tally report of health centers able to comply with
B. Output report standards. Sentrong Sigla Movement is a joint
C. Target/client list project of the DOH and local government units.
D. Individual health record Its main strategy is certification of health
centers that are able to comply with standards
Answer: (C) Target/client list. The MDT Client set by the DOH.
List is a record of clients enrolled in MDT and
other relevant data, such as dates when clients 18. Which of the following women should be
collected their monthly supply of drugs considered as special targets for family planning?

14. Civil registries are important sources of data. A. Those who have two children or more
Which law requires registration of births within 30 B. Those with medical conditions such as anemia
days from the occurrence of the birth? C. Those younger than 20 years and older than 35
years
D. Those who just had a delivery within the B. Riboflavin
past 15 months C. Folic acid
D. Thiamine
Answer: (D) Those who just had a delivery
within the past 15 months. The ideal birth Answer: (C) Folic acid. It is estimated that the
spacing is at least two years. 15 months plus 9 incidence of neural tube defects can be reduced
months of pregnancy = 2 years drastically if pregnant women have an adequate
intake of folic acid.
19. Freedom of choice is one of the policies of the
Family Planning Program of the Philippines. Which 23. You are in a client’s home to attend to a
of the following illustrates this principle? delivery. Which of the following will you do first?

A. Information dissemination about the need for A. Set up the sterile area.
family planning B. Put on a clean gown or apron.
B. Support of research and development in family C. Cleanse the client’s vulva with soap and water.
planning methods D. Note the interval, duration and intensity of
C. Adequate information for couples regarding labor contractions.
the different methods
D. Encouragement of couples to take family Answer: (D) Note the interval, duration and
planning as a joint responsibility intensity of labor contractions.. Assessment
of the woman should be done first to determine
Answer: (C) Adequate information forcouples whether she is having true labor and, if so,
regarding the different methods. To enable the what stage of labor she is in.
couple to choose freely among different methods of
family planning, they must be given full 24. In preparing a primigravida for breastfeeding,
information regarding the different methods that are which of the following will you do?
available to them, considering the availability of
quality services that can support their choice. A. Tell her that lactation begins within a day after
delivery.
20. A woman, 6 months pregnant, came to the B. Teach her nipple stretching exercises if her
center for consultation. Which of the following nipples are everted.
substances is contraindicated? C. Instruct her to wash her nipples before and
after each breastfeeding.
A. Tetanus toxoid D. Explain to her that putting the baby to breast
B. Retinol 200,000 IU will lessen blood loss after delivery.
C. Ferrous sulfate 200 mg
D. Potassium iodate 200 mg. capsule Answer: (D) Explain to her that putting the baby
to breast will lessen blood loss after
Answer: (B) Retinol 200,000 IU. Retinol 200,000 delivery. Suckling of the nipple stimulates the
IU is a form of megadose Vitamin A. This may release of oxytocin by the posterior pituitary gland,
have a teratogenic effect which causes uterine contraction. Lactation begins 1
to 3 days after delivery. Nipple stretching exercises
21. During prenatal consultation, a client asked you are done when the nipples are flat or inverted.
if she can have her delivery at home. After history Frequent washing dries up the nipples, making them
taking and physical examination, you advised her prone to the formation of fissures.
against a home delivery. Which of the following
findings disqualifies her for a home delivery? 25. A primigravida is instructed to offer her breast
to the baby for the first time within 30 minutes after
A. Her OB score is G5P3. delivery. What is the purpose of offering the breast
B. She has some palmar pallor. this early?
C. Her blood pressure is 130/80.
D. Her baby is in cephalic presentation. A. To initiate the occurrence of milk letdown
B. To stimulate milk production by the
Answer: (A) Her OB score is G5P3. Only women mammary acini
with less than 5 pregnancies are qualified for a C. To make sure that the baby is able to get the
home delivery. It is also advisable for a colostrum
primigravida to have delivery at a childbirth D. To allow the woman to practice breastfeeding
facility in the presence of the health worker
22. Inadequate intake by the pregnant woman of
which vitamin may cause neural tube defects? Answer: (B) To stimulate milk production by
the mammary acini. Suckling of the nipple
A. Niacin stimulates prolactin reflex (the release of prolactin
by the anterior pituitary gland), which initiates highly sensitive to heat, requiring storage in the
lactation freezer

26. In a mothers’ class, you discuss proper 30. Unused BCG should be discarded how many
breastfeeding technique. Which is of these is a sign hours after reconstitution?
that the baby has “latched on” to the breast properly?
A. 2
A. The baby takes shallow, rapid sucks. B. 4
B. The mother does not feel nipple pain. C. 6
C. The baby’s mouth is only partly open. D. At the end of the day
D. Only the mother’s nipple is inside the baby’s
mouth. Answer: (B) 4. While the unused portion of
other biologicals in EPI may be given until the
Answer: (B) The mother does not feel nipple end of the day, only BCG is discarded 4 hours
pain.. When the baby has properly latched on to the after reconstitution. This is why BCG
breast, he takes deep, slow sucks; his mouth is wide immunization is scheduled only in the morning.
open; and much of the areola is inside his mouth.
And, you’re right! The mother does not feel nipple
pain.

27. You explain to a breastfeeding mother that


breast milk is sufficient for all of the baby’s nutrient PNLE Community
needs only up to ____.

A. 3 months Health Nursing Exam


B. 6 months
C.
D.
1 year
2 years
4
1. In immunizing school entrants with BCG, you are
Answer: (B) 6 months. After 6 months, the baby’s not obliged to secure parental consent. This is
nutrient needs, especially the baby’s iron because of which legal document?
requirement, can no longer be provided by
mother’s milk alone A. P.D. 996
B. R.A. 7846
28. What is given to a woman within a month after C. Presidential Proclamation No. 6
the delivery of a baby? D. Presidential Proclamation No. 46

A. Malunggay capsule Answer: (A) P.D. 996. Presidential Decree 996,


B. Ferrous sulfate 100 mg. OD enacted in 1976, made immunization in the EPI
C. Retinol 200,000 I.U., 1 capsule compulsory for children under 8 years of age.
D. Potassium iodate 200 mg, 1 capsule Hepatitis B vaccination was made compulsory for
the same age group by R.A. 7846.
Answer: (C) Retinol 200,000 I.U., 1 capsule. A
capsule of Retinol 200,000 IU is given within 1 2. Which immunization produces a permanent scar?
month after delivery. Potassium iodate is given
during pregnancy; malunggay capsule is not A. DPT
routinely administered after delivery; and B. BCG
ferrous sulfate is taken for two months after C. Measles vaccination
delivery. D. Hepatitis B vaccination

29. Which biological used in Expanded Program on Answer: (B) BCG. BCG causes the formation of a
Immunization (EPI) is stored in the freezer? superficial abscess, which begins 2 weeks after
immunization. The abscess heals without treatment,
A. DPT with the formation of a permanent scar
B. Tetanus toxoid
C. Measles vaccine 3. A 4-week old baby was brought to the health
D. Hepatitis B vaccine center for his first immunization. Which can be
given to him?
Answer: (C) Measles vaccine. Among the
biologicals used in the Expanded Program on A. DPT1
Immunization, measles vaccine and OPV are B. OPV1
C. Infant BCG
D. Hepatitis B vaccine 1 D. Insignificant

Answer: (C) Infant BCG. Infant BCG may be Answer: (C) Normal. In IMCI, a respiratory
given at birth. All the other immunizations rate of 50/minute or more is fast breathing for
mentioned can be given at 6 weeks of age an infant aged 2 to 12 months.

4. You will not give DPT 2 if the mother says that


the infant had 8. Which of the following signs will indicate that a
young child is suffering from severe pneumonia?
A. Seizures a day after DPT 1.
B. Fever for 3 days after DPT 1. A. Dyspnea
C. Abscess formation after DPT 1. B. Wheezing
D. Local tenderness for 3 days after DPT 1. C. Fast breathing
D. Chest indrawing
Answer: (A) Seizures a day after DPT 1. Seizures
within 3 days after administration of DPT is an Answer: (D) Chest indrawing. In IMCI, chest
indication of hypersensitivity to pertussis vaccine, a indrawing is used as the positive sign of dyspnea,
component of DPT. This is considered a specific indicating severe pneumonia
contraindication to subsequent doses of DPT.
9. Using IMCI guidelines, you classify a child as
5. A 2-month old infant was brought to the health having severe pneumonia. What is the best
center for immunization. During assessment, the management for the child?
infant’s temperature registered at 38.1°C. Which is
the best course of action that you will take? A. Prescribe an antibiotic.
B. Refer him urgently to the hospital.
A. Go on with the infant’s immunizations. C. Instruct the mother to increase fluid intake.
B. Give Paracetamol and wait for his fever to D. Instruct the mother to continue breastfeeding.
subside.
C. Refer the infant to the physician for further Answer: (B) Refer him urgently to the
assessment. hospital. Severe pneumonia requires urgent referral
D. Advise the infant’s mother to bring him back to a hospital. Answers A, C and D are done for a
for immunization when he is well. client classified as having pneumonia.

Answer: (A) Go on with the infant’s 10. A 5-month old infant was brought by his mother
immunizations. In the EPI, fever up to 38.5°C is to the health center because of diarrhea occurring 4
not a contraindication to immunization. Mild acute to 5 times a day. His skin goes back slowly after a
respiratory tract infection, simple diarrhea and skin pinch and his eyes are sunken. Using the IMCI
malnutrition are not contraindications either. guidelines, you will classify this infant in which
category?
6. A pregnant woman had just received her 4th dose
of tetanus toxoid. Subsequently, her baby will have A. No signs of dehydration
protection against tetanus for how long? B. Some dehydration
C. Severe dehydration
A. 1 year D. The data is insufficient.
B. 3 years
C. 10 years Answer: (B) Some dehydration. Using the
D. Lifetime assessment guidelines of IMCI, a child (2 months
Answer: (A) 1 year. The baby will have passive to 5 years old) with diarrhea is classified as having
natural immunity by placental transfer of SOME DEHYDRATION if he shows 2 or more of
antibodies. The mother will have active artificial the following signs: restless or irritable, sunken
immunity lasting for about 10 years. 5 doses will eyes, the skin goes back slow after a skin pinch
give the mother lifetime protection
11. Based on assessment, you classified a 3-month
7. A 4-month old infant was brought to the health old infant with the chief complaint of diarrhea in the
center because of cough. Her respiratory rate is category of SOME DEHYDRATION. Based on
42/minute. Using the Integrated Management of IMCI management guidelines, which of the
Child Illness (IMCI) guidelines of assessment, her following will you do?
breathing is considered
A. Bring the infant to the nearest facility where IV
A. Fast fluids can be given.
B. Slow B. Supervise the mother in giving 200 to 400 ml.
C. Normal of Oresol in 4 hours.
C. Give the infant’s mother instructions on home D. Assess and treat the child for health problems
management. like infections and intestinal parasitism.
D. Keep the infant in your health center for close
observation. Answer: (A) Refer the child urgently to a
hospital for confinement. “Baggy pants” is a
Answer: (B) Supervise the mother in giving 200 sign of severe marasmus. The best management
to 400 ml. of Oresol in 4 hours. In the IMCI is urgent referral to a hospital.
management guidelines, SOME DEHYDRATION
is treated with the administration of Oresol within a 15. During the physical examination of a young
period of 4 hours. The amount of Oresol is best child, what is the earliest sign of xerophthalmia that
computed on the basis of the child’s weight (75 you may observe?
ml/kg body weight). If the weight is unknown, the
amount of Oresol is based on the child’s age. A. Keratomalacia
B. Corneal opacity
12. A mother is using Oresol in the management of C. Night blindness
diarrhea of her 3-year old child. She asked you what D. Conjunctival xerosis
to do if her child vomits. You will tell her to
Answer: (D) Conjunctival xerosis. The earliest
A. Bring the child to the nearest hospital for sign of Vitamin A deficiency (xerophthalmia) is
further assessment. night blindness. However, this is a functional
B. Bring the child to the health center for change, which is not observable during physical
intravenous fluid therapy. examination.The earliest visible lesion is
C. Bring the child to the health center for conjunctival xerosis or dullness of the conjunctiva
assessment by the physician. due to inadequate tear production.
D. Let the child rest for 10 minutes then
continue giving Oresol more slowly. 16. To prevent xerophthalmia, young children are
given Retinol capsule every 6 months. What is the
Answer: (D) Let the child rest for 10 minutes dose given to preschoolers?
then continue giving Oresol more slowly. If the
child vomits persistently, that is, he vomits A. 10,000 IU
everything that he takes in, he has to be referred B. 20,000 IU
urgently to a hospital. Otherwise, vomiting is C. 100,000 IU
managed by letting the child rest for 10 minutes D. 200,000 IU
and then continuing with Oresol administration. Answer: (D) 200,000 IU. Preschoolers are given
Teach the mother to give Oresol more slowly Retinol 200,000 IU every 6 months. 100,000 IU is
13. A 1 ½ year old child was classified as having given once to infants aged 6 to 12 months. The
3rd degree protein energy malnutrition, kwashiorkor. dose for pregnant women is 10,000 IU
Which of the following signs will be most apparent 17. The major sign of iron deficiency anemia is
in this child? pallor. What part is best examined for pallor?

A. Voracious appetite A. Palms


B. Wasting B. Nailbeds
C. Apathy C. Around the lips
D. Edema D. Lower conjunctival sac

Answer: (D) Edema. Edema, a major sign of Answer: (A) Palms. The anatomic
kwashiorkor, is caused by decreased colloidal characteristics of the palms allow a reliable and
osmotic pressure of the blood brought about by convenient basis for examination for pallor.
hypoalbuminemia. Decreased blood albumin level
is due a protein-deficient diet. 18. Food fortification is one of the strategies to
prevent micronutrient deficiency conditions. R.A.
14. Assessment of a 2-year old child revealed 8976 mandates fortification of certain food items.
“baggy pants”. Using the IMCI guidelines, how will Which of the following is among these food items?
you manage this child?
A. Sugar
A. Refer the child urgently to a hospital for B. Bread
confinement. C. Margarine
B. Coordinate with the social worker to enroll the D. Filled milk
child in a feeding program.
C. Make a teaching plan for the mother, focusing Answer: (A) Sugar. R.A. 8976 mandates
on menu planning for her child. fortification of rice, wheat flour, sugar and cooking
oil with Vitamin A, iron and/or iodine
19. What is the best course of action when there is a Answer: (B) Ask where the family
measles epidemic in a nearby municipality? resides. Because malaria is endemic, the first
question to determine malaria risk is where the
A. Give measles vaccine to babies aged 6 to 8 client’s family resides. If the area of residence is not
months. a known endemic area, ask if the child had traveled
B. Give babies aged 6 to 11 months one dose of within the past 6 months, where he/she was brought
100,000 I.U. of Retinol and whether he/she stayed overnight in that area.
C. Instruct mothers to keep their babies at home to
prevent disease transmission. 23. The following are strategies implemented by the
D. Instruct mothers to feed their babies adequately Department of Health to prevent mosquito-borne
to enhance their babies’ resistance. diseases. Which of these is most effective in the
control of Dengue fever?
Answer: (A) Give measles vaccine to babies aged
6 to 8 months. Ordinarily, measles vaccine is given A. Stream seeding with larva-eating fish
at 9 months of age. During an impending epidemic, B. Destroying breeding places of mosquitoes
however, one dose may be given to babies aged 6 to C. Chemoprophylaxis of non-immune persons
8 months. The mother is instructed that the baby going to endemic areas
needs another dose when the baby is 9 months old. D. Teaching people in endemic areas to use
chemically treated mosquito nets
20. A mother brought her daughter, 4 years old, to
the RHU because of cough and colds. Following the Answer: (B) Destroying breeding places of
IMCI assessment guide, which of the following is a mosquitoes. Aedes aegypti, the vector of Dengue
danger sign that indicates the need for urgent fever, breeds in stagnant, clear water. Its feeding
referral to a hospital? time is usually during the daytime. It has a cyclical
pattern of occurrence, unlike malaria which is
A. Inability to drink endemic in certain parts of the country
B. High grade fever 24. Secondary prevention for malaria includes
C. Signs of severe dehydration
D. Cough for more than 30 days A. Planting of neem or eucalyptus trees
B. Residual spraying of insecticides at night
Answer: (A) Inability to drink. A sick child aged C. Determining whether a place is endemic or
2 months to 5 years must be referred urgently to a not
hospital if he/she has one or more of the following D. Growing larva-eating fish in mosquito breeding
signs: not able to feed or drink, vomits everything, places
convulsions, abnormally sleepy or difficult to
awaken Answer: (C) Determining whether a place is
endemic or not. This is diagnostic and therefore
21. Management of a child with measles includes secondary level prevention. The other choices are
the administration of which of the following? for primary prevention

A. Gentian violet on mouth lesions 25. Scotch tape swab is done to check for which
B. Antibiotics to prevent pneumonia intestinal parasite?
C. Tetracycline eye ointment for corneal opacity
D. Retinol capsule regardless of when the last A. Ascaris
dose was given B. Pinworm
C. Hookworm
Answer: (D) Retinol capsule regardless of when D. Schistosoma
the last dose was given. An infant 6 to 12 months
classified as a case of measles is given Retinol Answer: (B) Pinworm. Pinworm ova are deposited
100,000 IU; a child is given 200,000 IU regardless around the anal orifice.
of when the last dose was given
26. Which of the following signs indicates the need
22. A mother brought her 10 month old infant for for sputum examination for AFB?
consultation because of fever, which started 4 days
prior to consultation. To determine malaria risk, A. Hematemesis
what will you do? B. Fever for 1 week
C. Cough for 3 weeks
A. Do a tourniquet test. D. Chest pain for 1 week
B. Ask where the family resides.
C. Get a specimen for blood smear. Answer: (C) Cough for 3 weeks. A client is
D. Ask if the fever is present everyday. considered a PTB suspect when he has cough for 2
weeks or more, plus one or more of the following A. 3 skin lesions, negative slit skin smear
signs: fever for 1 month or more; chest pain lasting B. 3 skin lesions, positive slit skin smear
for 2 weeks or more not attributed to other C. 5 skin lesions, negative slit skin smear
conditions; progressive, unexplained weight loss; D. 5 skin lesions, positive slit skin smear
night sweats; and hemoptysis.
Answer: (D) 5 skin lesions, positive slit skin
27. Which clients are considered targets for DOTS smear. A multibacillary leprosy case is one who
Category I? has a positive slit skin smear and at least 5 skin
lesions
A. Sputum negative cavitary cases
B. Clients returning after a default
C. Relapses and failures of previous PTB
PNLE Community
treatment regimens
D. Clients diagnosed for the first time through a
positive sputum exam
Health Nursing Exam
\
Answer: (D) Clients diagnosed for the first time
through a positive sputum exam. Category I is for
5
new clients diagnosed by sputum examination and . In the Philippines, which condition is the most
clients diagnosed to have a serious form of frequent cause of death associated with
extrapulmonary tuberculosis, such as TB schistosomiasis?
osteomyelitis.
A. Liver cancer
28. To improve compliance to treatment, what B. Liver cirrhosis
innovation is being implemented in DOTS? C. Bladder cancer
D. Intestinal perforation
A. Having the health worker follow up the client
at home Answer: (B) Liver cirrhosis. The etiologic agent
B. Having the health worker or a responsible of schistosomiasis in the Philippines is
family member monitor drug intake Schistosoma japonicum, which affects the small
C. Having the patient come to the health center intestine and the liver. Liver damage is a
every month to get his medications consequence of fibrotic reactions to schistosoma
D. Having a target list to check on whether the eggs in the liver
patient has collected his monthly supply of
drugs 2. What is the most effective way of controlling
schistosomiasis in an endemic area?
Answer: (B) Having the health worker or a
responsible family member monitor drug A. Use of molluscicides
intake. Directly Observed Treatment Short Course B. Building of foot bridges
is so-called because a treatment partner, preferably C. Proper use of sanitary toilets
a health worker accessible to the client, monitors D. Use of protective footwear, such as rubber
the client’s compliance to the treatment. boots

29. Diagnosis of leprosy is highly dependent on Answer: (C) Proper use of sanitary toilets. The
recognition of symptoms. Which of the following is ova of the parasite get out of the human body
an early sign of leprosy? together with feces. Cutting the cycle at this stage
is the most effective way of preventing the spread
A. Macular lesions of the disease to susceptible hosts.
B. Inability to close eyelids
C. Thickened painful nerves 3. When residents obtain water from an artesian
D. Sinking of the nosebridge well in the neighborhood, the level of this approved
type of water facility is
Answer: (C) Thickened painful nerves. The
lesion of leprosy is not macular. It is characterized A. I
by a change in skin color (either reddish or whitish) B. II
and loss of sensation, sweating and hair growth C. III
over the lesion. Inability to close the eyelids D. IV
(lagophthalmos) and sinking of the nosebridge are
late symptoms Answer: (B) II. A communal faucet or water
standpost is classified as Level II.
30. Which of the following clients should be
classified as a case of multibacillary leprosy?
4. For prevention of hepatitis A, you decided to When the foregoing measures are not possible or
conduct health education activities. Which of the effective, tehn urgent referral to the hospital is done.
following is IRRELEVANT?
8. A client was diagnosed as having Dengue fever.
A. Use of sterile syringes and needles You will say that there is slow capillary refill when
B. Safe food preparation and food handling by the color of the nailbed that you pressed does not
vendors return within how many seconds?
C. Proper disposal of human excreta and personal
hygiene A. 3
D. Immediate reporting of water pipe leaks and B. 5
illegal water connections C. 8
D. 10
Answer: (A) Use of sterile syringes and
needles. Hepatitis A is transmitted through the fecal Answer: (A) 3. Adequate blood supply to the area
oral route. Hepatitis B is transmitted through allows the return of the color of the nailbed within 3
infected body secretions like blood and semen. seconds.

5. Which biological used in Expanded Program on 9. A 3-year old child was brought by his mother to
Immunization (EPI) should NOT be stored in the the health center because of fever of 4-day duration.
freezer? The child had a positive tourniquet test result. In the
absence of other signs, which is the most
A. DPT appropriate measure that the PHN may carry out to
B. Oral polio vaccine prevent Dengue shock syndrome?
C. Measles vaccine
D. MMR A. Insert an NGT and give fluids per NGT.
B. Instruct the mother to give the child Oresol.
Answer: (A) DPT. DPT is sensitive to freezing. C. Start the patient on intravenous fluids STAT.
The appropriate storage temperature of DPT is 2 to D. Refer the client to the physician for appropriate
8° C only. OPV and measles vaccine are highly management.
sensitive to heat and require freezing. MMR is not
an immunization in the Expanded Program on Answer: (B) Instruct the mother to give the child
Immunization. Oresol. Since the child does not manifest any other
danger sign, maintenance of fluid balance and
6. You will conduct outreach immunization in a replacement of fluid loss may be done by giving the
barangay with a population of about 1500. Estimate client Oresol.
the number of infants in the barangay.
10. The pathognomonic sign of measles is Koplik’s
A. 45 spot. You may see Koplik’s spot by inspecting the
B. 50 _____.
C. 55
D. 60 A. Nasal mucosa
B. Buccal mucosa
Answer: (A) 45. To estimate the number of infants, C. Skin on the abdomen
multiply total population by 3% D. Skin on the antecubital surface

7. In Integrated Management of Childhood Illness, Answer: (B) Buccal mucosa. Koplik’s spot may be
severe conditions generally require urgent referral seen on the mucosa of the mouth or the throat.
to a hospital. Which of the following severe
conditions DOES NOT always require urgent 11. Among the following diseases, which is
referral to a hospital? airborne?

A. Mastoiditis A. Viral conjunctivitis


B. Severe dehydration B. Acute poliomyelitis
C. Severe pneumonia C. Diphtheria
D. Severe febrile disease D. Measles

Answer: (B) Severe dehydration. The order of Answer: (D) Measles. Viral conjunctivitis is
priority in the management of severe dehydration is transmitted by direct or indirect contact with
as follows: intravenous fluid therapy, referral to a discharges from infected eyes. Acute poliomyelitis
facility where IV fluids can be initiated within 30 is spread through the fecal-oral route and contact
minutes, Oresol/nasogastric tube, Oresol/orem. with throat secretions, whereas diphtheria is through
direct and indirect contact with respiratory
secretions. Answer: (C) Destruction of breeding places of
the mosquito vector. Anopheles mosquitoes breed
12. Among children aged 2 months to 3 years, the in slow-moving, clear water, such as mountain
most prevalent form of meningitis is caused by streams.
which microorganism?
16. A 4-year old client was brought to the health
A. Hemophilus influenzae center with the chief complaint of severe diarrhea
B. Morbillivirus and the passage of “rice water” stools. The client is
C. Steptococcus pneumoniae most probably suffering from which condition?
D. Neisseria meningitides
A. Giardiasis
Answer: (A) Hemophilus B. Cholera
influenzae. Hemophilus meningitis is unusual over C. Amebiasis
the age of 5 years. In developing countries, the D. Dysentery
peak incidence is in children less than 6 months of
age. Morbillivirus is the etiology of measles. Answer: (B) Cholera. Passage of profuse watery
Streptococcus pneumoniae and Neisseria stools is the major symptom of cholera. Both
meningitidis may cause meningitis, but age amebic and bacillary dysentery are characterized by
distribution is not specific in young children the presence of blood and/or mucus in the stools.
Giardiasis is characterized by fat malabsorption and,
13. Human beings are the major reservoir of malaria. therefore, steatorrhea.
Which of the following strategies in malaria control
is based on this fact? 17. In the Philippines, which specie of schistosoma
is endemic in certain regions?
A. Stream seeding
B. Stream clearing A. S. mansoni
C. Destruction of breeding places A.
D. Zooprophylaxis B. S. malayensis
C. S. haematobium
Answer: (D) Zooprophylaxis. Zooprophylaxis is
done by putting animals like cattle or dogs close to Answer: (B) S. japonicum. S. mansoni is found
windows or doorways just before nightfall. The mostly in Africa and South America; S.
Anopheles mosquito takes his blood meal from the haematobium in Africa and the Middle East; and S.
animal and goes back to its breeding place, thereby malayensis only in peninsular Malaysia.
preventing infection of humans
18. A 32-year old client came for consultation at the
14. The use of larvivorous fish in malaria control is health center with the chief complaint of fever for a
the basis for which strategy of malaria control? week. Accompanying symptoms were muscle pains
and body malaise. A week after the start of fever,
A. Stream seeding the client noted yellowish discoloration of his sclera.
B. Stream clearing History showed that he waded in flood waters about
C. Destruction of breeding places 2 weeks before the onset of symptoms. Based on his
D. Zooprophylaxis history, which disease condition will you suspect?

Answer: (A) Stream seeding. Stream seeding is A. Hepatitis A


done by putting tilapia fry in streams or other B. Hepatitis B
bodies of water identified as breeding places of the C. Tetanus
Anopheles mosquito D. Leptospirosis
Answer: (D) Leptospirosis. Leptospirosis is
15. Mosquito-borne diseases are prevented mostly transmitted through contact with the skin or mucous
with the use of mosquito control measures. Which membrane with water or moist soil contaminated
of the following is NOT appropriate for malaria with urine of infected animals, like rats.
control?

A. Use of chemically treated mosquito nets 19. MWSS provides water to Manila and other
B. Seeding of breeding places with larva-eating cities in Metro Manila. This is an example of which
fish level of water facility?
C. Destruction of breeding places of the
mosquito vector A. I
D. Use of mosquito-repelling soaps, such as those B. II
with basil or citronella C. III
D. IV B. Community survey
Answer: (C) III. Waterworks systems, such as C. Mass screening tests
MWSS, are classified as level III. D. Interview of suspects

20. You are the PHN in the city health center. A Answer: (A) Contact tracing. Contact tracing is
client underwent screening for AIDS using ELISA. the most practical and reliable method of finding
His result was positive. What is the best course of possible sources of person-to-person transmitted
action that you may take? infections, such as sexually transmitted diseases

A. Get a thorough history of the client, focusing 24. Antiretroviral agents, such as AZT, are used in
on the practice of high risk behaviors. the management of AIDS. Which of the following
B. Ask the client to be accompanied by a is NOT an action expected of these drugs.
significant person before revealing the result.
C. Refer the client to the physician since he is the A. They prolong the life of the client with AIDS.
best person to reveal the result to the client. B. They reduce the risk of opportunistic infections
D. Refer the client for a supplementary test, C. They shorten the period of communicability of
such as Western blot, since the ELISA result the disease.
may be false. D. They are able to bring about a cure of the
disease condition.
Answer: (D) Refer the client for a supplementary
test, such as Western blot, since the ELISA result Answer: (D) They are able to bring about a cure
may be false. A client having a reactive ELISA of the disease condition. There is no known
result must undergo a more specific test, such as treatment for AIDS. Antiretroviral agents reduce
Western blot. A negative supplementary test result the risk of opportunistic infections and prolong life,
means that the ELISA result was false and that, but does not cure the underlying
most probably, the client is not infected. immunodeficiency

21. Which is the BEST control measure for AIDS? 25. A barangay had an outbreak of German measles.
To prevent congenital rubella, what is the BEST
A. Being faithful to a single sexual partner advice that you can give to women in the first
B. Using a condom during each sexual contact trimester of pregnancy in the barangay?
C. Avoiding sexual contact with commercial sex
workers A. Advice them on the signs of German measles.
D. Making sure that one’s sexual partner does not B. Avoid crowded places, such as markets and
have signs of AIDS moviehouses.
\ C. Consult at the health center where rubella
Answer: (A) Being faithful to a single sexual vaccine may be given.
partner. Sexual fidelity rules out the possibility of D. Consult a physician who may give them
getting the disease by sexual contact with another rubella immunoglobulin.
infected person. Transmission occurs mostly
through sexual intercourse and exposure to blood Answer: (D) Consult a physician who may give
or tissues them rubella immunoglobulin. Rubella vaccine is
made up of attenuated German measles viruses.
22. The most frequent causes of death among This is contraindicated in pregnancy. Immune
clients with AIDS are opportunistic diseases. Which globulin, a specific prophylactic against German
of the following opportunistic infections is measles, may be given to pregnant women.
characterized by tonsillopharyngitis?
26. You were invited to be the resource person in a
A. Respiratory candidiasis training class for food handlers. Which of the
B. Infectious mononucleosis following would you emphasize regarding
C. Cytomegalovirus disease prevention of staphylococcal food poisoning?
D. Pneumocystis carinii pneumonia
Answer: (B) Infectious A. All cooking and eating utensils must be
mononucleosis. Cytomegalovirus disease is an thoroughly washed.
acute viral disease characterized by fever, sore B. Food must be cooked properly to destroy
throat and lymphadenopathy. staphylococcal microorganisms.
C. Food handlers and food servers must have a
23. To determine possible sources of sexually negative stool examination result.
transmitted infections, which is the BEST method D. Proper handwashing during food
that may be undertaken by the public health nurse? preparation is the best way of preventing the
condition.
A. Contact tracing
Answer: (D) Proper handwashing during food
preparation is the best way of preventing the
condition. Symptoms of this food poisoning are PNLE Medical
due to staphylococcal enterotoxin, not the
microorganisms themselves. Contamination is by
food handling by persons with staphylococcal skin
Surgical Nursing
or eye infections

27. In a mothers’ class, you discussed childhood


Exam 1
diseases such as chicken pox. Which of the
following statements about chicken pox is correct?
SITUATION : Arthur, A registered nurse,
witnessed an old woman hit by a motorcycle while
A. The older one gets, the more susceptible he
crossing a train railway. The old woman fell at the
becomes to the complications of chicken pox.
railway. Arthur rushed at the scene.
B. A single attack of chicken pox will prevent
1. As a registered nurse, Arthur knew that the first
future episodes, including conditions such as
thing that he will do at the scene is
shingles.
C. To prevent an outbreak in the community,
quarantine may be imposed by health A. Stay with the person, Encourage her to remain
authorities. still and Immobilize the leg while While
D. Chicken pox vaccine is best given when there waiting for the ambulance.
is an impending outbreak in the community. B. Leave the person for a few moments to call for
help.
Answer: (A) The older one gets, the more C. Reduce the fracture manually.
susceptible he becomes to the complications of D. Move the person to a safer place.
chicken pox. Chicken pox is usually more severe in
adults than in children. Complications, such as 2. Arthur suspects a hip fracture when he noticed
pneumonia, are higher in incidence in adults. that the old woman’s leg is

28. Complications to infectious parotitis (mumps) A. A. Lengthened, Abducted and Internally


may be serious in which type of clients? Rotated.
B. Shortened, Abducted and Externally Rotated.
A. Pregnant women C. Shortened, Adducted and Internally Rotated.
B. Elderly clients D. Shortened, Adducted and Externally
C. Young adult males Rotated.
D. Young infants
Answer: (C) Young adult males. Epididymitis and 3. The old woman complains of pain. John noticed
orchitis are possible complications of mumps. In that the knee is reddened, warm to touch and
post-adolescent males, bilateral inflammation of the swollen. John interprets that this signs and
testes and epididymis may cause sterility. symptoms are likely related to

A. Infection
B. Thrombophlebitis
C. Inflammation
D. Degenerative disease
4. The old woman told John that she has
osteoporosis; Arthur knew that all of the following
factors would contribute to osteoporosis except

A. Hypothyroidism
B. End stage renal disease
C. Cushing’s Disease
D. Taking Furosemide and Phenytoin.
5. Martha, The old woman was now Immobilized
and brought to the emergency room. The X-ray
shows a fractured femur and pelvis. The ER Nurse
would carefully monitor Martha for which of the
following sign and symptoms?

A. Tachycardia and Hypotension


B. Fever and Bradycardia
C. Bradycardia and Hypertension
D. Fever and Hypertension B. 5 mm Induration
SITUATION: Mr. D. Rojas, An obese 35 year old C. 10 mm Wheal
MS Professor of OLFU Lagro is admitted due to D. 10 mm Induration
pain in his weight bearing joint. The diagnosis was
Osteoarthritis. 12. The nurse told Alfred to come back after
6. As a nurse, you instructed Mr. Rojas how to use a
cane. Mr. Rojas has a weakness on his right leg due A. a week
to self immobilization and guarding. You plan to B. 48 hours
teach Mr. Rojas to hold the cane C. 1 day
D. 4 days
A. On his left hand, because his right side is weak.
B. On his left hand, because of reciprocal 13. Mang Alfred returns after the Mantoux Test.
motion. The test result read POSITIVE. What should be the
C. On his right hand, to support the right leg. nurse’s next action?
D. On his right hand, because only his right leg is
weak. A. Call the Physician
7. You also told Mr. Rojas to hold the cane B. Notify the radiology dept. for CXR evaluation
C. Isolate the patient
A. 1 Inches in front of the foot. D. Order for a sputum exam
B. 3 Inches at the lateral side of the foot.
C. 6 Inches at the lateral side of the foot. 14. Why is Mantoux test not routinely done in the
D. 12 Inches at the lateral side of the foot. Philippines?

8. Mr. Rojas was discharged and 6 months later, he A. It requires a highly skilled nurse to perform a
came back to the emergency room of the hospital Mantoux test
because he suffered a mild stroke. The right side of B. The sputum culture is the gold standard of PTB
the brain was affected. At the rehabilitative phase of Diagnosis and it will definitively determine the
your nursing care, you observe Mr. Rojas use a cane extent of the cavitary lesions
and you intervene if you see him C. Chest X Ray Can diagnose the specific
microorganism responsible for the lesions
A. Moves the cane when the right leg is moved. D. Almost all Filipinos will test positive for
B. Leans on the cane when the right leg swings Mantoux Test
through.
C. keeps the cane 6 Inches out to the side of the 15. Mang Alfred is now a new TB patient with an
right foot. active disease. What is his category according to the
D. Holds the cane on the right side. DOH?

SITUATION: Alfred, a 40 year old construction A. I


worker developed cough, night sweats and fever. He B. II
was brought to the nursing unit for diagnostic C. III
studies. He told the nurse he did not receive a BCG D. IV
vaccine during childhood
9. The nurse performs a Mantoux Test. The nurse 16. How long is the duration of the maintenance
knows that Mantoux Test is also known as phase of his treatment?

A. PPD A. 2 months
B. PDP B. 3 months
C. PDD C. 4 months
D. DPP D. 5 months

10. The nurse would inject the solution in what 17. Which of the following drugs is UNLIKELY
route? given to Mang Alfred during the maintenance phase?

A. IM A. Rifampicin
B. IV B. Isoniazid
C. ID C. Ethambutol
D. SC D. Pyridoxine

11. The nurse notes that a positive result for Alfred 18. According to the DOH, the most hazardous
is period for development of clinical disease is during
the first
A. 5 mm wheal
A. 6-12 months after 25. The maximum height of irrigation solution for
B. 3-6 months after colostomy is
C. 1-2 months after
D. 2-4 weeks after A. 5 inches
B. 12 inches
19. This is the name of the program of the DOH to C. 18 inches
control TB in the country D. 24 inches

A. DOTS 26. Which of the following behavior of the client


B. National Tuberculosis Control Program indicates the best initial step in learning to care for
C. Short Coursed Chemotherapy his colostomy?
D. Expanded Program for Immunization
A. Ask to defer colostomy care to another
20. Susceptibility for the disease [ TB ] is increased individual
markedly in those with the following condition B. Promises he will begin to listen the next day
except C. Agrees to look at the colostomy
D. States that colostomy care is the function of the
A. 23 Year old athlete with diabetes insipidus nurse while he is in the hospital
B. 23 Year old athlete taking long term Decadron
therapy and anabolic steroids 27. While irrigating the client’s colostomy, Michiel
C. 23 Year old athlete taking illegal drugs and suddenly complains of severe cramping. Initially,
abusing substances the nurse would
D. Undernourished and Underweight individual
who undergone gastrectomy A. Stop the irrigation by clamping the tube
B. Slow down the irrigation
21. Direct sputum examination and Chest X ray of C. Tell the client that cramping will subside and is
TB symptomatic is in what level of prevention? normal
D. Notify the physician
A. Primary
B. Secondary 28. The next day, the nurse will assess Michiel’s
C. Tertiary stoma. The nurse noticed that a prolapsed stoma is
D. Quarterly evident if she sees which of the following?

SITUATION: Michiel, A male patient diagnosed A. A sunken and hidden stoma


with colon cancer was newly put in colostomy. B. A dusky and bluish stoma
22. Michiel shows the BEST adaptation with the C. A narrow and flattened stoma
new colostomy if he shows which of the following? D. Protruding stoma with swollen appearance

A. Look at the ostomy site 29. Michiel asked the nurse, what foods will help
B. Participate with the nurse in his daily lessen the odor of his colostomy. The nurse best
ostomy care response would be
C. Ask for leaflets and contact numbers of ostomy
support groups A. Eat eggs
D. Talk about his ostomy openly to the nurse and B. Eat cucumbers
friends C. Eat beet greens and parsley
D. Eat broccoli and spinach
23. The nurse plans to teach Michiel about
colostomy irrigation. As the nurse prepares the 30. The nurse will start to teach Michiel about the
materials needed, which of the following item techniques for colostomy irrigation. Which of the
indicates that the nurse needs further instruction? following should be included in the nurse’s teaching
plan?
A. Plain NSS / Normal Saline
B. K-Y Jelly A. Use 500 ml to 1,000 ml NSS
C. Tap water B. Suspend the irrigant 45 cm above the stoma
D. Irrigation sleeve C. Insert the cone 4 cm in the stoma
24. The nurse should insert the colostomy tube for D. If cramping occurs, slow the irrigation
irrigation at approximately
31. The nurse knew that the normal color of
A. 1-2 inches Michiel’s stoma should be
B. 3-4 inches
C. 6-8 inches A. Brick Red
D. 12-18 inches B. Gray
C. Blue C. James’ neck veins are not engorged
D. Pale Pink D. Wilma measures the tie from the nose to the tip
SITUATION: James, A 27 basketball player of the earlobe and to the xiphoid process.
sustained inhalation burn that required him to have 38. Wilma knew that James have an adequate
tracheostomy due to massive upper airway edema. respiratory condition if she notices that
32. Wilma, His sister and a nurse is suctioning the
tracheostomy tube of James. Which of the following, A. James’ respiratory rate is 18
if made by Wilma indicates that she is committing B. James’ Oxygen saturation is 91%
an error? C. There are frank blood suction from the tube
D. There are moderate amount of tracheobronchial
A. Hyperventilating James with 100% oxygen secretions
before and after suctioning
B. Instilling 3 to 5 ml normal saline to loosen up 39. Wilma knew that the maximum time when
secretion suctioning James is
C. Applying suction during catheter withdrawal
D. Suction the client every hour A. 10 seconds
B. 20 seconds
33. What size of suction catheter would Wilma use C. 30 seconds
for James, who is 6 feet 5 inches in height and D. 45 seconds
weighing approximately 145 lbs?
SITUATION : Juan Miguel Lopez Zobel Ayala de
A. Fr. 5 Batumbakal was diagnosed with Acute Close Angle
B. Fr. 10 Glaucoma. He is being seen by Nurse Jet.
C. Fr. 12 40. What specific manifestation would nurse Jet see
D. Fr. 18 in Acute close angle glaucoma that she would not
see in an open angle glaucoma?
34. Wilma is using a portable suction unit at home,
What is the amount of suction required by James A. Loss of peripheral vision
using this unit? B. Irreversible vision loss
C. There is an increase in IOP
A. 2-5 mmHg D. Pain
B. 5-10 mmHg
C. 10-15 mmHg 41. Nurse jet knew that Acute close angle glaucoma
D. 20-25 mmHg is caused by

35. If a Wall unit is used, What should be the A. Sudden blockage of the anterior angle by the
suctioning pressure required by James? base of the iris
B. Obstruction in trabecular meshwork
A. 50-95 mmHg C. Gradual increase of IOP
B. 95-110 mmHg D. An abrupt rise in IOP from 8 to 15 mmHg
C. 100-120 mmHg
D. 155-175 mmHg 42. Nurse jet performed a TONOMETRY test to Mr.
36. Wilma was shocked to see that the Batumbakal. What does this test measures
Tracheostomy was dislodged. Both the inner and
outer cannulas was removed and left hanging on A. It measures the peripheral vision remaining on
James’ neck. What are the 2 equipment’s at james’ the client
bedside that could help Wilma deal with this B. Measures the Intra Ocular Pressure
situation? C. Measures the Client’s Visual Acuity
D. Determines the Tone of the eye in response to
A. New set of tracheostomy tubes and Oxygen the sudden increase in IOP.
tank
B. Theophylline and Epinephrine 43. The Nurse notices that Mr. Batumbakal cannot
C. Obturator and Kelly clamp anymore determine RED from BLUE. The nurse
D. Sterile saline dressing knew that which part of the eye is affected by this
37. Which of the following method if used by change?
Wilma will best assure that the tracheostomy ties
are not too tightly placed? A. IRIS
B. PUPIL
A. Wilma places 2 fingers between the tie and C. RODS [RETINA]
neck D. CONES [RETINA]
B. The tracheotomy can be pulled slightly away
from the neck
44. Nurse Jet knows that Aqueous Humor is A. Atropine Sulfate
produce where? B. Pindolol [Visken]
C. Naloxone Hydrochloride [Narcan]
A. In the sub arachnoid space of the meninges D. Mesoridazine Besylate [Serentil]
B. In the Lateral ventricles
C. In the Choroids SITUATION : Wide knowledge about the human
D. In the Ciliary Body ear, it’s parts and it’s functions will help a nurse
assess and analyze changes in the adult client’s
45. Nurse Jet knows that the normal IOP is health.

A. 8-21 mmHg 52. Nurse Anna is doing a caloric testing to his


B. 2-7 mmHg patient, Aida, a 55 year old university professor
C. 31-35 mmHg who recently went into coma after being mauled by
D. 15-30 mmHg her disgruntled 3rd year nursing students whom she
gave a failing mark. After instilling a warm water in
46. Nurse Jet wants to measure Mr. Batumbakal’s the ear, Anna noticed a rotary nystagmus towards
CN II Function. What test would Nurse Jet the irrigated ear. What does this means?
implement to measure CN II’s Acuity? A. Indicates a CN VIII Dysfunction
B. Abnormal
A. Slit lamp C. Normal
B. Snellen’s Chart D. Inconclusive
C. Wood’s light
D. Gonioscopy 53. Ear drops are prescribed to an infant, The most
appropriate method to administer the ear drops is
47. The Doctor orders pilocarpine. Nurse jet knows
that the action of this drug is to A. Pull the pinna up and back and direct the
solution towards the eardrum
A. Contract the Ciliary muscle B. Pull the pinna down and back and direct the
B. Relax the Ciliary muscle solution onto the wall of the canal
C. Dilate the pupils C. Pull the pinna down and back and direct the
D. Decrease production of Aqueous Humor solution towards the eardrum
D. Pull the pinna up and back and direct the
48. The doctor orders timolol [timoptic]. Nurse jet solution onto the wall of the canal
knows that the action of this drug is
54. Nurse Jenny is developing a plan of care for a
A. Reduce production of CSF patient with Menieres disease. What is the priority
B. Reduce production of Aquesous Humor nursing intervention in the plan of care for this
C. Constrict the pupil particular patient?
D. Relaxes the Ciliary muscle
A. Air, Breathing, Circulation
49. When caring for Mr. Batumbakal, Jet teaches B. Love and Belongingness
the client to avoid C. Food, Diet and Nutrition
D. Safety
A. Watching large screen TVs
B. Bending at the waist 55. After mastoidectomy, Nurse John should be
C. Reading books aware that the cranial nerve that is usually damage
D. Going out in the sun after this procedure is

50. Mr. Batumbakal has undergone eye angiography A. CN I


using an Intravenous dye and fluoroscopy. What B. CN II
activity is contraindicated immediately after C. CN VII
procedure? D. CN VI

A. Reading newsprint 56. The physician orders the following for the client
B. Lying down with Menieres disease. Which of the following
C. Watching TV should the nurse question?
D. Listening to the music
A. Dipenhydramine [Benadryl]
51. If Mr. Batumbakal is receiving pilocarpine, B. Atropine sulfate
what drug should always be available in any case C. Out of bed activities and ambulation
systemic toxicity occurs? D. Diazepam [Valium]
57. Nurse Anna is giving dietary instruction to a D. Body Image disturbance R/T the eye packing
client with Menieres disease. Which statement if after surgery
made by the client indicates that the teaching has 62. Nurse Joseph is performing a WEBERS TEST.
been successful? He placed the tuning fork in the patients forehead
after tapping it onto his knee. The client states that
A. I will try to eat foods that are low in sodium the fork is louder in the LEFT EAR. Which of the
and limit my fluid intake following is a correct conclusion for nurse Josph to
B. I must drink atleast 3,000 ml of fluids per day make?
C. I will try to follow a 50% carbohydrate, 30%
fat and 20% protein diet A. He might have a sensory hearing loss in the left
D. I will not eat turnips, red meat and raddish ear
B. Conductive hearing loss is possible in the right
58. Peachy was rushed by his father, Steven into the ear
hospital admission. Peachy is complaining of C. He might have a sensory hearing loss in the
something buzzing into her ears. Nurse Joemar right hear, and/or a conductive hearing loss
assessed peachy and found out It was an insect. in the left ear.
What should be the first thing that Nurse Joemar D. He might have a conductive hearing loss in the
should try to remove the insect out from peachy’s right ear, and/or a sensory hearing loss in the
ear? left ear.

A. Use a flashlight to coax the insect out of 63. Aling myrna has Menieres disease. What typical
peachy’s ear dietary prescription would nurse Oca expect the
B. Instill an antibiotic ear drops doctor to prescribe?
C. Irrigate the ear
D. Pick out the insect using a sterile clean forceps A. A low sodium , high fluid intake
B. A high calorie, high protein dietary intake
59. Following an ear surgery, which statement if C. low fat, low sodium and high calorie intake
heard by Nurse Oca from the patient indicates a D. low sodium and restricted fluid intake
correct understanding of the post operative
instructions? SITUATION : [ From DEC 1991 NLE ] A 45 year
old male construction worker was admitted to a
A. Activities are resumed within 5 days tertiary hospital for incessant vomiting. Assessment
B. I will make sure that I will clean my hair and disclosed: weak rapid pulse, acute weight loss
face to prevent infection of .5kg, furrows in his tongue, slow flattening of the
C. I will use straw for drinking skin was noted when the nurse released her pinch.
D. I should avoid air travel for a while Temperature: 35.8 C , BUN Creatinine ratio : 10 :
1, He also complains for postural hypotension.
60. Nurse Oca will do a caloric testing to a client There was no infection.
who sustained a blunt injury in the head. He
instilled a cold water in the client’s right ear and he 64. Which of the following is the appropriate
noticed that nystagmus occurred towards the left ear. nursing diagnosis?
What does this finding indicates? A. Fluid volume deficit R/T furrow tongue
B. Fluid volume deficit R/T uncontrolled
A. Indicating a Cranial Nerve VIII Dysfunction vomiting
B. The test should be repeated again because the C. Dehydration R/T subnormal body temperature
result is vague D. Dehydration R/T incessant vomiting
C. This is Grossly abnormal and should be
reported to the neurosurgeon 65. Approximately how much fluid is lost in acute
D. This indicates an intact and working weight loss of .5kg?
vestibular branch of CN VIII
A. 50 ml
61. A client with Cataract is about to undergo B. 750 ml
surgery. Nurse Oca is preparing plan of care. Which C. 500 ml
of the following nursing diagnosis is most D. 75 ml
appropriate to address the long term need of this
type of patient? 66. Postural Hypotension is

A. Anxiety R/T to the operation and its outcome A. A drop in systolic pressure less than 10 mmHg
B. Sensory perceptual alteration R/T Lens when patient changes position from lying to
extraction and replacement sitting.
C. Knowledge deficit R/T the pre operative and
post operative self care
B. A drop in systolic pressure greater than 10 A. Vitamin C rich food
mmHg when patient changes position from B. Vitamin E rich food
lying to sitting C. Thiamine rich food
C. A drop in diastolic pressure less than 10 mmHg D. Vitamin B6 rich food
when patient changes position from lying to
sitting 73. One day, the patient complained of difficulty in
D. A drop in diastolic pressure greater than 10 walking. Your response would be
mmHg when patient changes position from
lying to sitting A. You will need a cane for support
B. Walk erect with eyes on horizon
67. Which of the following measures will not help C. I’ll get you a wheelchair
correct the patient’s condition D. Don’t force yourself to walk

A. Offer large amount of oral fluid intake to SITUATION: Mr. Dela Isla, a client with early
replace fluid lost Dementia exhibits thought process disturbances.
B. Give enteral or parenteral fluid 74. The nurse will assess a loss of ability in which
C. Frequent oral care of the following areas?
D. Give small volumes of fluid at frequent interval
A. Balance
68. After nursing intervention, you will expect the B. Judgment
patient to have C. Speech
D. Endurance
1. Maintain body temperature at 36.5 C
2. Exhibit return of BP and Pulse to normal 75. Mr. Dela Isla said he cannot comprehend what
3. Manifest normal skin turgor of skin and tongue the nurse was saying. He suffers from:
4. Drinks fluids as prescribed
A. 1,3 A. Insomnia
B. 2,4 B. Aphraxia
C. 1,3,4 C. Agnosia
D. 2,3,4 D. Aphasia

SITUATION: A 65 year old woman was admitted 76. The nurse is aware that in communicating with
for Parkinson’s Disease. The charge nurse is going an elderly client, the nurse will
to make an initial assessment.
69. Which of the following is a characteristic of a A. Lean and shout at the ear of the client
patient with advanced Parkinson’s disease? B. Open mouth wide while talking to the client
C. Use a low-pitched voice
A. Disturbed vision D. Use a medium-pitched voice
B. Forgetfulness
C. Mask like facial expression 77. As the nurse talks to the daughter of Mr. Dela
D. Muscle atrophy Isla, which of the following statement of the
daughter will require the nurse to give further
70. The onset of Parkinson’s disease is between 50- teaching?
60 years old. This disorder is caused by
A. I know the hallucinations are parts of the
A. Injurious chemical substances disease
B. Hereditary factors B. I told her she is wrong and I explained to
C. Death of brain cells due to old age her what is right
D. Impairment of dopamine producing cells in C. I help her do some tasks he cannot do for
the brain himself
D. Ill turn off the TV when we go to another room
71. The patient was prescribed with levodopa. What
is the action of this drug? 78. Which of the following is most important
discharge teaching for Mr. Dela Isla
A. Increase dopamine availability
B. Activates dopaminergic receptors in the basal A. Emergency Numbers
ganglia B. Drug Compliance
C. Decrease acetylcholine availability C. Relaxation technique
D. Release dopamine and other catecholamine D. Dietary prescription
from neurological storage sites
72. You are discussing with the dietician what food
to avoid with patients taking levodopa?
SITUATION : Knowledge of the drug C. Coherence and sense of hearing
PROPANTHELINE BROMIDE [Probanthine] Is D. Patency of airway and adequacy of
necessary in treatment of various disorders. respiration
79. What is the action of this drug?
85. Considering Mr. Franco’s conditions, which of
A. Increases glandular secretion for clients the following is most important to include in
affected with cystic fibrosis preparing Franco’s bedside equipment?
B. Dissolve blockage of the urinary tract due to
obstruction of cystine stones A. Hand bell and extra bed linen
C. Reduces secretion of the glandular organ of B. Sandbag and trochanter rolls
the body C. Footboard and splint
D. Stimulate peristalsis for treatment of D. Suction machine and gloves
constipation and obstruction
86. What is the rationale for giving Mr. Franco
80. What should the nurse caution the client when frequent mouth care?
using this medication
A. He will be thirsty considering that he is doesn’t
A. Avoid hazardous activities like driving, drink enough fluids
operating machineries etc. B. To remove dried blood when tongue is bitten
B. Take the drug on empty stomach during a seizure
C. Take with a full glass of water in treatment of C. The tactile stimulation during mouth care will
Ulcerative colitis hasten return to consciousness
D. I must take double dose if I missed the D. Mouth breathing is used by comatose
previous dose patient and it’ll cause oral mucosa dying
and cracking.
81. Which of the following drugs are not
compatible when taking Probanthine? 87. One of the complications of prolonged bed rest
is decubitus ulcer. Which of the following can best
A. Caffeine prevent its occurrence?
B. NSAID
C. Acetaminophen A. Massage reddened areas with lotion or oils
D. Alcohol B. Turn frequently every 2 hours
C. Use special water mattress
82. What should the nurse tell clients when taking D. Keep skin clean and dry
Probanthine?
88. If Mr. Franco’s Right side is weak, What should
A. Avoid hot weathers to prevent heat strokes be the most accurate analysis by the nurse?
B. Never swim on a chlorinated pool
C. Make sure you limit your fluid intake to 1L a A. Expressive aphasia is prominent on clients
day with right sided weakness
D. Avoid cold weathers to prevent hypothermia B. The affected lobe in the patient is the Right
lobe
83. Which of the following disease would C. The client will have problems in judging
Probanthine exert the much needed action for distance and proprioception
control or treatment of the disorder? D. Clients orientation to time and space will be
much affected
A. Urinary retention
B. Peptic Ulcer Disease SITUATION : a 20 year old college student was
C. Ulcerative Colitis rushed to the ER of PGH after he fainted during
D. Glaucoma their ROTC drill. Complained of severe right iliac
pain. Upon palpation of his abdomen, Ernie jerks
SITUATION : Mr. Franco, 70 years old, suddenly even on slight pressure. Blood test was ordered.
could not lift his spoons nor speak at breakfast. He Diagnosis is acute appendicitis.
was rushed to the hospital unconscious. His
diagnosis was CVA. 89. Which result of the lab test will be significant to
84. Which of the following is the most important the diagnosis?
assessment during the acute stage of an unconscious A. RBC : 4.5 TO 5 Million / cu. mm.
patient like Mr. Franco? B. Hgb : 13 to 14 gm/dl.
C. Platelets : 250,000 to 500,000 cu.mm.
A. Level of awareness and response to pain D. WBC : 12,000 to 13,000/cu.mm
B. Papillary reflexes and response to sensory
stimuli
90. Stat appendectomy was indicated. Pre op care A. When patient requests for it
would include all of the following except? B. Abdomen is soft and patient asks for water
C. Abdomen is soft and flatus has been expelled
A. Consent signed by the father D. B and C only
B. Enema STAT
C. Skin prep of the area including the pubis Situation: Amanda is suffering from chronic
D. Remove the jewelries arteriosclerosis Brain syndrome she fell while
getting out of the bed one morning and was brought
91. Pre-anesthetic med of Demerol and atrophine to the hospital, and she was diagnosed to have
sulfate were ordered to : cerebrovascular thrombosis thus transferred to a
nursing home.
A. Allay anxiety and apprehension 98. What do you call a STROKE that manifests a
B. Reduce pain bizarre behavior?
C. Prevent vomiting
D. Relax abdominal muscle A. Inorganic Stroke
92. Common anesthesia for appendectomy is B. Inorganic Psychoses
C. Organic Stroke
A. Spinal D. Organic Psychoses
B. General
C. Caudal 99. The main difference between chronic and
D. Hypnosis organic brain syndrome is that the former

93. Post op care for appendectomy include the A. Occurs suddenly and reversible
following except B. Is progressive and reversible
C. tends to be progressive and irreversible
A. Early ambulation D. Occurs suddenly and irreversible
B. Diet as tolerated after fully conscious 100. Which behavior results from organic
C. Nasogastric tube connect to suction psychoses?
D. Deep breathing and leg exercise
A. Memory deficit
94. Peritonitis may occur in ruptured appendix and B. Disorientation
may cause serious problems which are C. Impaired Judgement
D. Inappropriate affect
1. Hypovolemia, electrolyte imbalance
2. Elevated temperature, weakness and
diaphoresis
3. Nausea and vomiting, rigidity of the abdominal
wall
4. Pallor and eventually shock
A. 1 and 2
B. 2 and 3
C. 1,2,3
D. All of the above

95. If after surgery the patient’s abdomen becomes


distended and no bowel sounds appreciated, what
would be the most suspected complication?

A. Intussusception
B. Paralytic Ileus
C. Hemorrhage
D. Ruptured colon

96. NGT was connected to suction. In caring for the


patient with NGT, the nurse must

A. Irrigate the tube with saline as ordered


B. Use sterile technique in irrigating the tube
C. advance the tube every hour to avoid kinks
D. Offer some ice chips to wet lips

97. When do you think the NGT tube be removed?


colored stools are seen with biliary disease
PNLE Medical when bile flow is blocked. Celiac disease
doesn’t cause a widened pulse pressure.

Surgical Nursing 4. A client is hospitalized with a diagnosis of


chronic glomerulonephritis. The client mentions
that she likes salty foods. The nurse should warn her
Exam 2 to avoid foods containing sodium because:

1. After a cerebrovascular accident, a 75 yr old A. reducing sodium promotes urea nitrogen


client is admitted to the health care facility. The excretion
client has left-sided weakness and an absent gag B. reducing sodium improves her glomerular
reflex. He’s incontinent and has a tarry stool. His filtration rate
blood pressure is 90/50 mm Hg, and his hemoglobin C. reducing sodium increases potassium
is 10 g/dl. Which of the following is a priority for absorption
this client? D. reducing sodium decreases edema

A. checking stools for occult blood ANS: D


B. performing range-of-motion exercises to the Reducing sodium intake reduces fluid retention.
left side Fluid retention increases blood volume, which
C. keeping skin clean and dry changes blood vessel permeability and allows
D. elevating the head of the bed to 30 degrees plasma to move into interstitial tissue, causing
ANS: D edema. Urea nitrogen excretion can be increased
Because the client’s gag reflex is absent, only by improved renal function. Sodium intake
elevating the head of the bed to 30 degrees doesn’t affect the glomerular filtration rate.
helps minimize the client’s risk of aspiration. Potassium absorption is improved only by
Checking the stools, performing ROM increasing the glomerular filtration rate; it isn’t
exercises, and keeping the skin clean and dry affected by sodium intake.
are important, but preventing aspiration
through positioning is the priority 5. The nurse is caring for a client with a cerebral
injury that impaired his speech and hearing. Most
2. The nurse is caring for a client with a colostomy. likely, the client has experienced damage to the:
The client tells the nurse that he makes small pin
holes in the drainage bag to help relieve gas. The A. frontal lobe
nurse should teach him that this action: B. parietal lobe
C. occipital lobe
A. destroys the odor-proof seal D. temporal lobe
B. wont affect the colostomy system
C. is appropriate for relieving the gas in a ANS: D
colostomy system The portion of the cerebrum that controls speech
D. destroys the moisture barrier seal and hearing is the temporal lobe. Injury to the
frontal lobe causes personality changes, difficulty
ANS: A speaking, and disturbance in memory, reasoning,
Any hole, no matter how small, will destroy the and concentration. Injury to the parietal lobe
odor-proof seal of a drainage bag. Removing the causes sensory alterations and problems with
bag or unclamping it is the only appropriate spatial relationships. Damage to the occipital lobe
method for relieving gas. causes vision disturbances

3. When assessing the client with celiac disease, the 6. The nurse is assessing a postcraniotomy client
nurse can expect to find which of the following? and finds the urine output from a catheter is 1500
ml for the 1st hour and the same for the 2nd hour.
A. steatorrhea The nurse should suspect:
B. jaundiced sclerae
C. clay-colored stools A. Cushing’s syndrome
D. widened pulse pressure B. Diabetes mellitus
C. Adrenal crisis
ANS: A D. Diabetes insipidus
because celiac disease destroys the absorbing
surface of the intestine, fat isn’t absorbed but is ANS: D
passed in the stool. Steatorrhea is bulky, fatty Diabetes insipidus is an abrupt onset of
stools that have a foul odor. Jaundiced sclerae extreme polyuria that commonly occurs in
result from elevated bilirubin levels. Clay- clients after brain surgery. Cushing’s syndrome
is excessive glucocorticoid secretion resulting life threatening, eliminating ineffective
in sodium and water retention. Diabetes thermoregulation as the top priority.
mellitus is a hyperglycemic state marked by
polyuria, polydipsia, and polyphagia. Adrenal 9. Capillary glucose monitoring is being performed
crisis is undersecretion of glucocorticoids every 4 hours for a client diagnosed with diabetic
resulting in profound hypoglycemia, ketoacidosis. Insulin is administered using a scale of
hypovolemia, and hypotension regular insulin according to glucose results. At 2
p.m., the client has a capillary glucose level of 250
7. The nurse is providing postprocedure care for a mg/dl for which he receives 8 U of regular insulin.
client who underwent percutaneous lithotripsy. In The
this procedure, an ultrasonic probe inserted through nurse should expect the dose’s:
a nephrostomy tube into the renal pelvis generates
ultra-high-frequency sound waves to shatter renal A. onset to be at 2 p.m. and its peak at 3 p.m.
calculi. The nurse should instruct the client to: B. onset to be at 2:15 p.m. and its peak at 3 p.m.
C. onset to be at 2:30 p.m. and its peak at 4 p.m.
A. limit oral fluid intake for 1 to 2 weeks D. onset to be at 4 p.m. and its peak at 6 p.m.
B. report the presence of fine, sandlike particles
through the nephrostomy tube. ANS: C
C. Notify the physician about cloudy or foul Regular insulin, which is a short-acting insulin, has
smelling urine an onset of 15 to 30 minutes and a peak of 2 to 4
D. Report bright pink urine within 24 hours after hours. Because the nurse gave the insulin at 2 p.m.,
the procedure the expected onset would be from 2:15 to 2:30 p.m.
and the peak from 4 p.m. to 6 p.m.
ANS: C
The client should report the presence of 10. A client with a head injury is being monitored
foulsmelling or cloudy urine. Unless for increased intracranial pressure (ICP). His blood
contraindicated, the client should be instructed to pressure is 90/60 mmHG and the ICP is 18 mmHg;
drink large quantities of fluid each day to flush the therefore his cerebral perfusion pressure (CPP) is:
kidneys. Sand-like debris is normal because of
residual stone products. Hematuria is common after A. 52 mm Hg
lithotripsy. B. 88 mm Hg
C. 48 mm Hg
8. A client with a serum glucose level of 618 mg/dl D. 68 mm Hg
is admitted to the facility. He’s awake and oriented,
has hot dry skin, and has the following vital signs: ANS: A
temperature of 100.6º F (38.1º C), heart rate of 116 CPP is derived by subtracting the ICP from the
beats/minute, and blood pressure of 108/70 mm Hg. mean arterial pressure (MAP). For adequate
Based on these assessment findings, which nursing cerebral perfusion to take place, the minimum goal
diagnosis takes the highest priority? is 70 mmHg. The MAP is derived using the
following formula:
A. deficient fluid volume related to osmotic MAP = ((diastolic blood pressure x 2) + systolic
diuresis blood pressure) / 3
B. decreased cardiac output related to elevated MAP = ((60 x2) + 90) / 3
heart rate MAP = 70 mmHg
C. imbalanced nutrition: Less than body To find the CPP, subtract the client’s ICP from the
requirements related to insulin deficiency MAP; in this case , 70 mmHg – 18 mmHg = 52
D. ineffective thermoregulation related to mmHg.
dehydration
11. A 52 yr-old female tells the nurse that she has
ANS: A found a painless lump in her right breast during her
A serum glucose level of 618 mg/dl indicates monthly self-examination. Which assessment
hyperglycemia, which causes polyuria and deficient finding would strongly suggest that this client’s
fluid volume. In this client, tachycardia is more lump is cancerous?
likely to result from deficient fluid volume than
from decreased cardiac output because his blood A. eversion of the right nipple and a mobile mass
pressure is normal. Although the client’s serum B. nonmobile mass with irregular edges
glucose is elevated, food isn’t a priority because C. mobile mass that is oft and easily delineated
fluids and insulin should be administered to lower D. nonpalpable right axillary lymph nodes
the serum glucose level. Therefore, a diagnosis of
Imbalanced Nutrition: Less then body requirements ANS: B
isn’t appropriate. A temperature of 100.6º F isn’t Breast cancer tumors are fixed, hard, and poorly
delineated with irregular edges. Nipple retraction —
not eversion—may be a sign of cancer. A mobile B. baseline arterial blood gas (ABG) levels
mass that is soft and easily delineated is most often C. prior outcomes of weaning
a fluid-filled benigned cyst. Axillary lymph nodes D. electrocardiogram (ECG) results
may or may not be palpable on initial detection of a
cancerous mass. ANS: B
Before weaning a client from mechanical
ventilation, it’s most important to have a baseline
12. A Client is scheduled to have a descending ABG levels. During the weaning process, ABG
colostomy. He’s very anxious and has many levels will be checked to assess how the client is
questions regarding the surgical procedure, care of tolerating the procedure. Other assessment
stoma, and lifestyle changes. It would be most parameters are less critical. Measuring fluid volume
appropriate for the nurse to make a referral to which intake and output is always important when a client
member of the health care team? is being mechanically ventilated. Prior attempts at
weaning and ECG results are documented on the
A. Social worker client’s record, and the nurse can refer to them
B. registered dietician before the weaning process begins.
C. occupational therapist
D. enterostomal nurse therapist
16. The nurse is speaking to a group of women
ANS: D about early detection of breast cancer. The average
An enterostomal nurse therapist is a registered nurse age of the women in the group is 47. Following the
who has received advance education in an American Cancer Society (ACS) guidelines, the
accredited program to care for clients with stomas. nurse should recommend that the women:
The enterostomal nurse therapist can assist with
selection of an appropriate stoma site, teach about A. perform breast self-examination annually
stoma care, and provide emotional support. B. have a mammogram annually
C. have a hormonal receptor assay annually
13. Ottorrhea and rhinorrhea are most commonly D. have a physician conduct a clinical evaluation
seen with which type of skull fracture? every 2 years

A. basilar ANS: B
B. temporal According to the ACS guidelines, “Women older
C. occipital than age 40 should perform breast selfexamination
D. parietal monthly (not annually).” The hormonal receptor
assay is done on a known breast tumor to determine
ANS: A whether the tumor is estrogen- or progesterone-
Ottorrhea and rhinorrhea are classic signs of dependent.
basilar skull fracture. Injury to the dura commonly
occurs with this fracture, resulting in cerebrospinal 17. When caring for a client with esophageal
fluid (CSF) leaking through the ears and nose. Any varices, the nurse knows that bleeding in this
fluid suspected of being CSF should be checked disorder usually stems from:
for glucose or have a halo test done
A. esophageal perforation
14. A male client should be taught about testicular B. pulmonary hypertension
examinations: C. portal hypertension
D. peptic ulcers
A. when sexual activity starts
B. after age 60 ANS: C
C. after age 40 Increased pressure within the portal veins causes
D. before age 20 them to bulge, leading to rupture and bleeding into
the lower esophagus. Bleeding associated with
ANS: D esophageal varices doesn’t stem from esophageal
Testicular cancer commonly occurs in men between perforation, pulmonary hypertension, or peptic
ages 20 and 30. A male client should be taught how ulcers
to perform testicular self-examination before age 20,
preferably when he enters his teens. 18. A 49-yer-old client was admitted for surgical
repair of a Colles’ fracture. An external fixator was
15. Before weaning a client from a ventilator, which placed during surgery. The surgeon explains that
assessment parameter is most important for the this method of repair:
nurse to review?
A. has very low complication rate
A. fluid intake for the last 24 hours
B. maintains reduction and overall hand B. Peripheral edema.
function C. A dry cough.
C. is less bothersome than a cast D. Postural hypotension.
D. is best for older people 23. Which assessment finding indicates dehydration?

ANS: B A. Tenting of chest skin when pinched.


Complex intra-articular fractures are repaired with B. Rapid filling of hand veins.
external fixators because they have a better long- C. A pulse that isn’t easily obliterated.
term outcome than those treated with casting. This D. Neck vein distention
is especially true in a young client. The incidence of 24. The nurse is teaching a client with a history of
complications, such as pin tract infections and atherosclerosis. To decrease the risk of
neuritis, is 20% to 60%. Clients must be taught how atherosclerosis, the nurse should encourage the
to do pin care and assess for development of client to:
neurovascular complications.
A. Avoid focusing on his weight.
19. A client is hospitalized with a diagnosis of B. Increase his activity level.
chronic renal failure. An arteriovenous fistula was C. Follow a regular diet.
created in his left arm for hemodialysis. When D. Continue leading a high-stress lifestyle.
preparing the client for discharge, the nurse should 25. For a client newly diagnosed with
reinforce which dietary instruction? radiationinduced thrombocytopenia, the nurse
should include which intervention in the plan of
A. “Be sure to eat meat at every meal.” care?
B. “Monitor your fruit intake and eat plenty of
bananas.” A. Administer aspirin if the temperature exceeds
C. “Restrict your salt intake.” 38.8º C.
D. “Drink plenty of fluids.” B. Inspect the skin for petechiae once every shift.
20. The nurse is caring for a client who has just had C. Provide for frequent periods of rest.
a modified radical mastectomy with immediate D. Place the client in strict isolation.
reconstruction. She’s in her 30s and has tow 26. A client is chronically short of breath and yet
children. Although she’s worried about her future, has normal lung ventilation, clear lungs, and an
she seems to be adjusting well to her diagnosis. arterial oxygen saturation (SaO2) 96% or better.
What should the nurse do to support The client most likely has:
her coping?
A. poor peripheral perfusion
A. Tell the client’s spouse or partner to be B. a possible Hematologic problem
supportive while she recovers. C. a psychosomatic disorder
B. Encourage the client to proceed with the next D. left-sided heart failure
phase of treatment. 27. For a client in addisonian crisis, it would be
C. Recommend that the client remain cheerful for very risky for a nurse to administer:
the sake of her children.
D. Refer the client to the American Cancer A. potassium chloride
Society’s Reach for Recovery program or B. normal saline solution
another support program. C. hydrocortisone
21. A 21 year-old male has been seen in the clinic D. fludrocortisone
for a thickening in his right testicle. The physician 28. The nurse is reviewing the laboratory report of a
ordered a human chorionic gonadotropin (HCG) client who underwent a bone marrow biopsy. The
level. The nurse’s explanation to the client should finding that would most strongly support a
include the fact that: diagnosis of acute leukemia is the existence of a
large number of immature:
A. The test will evaluate prostatic function.
B. The test was ordered to identify the site of a A. lymphocytes
possible infection. B. thrombocytes
C. The test was ordered because clients who have C. reticulocytes
testicular cancer has elevated levels of HCG. D. leukocytes
D. The test was ordered to evaluate the 29. The nurse is performing wound care on a foot
testosterone level. ulcer in a client with type 1 diabetes mellitus.
22. A client is receiving captopril (Capoten) for Which technique demonstrates surgical asepsis?
heart failure. The nurse should notify the physician
that the medication therapy is ineffective if an A. Putting on sterile gloves then opening a
assessment reveals: container of sterile saline.
B. Cleaning the wound with a circular motion,
A. A skin rash. moving from outer circles toward the center.
C. Changing the sterile field after sterile water is B. Weight the client daily.
spilled on it. C. Ask the client to list what she eats during a
D. Placing a sterile dressing ½” (1.3 cm) from the typical day.
edge of the sterile field. D. Place the client on I & O status and draw blood
30. A client with a forceful, pounding heartbeat is for electrolyte levels.
diagnosed with mitral valve prolapse. This client 35. When instructions should be included in the
should avoid which of the following? discharge teaching plan for a client after
thyroidectomy for Grave’s disease?
A. high volumes of fluid intake
B. aerobic exercise programs A. Keep an accurate record of intake and output.
C. caffeine-containing products B. Use nasal desmopressin acetate DDAVP).
D. foods rich in protein C. Be sure to get regulate follow-up care.
31. A client with a history of hypertension is D. Be sure to exercise to improve cardiovascular
diagnosed with primary hyperaldosteronism. This fitness.
diagnosis indicates that the client’s hypertension is 36. A client comes to the emergency department
caused by excessive hormone secretion from which with chest pain, dyspnea, and an irregular heartbeat.
organ? An electrocardiogram shows a heart rate of 110
beats/minute (sinus tachycardia) with frequent
A. adrenal cortex premature ventricular contractions. Shortly after
B. pancreas admission, the client has ventricular tachycardia and
C. adrenal medulla becomes unresponsive. After successful
D. parathyroid resuscitation, the client is taken to the intensive care
32. A client has a medical history of rheumatic unit. Which nursing diagnosis is appropriate at this
fever, type 1 (insulin dependent) diabetes mellitus, time?
hypertension, pernicious anemia, and appendectomy.
She’s admitted to the hospital and undergoes mitral A. Deficient knowledge related to interventions
valve replacement surgery. After discharge, the used to treat acute illness
client is scheduled for a tooth extraction. Which B. Impaired physical mobility related to complete
history finding is a major risk factor for infective bed rest
endocarditis? C. Social isolation related to restricted visiting
hours in the intensive care unit
A. appendectomy D. Anxiety related to the threat of death
B. pernicious anemia 37. A client is admitted to the health care facility
C. diabetes mellitus with active tuberculosis. The nurse should include
D. valve replacement which intervention in the plan of care?
33. A 62 yr-old client diagnosed with pyelonephritis
and possible septicemia has had five urinary tract A. Putting on a mask when entering the client’s
infections over the past two years. She’s fatigued room.
from lack of sleep; urinates frequently, even during B. Instructing the client to wear a mask at all
the night; and has lost weight recently. Test reveal times
the following: sodium level 152 mEq/L, osmolarity C. Wearing a gown and gloves when providing
340 mOsm/L, glucose level 125 mg/dl, and direct care
potassium level 3.8 mEq/L. which of the following D. Keeping the door to the client’s room open to
nursing diagnoses is most appropriate for this client? observe the client
38. The nurse is caring for a client who underwent a
A. Deficient fluid volume related to inability to subtotal gastrectomy 24 hours earlier. The client has
conserve water a nasogastric (NG) tube. The nurse should:
B. Imbalanced nutrition: less than body
requirements related to hypermetabolic state A. Apply suction to the NG tube every hour.
C. Deficient fluid volume related to osmotic B. Clamp the NG tube if the client complains of
diuresis induced by hypernatremia nausea.
D. Imbalanced nutrition: less than body C. Irrigate the NG tube gently with normal saline
requirements related to catabolic effects of solution.
insulin deficiency D. Reposition the NG tube if pulled out.
34. A 20 yr-old woman has just been diagnosed 39. Which statement about fluid replacement is
with Crohn’s disease. She has lost 10 lb (4.5 kg) accurate for a client with hyperosmolar
and has cramps and occasional diarrhea. The nurse hyperglycemic nonketotic syndrome (HHNS)?
should include which of the following when doing a
nutritional assessment? A. administer 2 to 3 L of IV fluid rapidly
B. administer 6 L of IV fluid over the first 24
A. Let the client eat as desired during the hours
hospitalization.
C. administer a dextrose solution containing C. high pressure and continuous
normal saline solution D. high pressure and intermittent
D. administer IV fluid slowly to prevent 46. Which nursing diagnosis is most appropriate for
circulatory overload and collapse an elderly client with osteoarthritis?
40. Which of the following is an adverse reaction to
glipizide (Glucotrol)? A. Risk for injury
B. Impaired urinary elimination
A. headache C. Ineffective breathing pattern
B. constipation D. Imbalanced nutrition: less than body
C. hypotension requirements
D. photosensitivity 47. Parathyroid hormone (PTH) has which effects
41. The nurse is caring for four clients on a on the kidney?
stepdown intensive care unit. The client at the
highest risk for developing nosocomial pneumonia A. Stimulation of calcium reabsorption and
is the one who: phosphate excretion
B. Stimulation of phosphate reabsorption and
A. has a respiratory infection calcium excretion
B. is intubated and on a ventilator C. Increased absorption of vit D and excretion of
C. has pleural chest tubes vit E
D. is receiving feedings through a jejunostomy D. Increased absorption of vit E and excretion of
tube Vit D
42. The nurse is teaching a client with chronic 48. A visiting nurse is performing home assessment
bronchitis about breathing exercises. Which of the for a 59-yr old man recently discharged after hip
following should the nurse include in the teaching? replacement surgery. Which home assessment
finding warrants health promotion teaching from the
A. Make inhalation longer than exhalation. nurse?
B. Exhale through an open mouth.
C. Use diaphragmatic breathing. A. A bathroom with grab bars for the tub and
D. Use chest breathing. toilet
43. A client is admitted to the hospital with an B. Items stored in the kitchen so that reaching up
exacerbation of her chronic systemic lupus and bending down aren’t necessary
erythematosus (SLE). She gets angry when her call C. Many small, unsecured area rugs
bell isn’t answered immediately. The most D. Sufficient stairwell lighting, with switches to
appropriate response to her would be: the top and bottom of the stairs
49. A client with autoimmune thrombocytopenia
A. “You seem angry. Would you like to talk about and a platelet count of 800/uL develops epistaxis
it?” and melena. Treatment with corticosteroids and
B. “Calm down. You know that stress will make immunoglobulins has been unsuccessful, and the
your symptoms worse.” physician recommends a splenectomy. The client
C. “Would you like to talk about the problem with states, “I don’t need surgery—this will go away on
the nursing supervisor?” its own.” In considering her response to the client,
D. “I can see you’re angry. I’ll come back when the nurse must depend on the ethical principle of:
you’ve calmed down.”
44. On a routine visit to the physician, a client with A. beneficence
chronic arterial occlusive disease reports stopping B. autonomy
smoking after 34 years. To relive symptoms of C. advocacy
intermittent claudication, a condition associated D. justice
with chronic arterial occlusive disease, the nurse 50. Which of the following is t he most critical
should recommend which additional measure? intervention needed for a client with myxedema
coma?
A. Taking daily walks.
B. Engaging in anaerobic exercise. A. Administering and oral dose of levothyroxine
C. Reducing daily fat intake to less than 45% of (Synthroid)
total calories B. Warming the client with a warming blanket
D. Avoiding foods that increase levels of C. Measuring and recording accurate intake and
highdensity lipoproteins (HDLs) output
45. A physician orders gastric decompression for a D. Maintaining a patent airway
client with small bowel obstruction. The nurse 51. Because diet and exercise have failed to control
should plan for the suction to be: a 63 yr-old client’s blood glucose level, the client is
prescribed glipizide (Glucotrol). After oral
A. low pressure and intermittent administration, the onset of action is:
B. low pressure and continuous
A. 15 to 30 minutes A. hypocalcemia
B. 30 to 60 minutes B. hypercalcemia
C. 1 to 1 ½ hours C. hypokalemia
D. 2 to 3 hours D. Hyperkalemia
52. A client with pneumonia is receiving 58. In a client with enteritis and frequent diarrhea,
supplemental oxygen, 2 L/min via nasal cannula. the nurse should anticipate an acidbase imbalance
The client’s history includes chronic obstructive of:
pulmonary disease (COPD) and coronary artery
disease. Because of these findings, the nurse closely A. respiratory acidosis
monitors the oxygen flow and the client’s B. respiratory alkalosis
respiratory status. Which complication may arise if C. metabolic acidosis
the client receives a high oxygen concentration? D. metabolic alkalosis
59. When caring for a client with the nursing
A. Apnea diagnosis Impaired swallowing related to
B. Anginal pain neuromuscular impairment, the nurse should:
C. Respiratory alkalosis
D. Metabolic acidosis A. position the client in a supine position
53. A client with type 1 diabetes mellitus has been B. elevate the head of the bed 90 degrees during
on a regimen of multiple daily injection therapy. meals
He’s being converted to continuous subcutaneous C. encourage the client to remove dentures
insulin therapy. While teaching the client bout D. encourage thin liquids for dietary intake
continuous subcutaneous insulin therapy, the nurse 60. A nurse is caring for a client who has a
would be accurate in telling him the regimen tracheostomy and temperature of 39º C. which
includes the use of: intervention will most likely lower the client’s
arterial blood oxygen saturation?
A. intermediate and long-acting insulins
B. short and long-acting insulins A. Endotracheal suctioning
C. short-acting only B. Encouragement of coughing
D. short and intermediate-acting insulins C. Use of cooling blanket
54. a client who recently had a cerebrovascular D. Incentive spirometry
accident requires a cane to ambulate. When 61. A client with a solar burn of the chest, back,
teaching about cane use, the rationale for holding a face, and arms is seen in urgent care. The nurse’s
cane on the uninvolved side is to: primary concern should be:

A. prevent leaning A. fluid resuscitation


B. distribute weight away from the involved side B. infection
C. maintain stride length C. body image
D. prevent edema D. pain management
55. A client with a history of an anterior wall 62. Which statement is true about crackles?
myocardial infarction is being transferred from the
coronary care unit (CCU) to the cardiac stepdown A. They’re grating sounds.
unit (CSU). While giving report to the CSU nurse, B. They’re high-pitched, musical squeaks.
the CCU nurse says, “His pulmonary artery wedge C. They’re low-pitched noises that sound like
pressures have been in the high normal range.” The snoring.
CSU nurse should be especially observant for: D. They may be fine, medium, or course.
63. A woman whose husband was recently
A. hypertension diagnosed with active pulmonary tuberculosis (TB)
B. high urine output is a tuberculin skin test converter. Management of
C. dry mucous membranes her care would include:
D. pulmonary crackles
56. The nurse is caring for a client with a fractures A. scheduling her for annual tuberculin skin
hip. The client is combative, confused, and trying to testing
get out of bed. The nurse should: B. placing her in quarantine until sputum cultures
are negative
A. leave the client and get help C. gathering a list of persons with whom she has
B. obtain a physician’s order to restrain the client had recent contact
C. read the facility’s policy on restraints D. advising her to begin prophylactic therapy with
D. order soft restraints from the storeroom isoniazid (INH)
57. For the first 72 hours after thyroidectomy 64. The nurse is caring for a client who ahs had an
surgery, the nurse would assess the client for above the knee amputation. The client refuses to
Chvostek’s sign and Trousseau’s sign because they look at the stump. When the nurse attempts to speak
indicate which of the following? with the client about his surgery, he tells the nurse
that he doesn’t wish to discuss it. The client also 70. When assessing a client with partial thickness
refuses to have his family visit. The nursing burns over 60% of the body, which of the following
diagnosis that best describes the client’s problem is: should the nurse report immediately?

A. Hopelessness A. Complaints of intense thirst


B. Powerlessness B. Moderate to severe pain
C. Disturbed body image C. Urine output of 70 ml the 1st hour
D. Fear D. Hoarseness of the voice
65. A client with three children who is still I the 71. A client is admitted to the hospital following a
child bearing years is admitted for surgical repair of burn injury to the left hand and arm. The client’s
a prolapsed bladder. The nurse would find that the burn is described as white and leathery with no
client understood the surgeon’s preoperative blisters. Which degree of severity is this burn?
teaching when the client states:
A. first-degree burn
A. “If I should become pregnant again, the child B. second-degree burn
would be delivered by cesarean delivery.” C. third-degree burn
B. “If I have another child, the procedure may D. fourth-degree burn
need to be repeated.” 72. The nurse is caring for client with a new donor
C. “This surgery may render me incapable of site that was harvested to treat a new burn. The
conceiving another child.” nurse position the client to:
D. “This procedure is accomplished in two
separate surgeries.” A. allow ventilation of the site
66. A client experiences problems in body B. make the site dependent
temperature regulation associated with a skin C. avoid pressure on the site
impairment. Which gland is most likely involved? D. keep the site fully covered
73. a 45-yr-old auto mechanic comes to the
A. Eccrine physician’s office because an exacerbation of his
B. Sebaceous psoriasis is making it difficult to work. He tells the
C. Apocrine nurse that his finger joints are stiff and sore in the
D. Endocrine morning. The nurse should respond by:
67. A school cafeteria worker comes to the
physician’s office complaining of severe scalp A. Inquiring further about this problem because
itching. On inspection, the nurse finds nail marks on psoriatic arthritis can accompany psoriasis
the scalp and small light-colored round specks vulgaris
attached to the hair shafts close to the scalp. These B. Suggesting he take aspirin for relief because
findings suggest that the client suffers from: it’s probably early rheumatoid arthritis
C. Validating his complaint but assuming it’s an
A. scabies adverse effect of his vocation
B. head lice D. Asking him if he has been diagnosed or treated
C. tinea capitis for carpal tunnel syndrome
D. impetigo 74. The nurse is providing home care instructions to
68. Following a small-bowel resection, a client a client who has recently had a skin graft. Which
develops fever and anemia. The surface surrounding instruction is most important for the client to
the surgical wound is warm to touch and necrotizing remember?
fasciitis is suspected. Another manifestation that
would most suggest necrotizing fasciitis is: A. Use cosmetic camouflage techniques.
B. Protect the graft from direct sunlight.
A. erythema C. Continue physical therapy.
B. leukocytosis D. Apply lubricating lotion to the graft site.
C. pressure-like pain 75. a 28 yr-old female nurse is seen in the employee
D. swelling health department for mild itching and rash of both
69. A 28 yr-old nurse has complaints of itching and hands. Which of the following could be causing this
a rash of both hands. Contact dermatitis is initially reaction?
suspected. The diagnosis is confirmed if the rash
appears: A. possible medication allergies
B. current life stressors she may be experiencing
A. erythematous with raised papules C. chemicals she may be using and use of latex
B. dry and scaly with flaking skin gloves
C. inflamed with weeping and crusting lesions D. recent changes made in laundry detergent or
D. excoriated with multiple fissures bath soap.
76. The nurse assesses a client with urticaria. The
nurse understands that urticaria is another name for:
A. hives A. “All family members will need to be treated.”
B. a toxin B. “If someone develops symptoms, tell him to
C. a tubercle see a physician right away.”
D. a virus C. “Just be careful not to share linens and towels
77. A client with psoriasis visits the dermatology with family members.”
clinic. When inspecting the affected areas, the nurse D. “After you’re treated, family members won’t
expects to see which type of secondary lesion? be at risk for contracting scabies.”
83. In an industrial accident, client who weighs 155
A. scale lb (70.3 kg) sustained full-thickness burns over 40%
B. crust of his body. He’s in the burn unit receiving fluid
C. ulcer resuscitation. Which observation shows that the
D. scar fluid resuscitation is benefiting the client?
78. The nurse is caring for a bedridden, elderly
adult. To prevent pressure ulcers, which A. A urine output consistently above 100 ml/hour.
intervention should the nurse include in the plan of B. A weight gain of 4 lb (1.8 kg) in 24 hours.
care? C. Body temperature readings all within normal
limits
A. Turn and reposition the client a minimum of D. An electrocardiogram (ECG) showing no
every 8 hours. arrhythmias.
B. Vigorously massage lotion into bony 84. The nurse is reviewing the laboratory results of
prominences. a client with rheumatoid arthritis. Which of the
C. Post a turning schedule at the client’s bedside. following laboratory results should the nurse expect
D. Slide the client, rather than lifting when turning. to find?
79. Following a full-thickeness (3rd degree) burn of
his left arm, a client is treated with artificial skin. A. Increased platelet count
The client understands postoperative care of the B. Elevated erythrocyte sedimentation rate (ESR)
artificial skin when he states that during the first 7 C. Electrolyte imbalance
days after the procedure, he’ll restrict: D. Altered blood urea nitrogen (BUN) and
creatinine levels
A. range of motion 85. Which nursing diagnosis takes the highest
B. protein intake priority for a client with Parkinson’s crisis?
C. going outdoors
D. fluid ingestion A. Imbalanced nutrition: less than body
80. A client received burns to his entire back and requirements
left arm. Using the Rule of Nines, the nurse can B. Ineffective airway clearance
calculate that he has sustained burns on what C. Impaired urinary elimination
percentage of his body? D. Risk for injury
86. A client with a spinal cord injury and
A. 9% subsequent urine retention receives intermittent
B. 18% catheterization every 4 hours. The average
C. 27% catheterized urine volume has been 550 ml. The
D. 36% nurse should plan to:
81. The nurse is providing care for a client who has
a sacral pressure ulcer with wet-to-dry dressing. A. Increase the frequency of the catheterizations.
Which guideline is appropriate for a wet-to-dry B. Insert an indwelling urinary catheter
dressing? C. Place the client on fluid restrictions
D. Use a condom catheter instead of an invasive
A. The wound should remain moist form the one.
dressing. 87.The nurse is caring for a client who is to undergo
B. The wet-to-dry dressing should be tightly a lumbar puncture to assess for the presence of
packed into the wound. blood in the cerebrospinal fluid (CSF) and to
C. The dressing should be allowed to dry out measure CSF pressure. Which result would indicate
before removal. n abnormality?
D. A plastic sheet-type dressing should cover the
wet dressing. A. The presence of glucose in the CSF.
82. While in skilled nursing facility, a client B. A pressure of 70 to 200 mm H2O
contracted scabies, which is diagnosed the day after C. The presence of red blood cells (RBCs) in the
discharge. The client is living at her daughter’s first specimen tube
home with six other persons. During her visit to the D. A pressure of 00 to 250 mmH2O
clinic, she asks a staff nurse, “What should my 88. The nurse is administering eyedrops to a client
family do?” the most accurate response from the with glaucoma. To achieve maximum absorption,
nurse is: the nurse should instill the eyedrop into the:
A. conjunctival sac 95. A 70 yr-old client with a diagnosis of leftsided
B. pupil cerebrovascular accident is admitted to the facility.
C. sclera To prevent the development of diffuse osteoporosis,
D. vitreous humor which of the following objectives is most
89. A 52 yr-old married man with two adolescent appropriate?
children is beginning rehabilitation following a
cerebrovascular accident. As the nurse is planning A. Maintaining protein levels.
the client’s care, the nurse should recognize that his B. Maintaining vitamin levels.
condition will affect: C. Promoting weight-bearing exercises
D. Promoting range-of-motion (ROM) exercises
A. only himself 96. A client is admitted with a diagnosis of
B. only his wife and children meningitis caused by Neisseria meningitides. The
C. him and his entire family nurse should institute which type of isolation
D. no one, if he has complete recovery precautions?
90. Which action should take the highest priority
when caring for a client with hemiparesis caused by A. Contact precautions
a cerebrovascular accident (CVA)? B. Droplet precautions
C. Airborne precautions
A. Perform passive range-of-motion (ROM) D. Standard precautions
exercises. 97. A young man was running along an ocean pier,
B. Place the client on the affected side. tripped on an elevated area of the decking, and
C. Use hand rolls or pillows for support. struck his head on the pier railing. According to his
D. Apply antiembolism stockings friends, “He was unconscious briefly and then
91. The nurse is formulating a teaching plan for a became alert and behaved as though nothing had
client who has just experienced a transient ischemic happened.” Shortly afterward, he began
attack (TIA). Which fact should the nurse include in complaining of a headache and asked to be taken to
the teaching plan? the emergency department. If the client’s
intracranial pressure (ICP) is increasing, the nurse
A. TIA symptoms may last 24 to 48 hours. would expect to observe which of the
B. Most clients have residual effects after having following signs first?
a TIA.
C. TIA may be a warning that the client may have A. pupillary asymmetry
cerebrovascular accident (CVA) B. irregular breathing pattern
D. The most common symptom of TIA is the C. involuntary posturing
inability to speak. D. declining level of consciousness
92. The nurse has just completed teaching about 98. Emergency medical technicians transport a 28
postoperative activity to a client who is going to yr-old iron worker to the emergency department.
have a cataract surgery. The nurse knows the They tell the nurse, “He fell from a two-story
teaching has been effective if the client: building. He has a large contusion on his left chest
and a hematoma in the left parietal area. He has
A. coughs and deep breathes postoperatively compound fracture of his left femur and he’s
B. ties his own shoes comatose. We intubated him and he’s maintaining
C. asks his wife to pick up his shirt from the floor an arterial oxygen saturation of 92% by pulse
after he drops it. oximeter with a manual-resuscitation bag.” Which
D. States that he doesn’t need to wear an eyepatch intervention by the nurse has the highest priority?
or guard to bed
93. The least serious form of brain trauma, A. Assessing the left leg
characterized by a brief loss of consciousness and B. Assessing the pupils
period of confusion, is called: C. Placing the client in Trendelenburg’s position
D. Assessing the level of consciousness
A. contusion 99. Alzheimer’s disease is the secondary diagnosis
B. concussion of a client admitted with myocardial infarction.
C. coup Which nursing intervention should appear on this
D. contrecoup client’s plan of care?
94. When the nurse performs a neurologic
assessment on Anne Jones, her pupils are dilated A. Perform activities of daily living for the client
and don’t respond to light. to decease frustration.
B. Provide a stimulating environment.
A. glaucoma C. Establish and maintain a routine.
B. damage to the third cranial nerve D. Try to reason with the client as much as
C. damage to the lumbar spine possible.
D. Bell’s palsy
100. For a client with a head injury whose neck has 7. ANS: C
been stabilized, the preferred bed position is: The client should report the presence of
foulsmelling or cloudy urine. Unless
A. Trendelenburg’s contraindicated, the client should be instructed
B. 30-degree head elevation to drink large quantities of fluid each day to
C. flat flush the kidneys. Sand-like debris is normal
D. side-lying because of residual stone products. Hematuria
Answers and Rationales is common after lithotripsy.
8. ANS: A
1. ANS: D A serum glucose level of 618 mg/dl indicates
Because the client’s gag reflex is absent, hyperglycemia, which causes polyuria and
elevating the head of the bed to 30 degrees deficient fluid volume. In this client,
helps minimize the client’s risk of aspiration. tachycardia is more likely to result from
Checking the stools, performing ROM deficient fluid volume than from decreased
exercises, and keeping the skin clean and dry cardiac output because his blood pressure is
are important, but preventing aspiration normal. Although the client’s serum glucose is
through positioning is the priority. elevated, food isn’t a priority because fluids
2. ANS: A and insulin should be administered to lower the
Any hole, no matter how small, will destroy serum glucose level. Therefore, a diagnosis of
the odor-proof seal of a drainage bag. Imbalanced Nutrition: Less then body
Removing the bag or unclamping it is the only requirements isn’t appropriate. A temperature
appropriate method for relieving gas. of 100.6º F isn’t life threatening, eliminating
3. ANS: A ineffective thermoregulation as the top priority.
because celiac disease destroys the absorbing 9. ANS: C
surface of the intestine, fat isn’t absorbed but is Regular insulin, which is a short-acting insulin,
passed in the stool. Steatorrhea is bulky, fatty has an onset of 15 to 30 minutes and a peak of
stools that have a foul odor. Jaundiced sclerae 2 to 4 hours. Because the nurse gave the insulin
result from elevated bilirubin levels. Clay- at 2 p.m., the expected onset would be from
colored stools are seen with biliary disease 2:15 to 2:30 p.m. and the peak from 4 p.m. to 6
when bile flow is blocked. Celiac disease p.m.
doesn’t cause a widened pulse pressure. 10. ANS: A
4. ANS: D CPP is derived by subtracting the ICP from the
Reducing sodium intake reduces fluid retention. mean arterial pressure (MAP). For adequate
Fluid retention increases blood volume, which cerebral perfusion to take place, the minimum
changes blood vessel permeability and allows goal is 70 mmHg. The MAP is derived using
plasma to move into interstitial tissue, causing the following formula:
edema. Urea nitrogen excretion can be MAP = ((diastolic blood pressure x 2) +
increased only by improved renal function. systolic blood pressure) / 3
Sodium intake doesn’t affect the glomerular MAP = ((60 x2) + 90) / 3
filtration rate. Potassium absorption is MAP = 70 mmHg
improved only by increasing the glomerular To find the CPP, subtract the client’s ICP from
filtration rate; it isn’t affected by sodium intake. the MAP; in this case , 70 mmHg – 18 mmHg
5. ANS: D = 52 mmHg.
The portion of the cerebrum that controls 11. ANS: B
speech and hearing is the temporal lobe. Injury Breast cancer tumors are fixed, hard, and
to the frontal lobe causes personality changes, poorly delineated with irregular edges. Nipple
difficulty speaking, and disturbance in memory, retraction —not eversion—may be a sign of
reasoning, and concentration. Injury to the cancer. A mobile mass that is soft and easily
parietal lobe causes sensory alterations and delineated is most often a fluid-filled benigned
problems with spatial relationships. Damage to cyst. Axillary lymph nodes may or may not be
the occipital lobe causes vision disturbances. palpable on initial detection of a cancerous
6. ANS: D mass.
Diabetes insipidus is an abrupt onset of 12. ANS: D
extreme polyuria that commonly occurs in An enterostomal nurse therapist is a registered
clients after brain surgery. Cushing’s syndrome nurse who has received advance education in
is excessive glucocorticoid secretion resulting an accredited program to care for clients with
in sodium and water retention. Diabetes stomas. The enterostomal nurse therapist can
mellitus is a hyperglycemic state marked by assist with selection of an appropriate stoma
polyuria, polydipsia, and polyphagia. Adrenal site, teach about stoma care, and provide
crisis is undersecretion of glucocorticoids emotional support.
resulting in profound hypoglycemia, 13. ANS: A
hypovolemia, and hypotension. Ottorrhea and rhinorrhea are classic signs of
basilar skull fracture. Injury to the dura nurse can probably safely approach her about
commonly occurs with this fracture, resulting talking with others who have had similar
in cerebrospinal fluid (CSF) leaking through experiences, either through Reach for
the ears and nose. Any fluid suspected of being Recovery or another formal support group. The
CSF should be checked for glucose or have a nurse may educate the client’s spouse or
halo test done. partner to listen to concerns, but the nurse
14. ANS: D shouldn’t tell the client’s spouse what to do.
Testicular cancer commonly occurs in men The client must consult with her physician and
between ages 20 and 30. A male client should make her own decisions
be taught how to perform testicular self- about further treatment. The client needs to
examination before age 20, preferably when he express her sadness, frustration, and fear. She
enters his teens. can’t be expected to be cheerful at all times.
15. ANS: B 21. ANS: C
Before weaning a client from mechanical HCG is one of the tumor markers for testicular
ventilation, it’s most important to have a cancer. The HCG level won’t identify the site
baseline ABG levels. During the weaning of an infection or evaluate prostatic function or
process, ABG levels will be checked to assess testosterone level.
how the client is tolerating the procedure. 22. ANS: B
Other assessment parameters are less critical. Peripheral edema is a sign of fluid volume
Measuring fluid volume intake and output is overload and worsening heart failure. A skin
always important when a client is being rash, dry cough, and postural hypotension are
mechanically ventilated. Prior attempts at adverse reactions to captopril, but the don’t
weaning and ECG results are documented on indicate that therapy isn’t effective.
the client’s record, and the nurse can refer to 23. ANS: A
them before the weaning process begins. Tenting of chest skin when pinched indicates
16. ANS: B decreased skin elasticity due to dehydration.
According to the ACS guidelines, “Women Hand veins fill slowly with dehydration, not
older than age 40 should perform breast rapidly. A pulse that isn’t easily obliterated and
selfexamination monthly (not annually).” The neck vein distention indicate fluid overload,
hormonal receptor assay is done on a known not dehydration.
breast tumor to determine whether the tumor is 24. ANS: B
estrogen- or progesterone-dependent. The client should be encouraged to increase his
17. ANS: C activity level. Maintaining an ideal weight;
Increased pressure within the portal veins following a low-cholesterol, low-sodium diet;
causes them to bulge, leading to rupture and and avoiding stress are all important factors in
bleeding into the lower esophagus. Bleeding decreasing the risk of atherosclerosis.
associated with esophageal varices doesn’t 25. ANS: B
stem from esophageal perforation, pulmonary Because thrombocytopenia impairs blood
hypertension, or peptic ulcers. clotting, the nurse should assess the client
18. ANS: B regularly for signs of bleeding, such as
Complex intra-articular fractures are repaired petechiae, purpura, epistaxis, and bleeding
with external fixators because they have a gums. The nurse should avoid administering
better long-term outcome than those treated aspirin because it can increase the risk of
with casting. This is especially true in a young bleeding. Frequent rest periods are indicated
client. The incidence of complications, such as for clients with anemia, not thrombocytopenia.
pin tract infections and neuritis, is 20% to 60%. Strict isolation is indicated only for clients who
Clients must be taught how to do pin care and have highly contagious or virulent infections
assess for development of neurovascular that are spread by air or physical contact.
complications. 26. ANS: B
19. ANS: C SaO2 is the degree to which hemoglobin is
In a client with chronic renal failure, saturated with oxygen. It doesn’t indicate the
unrestricted intake of sodium, protein, client’s overall Hgb adequacy. Thus, an
potassium, and fluids may lead to a dangerous individual with a subnormal Hgb level could
accumulation of electrolytes and protein have normal SaO2 and still be short of breath.
metabolic products, such as amino acids and In this case, the nurse could assume that the
ammonia. Therefore, the client must limit his client has a Hematologic problem. Poor
intake of sodium, meat (high in Protein), peripheral perfusion would cause subnormal
bananas (high in potassium), and fluid because SaO2. There isn’t enough data to assume that
the kidneys can’t secrete adequate urine. the client’s problem is psychosomatic. If the
20. ANS: D problem were
The client isn’t withdrawn or showing other left-sided heart failure, the client would exhibit
signs of anxiety or depression. Therefore, the pulmonary crackles.
27. ANS: A The hypernatremia is secondary to her water
Addisonian crisis results in Hyperkalemia; loss. Imbalanced nutrition related to
therefore, administering potassium chloride is hypermetabolic state or catabolic effect of
contraindicated. Because the client will be insulin deficiency is an inappropriate nursing
hyponatremic, normal saline solution is diagnosis for the client.
indicated. Hydrocortisone and fludrocortisone 34. ANS: C
are both useful in replacing deficient adrenal When performing a nutritional assessment, one
cortex hormones. of the first things the nurse should do is to
28. ANS: D assess what the client typically eats. The client
Leukemia is manifested by an abnormal shouldn’t be permitted to eat as desired.
overpopulation of immature leukocytes in the Weighing the client daily, placing her on I & O
bone marrow. status, and drawing blood to determine
29. ANS: C electrolyte level aren’t part of a nutritional
A sterile field is considered contaminated when assessment.
it becomes wet. Moisture can act as a wick, 35. Ans. C
allowing microorganisms to contaminate the Regular follow-up care for the client with
field. The outside of containers, such as sterile Grave’s disease is critical because most cases
saline bottles, aren’t sterile. The containers eventually result in hypothyroidism. Annual
should be opened before sterile gloves are put thyroid-stimulating hormone tests and the
on and the solution poured over the sterile client’s ability to recognize signs and
dressings placed in a sterile basin. Wounds symptoms of thyroid dysfunction will help
should be cleaned from the most contaminated detect thyroid abnormalities early. Intake and
area to the least contaminated area—for output is important for clients with fluid and
example, from the center outward. The outer electrolyte imbalances but not thyroid disorders.
inch of a sterile field shouldn’t be considered DDAVP is used to treat diabetes insipidus.
sterile. While exercise to improve cardiovascular
30. ANS: C fitness is important, for this client the
Caffeine is a stimulant, which can exacerbate importance of regular follow-up is most critical.
palpitations and should be avoided by a client 36. ANS: D
with symptomatic mitral valve prolapse. High Anxiety related to the threat of death is an
fluid intake helps maintain adequate preload appropriate nursing diagnosis because the
and cardiac output. Aerobic exercise helps in client’s anxiety can adversely affect hear rate
increase cardiac output and decrease heart rate. and rhythm by stimulating the autonomic
Protein-rich foods aren’t restricted but high nervous system. Also, because the client
calorie foods are. required resuscitation, the threat of death is a
31. ANS: A real and immediate concern. Unless anxiety is
Excessive of aldosterone in the adrenal cortex dealt with first, the client’s emotional state will
is responsible for the client’s hypertension. impede learning. Client teaching should be
This hormone acts on the renal tubule, where it limited to clear concise explanations that
promotes reabsorption of sodium and excretion reduce anxiety and promote cooperation. An
of potassium and hydrogen ions. The pancreas anxious client has difficulty learning, so the
mainly secretes hormones involved in fuel deficient knowledge would continue despite
metabolism. The adrenal medulla secretes the attempts teaching. Impaired physical mobility
cathecolamines—epinephrine and and social isolation are necessitated by the
norepinephrine. The parathyroids secrete client’s critical condition; therefore, they aren’t
parathyroid hormone. considered problems warranting nursing
32. ANS: D diagnoses.
A heart valve prosthesis, such as a mitral valve 37. ANS: A
replacement, is a major risk factor for infective Because tuberculosis is transmitted by droplet
endocarditis. Other risk factors include a nuclei from the respiratory tract, the nurse
history of heart disease (especially mitral valve should put on a mask when entering the
prolapse), chronic debilitating disease, IV drug client’s room. Having the client wear a mask at
abuse, and immunosuppression. Although all the times would hinder sputum
diabetes mellitus may predispose a person to expectoration and make the mask moist from
cardiovascular disease, it isn’t a major risk respirations. If no contact with the client’s
factor for infective endocarditis, nor is an blood or body fluids is anticipated, the nurse
appendectomy or pernicious anemia. need not wear a gown or gloves when
33. ANS: A providing direct care. A client with
The client has signs and symptoms of diabetes tuberculosis should be in a room with laminar
insipidus, probably caused by the failure of her air flow, and the door should be closed at all
renal tubules to respond to antidiuretic times.
hormone as a consequence of pyelonephritis.
38. ANS: C the client’s feelings. Ignoring the client’s
The nurse can gently irrigate the tube but must feelings suggest that the nurse has no interest
take care not to reposition it. Repositioning can in what the client has said.
cause bleeding. Suction should be applied 44. ANS: A
continuously, not every hour. The NG tube Daily walks relieve symptoms of intermittent
shouldn’t be clamped postoperatively because claudication, although the exact mechanism is
secretions and gas will accumulate, stressing unclear. Anaerobic exercise may exacerbate
the suture line. these symptoms. Clients with chronic arterial
39. ANS: A occlusive disease must reduce daily fat intake
Regardless of the client’s medical history, to 30% or less of total calories. The client
rapid fluid resuscitation is critical for should limit dietary cholesterol because
maintaining cardiovascular integrity. Profound hyperlipidemia is associated with
intravascular depletion requires aggressive atherosclerosis, a known cause of arterial
fluid replacement. A typical fluid resuscitation occlusive disease. However, HDLs have the
protocol is 6 L of fluid over the first 12 hours, lowest cholesterol concentration,
with more fluid to follow over the next 24 so this client should eat foods that raise HDL
hours. Various fluids can be used, depending levels.
on the degree of hypovolemia. Commonly 45. ANS: A
prescribed fluids include dextran (in case of Gastric decompression is typically low
hypovolemic shock), isotonic normal saline pressure and intermittent. High pressure and
solution and, when the client is stabilized, continuous gastric suctioning predisposes the
hypotonic half-normal saline solution. gastric mucosa to injury and ulceration.
40. ANS: D 46. ANS: A
Glipizide may cause adverse skin reactions, In osteoarthritis, stiffness is common in large,
such as pruritus, and photosensitivity. It weight bearing joints such as the hips. This
doesn’t cause headache, constipation, or joint stiffness alters functional ability and
hypotension. range of motion, placing the client at risk for
41. ANS: B falling and injury. Therefore, client safety is in
When clients are on mechanical ventilation, the jeopardy. Osteoporosis doesn’t affect urinary
artificial airway impairs the gag and cough elimination, breathing, or nutrition.
reflexes that help keep organisms out of the 47. ANS: A
lower respiratory tract. The artificial airway PTH stimulates the kidneys to reabsorb
also prevents the upper respiratory system from calcium and excrete phosphate and converts vit
humidifying and heating air to enhance D to its active form: 1 , 25 dihydroxy vitamin
mucociliary clearance. Manipulations of the D. PTH doesn’t have a role in the metabolism
artificial airway sometimes allow secretions of Vit E.
into the lower airways. Whit standard 48. ANS: C
procedures the other choices wouldn’t be at The presence of unsecured area rugs poses a
high risk. hazard in all homes, particularly in one with a
42. ANS: C resident at high risk for falls.
In chronic bronchitis, the diaphragmatic is flat 49. ANS: B
and weak. Diaphragmatic breathing helps to Autonomy ascribes the right of the individual
strengthen the diaphragm and maximizes to make his own decisions. In this case, the
ventilation. Exhalation should longer than client is capable of making his own decision
inhalation to prevent collapse of the and the nurse should support his autonomy.
bronchioles. The client with chronic bronchitis Beneficence and justice aren’t the principles
should exhale through pursed lips to prolong that directly relate to the situation. Advocacy is
exhalation, keep the bronchioles from the nurse’s role in supporting the principle of
collapsing, and prevent air trapping. autonomy.
Diaphragmatic breathing—not chest 50. ANS: D
breathing—increases lung expansion. Because respirations are depressed in
43. ANS: A myxedema coma, maintaining a patent airway
Verbalizing the observed behavior is a is the most critical nursing intervention.
therapeutic communication technique in which Ventilatory support is usually needed. Thyroid
the nurse acknowledges what the client is replacement will be administered IV. Although
feeling. Offering to listen to the client express myxedema coma is associated with severe
her anger can help the nurse and the client hypothermia, a warming blanket shouldn’t be
understand its cause and begin to deal with it. used because it may cause vasodilation and
Although stress can exacerbate the symptoms shock. Gradual warming blankets would be
of SLE, telling the client to calm down doesn’t appropriate. Intake and output are very
acknowledge her feelings. Offering to get the important but aren’t critical
nursing supervisor also doesn’t acknowledge interventions at this time.
51. ANS: A 58. ANS: C
Glipizide begins to act in 15 to 30 minutes. The Diarrhea causes a bicarbonate deficit. With loss
other options are incorrect. of the relative alkalinity of the lower GI tract,
52. ANS: A the relative acidity of the upper GI tract
Hypoxia is the main breathing stimulus for a predominates leading to metabolic acidosis.
client with COPD. Excessive oxygen Diarrhea doesn’t lead to respiratory acid-base
administration may lead to apnea by removing imbalances, such as respiratory acidosis and
that stimulus. Anginal pain results from a respiratory alkalosis. Loss of acid, which
reduced myocardial oxygen supply. A client occurs with severe vomiting, may lead to
with COPD may have anginal pain from metabolic alkalosis.
generalized vasoconstriction secondary to 59. ANS: B
hypoxia; however, administering oxygen at any The head of the bed must be elevated while the
concentration dilates blood vessels, easing client is eating. The client should be placed in a
anginal pain. Respiratory alkalosis results from recumbent position—not a supine position—
alveolar hyperventilation, not excessive oxygen when lying down to reduce the risk of
administration. In a client with COPD, high aspiration. Encourage the client to wear
oxygen concentrations decrease the ventilatory properly fitted dentures to enhance his chewing
drive, leading to respiratory acidosis, not ability. Thickened liquids, not thin liquids,
alkalosis. High oxygen concentrations don’t decrease aspiration risk.
cause metabolic acidosis. 60. ANS: A
53. ANS: C Endotracheal suctioning secretions as well as
Continuous subcutaneous insulin regimen uses gases from the airway and lowers the arterial
a basal rate and boluses of short-acting insulin. oxygen saturation (SaO2) level. Coughing and
Multiple daily injection therapy uses a incentive spirometry improve oxygenation and
combination of short-acting and intermediate should raise or maintain oxygen saturation.
or long-acting insulins. Because of superficial vasoconstriction, using a
54. ANS: B cooling blanket can lower peripheral oxygen
Holding a cane on the uninvolved side saturation readings, but SaO2 levels wouldn’t
distributes weight away from the involved side. be affected.
Holding the cane close to the body prevents 61. ANS: D
leaning. Use of a cane won’t maintain stride With a superficial partial thickness burn such
length or prevent edema. as a solar burn (sunburn), the nurse’s main
55. ANS: D concern is pain management. Fluid
High pulmonary artery wedge pressures are resuscitation and infection become concerns if
diagnostic for left-sided heart failure. With the burn extends to the dermal and
leftsided heart failure, pulmonary edema can subcutaneous skin layers. Body image
develop causing pulmonary crackles. In disturbance is a concern that has a lower
leftsided heart failure, hypotension may result priority than pain management.
and urine output will decline. Dry mucous 62. ANS: D
membranes aren’t directly associated with Crackles result from air moving through
elevated pulmonary artery wedge pressures. airways that contain fluid. Heard during
56. ANS: B inspiration and expiration, crackles are discrete
It’s mandatory in most settings to have a sounds that vary in pitch and intensity. They’re
physician’s order before restraining a client. A classified as fine, medium, or coarse. Pleural
client should never be left alone while the friction rubs have a distinctive grating sound.
nurse summons assistance. All staff members As the name indicates, these breath sounds
require annual instruction on the use of result when inflamed pleurae rub together.
restraints, and the nurse should be familiar with Continuous, highpitched, musical squeaks,
the facility’s policy. called wheezes, result when air moves rapidly
57. ANS: A through airways narrowed by asthma or
The client who has undergone a thyroidectomy infection or when an airway is partially
is t risk for developing hypocalcemia from obstructed by a tumor or foreign body.
inadvertent removal or damage to the Wheezes, like gurgles, occur on expiration and
parathyroid gland. The client with sometimes on inspiration. Loud, coarse, low-
hypocalcemia will exhibit a positive pitched sounds resembling snoring are called
Chvostek’s sign (facial muscle contraction gurgles. These sounds develop when thick
when the facial nerve in front of the ear is secretions partially obstruct airflow through the
tapped) and a positive Trousseau’s sign (carpal large upper airways.
spasm when a blood pressure cuff is inflated 63. Ans. D
for few minutes). These signs aren’t present Individuals who are tuberculin skin test
with hypercalcemia, hypokalemia, or converters should begin a 6-month regimen of
Hyperkalemia. an antitubercular drug such as INH, and they
should never have another skin test. After an raised papules occur at the site of the exposure
individual has a positive tuberculin skin test, and can begin within 1 hour of exposure.
subsequent skin tests will cause severe skin Allergic reactions tend to be red and not scaly
reactions but won’t provide new information or flaky. Weeping, crusting lesions are also
about the client’s TB status. The client doesn’t uncommon unless the reaction is quite severe
have active TB, so can’t transmit, or spread, or has been present for a long time. Excoriation
the bacteria. Therefore, she shouldn’t be is more common in skin disorders associated
quarantined or asked for information about with a moist environment.
recent contacts. 70. ANS: D
64. ANS: C Hoarseness indicate injury to the respiratory
Disturbed body image is a negative perception system and could indicate the need for
of the self that makes healthful functioning immediate intubation. Thirst following burns is
more difficult. The defining characteristics for expected because of the massive fluid shifts
this nursing diagnosis include undergoing a and resultant loss leading to dehydration. Pain,
change in body structure or function, hiding or either severe or moderate, is expected with a
overexposing a body part, not looking at a burn injury. The client’s output is adequate.
body part, and responding verbally or 71. ANS: C
nonverbally to the actual or perceived change Third-degree burn may appear white, red, or
in structure or function. This client may have black and are dry and leathery with no blisters.
any of the other diagnoses, but the signs and There may be little pain because nerve endings
symptoms described in he case most closely have been destroyed. First-degree burns are
match the defining characteristics for disturbed superficial and involve the epidermis only.
body image. There is local pain and redness but no
65. ANS: B blistering. Second-degree burn appear red and
Because the pregnant uterus exerts a lot of moist with blister formation and are painful.
pressure on the urinary bladder, the bladder Fourth-degree burns involve underlying muscle
repair may need to be repeated. These clients and bone tissue.
don’t necessarily have to have a cesarean 72. ANS: C
delivery if they become pregnant, and this A universal concern I the care of donor sites
procedure doesn’t render them sterile. This for burn care is to keep the site away from
procedure is completed in one surgery. sources of pressure. Ventilation of the site and
66. ANS: A keeping the site fully covered are practices in
Eccrine glands are associated with body some institutions but aren’t hallmarks of donor
temperature regulation. Sebaceous glands site care. Placing the site in a position of
lubricate the skin and hairs, and apocrine dependence isn’t a justified aspect of donor site
glands are involved in bacteria decomposition. care.
Endocrine glands secrete hormones responsible 73. ANS: A
for the regulation of body processes, such as Anyone with psoriasis vulgaris who reports
metabolism and glucose regulation. joint pain should be evaluated for psoriaic
67. ANS: B arthritis. Approximately 15% to 20% of
The light-colored spots attached to the hair individuals with psoriasis will also develop
shafts are nits, which are the eggs of head lice. psoriatic arthritis, which can be painful and
They can’t be brushed off the hair shaft like cause deformity. It would be incorrect to
dandruff. Scabies is a contagious dermatitis assume that his pain is caused by early
caused by the itch mite, Sacoptes scabiei, rheumatoid arthritis or his vocation without
which lives just beneath the skin. Tinea capitis, asking more questions or performing
or ringworm, causes patchy hair loss and diagnostic studies. Carpal tunnel syndrome
circular lesions with healing centers. Impetigo causes sensory and motor changes in the
is an infection caused by Staphylococcus or fingers rather than localized pain in the joints.
Sterptococcus, manifested by vesicles or 74. ANS: B
pustules that form a thick, honey-colored crust. To avoid burning and sloughing, the client
68. ANS: C must protect the graft from sunlight. The other
Severe pressure-like pain out of proportion to three interventions are all helpful to the client
visible signs distinguishes necrotizing fasciitis and his recovery but are less important.
from cellulites. Erythema, leukocytosis, and 75. ANS: C
swelling are present in both cellulites and Because the itching and rash are localized, an
necrotizing fasciitis. environmental cause in the workplace should
69. ANS: A be suspected. With the advent of universal
Contact dermatitis is caused by exposure to a precautions, many nurses are experiencing
physical or chemical allergen, such as cleaning allergies to latex gloves. Allergies to
products, skin care products, and latex gloves. medications, laundry detergents, or bath soaps
Initial symptoms of itching, erythema, and
or a dermatologic reaction to stress usually output consistently above 100 ml/hour is more
elicit a more generalized or widespread rash. than adequate. Weight gain from fluid
76. ANS: A resuscitation isn’t a goal. In fact, a 4 lb weight
Hives and urticaria are two names for the same gain in 24 hours suggests third spacing. Body
skin lesion. Toxin is a poison. A tubercle is a temperature readings and ECG interpretations
tiny round nodule produced by the tuberculosis may demonstrate secondary benefits of fluid
bacillus. A virus is an infectious parasite. resuscitation but aren’t primary indicators.
77. ANS: A 84. ANS: B
A scale is the characteristic secondary lesion The ESR test is performed to detect
occurring in psoriasis. Although crusts, ulcers, inflammatory processes in the body. It’s a
and scars also are secondary lesions in skin nonspecific test, so the health care professional
disorders, they don’t accompany psoriasis. must view results in conjunction with physical
78. ANS: C signs and symptoms. Platelet count,
A turning schedule with a signing sheet will electrolytes, BUN, and creatinine levels aren’t
help ensure that the client gets turned and thus, usually affected by the inflammatory process.
help prevent pressure ulcers. Turning should 85. ANS: B
occur every 1-2 hours—not every 8 hours—for In Parkinson’s crisis, dopamine-related
clients who are in bed for prolonged periods. symptoms are severely exacerbated, virtually
The nurse should apply lotion to keep the skin immobilizing the client. A client who is
moist but should avoid vigorous massage, confined to bed during a crisis is at risk for
which could damage capillaries. When moving aspiration and pneumonia. Also, excessive
the client, the nurse should lift rather than slide drooling increases the risk of airway
the client to void shearing. obstruction. Because of these concerns,
79. ANS: A ineffective airway clearance is the priority
To prevent disruption of the artificial skin’s diagnosis for this client. Although imbalanced
adherence to the wound bed, the client should nutrition:less than body requirements, impaired
restrict range of motion of the involved limb. urinary elimination and risk for injury also are
Protein intake and fluid intake are important appropriate diagnoses for this client, they
for healing and regeneration and shouldn’t be aren’t immediately lifethreatening and thus are
restricted. Going outdoors is acceptable as long less urgent.
as the left arm is protected from direct sunlight. 86. ANS: A
80. ANS: C As a rule of practice, if intermittent
According to the Rule of Nines, the posterior catheterization for urine retention typically
and anterior trunk, and legs each make up 18% yields 500 ml or more, the frequency of
of the total body surface. The head, neck, and catheterization should be increased. Indwelling
arms each make up 9% of total body durface, catheterization is less preferred because of the
and the perineum makes up 1%. In this case, risk of urinary tract infection and the loss of
the client received burns to his back (18%) and bladder tone. Fluid restrictions aren’t indicated
one arm (9%), totaling 27%. for this case; the problem isn’t overhydration,
81. ANS: A rather it’s urine retention. A condom catheter
A wet-to-dry saline dressing should always doesn’t help empty the bladder of a client with
keep the wound moist. Tight packing or dry urine retention.
packing can cause tissue damage and pain. A 87. ANS: D
dry gauze —not a plastic-sheet-type dressing— The normal pressure is 70 to 200 mm H2O are
should cover the wet dressing. considered abnormal. The presence of glucose
82. ANS: A is an expected finding in CSF, and RBCs
When someone in a group of persons sharing a typically occur in the first specimen tube from
home contracts scabies, each individual in the the trauma caused by the procedure.
same home needs prompt treatment whether 88. ANS: A
he’s symptomatic or not. Towels and linens The nurse should instill the eyedrop into the
should be washed in hot water. Scabies can be conjunctival sac where absorption can best take
transmitted from one person to another before place. The pupil permits light to enter the eye.
symptoms develop The sclera maintains the eye’s shape and size.
83. ANS: A The vitreous humor maintains the retina’s
In a client with burns, the goal of fluid placement and the shape of the eye.
resuscitation is to maintain a mean arterial 89. ANS: C
blood pressure that provides adequate According to family theory, any change in a
perfusion of vital structures. If the kidneys are family member, such as illness, produces role
adequately perfused, they will produce an changes in all family members and affects the
acceptable urine output of at least 0.5 entire family, even if the client eventually
ml/kg/hour. Thus, the expected urine output of recovers completely.
a 155-lb client is 35 ml/hour, and a urine
90. ANS: B 97. ANS: D
To help prevent airway obstruction and reduce With a brain injury such as an epidural
the risk of aspiration, the nurse should position hematoma (a diagnosis that is most likely
a client with hemiparesis on the affected side. based on this client’s symptoms), the initial
Although performing ROM exercises, sign of increasing ICP is a change in the level
providing pillows for support, and applying of consciousness. As neurologic deterioration
antiembolism stockings can be appropriate for progresses, manifestations involving pupillary
a client with CVA, the first concern is to symmetry, breathing patterns, and posturing
maintain a patent airway. will
91. ANS: C occur.
TIA may be a warning that the client will 98. ANS: A
experience a CVA, or stroke, in the near future. In the scenario, airway and breathing are
TIA aymptoms last no longer than 24 hours established so the nurse’s next priority should
and clients usually have complete recovery be circulation. With a compound fracture of the
after TIA. The most common symptom of TIA femur, there is a high risk of profuse bleeding;
is sudden, painless loss of vision lasting up to therefore, the nurse should assess the site.
24 hours. Neurologic assessment is a secondary concern
92. ANS: C to airway, breathing and circulation. The nurse
Bending to pick up something from the floor doesn’t have enough data to warrant putting the
would increase intraocular pressure, as would client in Trendelenburg’s position.
bending to tie his shoes. The client needs to 99. ANS: C
wear eye protection to bed to prevent Establishing and maintaining a routine is
accidental injury during sleep. essential to decreasing extraneous stimuli. The
93. ANS: B client should participate in daily care as much
Concussions are considered minor with no as possible. Attempting to reason with such
structural signs of injury. A contusion is clients isn’t successful, because they can’t
bruising of the brain tissue with small participate in abstract thinking.
hemorrhages in the tissue. Coup and 100. ANS: B
contrecoup are type of injuries in which the For clients with increased intracranial pressure
damaged area on the brain forms directly (ICP), the head of the bed is elevated to
below that site of impact (coup) or at the promote venous outflow. Trendelenburg’s
site opposite the injury (contrecoup) due to position is contraindicated because it can raise
movement of the brain within the skull. ICP. Flat or neutral positioning is indicated
94. ANS: B when elevating the head of the bed would
The third cranial nerve (oculomotor) is increase the risk of neck injury or airway
responsible for pupil constriction. When there obstruction. Sidelying isn’t specifically a
is damage to the nerve, the pupils remain therapeutic treatment for increased ICP.
dilated and don’t respond to light. Glaucoma,
lumbar spine injury, and Bell’s palsy won’t
affect pupil constriction.
95. ANS: C
When the mechanical stressors of weight
bearing are absent, diffuse osteoporosis can PNLE Medical
occur. Therefore, if the client does weight-
bearing exercises, disuse complications can be
prevented. Maintaining protein and vitamins Surgical Nursing
levels is important, but neither will prevent
osteoporosis. ROM exercises will help prevent
muscle atrophy and contractures.
Exam 3
96. ANS: B 1. Lisa is newly diagnosed with asthma and is being
This client requires droplet precautions because discharged from the hospital after an episode of
the organism can be transmitted through status asthmaticus. Discharge teaching should
airborne droplets when the client coughs, include which of the following:
sneezes, or doesn’t cover his mouth. Airborne
precautions would be instituted for a client A. Limitations in sports that will be imposed by
infected with tuberculosis. Standard the illness
precautions would be instituted for a client B. Specific instructions on staying cal during an
when contact with body substances is likely. attack
Contact precautions would be instituted for a C. The relationship of symptoms and a specific
client infected with an organism that is trigger such as physical exercise
transmitted through skin-to-skin D. Incidence of status asthmaticus in children and
contact. teens
2. Which of the following symptoms is most D. Aneurysm
characteristic of a client with a cancer of the lung? 9. Nurse Fiona is caring a patient with Raynaud’s
disease. Which of the following outcomes
A. air hunger concerning medication regimen is of highest
B. exertional dyspnea priority?
C. cough with night sweats
D. persistent changing cough A. Controlling the pain once vasospasm occur
3. The client has ST segment depression on his 12- B. Relaxing smooth muscle to avoid vasospasms
lead ECG. The nurse determines that this would C. Preventing major disabilities that may occur
indicate the following: D. Avoiding lesions on the feet
10. Mr. Roberto Robles complains of a severe
A. necrosis headache and is extremely anxious. The nurse
B. injury checks his vital signs and finds him to have a heart
C. ischemia rate of 57 bpm and a blood pressure of 230/110
D. nothing significant mmHg. The nurse should also assess for?
4. Red has just returned from the postanesthesia
care unit (PACU) from a hemorrhidectomy. His A. presence of bowel sounds
postoperative orders include sitz baths every B. presence of babinski reflex
morning. The nurse understands that sitz bath is use C. fecal incontinence
for: D. urinary catheter patency
11. A 40n year old male patient is complaining of
A. promote healing chronic progressive and mental deterioration is
B. relive tension admitted to the unit. The nurse recognizes that these
C. lower body temperature characteristics indicate a disease that results in
D. cause swelling degeneration of the basal ganglia and cerebral
5. Trousseau’s sign is associated with which cortex. The disease is called:
electrolyte imbalance?
A. multiple sclerosis
A. hyponatremia B. myasthenia gravis
B. hypocalcemia C. Huntington’s disease
C. hypernatremia D. Guillain-Barre syndrome
D. hypercalcemia 12. Dianne Hizon is a 27 year old woman who has
6. A 36 year old female complains of headache and been admitted to the ER due to severe vomiting.
neck pain. The nurse’s assessments reveal painful Her ABG values are pH= 7.50, PaCO2= 85,
flexion of the neck to the chest. The nurse HCO3= 31, and SaO2= 93%. The nurse
understands that nuchal rigidity is associated with: interpretation of this ABG analysis is:

A. brain tumor A. respiratory acidosis


B. CVA B. respiratory alkalosis
C. meningitis C. metabolic acidosis
D. subdural hematoma D. metabolic alkalosis
7. The nurse teaching the client about behavioral 13. Mr. Perkson has a parkinson’s disease and he
changes, which can affect development of finds the resting tremor he is experiencing in his
atherosclerosis, should discuss which of the right hand very frustrating. The nurse advises him
following as a non-modifiable risk factor for to:
atherosclerosis?
A. take a warm bath
A. cigarette smoking B. hold an object
B. hyperlipidemia C. practice deep breathing
C. female over 55 years of age D. take diazepam as needed
D. sedentary lifestyle 14. A shuffling gait is typically associated with the
8. A 76 year old man enters the ER with complaints patient who has:
of back pain and feeling fatigued. Upon
examination, his blood pressure is 190/100, pulse is A. Parkinson’s disease
118, and hematocrit and hemoglobin are both low. B. Multiple sclerosis
The nurse palpates the abdomen which is soft, non- C. Raynaud’s disease
tender and auscultates an abdominal pulse. The D. Myasthenia gravis
most likely diagnosis is: 15. The priority in preparing the room for a client
with a C7 spinal cord injury is having:
A. Buerger’s disease
B. CHF A. the halo brace device
C. Secondary hypertension B. a catheterization tray
C. a ventilator on stand by D. “The required medications reduce clotting
D. the spinal kinetic bed factors.”
16. A 47 year old man with liver failure who has 22. Betty Lee is a 58 year old woman who is being
developed ascites. The nurse understands that admitted to the medical ward with trigeminal
ascites is due to: neuralgia. The nurse anticipates that Mr. Lee will
demonstrate which of the following major
A. dehydration complaints?
B. protein deficiency
C. bleeding disorders A. excruciating, intermittent, paroxysmal facial
D. vitamin deficiency pain
17. A client with rheumatoid arthritis may reveal B. unilateral facial droop
which of the following assessment data: C. painless eye spasm
D. mildly painful unilateral eye twitching
A. Heberden’s nodes 23. A 38 year old woman returns from a subtotal
B. Morning stiffness no longer than 30 minutes thryroidectomy for the treatment of
C. Asymmetric joint swelling hyperthyroidism. Upon assessment, the immediate
D. Swan neck deformities priority that the nurse would include is:
18. Elsa Santos is a 18 year old student admitted to
the ward with a diagnosis of epilepsy. She tells the A. Assess for pain
nurse that she is experiencing a generalized tingling B. Assess for neurological status
sensation and is “smelling roses”. The nurse C. Assess fluid volume status
understands that Esla is probably experiencing: D. Assess for respiratory distress
24. Nurse Shiela is teaching self-care to a client
A. an acute alcohol withdrawal with psoriasis. The nurse should encourage which
B. an acute CVA of the following for his scaled lesion?
C. an aura
D. an olfactory hallucination A. Importance of follow-up appointments
19. Mr. Lucas, a 63 year old, went to the clinic B. Emollients and moisturizers to soften scales
complaining of hoarseness of voice and a cough. C. Keep occlusive dressings on the lesions 24
His wife states that his voice has changed in the last hours a day
few months. The nurse interprets that Mr. Lucas’s D. Use of a clean razor blade each time he shaves
symptoms are consistent with which of the 25. A 48 year old woman presents to the hospital
following disorders: complaining of chest pain, tachycardia and dyspnea.
On exam, heart sounds are muffled. Which of the
A. chronic sinusitis following assessment findings would support a
B. laryngeal cancer diagnosis of cardiac tamponade?
C. gastroesophageal reflux disease
D. coronary artery disease A. A deviated trachea
20. Sarah complains of a nursing sensation, B. Absent breath sounds to the lower lobes
cramping pain in the top part of her abdomen that C. Pulse 40 with inspiration
becomes worse in the afternoon and sometimes D. Blood pressure 140/80
awakes her at night. She reports that when she eats, Answers and Rationales
it helps the pain go away but that pain is now
becoming more intense. Which of the following is 1. C. The relationship of symptoms and a specific
the best condition for the nurse to draw: trigger such as physical exercise. COPD clients
have low oxygen and high carbon dioxide
A. these symptoms are consistent with an ulcer levels. Therefore, hypoxia is the main stimulus
B. The client probably has indigestion for ventilation is persons with chronic
C. A snack before going to bed should be advised hypercapnea. Increasing the level of oxygen
D. The client probably developing cholelithiasis would decrease the stimulus to breathe.
21. Nurse Cynthia is providing a discharge teaching 2. D. persistent changing cough. The most
to a client with chronic cirrhosis. His wife asks her common sign of lung cancer is a persistent
to explain why there is so much emphasis on cough that changes. Other signs are dyspnea,
bleeding precautions. Which of the following bloody sputum and long term pulmonary
provides the most appropriate response? infection. Option A is common with asthma,
option B is common with COPD and option C
A. “The low protein diet will result in reduced is common with TB.
clotting.” 3. C. ischemia. Depressed ST segment and
B. “The increased production of bile decreases inverted T-waves represent myocardial
clotting factors.” ischemia. Injury has a ST segment elevation.
C. “The liver affected by cirrhosis is unable to 4. A. promote healing. Sitz bath provides moist
produce clotting factors.” heat to the perineal and anal area to clean,
promote healing and drainage and reduce associated with generalized weakness; a
soreness to the area. Sitz bath helps healing shuffling gait is usually not observed in stroke
with cleaning action and promotion of patient.
circulation, thereby reducing swelling. Sitz 15. C. a ventilator on stand by. Although a
bath usually has no therapeutic value in ventilator is not required for injury below C3,
lowering body temperature. Although relief of the innervation of intercostal muscles is
tension can occur, this effect is secondary to affected. Hemorrhage and cord swelling
the promotion of healing. extends the level of injury making it likely that
5. B. hypocalcemia. Trousseau’s sign is a carpal this client will need a ventilator.
pedal spasm elicited when a blood pressure 16. B. protein deficiency. Protein deficiency allows
cuff is inflated on the arm of a patient with fluid to leak out of the vascular system and
hypocalcemia. third space into the tissues and spaces in the
6. C. meningitis. A patient with meningitis will body such as the peritoneal space. Bleeding
exhibit signs that include photophobia and tendencies, dehydration and vitamin deficiency
nuchal rigidity, which is pain on the flexion of can occur but don’t cause ascites.
the chin to chest. 17. D. Swan neck deformities. Swan neck
7. C. female over 55 years of age. Lifestyle, deformities of the hand are classic deformities
cigarette smoking and hyperlipidemia can be associated with rheumatoid arthritis secondary
changed by changing behaviors. to the presence of fibrous connective tissue
8. D. Aneurysm. The symptoms exhibited by the within the joint space. Clients with RA do
client are typical of an abdominal aortic experience morning stiffness, but it can last
aneurysm. The most significant sign is the from 30 minutes up to several hours. RA is
audible pulse in the abdominal area. If characterized by symmetrical joint movement,
hemorrhage were present, the abdomen would and heberden’s nodes are characteristic of
be tender and firm. osteoarthritis.
9. B. Relaxing smooth muscle to avoid 18. C. an aura. An aura frequently precedes an
vasospasms. The major task of the health care epileptic seizure and may manifest as vague
team is to medicate the client drugs that physic discomfort or specific aromas. Patients
produce smooth muscle relaxation, which will experiencing auras aren’t having a CVA,
decrease the vasospasm and increase the substance withdrawal or hallucination.
arterial flow to the affected part. The drugs 19. B. laryngeal cancer. These symptoms, along
used are calcium antagonists. with dysphagia, foul-smelling breath, and pain
10. D. urinary catheter patency. The patient is when drinking hot or acidic, are common signs
complaining of symptoms of autonomic of laryngeal cancer.
dysreflexia, which consists of the triad of 20. A. these symptoms are consistent with an
hypertension, bradycardia and a headache. ulcer. The description of pain is consistent with
Major causes of autonomic dysreflexia include ulcer pain. The pain is epigastric and is worse
urinary bladder distention and fecal impaction. when the stomach is empty and is relived by
Checking the patency of the urinary catheter food.
will check for bladder distention. 21. C. “The liver affected by cirrhosis is unable to
11. C. Huntington’s disease. Huntington’s disease produce clotting factors.” When bile
is a hereditary disease in which degeneration of production is reduced, the body has reduced
the basal ganglia and cerebral cortex causes ability to absorb fat-soluble vitamins. Without
chronic progressive chorea (muscle twitching) adequate Vitamin K absorption, clotting factors
and mental deterioration, ending in dementia. II, VII, IX, and X are not produced in sufficient
Huntington’s disease usually strikes people amounts.
ages 25 to 55. 22. A. excruciating, intermittent, paroxysmal facial
12. D. metabolic alkalosis. Ms. Hizon’s pH is pain. Trigeminal neuralgia is a syndrome of
above 7.45, which makes it alkalatic, and her excruciating, intermittent, paroxysmal facial
bicarbonate is high which is also makes it basic. pain. It manifests as intense, periodic pain in
Thus, the diagnosis is metabolic alkalosis. the lips, gums, teeth or chin. The other
13. B. hold an object. The resting or non- symptoms aren’t characteristic of trigeminal
intentional tremor may be controlled with neuralgia.
purposeful movement such as holding an 23. D. Assess for respiratory distress. Though fluid
object. A warm bath, deep breathing and volume status, neurological status and pain are
diazepam will promote relaxation but are not all important assessment, the immediate
specific interventions for tremor. priority for postoperative is the airway
14. A. Parkinson’s disease. A shuffling gait from management. Respiratory distress may result
the musculoskeletal rigidity of the patient with from hemorrhage, edema, laryngeal damage or
Parkinson’s disease is common. Patients tetany.
experiencing a stroke usually exhibit loss of 24. B. Emollients and moisturizers to soften
voluntary control over motor movements scales. Emollients will ease dry skin that
increases pruritus and causes psoriasis to be A. away from the body
worse. Washing and drying the skin with rough B. to the left of the body
linens or pressure may cause excoriation. C. to the right of the body
Constant occlusion may increase the effects of D. toward the body
the medication and increase the risk of 5. Assessment of a client with possible
infection. thrombophlebitis to the left leg and a deep vein
25. C. Pulse 40 with inspiration. Paradoxical pulse thrombosis is done by pulling up on the toes while
is a hallmark symptom of cardiac tamponade. gently holding down on the knee. The client
As pressure is exerted on the left ventricle from complains of extreme pain in the calf. This should
fluid, the natural increase in pressure from the be documented as:
right ventricle during inspiration creates even
more pressure, diminishing cardiac output. A. positive tourniquet test
B. positive homan’s sign
PNLE Medical C. negative homan’s sign
D. negative tourniquet test
6. Thomas Elison is a 79 year old man who is
Surgical Nursing admitted with diagnosis of dementia. The doctor
orders a series of laboratory tests to determine
whether Mr. Elison’s dementia is treatable. The
Exam 4 nurse understands that the most common cause of
dementia in this population is:

A. AIDS
1. A patient is admitted to the medical surgical unit B. Alzheimer’s disease
following surgery. Four days after surgery, the C. Brain tumors
patient spikes a 38.9 degrees C oral temperature and D. Vascular disease
exhibits a wet, productive cough. The nurse 7. Which of the following nursing interventions is
assesses the patient with understanding that an contraindicated in the care of a client with acute
infection that is acquired during hospitalization is osteomyelitis?
known as:
A. Apply heat compress to the affected area
A. a community acquired infection B. Immobilize the affected area
B. an iatrogenic infection C. Administer narcotic analgesics for pain
C. a nosocomial infection D. Administer OTC analgesics for pain
D. an opportunistic infection 8. A client with congestive heart failure has digoxin
2. A client with anemia has a hemoglobin of 6.5 (Lanoxin) ordered everyday. Prior to giving the
g/dL. The client is experiencing symptoms of medication, the nurse checks the digoxin level
cerebral tissue hypoxia. Which of the following which is therapeutic and ausculates an apical pulse.
nursing interventions would be most important in The apical pulse is 63 bpm for 1 full minute. The
providing care? nurse should:

A. Providing rest periods throughout the day A. Hold the Lanoxin


B. Instituting energy conservation techniques B. Give the half dose now, wait an hour and give
C. Assisting in ambulation to the bathroom the other half
D. Checking temperature of water prior to bathing C. Call the physician
3. A client was involved in a motor vehicular D. Give the Lanoxin as ordered
accident in which the seat belt was not worn. The 9. Nurse Marian is caring for a client with haital
client is exhibiting crepitus, decrease breath sounds hernia, which of the following should be included in
on the left, complains of shortness of breath, and her teaching plan regarding causes:
has a respiratory rate of 34 breaths per minute.
Which of the following assessment findings would A. To avoid heavy lifting
concern the nurse most? B. A dietary plan based on soft foods
C. Its prevalence in young adults
A. Temperature of 102 degrees F and productive D. Its prevalence in fair-skinned individuals
cough 10. Joseph has been diagnosed with hepatic
B. ABG with PaO2 of 92 and PaCO2 of 40 encephalopathy. The nurse observes flapping
mmHg tremors. The nurse understands that flapping
C. Trachea deviating to the right tremors associated with hepatic encephalopathy are
D. Barrel-chested appearance also known as:
4. The proper way to open an envelop-wrapped
sterile package after removing the outer package or A. aphasia
tape is to open the first position of the wrapper: B. ascites
C. astacia A. Encourage him to maintain bed rest for several
D. asterixis days
11. Hyperkalemia can be treated with B. Monitor temperature every 4 hours
administration of 50% dextrose and insulin. The 50% C. Instruct him to avoid sexual contact during
dextrose: acute phases of illness
D. Encourage him to use antifungal agents
A. causes potassium to be excreted regularly
B. causes potassium to move into the cell 19. An 8 year old boy is brought to the trauma unit
C. causes potassium to move into the serum with a chemical burn to the face. Priority
D. counteracts the effects of insulin assessment would include which of the following?
12. Which of the following findings would strongly
indicate the possibility of cirrhosis? A. Skin integrity
B. BP and pulse
A. dry skin C. Patency of airway
B. hepatomegaly D. Amount of pain
C. peripheral edema 20. A client with anemia due to chemotherapy has a
D. pruritus hemoglobin of 7.0 g/dL. Which of the following
13. Aling Puring has just been diagnosed with complaints would be indicative of tissue hypoxia
close-angle (narrow-angle) glaucoma. The nurse related to anemia?
assesses the client for which of the following
common presenting symptoms of the disorder? A. dizziness
B. fatigue relieved by rest
A. halo vision C. skin that is warm and dry to the touch
B. dull eye pain D. apathy
C. severe eye and face pain 21. Hazel Murray, 32 years old complains of abrupt
D. impaired night vision onset of chest and back pain and loss of radial
14. Chvostek’s sign is associated with which pulses. The nurse suspects that Mrs. Murray may
electrolyte impabalnce? have:

A. hypoclacemia A. Acute MI
B. hypokalemia B. CVA
C. hyponatremia C. Dissecting abdominal aorta
D. hypophosphatenia D. Dissecting thoracic aneurysm
15. What laboratory test is a common measure of 22. Nurse Alexandra is establishing a plan of care
the renal function? for a client newly admitted with SIADH. The
priority diagnosis for this client would be which of
A. CBC the following?
B. BUN/Crea
C. Glucose A. Fluid volume deficit
D. Alanine amino transferase (ALT) B. Anxiety related to disease process
16. Nurse Edward is performing discharge teaching C. Fluid volume excess
for a newly diagnosed diabetic patient scheduled for D. Risk for injury
a fasting blood glucose test. The nurse explains to 23. Nursing management of the client with a UTI
the patient that hyperglycemia is defined as a blood should include:
glucose level above:
A. Taking medication until feeling better
A. 100 mg/dl B. Restricting fluids
B. 120 mg/dl C. Decreasing caffeine drinks and alcohol
C. 130 mg/dl D. Douching daily
D. 150 mg/dl 24. Felicia Gomez is 1 day postoperative from
17. Mang Edison is on bed rest has developed an coronary artery bypass surgery. The nurse
ulcer that is full thickness and is penetrating the understands that a postoperative patient who’s
subcutaneous tissue. The nurse documents that this maintained on bed rest is at high risk for developing:
ulcer is in which of the following stages?
A. angina
A. Stage 1 B. arterial bleeding
B. Stage 2 C. deep vein thrombosis (DVT)
C. Stage 3 D. dehiscence of the wound
D. Stage 4 25. Which of the following statement is true
18. A 24 year old male patient comes to the clinic regarding the visual changes associated with
after contracting genital herpes. Which of the cataracts?
following intervention would be most appropriate?
A. Both eyes typically cataracts at the same time diagnosed with osteomyelitis. The application
B. The loss of vision is experienced as a painless, of heat can increase edema and pain in the
gradual blurring affected area and spread bacteria through
C. The patient is suddenly blind vasodilation.
D. The patient is typically experiences a painful, 8. D. Give the Lanoxin as ordered. The Lanoxin
sudden blurring of vision. should be held for a pulse of 60 bpm. Nurses
Answers and Rationales cannot arbitrarily give half of a dose without a
physician’s order. Unless specific parameters
1. C. a nosocomial infection. Nosocomial, or are given concerning pulse rate, most resources
hospital-acquired are infections acquired identify 60 as the reference pulse.
during hospitalization for which the patient 9. A. To avoid heavy lifting. Heavy lifting is one
isn’t being primarily treated. Community factor that leads to development of a hiatal
acquired or opportunistic infections may not be hernia. Dietary factors involve limiting fat
acquired during hospitalization. An iatrogenic intake, not restricting client to soft foods. It is
infection is caused by the doctor or by medical more prevalent in individuals who are middle-
therapy. And an opportunistic infection affects aged or older. Fair-skinned individuals are not
a compromised host. prone to this condition.
2. C. Assisting in ambulation to the 10. D. asterixis. Flapping tremors associated with
bathroom. Cerebral tissue hypoxia is hepatic encephalophaty are asterixis. Aphasia
commonly associated with dizziness. The is the inability to speak. Ascites is an
greatest potential risk to the client with accumulation of fluid in the peritoneal cavity.
dizziness is injury, especially with changes in Astacia is the inability to stand or sit still.
position. Planning for periods of rest and 11. D. counteracts the effects of insulin. The 50%
conserving energy are important with someone dextrose is given to counteract the effects of
with anemia because of his or her fatigue level insulin. Insulin drives the potassium into the
but most important is safety. cell, thereby lowering the serum potassium
3. C. Trachea deviating to the right. A mediastinal levels. The dextrose doesn’t directly cause
shift is indicative of a tension pneumothorax potassium excretion or any movement of
along with the other symptoms in the question. potassium.
Since the individual was involved in a MVA, 12. B. hepatomegaly. Although option D is correct,
assessment would be targeted at acute it is not a strong indicator of cirrhosis. Pruritus
traumatic injuries to the lungs, heart or chest can occur for many reasons. Options A and C
wall rather than other conditions indicated in are incorrect, fluid accumulations is usually in
the other answers. Option A is common with the form of ascites in the abdomen.
pneumonia; values in option B are not alarming; Hepatomegaly is an enlarged liver, which is
and option D is typical of someone with COPD. correct. The spleen may also be enlarged.
4. A. away from the body. When opening an 13. C. severe eye and face pain. Narrow-angle
envelop-wrapped sterile package, reaching glaucoma develops abruptly and manifests with
across the package and using the first motion to acute face and eye pain and is a medial
open the top cover away from the body emergency. Halo vision, dull eye pain and
eliminates the need to later reach across the impaired night vision are symptoms associated
steri9le field while opening the package. To with open-angle glaucoma.
remove equipment from the package, opening 14. A. hypoclacemia. Chvostek’s sign is a spasm
the first portion of the package toward, to the of the facial muscles elicited by tapping the
left, or to the right of the body would require facial nerve and is associated with
reaching across a sterile field. hypocalcemia. Clinical signs of hypokalemia
5. B. positive homan’s sign. Pain in the calf while are muscle weakness, leg cramps, fatigue,
pulling up on the toes is abnormal and nausea and vomiting. Muscle cramps, anorexia,
indicates a positive test. If the client feels nausea and vomiting are clinical signs of
nothing or just feels like the calf muscle is hyponatremia. Clinical manifestations
stretching, it is considered negative. A associated with hypophosphatemia include
tourniquet test is used to measure for varicose muscle pain, confusion, seizures and coma.
veins. 15. B. BUN/Crea. The BUN is primarily used as
6. B. Alzheimer’s disease. Alzheimer’s disease is indicator of kidney function because most renal
the most common cause of dementia in the diseases interfere with its excretion and cause
elderly population. AIDS, brain tumors and blood vessels to rise. Creatinine is produced in
vascular disease are all less common causes of relatively constant amounts, according to the
progressive loss of mental function in elderly amount of muscle mass and is excreted entirely
patients. by the kidneys making it a good indicator of
7. A. Apply heat compress to the affected renal function.
area. Options B, C and D are appropriate 16. B. 120 mg/dl. Hyperglycemia is defined as a
nursing interventions when caring for a client blood glucose level greater than 120 mg/dl.
Blood glucose levels of 120 mg/dl, 130 mg/dl
and 150 mg/dl are considered hyperglycemic.
A blood glucose of 100 mg/dl is normal.
17. C. Stage 3. A stage 3 ulcer is full thickness
involving the subcutaneous tissue. A stage 1
ulcer has a defined area of persistent redness in
lightly pigmented skin. A stage 2 ulcer
involves partial thickness skin loss. Stage 4
ulcers extend through the skin and exhibit
tissue necrosis and muscle or bone
involvement.
18. C. Instruct him to avoid sexual contact during
acute phases of illness. Herpes is a virus and is
spread through direct contact. An antifungal
would not be useful; bed rest and temperature
measurement are usually not necessary.
19. C. Patency of airway. A burn face, neck or
chest may cause airway closure because of the
edema that occurs within hours. Remember the
ABC’s: airway, breathing and circulation.
Airway always comes first, even before pain.
The nurse will also assess options B and D, but
these are not the highest priority assessments.
20. A. dizziness. Central tissue hypoxia is
commonly associated with dizziness.
Recognition of cerebral hypoxia is critical
since the body will attempt to shunt
oxygenated blood to vital organs.
21. D. Dissecting thoracic aneurysm. A dissecting
thoracic aneurysm may cause loss of radical
pulses and severe chest and back pain. An MI
typically doesn’t cause loss of radial pulses or
severe back pain. CVA and dissecting
abdominal aneurysm are incorrect responses.
22. C. Fluid volume excess. SIADH results in fluid
retention and hyponatremia. Correction is
aimed at restoring fluid and electrolyte balance.
Anxiety and risk for injury should be addressed
following fluid volume excess.
23. C. Decreasing caffeine drinks and
alcohol. Caffeine and alcohol can increase
PNLE Exam 1
bladder spasms and mucosal irritation, thus 1. A 10 year old who has sustained a head injury is
increase the signs and symptoms of UTI. All brought to the emergency department by his mother.
antibiotics should be taken completely to A diagnosis of a mild concussion is made. At the
prevent resistant strains of organisms. time of discharge, nurse Ron should instruct the
24. C. deep vein thrombosis (DVT). DVT, is the mother to:
most probable complication for postoperative
patients on bed rest. Options A, B and D aren’t A. Withhold food and fluids for 24 hours.
likely complications of the post operative B. Allow him to play outdoors with his friends.
period. C. Arrange for a follow up visit with the child’s
25. B. The loss of vision is experienced as a primary care provider in one week.
painless, gradual blurring. Typically, a patient D. Check for any change in responsiveness every
with cataracts experiences painless, gradual two hours until the follow-up visit.
loss of vision. Although both eyes may develop 2. A male client has suffered a motor accident and
at different rates. is now suffering from hypovolemic shock. Nurse
Helen should frequency assess the client’s vital
signs during the compensatory stage of shock,
because:

A. Arteriolar constriction occurs


B. The cardiac workload decreases
C. Decreased contractility of the heart occurs
D. The parasympathetic nervous system is A. Eating patterns are altered
triggered B. Fats are limited in the diet
3. A paranoid male client with schizophrenia is C. Carbohydrates are regulated
losing weight, reluctant to eat, and voicing concerns D. Exercise is a major component
about being poisoned. The best intervention by 9. As a very anxious female client is talking to the
nurse Dina would be to: nurse May, she starts crying. She appears to be
upset that she cannot control her crying. The most
A. Allow the client to open canned or pre- appropriate response by the nurse would be:
packaged food
B. Restrict the client to his room until 2 lbs are A. “Is talking about your problem upsetting you?”
gained B. “It is Ok to cry; I’ll just stay with you for now”
C. Have a staff member personally taste all of the C. “You look upset; lets talk about why you are
client’s food crying.”
D. Tell the client the food has been x-rayed by the D. “Sometimes it helps to get it out of your
staff and is safe system.”
4. One day the mother of a young adult confides to 10. A patient has partial-thickness burns to both
nurse Frida that she is very troubled by he child’s legs and portions of his trunk. Which of the
emotional illness. The nurse’s most therapeutic following I.V. fluids is given first?
initial response would be:
A. Albumin
A. “You may be able to lessen your feelings of B. D5W
guilt by seeking counseling” C. Lactated Ringer’s solution
B. “It would be helpful if you become involved in D. 0.9% sodium chloride solution with 2 mEq of
volunteer work at this time” potassium per 100 ml
C. “I recognize it’s hard to deal with this, but try 11. During the first 48 hours after a severe burn of
to remember that this too shall pass” 40% of the clients body surface, the nurse’s
D. “Joining a support group of parents who are assessment should include observations for water
coping with this problem can be quite helpful. intoxication. Associated adaptations include:
5. To check for wound hemorrhage after a client has
had a surgery for the removal of a tumor in the neck, A. Sooty-colored sputum
nurse grace should: B. Frothy pink-tinged sputum
C. Twitching and disorientation
A. Loosen an edge of the dressing and lift it to see D. Urine output below 30ml per hour
the wound 12. After a muscle biopsy, nurse Willy should teach
B. Observe the dressing at the back of the neck for the client to:
the presence of blood
C. Outline the blood as it appears on the dressing A. Change the dressing as needed
to observe any progression B. Resume the usual diet as soon as desired
D. Press gently around the incision to express C. Bathe or shower according to preference
accumulated blood from the wound D. Expect a rise in body temperature for 48 hours
6. A 16-year-old primigravida arrives at the labor 13. Before a client whose left hand has been
and birthing unit in her 38th week of gestation and amputated can be fitted for a prosthesis, nurse Joy is
states that she is labor. To verify that the client is in aware that:
true labor nurse Trina should:
A. Arm and shoulder muscles must be developed
A. Obtain sides for a fern test B. Shrinkage of the residual limb must be
B. Time any uterine contractions completed
C. Prepare her for a pelvic examination C. Dexterity in the other extremity must be
D. Apply nitrazine paper to moist vaginal tissue achieved
7. As part of the diagnostic workup for pulmonic D. Full adjustment to the altered body image must
stenosis, a child has cardiac catheterization. Nurse have occurred
Julius is aware that children with pulmonic stenosis 14. Nurse Cathy applies a fetal monitor to the
have increased pressure: abdomen of a client in active labor. When the client
has contractions, the nurse notes a 15 beat per
A. In the pulmonary vein minute deceleration of the fetal heart rate below the
B. In the pulmonary artery baseline lasting 15 seconds. Nurse Cathy should:
C. On the left side of the heart
D. On the right side of the heart A. Change the maternal position
8. An obese client asks nurse Julius how to lose B. Prepare for an immediate birth
weight. Before answering, the nurse should C. Call the physician immediately
remember that long-term weight loss occurs best D. Obtain the client’s blood pressure
when:
15. A male client receiving prolonged steroid demonstrates multiple rib fraactures, resulting in a
therapy complains of always being thirsty and flail chest. The complication the nurse should
urinating frequently. The best initial action by the carefully observe for would be:
nurse would be to:
A. Mediastinal shift
A. Perform a finger stick to test the client’s blood B. Tracheal laceration
glucose level C. Open pneumothorax
B. Have the physician assess the client for an D. Pericardial tamponade
enlarged prostate 22. When planning care for a client at 30-weeks
C. Obtain a urine specimen from the client for gestation, admitted to the hospital after vaginal
screening purposes bleeding secondary to placenta previa, the nurse’s
D. Assess the client’s lower extremities for the primary objective would be:
presence of pitting edema
16. Nurse Bea recognizes that a pacemaker is A. Provide a calm, quiet environment
indicated when a client is experiencing: B. Prepare the client for an immediate cesarean
birth
A. Angina C. Prevent situations that may stimulate the cervix
B. Chest pain or uterus
C. Heart block D. Ensure that the client has regular cervical
D. Tachycardia examinations assess for labor
17. When administering pancrelipase (Pancreases 23. When planning discharge teaching for a young
capsules) to child with cystic fibrosis, nurse Faith female client who has had a pneumothorax, it is
knows they should be given: important that the nurse include the signs and
symptoms of a pneumothorax and teach the client to
A. With meals and snacks seek medical assistance if she experiences:
B. Every three hours while awake
C. On awakening, following meals, and at A. Substernal chest pain
bedtime B. Episodes of palpitation
D. After each bowel movement and after postural C. Severe shortness of breath
draianage D. Dizziness when standing up
18. A preterm neonate is receiving oxygen by an 24. After a laryngectomy, the most important
overhead hood. During the time the infant is under equipment to place at the client’s bedside would be:
the hood, it would be appropriate for nurse Gian to:
A. Suction equipment
A. Hydrate the infant q15 min B. Humidified oxygen
B. Put a hat on the infant’s head C. A nonelectric call bell
C. Keep the oxygen concentration consistent D. A cold-stream vaporizer
D. Remove the infant q15 min for stimulation 25. Nurse Oliver interviews a young female client
19. A client’s sputum smears for acid fast bacilli with anorexia nervosa to obtain information for the
(AFB) are positive, and transmission-based airborne nursing history. The client’s history is likely to
precautions are ordered. Nurse Kyle should instruct reveal a:
visitors to:
A. Strong desire to improve her body image
A. Limit contact with non-exposed family B. Close, supportive mother-daughter relationship
members C. Satisfaction with and desire to maintain her
B. Avoid contact with any objects present in the present weight
client’s room D. Low level of achievement in school, with little
C. Wear an Ultra-Filter mask when they are in the concerns for grades
client’s room 26. Nurse Bea should plan to assist a client with an
D. Put on a gown and gloves before going into the obsessive-compulsive disorder to control the use of
client’s room ritualistic behavior by:
20. A client with a head injury has a fixed, dilated A. Providing repetitive activities that require little
right pupil; responds only to painful stimuli; and thought
exhibits decorticate posturing. Nurse Kate should B. Attempting to reduce or limit situations that
recognize that these are signs of: increase anxiety
C. Getting the client involved with activities that
A. Meningeal irritation will provide distraction
B. Subdural hemorrhage D. Suggesting that the client perform menial tasks
C. Medullary compression to expiate feelings of guilt
D. Cerebral cortex compression 27. A 2 ½ year old child undergoes a
21. After a lateral crushing chest injury, obvious ventriculoperitoneal shunt revision. Before
right-sided paradoxic motion of the client’s chest discharge, nurse John, knowing the expected
developmental behaviors for this age group, should 34. Nurse Wilma recognizes that failure of a
tell the parents to call the physician if the child: newborn to make the appropriate adaptation to
extrauterine life would be indicated by:
A. Tries to copy all the father’s mannerisms
B. Talks incessantly regardless of the presence of A. flexed extremities
others B. Cyanotic lips and face
C. Becomes fussy when frustrated and displays a C. A heart rate of 130 beats per minute
shortened attention span D. A respiratory rate of 40 breath per minute
D. Frequently starts arguments with playmates by 35. The laboratory calls to state that a client’s
claiming all toys are “mine” lithium level is 1.9 mEq/L after 10 days of lithium
28. A urinary tract infection is a potential danger therapy. Nurse Reese should:
with an indwelling catheter. Nurse Gina can best
plan to avoid this complication by: A. Notify the physician of the findings because
the level is dangerously high
A. Assessing urine specific gravity B. Monitor the client closely because the level of
B. Maintaining the ordered hydration lithium in the blood is slightly elevated
C. Collecting a weekly urine specimen C. Continue to administer the medication as
D. Emptying the drainage bag frequently ordered because the level is within the
29. A client has sustained a fractured right femur in therapeutic range
a fall on stairs. Nurse Troy with the emergency D. Report the findings to the physician so the
response team assess for signs of circulatory dosage can be increased because the level is
impairment by: below therapeutic range
36. A client has a regular 30-day menstrual cycles.
A. Turning the client to side lying position When teaching about the rhythm method, Which the
B. Asking the client to cough and deep breathe client and her husband have chosen to use for
C. Taking the client’s pedal pulse in the affected family planning, nurse Dianne should emphasize
limb that the client’s most fertile days are:
D. Instructing the client to wiggle the toes of the
right foot A. Days 9 to 11
30. To assess orientation to place in a client B. Days 12 to 14
suspected of having dementia of the alzheimers type, C. Days 15 to 17
nurse Chris should ask: D. Days 18 to 20
37. Before an amniocentesis, nurse Alexandra
A. “Where are you?” should:
B. “Who brought you here?”
C. “Do you know where you are?” A. Initiate the intravenous therapy as ordered by
D. “How long have you been there?” the physiscian
31. Nurse Mary assesses a postpartum client who B. Inform the client that the procedure could
had an abruption placentae and suspects that precipitate an infection
disseminated intravascular coagulation (DIC) is C. Assure that informed consent has been
occurring when assessments demonstrate: obtained from the client
D. Perform a vaginal examination on the client to
A. A boggy uterus assess cervical dilation
B. Multiple vaginal clots 38. While a client is on intravenous magnesium
C. Hypotension and tachycardia sulfate therapy for preeclampsia, it is essential for
D. Bleeding from the venipuncture site nurse Amy to monitor the client’s deep tendon
32. When a client on labor experiences the urge to reflexes to:
push a 9cm dilation, the breathing pattern that nurse
Rhea should instruct the client to use is the: A. Determine her level of consciousness
B. Evaluate the mobility of the extremities
A. Expulsion pattern C. Determine her response to painful stimuli
B. Slow paced pattern D. Prevent development of respiratory distress
C. Shallow chest pattern 39. A preschooler is admitted to the hospital with a
D. blowing pattern diagnosis of acute glomerulonephritis. The child’s
33. Nurse Ronald should explain that the most history reveals a 5-pound weight gain in one week
beneficial between-meal snack for a client who is and peritoneal edema. For the most accurate
recovering from the full-thickness burns would be a: information on the status of the child’s edema,
nursing intervention should include:
A. Cheeseburger and a malted
B. Piece of blueberry pie and milk A. Obtaining the child’s daily weight
C. Bacon and tomato sandwich and tea B. Doing a visual inspection of the child
D. Chicken salad sandwich and soft drink C. Measuring the child’s intake and output
D. Monitoring the child’s electrolyte values 47. When assessing a newborn suspected of having
40. Nurse Mickey is administering dexamethasome Down syndrome, nurse Rey would expect to
(Decadron) for the early management of a client’s observe:
cerebral edema. This treatment is effective because:
A. long thin fingers
A. Acts as hyperosmotic diuretic B. Large, protruding ears
B. Increases tissue resistance to infection C. Hypertonic neck muscles
C. Reduces the inflammatory response of tissues D. Simian lines on the hands
D. Decreases the information of cerebrospinal 48. A 10 year old girl is admitted to the pediatric
fluid unit for recurrent pain and swelling of her joints,
41. During newborn nursing assessment, a positive particularly her knees and ankles. Her diagnosis is
Ortolani’s sign would be indicated by: juvenile rheumatoid arthritis. Nurse Janah
recognizes that besides joint inflammation, a unique
A. A unilateral droop of hip manifestation of the rheumatoid process involves
B. A broadening of the perineum the:
C. An apparent shortening of one leg
D. An audible click on hip manipulation A. Ears
42. When caring for a dying client who is in the B. Eyes
denial stage of grief, the best nursing approach C. Liver
would be to: D. Brain
49. A disturbed client is scheduled to begin group
A. Agree and encourage the client’s denial therapy. The client refuses to attend. Nurse Lolit
B. Allow the denial but be available to discuss should:
death
C. Reassure the client that everything will be OK A. Accept the client’s decision without discussion
D. Leave the client alone to confront the feelings B. Have another client to ask the client to consider
of impending loss C. Tell the client that attendance at the meeting is
43. To decrease the symptoms of gastroesophageal required
reflux disease (GERD), the physician orders dietary D. Insist that the client join the group to help the
and medication management. Nurse Helen should socialization process
teach the client that the meal alteration that would 50. Because a severely depressed client has not
be most appropriate would be: responded to any of the antidepressant medications,
the psychiatrist decides to try electroconvulsive
A. Ingest foods while they are hot therapy (ECT). Before the treatment the nurse
B. Divide food into four to six meals a day should:
C. Eat the last of three meals daily by 8pm
D. Suck a peppermint candy after each meal A. Have the client speak with other clients
44. After a mastectomy or hysterectomy, clients receiving ECT
may feel incomplete as women. The statement that B. Give the client a detailed explanation of the
should alert nurse Gina to this feeling would be: entire procedure
C. Limit the client’s intake to a light breakfast on
A. “I can’t wait to see all my friends again” the days of the treatment
B. “I feel washed out; there isn’t much left” D. Provide a simple explanation of the procedure
C. “I can’t wait to get home to see my grandchild” and continue to reassure the client
D. “My husband plans for me to recuperate at our 51. Nurse Vicky is aware that teaching about
daughter’s home” colostomy care is understood when the client states,
45. A client with obstruction of the common bile “I will contact my physician and report ____”:
duct may show a prolonged bleeding and clotting
time because: A. If I notice a loss of sensation to touch in the
stoma tissue”
A. Vitamin K is not absorbed B. When mucus is passed from the stoma between
B. The ionized calcium levels falls irrigations”
C. The extrinsic factor is not absorbed C. The expulsion of flatus while the irrigating
D. Bilirubin accumulates in the plasma fluid is running out”
46. Realizing that the hypokalemia is a side effect D. If I have difficulty in inserting the irrigating
of steroid therapy, nurse Monette should monitor a tube into the stoma”
client taking steroid medication for: 52. The client’s history that alerts nurse Henry to
assess closely for signs of postpartum infection
A. Hyperactive reflexes would be:
B. An increased pulse rate
C. Nausea, vomiting, and diarrhea A. Three spontaneous abortions
D. Leg weakness with muscle cramps B. negative maternal blood type
C. Blood loss of 850 ml after a vaginal birth A. Sit alone, display pincer grasp, wave bye bye
D. Maternal temperature of 99.9° F 12 hours after B. Pull self to a standing position, release a toy by
delivery choice, play peek-a-boo
53. A client is experiencing stomatitis as a result of C. Crawl, transfer toy from one hand to the other,
chemotherapy. An appropriate nursing intervention display of fear of strangers
related to this condition would be to: D. Turn completely over, sit momentarily without
support, reach to be picked up
A. Provide frequent saline mouthwashes 59. A breastfeeding mother asks the nurse what she
B. Use karaya powder to decrease irritation can do to ease the discomfort caused by a cracked
C. Increase fluid intake to compensate for the nipple. Nurse Tina should instruct the client to:
diarrhea
D. Provide meticulous skin care of the abdomen A. Manually express milk and feed it to the baby
with Betadine in a bottle
54. During a group therapy session, one of the B. Stop breastfeeding for two days to allow the
clients ask a male client with the diagnosis of nipple to heal
antisocial personality disorder why he is in the C. Use a breast shield to keep the baby from direct
hospital. Considering this client’s type of contact with the nipple
personality disorder, the nurse might expect him to D. Feed the baby on the unaffected breast first
respond: until the affected breast heals
60. Nurse Sandy observes that there is blood
A. “I need a lot of help with my troubles” coming from the client’s ear after head injury.
B. “Society makes people react in old ways” Nurse Sandy should:
C. “I decided that it’s time I own up to my
problems” A. Turn the client to the unaffected side
D. “My life needs straightening out and this might B. Cleanse the client’s ear with sterile gauze
help” C. Test the drainage from the client’s ear with
55. A child visits the clinic for a 6-week checkup Dextrostix
after a tonsillectomy and adenoidectomy. In D. Place sterile cotton loosely in the external ear
addition to assessing hearing, the nurse should of the client
include an assessment of the child’s: 61. Nurse Gio plans a long term care for parents of
children with sickle-cell anemia, which includes
A. Taste and smell periodic group conferences. Some of the
B. Taste and speech discussions should be directed towards:
C. Swallowing and smell
D. Swallowing and speech A. Finding special school facilities for the child
56. A client is diagnosed with cancer of the jaw. A B. Making plans for moving to a more therapeutic
course of radiation therapy is to be followed by climate
surgery. The client is concerned about the side C. Choosing a means of birth control to avoid
effects related to the radiation treaments. Nurse Ria future pregnancies
should explain that the major side effects that will D. Airing their feelings regarding the transmission
experienced is: of the disease to the child
62. The central problem the nurse might face with a
A. Fatigue disturbed schizophrenic client is the client’s:
B. Alopecia
C. Vomiting A. Suspicious feelings
D. Leucopenia B. Continuous pacing
57. Nurse Katrina prepares an older-adult client for C. Relationship with the family
sleep, actions are taken to help reduce the likelihood D. Concern about working with others
of a fall during the night. Targeting the most 63. When planning care with a client during the
frequent cause of falls, the nurse should: postoperative recovery period following an
abdominal hysterectomy and bilateral salpingo-
A. Offer the client assistance to the bathroom oophorectomy, nurse Frida should include the
B. Move the bedside table closer to the client’s explanation that:
bed
C. Encourage the client to take an available A. Surgical menopause will occur
sedative B. Urinary retention is a common problem
D. Assist the client to telephone the spouse to say C. Weight gain is expected, and dietary plan are
“goodnight” needed
58. When evaluating a growth and development of a D. Depression is normal and should be expected
6 month old infant, nurse Patty would expect the 64. An adolescent client with anorexia nervosa
infant to be able to: refuses to eat, stating, “I’ll get too fat.” Nurse
Andrea can best respond to this behavior initially by:
A. Not talking about the fact that the client is not 70. When caring for the a client with a
eating pneumothorax, who has a chest tube in place, nurse
B. Stopping all of the client’s priviledges until Kate should plan to:
food is eaten
C. Telling the client that tube feeding will A. Administer cough suppressants at appropriate
eventually be necessary intervals as ordered
D. Pointing out to the client that death can occur B. Empty and measure the drainage in the
with malnutrition. collection chamber each shift
65. A pain scale is used to assess the degree of pain. C. Apply clamps below the insertion site when
The client rates the pain as an 8 on a scale of 10 ever getting the client out of bed
before medication and a 7 on a scale of 10 after D. Encourage coughing, deep breathing, and range
being medicated. Nurse Glenda determines that the: of motion to the arm on the affected side
71. According to C.E.Winslow, which of the
A. Client has a low pain tolerance following is the goal of Public Health?
B. Medication is not adequately effective
C. Medication has sufficiently decreased the pain A. For people to attain their birthrights of health
level and longevity
D. Client needs more education about the use of B. For promotion of health and prevention of
the pain scale disease
66. To enhance a neonate’s behavioral development, C. For people to have access to basic health
therapeutic nursing measures should include: services
D. For people to be organized in their health
A. Keeping the baby awake for longer periods of efforts
time before each feeding 72. What other statistic may be used to determine
B. Assisting the parents to stimulate their baby attainment of longevity?
through touch, sound, and sight.
C. Encouraging parental contact for at least one A. Age-specific mortality rate
15-minute period every four hours. B. Proportionate mortality rate
D. Touching and talking to the baby at least C. Swaroop’s index
hourly, beginning within two to four hours D. Case fatality rate
after birth 73. Which of the following is the most prominent
67. Before formulating a plan of care for a 6 year feature of public health nursing?
old boy with attention deficit hyperactivity disorder
(ADHD), nurse Kyla is aware that the initial aim of A. It involves providing home care to sick people
therapy is to help the client to: who are not confined in the hospital
B. Services are provided free of charge to people
A. Develop language skills within the catchment area.
B. Avoid his own regressive behavior C. The public health nurse functions as part of a
C. Mainstream into a regular class in school team providing a public health nursing services.
D. Recognize himself as an independent person of D. Public health nursing focuses on preventive,
worth not curative, services.
68. Nurse Wally knows that the most important 74. Which of the following is the mission of the
aspect of the preoperative care for a child with Department of Health?
Wilms’ tumor would be:
A. Health for all Filipinos
A. Checking the size of the child’s liver B. Ensure the accessibility and quality of health
B. Monitoring the child’s blood pressure care
C. Maintaining the child in a prone position C. Improve the general health status of the
D. Collecting the child’s urine for culture and population
sensitivity D. Health in the hands of the Filipino people by
69. At 11:00 pm the count of hydrocodone (Vicodin) the year 2020
is incorrect. After several minutes of searching the 75. Nurse Pauline determines whether resources
medication cart and medication administration were maximized in implementing Ligtas Tigdas,
records, no explanation can be found. The primary she is evaluating:
nurse should notify the:
A. Effectiveness
A. Nursing unit manager B. Efficiency
B. Hospital administrator C. Adequacy
C. Quality control manager D. Appropriateness
D. Physician ordering the medication 76. Lissa is a B.S.N. graduate. She want to become
a Public Health Nurse. Where will she apply?
A. Department of Health 83. Functional nursing has some advantages, which
B. Provincial Health Office one is an EXCEPTION?
C. Regional Health Office
D. Rural Health Unit A. Psychological and sociological needs are
77. As an epidemiologist, Nurse Celeste is emphasized.
responsible for reporting cases of notifiable diseases. B. Great control of work activities.
What law mandates reporting of cases of notifiable C. Most economical way of delivering nursing
diseases? services.
D. Workers feel secure in dependent role
A. Act 3573 84. Which of the following is the best guarantee that
B. R.A. 3753 the patient’s priority needs are met?
C. R.A. 1054
D. R.A. 1082 A. Checking with the relative of the patient
78. Nurse Fay is aware that isolation of a child with B. Preparing a nursing care plan in collaboration
measles belongs to what level of prevention? with the patient
C. Consulting with the physician
A. Primary D. Coordinating with other members of the team
B. Secondary 85. Nurse Tony stresses the need for all the
C. Intermediate employees to follow orders and instructions from
D. Tertiary him and not from anyone else. Which of the
79. Nurse Gina is aware that the following is an following principles does he refer to?
advantage of a home visit?
A. Scalar chain
A. It allows the nurse to provide nursing care to a B. Discipline
greater number of people. C. Unity of command
B. It provides an opportunity to do first hand D. Order
appraisal of the home situation. 86. Nurse Joey discusses the goal of the department.
C. It allows sharing of experiences among people Which of the following statements is a goal?
with similar health problems.
D. It develops the family’s initiative in providing A. Increase the patient satisfaction rate
for health needs of its members. B. Eliminate the incidence of delayed
80. The PHN bag is an important tool in providing administration of medications
nursing care during a home visit. The most C. Establish rapport with patients
important principle of bag technique states that it: D. Reduce response time to two minutes
87. Nurse Lou considers shifting to transformational
A. Should save time and effort. leadership. Which of the following statements best
B. Should minimize if not totally prevent the describes this type of leadership?
spread of infection.
C. Should not overshadow concern for the patient A. Uses visioning as the essence of leadership
and his family. B. Serves the followers rather than being served
D. May be done in a variety of ways depending on C. Maintains full trust and confidence in the
the home situation, etc. subordinates
81. Nurse Willy reads about Path Goal theory. D. Possesses innate charisma that makes others
Which of the following behaviors is manifested by feel good in his presence.
the leader who uses this theory? 88. Nurse Mae tells one of the staff, “I don’t have
time to discuss the matter with you now. See me in
A. Recognizes staff for going beyond expectations my office later” when the latter asks if they can talk
by giving them citations about an issue. Which of the following conflict
B. Challenges the staff to take individual resolution strategies did she use?
accountability for their own practice
C. Admonishes staff for being laggards A. Smoothing
D. Reminds staff about the sanctions for non B. Compromise
performance C. Avoidance
82. Nurse Cathy learns that some leaders are D. Restriction
transactional leaders. Which of the following does 89. Nurse Bea plans of assigning competent people
NOT characterize a transactional leader? to fill the roles designed in the hierarchy. Which
process refers to this?
A. Focuses on management tasks
B. Is a caretaker A. Staffing
C. Uses trade-offs to meet goals B. Scheduling
D. Inspires others with vision C. Recruitment
D. Induction
90. Nurse Linda tries to design an organizational to a hospital. Which of the following severe
structure that allows communication to flow in all conditions DOES NOT always require urgent
directions and involve workers in decision making. referral to a hospital?
Which form of organizational structure is this?
A. Mastoiditis
A. Centralized B. Severe dehydration
B. Decentralized C. Severe pneumonia
C. Matrix D. Severe febrile disease
D. Informal 98. A mother brought her daughter, 4 years old, to
91. When documenting information in a client’s the RHU because of cough and colds. Following the
medical record, the nurse should: IMCI assessment guide, which of the following is a
danger sign that indicates the need for urgent
A. erase any errors. referral to a hospital?
B. use a #2 pencil.
C. leave one line blank before each new entry. A. Inability to drink
D. end each entry with the nurse’s signature and B. High grade fever
title. C. Signs of severe dehydration
92. Which of the following factors are major D. Cough for more than 30 days
components of a client’s general background drug 99. Food fortification is one of the strategies to
history? prevent micronutrient deficiency conditions. R.A.
8976 mandates fortification of certain food items.
A. Allergies and socioeconomic status Which of the following is among these food items?
B. Urine output and allergies
C. Gastric reflex and age A. Sugar
D. Bowel habits and allergies B. Bread
93. Which procedure or practice requires surgical C. Margarine
asepsis? D. Filled milk
100. The major sign of iron deficiency anemia is
A. Hand washing pallor. What part is best examined for pallor?
B. Nasogastric tube irrigation
C. I.V. cannula insertion A. Palms
D. Colostomy irrigation B. Nailbeds
94. The nurse is performing wound care using C. Around the lips
surgical asepsis. Which of the following practices D. Lower conjunctival sac
violates surgical asepsis? Answers and Rationales
A. Holding sterile objects above the waist 1. C. Check for any change in responsiveness
B. Pouring solution onto a sterile field cloth every two hours until the follow-up visit. Signs
C. Considering a 1″ (2.5-cm) edge around the of an epidural hematoma in children usually do
sterile field contaminated not appear for 24 hours or more hours; a
D. Opening the outermost flap of a sterile package follow-up visit usually is arranged for one to
away from the body two days after the injury.
95. On admission, a client has the following arterial 2. A. Arteriolar constriction occurs.The early
blood gas (ABG) values: PaO2, 50 mm Hg; PaCO2, compensation of shock is cardiovascular and is
70 mm Hg; pH, 7.20; HCO3–, 28 mEq/L. Based on seen in changes in pulse, BP, and pulse
these values, pressure; blood is shunted to vital centers,
the nurse should formulate which nursing diagnosis particularly heart and brain.
for this client? 3. A. Allow the client to open canned or pre-
packaged food. The client’s comfort, safety,
A. Risk for deficient fluid volume and nutritional status are the priorities; the
B. Deficient fluid volume client may feel comfortable to eat if the food
C. Impaired gas exchange has been sealed before reaching the mental
D. Metabolic acidosis health facility.
96. The use of larvivorous fish in malaria control is 4. D. “Joining a support group of parents who are
the basis for which strategy of malaria control? coping with this problem can be quite
helpful. Taking with others in similar
circumstances provides support and allows for
A. Stream seeding
sharing of experiences.
B. Stream clearing
5. B. Observe the dressing at the back of the neck
C. Destruction of breeding places
for the presence of blood. Drainage flows by
D. Zooprophylaxis
gravity.
97. In Integrated Management of Childhood Illness,
severe conditions generally require urgent referral
6. C. Prepare her for a pelvic examination. Pelvic 18. B. Put a hat on the infant’s head. Oxygen has
examination would reveal dilation and cooling effect, and the baby should be kept
effacement warm so that metabolic activity and oxygen
7. D. On the right side of the heart. Pulmonic demands are not increased.
stenosis increases resistance to blood flow, 19. C. Wear an Ultra-Filter mask when they are in
causing right ventricular hyperthropy; with the client’s room. Tubercle bacilli are
right ventricular failure there is an increase in transmitted through air currents; therefore
pressure on the right side of the heart. personal protective equipment such as an
8. A. Eating patterns are altered. A new dietary Ultra-Filter mask is necessary.
regimen, with a balance of foods from the food 20. D. Cerebral cortex compression. Cerebral
pyramid, must be established and continued for compression affects pyramidal tracts, resulting
weight reduction to occur and be maintained. in decorticate rigidity and cranial nerve injury,
9. B. “It is Ok to cry; I’ll just stay with you for which cause pupil dilation.
now”. This portrays a nonjudgmental attitude 21. A.Mediastinal shift. Mediastinal structures
that recognizes the client’s needs. move toward the uninjured lung, reducing
10. C. Lactated Ringer’s solution. Lactated oxygenation and venous return.
Ringer’s solution replaces lost sodium and 22. C. Prevent situations that may stimulate the
corrects metabolic acidosis, both of which cervix or uterus. Stimulation of the cervix or
commonly occur following a burn. Albumin is uterus may cause bleeding or hemorrhage and
used as adjunct therapy, not primary fluid should be avoided.
replacement. Dextrose isn’t given to burn 23. C. Severe shortness of breath. This could
patients during the first 24 hours because it can indicate a recurrence of the pneumothorax as
cause pseudodiabetes. The patient is one side of the lung is inadequate to meet the
hyperkalemic from the potassium shift from oxygen demands of the body.
the intracellular space to the plasma, so 24. A. Suction equipment. Respiratory
potassium would be detrimental. complications can occur because of edema of
11. C. Twitching and disorientation. Excess the glottis or injury to the recurrent laryngeal
extracellular fluid moves into cells (water nerve.
intoxication); intracellular fluid excess in 25. A. Strong desire to improve her body
sensitive brain cells causes altered mental image. Clients with anorexia nervosa have a
status; other signs include anorexia nervosa, disturbed self image and always see themselves
nausea, vomiting, twitching, sleepiness, and as fat and needing further reducing.
convulsions. 26. B. Attempting to reduce or limit situations that
12. B. Resume the usual diet as soon as desired. As increase anxiety. Persons with high anxiety
long as the client has no nausea or vomiting, levels develop various behaviors to relieve
there are no dietary restriction. their anxiety; by reducing anxiety, the need for
13. B. Shrinkage of the residual limb must be these obsessive-compulsive action is reduced.
completed. Shrinkage of the residual limb, 27. C. Becomes fussy when frustrated and displays
resulting from reduction of subcutaneous fat a shortened attention span. Shortened attention
and interstitial fluid, must occur for an span and fussy behavior may indicate a change
adequate fit between the limb and the in intracranial pressure and/or shunt
prosthesis. malfunction.
14. A. Change the maternal position. Stimulation 28. B. Maintaining the ordered
of the sympathetic nervous system is an initial hydration. Promoting hydration maintains urine
response to mild hypoxia that accompanies production at a higher rate, which flushes the
partial cord compression (umbilical vein) bladder and prevents urinary stasis and
during contractions; changing the maternal possible infection.
position can alleviate the compression. 29. C. Taking the client’s pedal pulse in the
15. A. Perform a finger stick to test the client’s affected limb. Monitoring a pedal pulse will
blood glucose level. The client has signs of assess circulation to the foot.
diabetes, which may result from steroid therapy, 30. A. “Where are you?”. “Where are you?” is the
testing the blood glucose level is a method of best question to elicit information about the
screening for diabetes, thus gathering more client’s orientation to place because it
data. encourages a response that can be assessed.
16. C. Heart block. This is the primary indication 31. D. Bleeding from the venipuncture site. This
for a pacemaker because there is an interfere indicates a fibrinogenemia; massive clotting in
with the electrical conduction system of the the area of the separation has resulted in a
heart. lowered circulating fibrinogen.
17. A. With meals and snacks. Pancreases capsules 32. D. blowing pattern. Clients should use a
must be taken with food and snacks because it blowing pattern to overcome the premature
acts on the nutrients and readies them for urge to push.
absorption.
33. A. Cheeseburger and a malted. Of the 47. D. Simian lines on the hands. This is
selections offered, this is the highest in calories characteristic finding in newborns with Down
and protein, which are needed for increased syndrome.
basal metabolic rate and for tissue repair. 48. B. Eyes. Rheumatoid arthritis can cause
34. B. Cyanotic lips and face. Central cyanosis inflammation of the iris and ciliary body of the
(blue lips and face) indicates lowered eyes which may lead to blindness.
oxygenation of the blood, caused by either 49. A. Accept the client’s decision without
decreased lung expansion or right to left discussion. This is all the nurse can do until
shunting of blood. trust is established; facing the client to attend
35. A. Notify the physician of the findings because will disrupt the group.
the level is dangerously high. Levels close to 2 50. D. Provide a simple explanation of the
mEq/L are dangerously close to the toxic level; procedure and continue to reassure the
immediate action must be taken. client. The nurse should offer support and use
36. C. Days 15 to 17. Ovulation occurs clear, simple terms to allay client’s anxiety.
approximately 14 days before the next menses, 51. D. If I have difficulty in inserting the irrigating
about the 16th day in 30 day cycle; the 15th to tube into the stoma”. This occurs with stenosis
17th days would be the best time to avoid of the stoma; forcing insertion of the tube
sexual intercourse. could cause injury.
37. C. Assure that informed consent has been 52. C. Blood loss of 850 ml after a vaginal
obtained from the client. An invasive birth. Excessive blood loss predisposes the
procedure such as amniocentesis requires client to an increased risk of infection because
informed consent. of decreased maternal resistance; they expected
38. D. Prevent development of respiratory blood loss is 350 to 500 ml.
distress. Respiratory distress or arrest may 53. A. Provide frequent saline mouthwashes. This
occur when the serum level of magnesium is soothing to the oral mucosa and helps
sulfate reaches 12 to 15 mg/dl; deep tendon prevent infection.
reflexes disappear when the serum level is 10 54. B. “Society makes people react in old
to 12 mg/dl; the drug is withheld in the absence ways”. The client is incapable of accepting
of deep tendon reflexes; the therapeutic serum responsibility for self-created problems and
level is 5 to 8 mg/dl. blames society for the behavior.
39. A. Obtaining the child’s daily weight. Weight 55. A. Taste and smell. Swelling can obstruct nasal
monitoring is the most useful means of breathing, interfering with the senses of taste
assessing fluid balance and changes in the and smell.
edematous state; 1 liter of fluid weighs about 56. A. Fatigue. Fatigue is a major problem caused
2.2 pounds. by an increase in waste products because of
40. C. Reduces the inflammatory response of catabolic processes.
tissues. Corticosteroids act to decrease 57. A. Offer the client assistance to the
inflammation which decreases edema. bathroom. Statistics indicate that the most
41. D. An audible click on hip manipulation. With frequent cause of falls by hospitalized clients is
specific manipulation, an audible click may be getting up or attempting to get up to the
heard of felt as he femoral head slips into the bathroom unassisted.
acetabulum. 58. D. Turn completely over, sit momentarily
42. B. Allow the denial but be available to discuss without support, reach to be picked up. These
death. This does not remove client’s only way abilities are age-appropriate for the 6 month
of coping, and it permits future movement old child.
through the grieving process when the client is 59. D. Feed the baby on the unaffected breast first
ready. until the affected breast heals. The most
43. B. Divide food into four to six meals a vigorous sucking will occur during the first few
day. The volume of food in the stomach should minutes of breastfeeding when the infant
be kept small to limit pressure on the cardiac would be on the unaffected breast; later
sphincter. suckling is less traumatic.
44. B. “I feel washed out; there isn’t much 60. D. Place sterile cotton loosely in the external
left”. The client’s statement infers an emptiness ear of the client. This would absorb the
with an associated loss. drainage without causing further trauma.
45. A. Vitamin K is not absorbed. Vitamin K, a fat 61. D. Airing their feelings regarding the
soluble vitamin, is not absorbed from the GI transmission of the disease to the
tract in the absence of bile; bile enters the child. Discussion with parents who have
duodenum via the common bile duct. children with similar problems helps to reduce
46. D. Leg weakness with muscle cramps. Impulse some of their discomfort and guilt.
conduction of skeletal muscle is impaired with 62. A. Suspicious feelings. The nurse must deal
decreased potassium levels, muscular weakness with these feelings and establish basic trust to
and cramps may occur with hypokalemia. promote a therapeutic milieu.
63. A. Surgical menopause will occur. When a 77. A. Act 3573. Act 3573, the Law on Reporting
bilateral oophorectomy is performed, both of Communicable Diseases, enacted in 1929,
ovaries are excised, eliminating ovarian mandated the reporting of diseases listed in the
hormones and initiating response. law to the nearest health station.
64. D. Pointing out to the client that death can 78. A. Primary. The purpose of isolating a client
occur with malnutrition. The client expects the with a communicable disease is to protect those
nurse to focus on eating, but the emphasis who are not sick (specific disease prevention).
should be placed on feelings rather than actions. 79. B. It provides an opportunity to do first hand
65. B. Medication is not adequately effective. The appraisal of the home situation. Choice A is not
expected effect should be more than a one correct since a home visit requires that the
point decrease in the pain level. nurse spend so much time with the family.
66. B. Assisting the parents to stimulate their baby Choice C is an advantage of a group
through touch, sound, and sight. Stimuli are conference, while choice D is true of a clinic
provided via all the senses; since the infant’s consultation.
behavioral development is enhanced through 80. B. Should minimize if not totally prevent the
parent-infant interactions, these interactions spread of infection. Bag technique is performed
should be encouraged. before and after handling a client in the home
67. D. Recognize himself as an independent person to prevent transmission of infection to and
of worth. Academic deficits, an inability to from the client.
function within constraints required of certain 81. A. Recognizes staff for going beyond
settings, and negative peer attitudes often lead expectations by giving them citations. Path
to low self-esteem. Goal theory according to House and associates
68. B. Monitoring the child’s blood rewards good performance so that others would
pressure. Because the tumor is of renal origin, do the same.
the rennin angiotensin mechanism can be 82. D. Inspires others with vision. Inspires others
involved, and blood pressure monitoring is with a vision is characteristic of a
important. transformational leader. He is focused more on
69. A. Nursing unit manager. Controlled substance the day-to-day operations of the
issues for a particular nursing unit are the department/unit.
responsibility of that unit’s nurse manager. 83. A. Psychological and sociological needs are
70. D. Encourage coughing, deep breathing, and emphasized. When the functional method is
range of motion to the arm on the affected used, the psychological and sociological needs
side. All these interventions promote aeration of the patients are neglected; the patients are
of the re-expanding lung and maintenance of regarded as ‘tasks to be done”
function in the arm and shoulder on the 84. B. Preparing a nursing care plan in
affected side. collaboration with the patient. The best source
71. A. For people to attain their birthrights of of information about the priority needs of the
health and longevity. According to Winslow, patient is the patient himself. Hence using a
all public health efforts are for people to realize nursing care plan based on his expressed
their birthrights of health and longevity. priority needs would ensure meeting his needs
72. C. Swaroop’s index. Swaroop’s index is the effectively.
percentage of the deaths aged 50 years or older. 85. C. Unity of command. The principle of unity of
Its inverse represents the percentage of command means that employees should receive
untimely deaths (those who died younger than orders coming from only one manager and not
50 years). from two managers. This averts the possibility
73. D. Public health nursing focuses on preventive, of sowing confusion among the members of the
not curative, services.. The catchment area in organization.
PHN consists of a residential community, 86. A. Increase the patient satisfaction rate. Goal is
many of whom are well individuals who have a desired result towards which efforts are
greater need for preventive rather than curative directed. Options AB, C and D are all
services. objectives which are aimed at specific end.
74. B. Ensure the accessibility and quality of 87. A. Uses visioning as the essence of
health care. Ensuring the accessibility and leadership. Transformational leadership relies
quality of health care is the primary mission of heavily on visioning as the core of leadership.
DOH. 88. C. Avoidance. This strategy shuns discussing
75. B. Efficiency. Efficiency is determining the issue head-on and prefers to postpone it to a
whether the goals were attained at the least later time. In effect the problem remains
possible cost. unsolved and both parties are in a lose-lose
76. D. Rural Health Unit. R.A. 7160 devolved situation.
basic health services to local government units 89. A. Staffing. Staffing is a management function
(LGU’s ). The public health nurse is an involving putting the best people to accomplish
employee of the LGU.
tasks and activities to attain the goals of the water identified as breeding places of the
organization. Anopheles mosquito.
90. B. Decentralized. Decentralized structures 97. B. Severe dehydration. The order of priority in
allow the staff to make decisions on matters the management of severe dehydration is as
pertaining to their practice and communicate in follows: intravenous fluid therapy, referral to a
downward, upward, lateral and diagonal flow. facility where IV fluids can be initiated within
91. D. end each entry with the nurse’s signature 30 minutes, Oresol/nasogastric tube,
and title. The end of each entry should include Oresol/orem. When the foregoing measures are
the nurse’s signature and title; the signature not possible or effective, tehn urgent referral to
holds the nurse accountable for the recorded the hospital is done.
information. Erasing errors in documentation 98. A. Inability to drink. A sick child aged 2
on a legal document such as a client’s chart months to 5 years must be referred urgently to
isn’t permitted by law. Because a client’s a hospital if he/she has one or more of the
medical record is considered a legal document, following signs: not able to feed or drink,
the nurse should make all entries in ink. The vomits everything, convulsions, abnormally
nurse is accountable for the information sleepy or difficult to awaken.
recorded and therefore shouldn’t leave any 99. A. Sugar. R.A. 8976 mandates fortification of
blank lines in which another health care worker rice, wheat flour, sugar and cooking oil with
could make additions. Vitamin A, iron and/or iodine.
92. A. Allergies and socioeconomic status. General 100. A. Palms. The anatomic characteristics of
background data consist of such components as the palms allow a reliable and convenient basis
allergies, medical history, habits, for examination for pallor.
socioeconomic status, lifestyle, beliefs, and
sensory deficits. Urine output, gastric reflex,
and bowel habits are significant only if a
disease affecting these functions is present.
93. C. I.V. cannula insertion. Caregivers must use
surgical asepsis when performing wound care
or any procedure in which a sterile body cavity
is entered or skin integrity is broken. To
achieve surgical asepsis, objects must be
rendered or kept free of all pathogens. Inserting
an I.V. cannula requires surgical asepsis
because it disrupts skin integrity and involves
entry into a sterile cavity (a vein). The other
options are used to ensure medical asepsis or
clean technique to prevent the spread of
infection. The GI tract isn’t sterile; therefore,
irrigating a nasogastric tube or a colostomy
requires only clean technique.
94. B. Pouring solution onto a sterile field
cloth. Pouring solution onto a sterile field cloth
violates surgical asepsis because moisture
penetrating the cloth can carry microorganisms
to the sterile field via capillary action. The
other options are practices that help ensure
surgical asepsis.
95. C. Impaired gas exchange. The client has a
below-normal value for the partial pressure of
arterial oxygen (PaO2) and an above-normal
value for the partial pressure of arterial carbon
dioxide (PaCO2), supporting the nursing
diagnosis of Impaired gas exchange. ABG
values can’t indicate a diagnosis of Fluid
volume deficit (or excess) or Risk for deficient
fluid volume. Metabolic acidosis is a medical,
not nursing, diagnosis; in any event, these
ABG values indicate respiratory, not metabolic,
acidosis.
96. A. Stream seeding. Stream seeding is done by
putting tilapia fry in streams or other bodies of
A. WBC count
B. Urinalysis
C. ELISA
D. Peripheral blood smear

PNLE Exam 2
1. A woman in a child bearing age receives a 7. The Nurse supervisor is planning for patient’s
rubella vaccination. Nurse Joy would give her assignment for the AM shift. The nurse supervisor
which of the following instructions? avoids assigning which of the following staff
members to a client with herpes zoster?
A. Refrain from eating eggs or egg products for
24 hours A. Nurse who never had chicken pox
B. Avoid having sexual intercourse B. Nurse who never had roseola
C. Don’t get pregnant at least 3 months C. Nurse who never had german measles
D. Avoid exposure to sun D. Nurse who never had mumps

2. Jonas who is diagnosed with encephalitis is under 8. Clarissa is 7 weeks pregnant. Further
the treatment of Mannitol. Which of the following examination revealed that she is susceptible to
patient outcomes indicate to Nurse Ronald that the rubella. When would be the most appropriate for
treatment of Mannitol has been effective for a her to receive rubella immunization?
patient that has increased intracranial pressure?
A. At once
A. Increased urinary output B. During 2nd trimester
B. Decreased RR C. During 3rd trimester
C. Slowed papillary response D. After the delivery of the baby
D. Decreased level of consciousness
9. A female child with rubella should be isolated
3. Mary asked Nurse Maureen about the incubation from a:
period of rabies. Which statement by the Nurse
Maureen is appropriate? A. 21 year old male cousin living in the same
house
A. Incubation period is 6 months B. 18 year old sister who recently got married
B. Incubation period is 1 week C. 11 year old sister who had rubeola during
C. Incubation period is 1 month childhood
D. Incubation period varies depending on the D. 4 year old girl who lives next door
site of the bite
10. What is the primary prevention of leprosy?

4. Which of the following should Nurse Cherry do A. Nutrition


first in taking care of a male client with rabies? B. Vitamins
C. BCG vaccination
A. Encourage the patient to take a bath D. DPT vaccination
B. Cover IV bottle with brown paper bag 11. A bacteria which causes diphtheria is also
C. Place the patient near the comfort room known as?
D. Place the patient near the door
A. Amoeba
5. Which of the following is the screening test for B. Cholera
dengue hemorrhagic fever? C. Klebs-loeffler bacillus
D. Spirochete
A. Complete blood count 12. Nurse Ron performed mantoux skin test today
B. ELISA (Monday) to a male adult client. Which statement
C. Rumpel-leede test by the client indicates that he understood the
D. Sedimentation rate instruction well?

A. I will come back later


6. Mr. Dela Rosa is suspected to have malaria after B. I will come back next month
a business trip in Palawan. The most important C. I will come back on Friday
diagnostic test in malaria is: D. I will come back on Wednesday, same time, to
read the result
13. A male client had undergone Mantoux skin test. B. OPV
Nurse Ronald notes an 8mm area of indurations at C. Measles
the site of the skin test. The nurse interprets the D. None of the above
result as: 22. EPI is based on?

A. Negative A. Basic health services


B. Uncertain and needs to be repeated B. Scope of community affected
C. Positive C. Epidemiological situation
D. Inconclusive D. Research studies
14. Tony will start a 6 month therapy with Isoniazid 23. TT? provides how many percentage of
(INH). Nurse Trish plans to teach the client to: protection against tetanus?

A. Use alcohol moderately A. 100


B. Avoid vitamin supplements while o therapy B. 99
C. Incomplete intake of dairy products C. 80
D. May be discontinued if symptoms subsides D. 90
15. Which is the primary characteristic lesion of 24. Temperature of refrigerator to maintain potency
syphilis? of measles and OPV vaccine is:

A. Sore eyes A. -3c to -8c


B. Sore throat B. -15c to -25c
C. Chancroid C. +15c to +25c
D. Chancre D. +3c to +8c
16. What is the fast breathing of Jana who is 3 25. Diptheria is a:
weeks old?
A. Bacterial toxin
A. 60 breaths per minute B. Killed bacteria
B. 40 breaths per minute C. Live attenuated
C. 10 breaths per minute D. Plasma derivatives
D. 20 breaths per minute 26. Budgeting is under in which part of
17. Which of the following signs and symptoms management process?
indicate some dehydration?
A. Directing
A. Drinks eagerly B. Controlling
B. Restless and irritable C. Organizing
C. Unconscious D. Planning
D. A and B 27. Time table showing planned work days and
18. What is the first line for dysentery? shifts of nursing personnel is:

A. Amoxicillin A. Staffing
B. Tetracycline B. Schedule
C. Cefalexin C. Scheduling
D. Cotrimoxazole D. Planning
19. In home made oresol, what is the ratio of salt 28. A force within an individual that influences the
and sugar if you want to prepare with 1 liter of strength of behavior?
water?
A. Motivation
A. 1 tbsp. salt and 8 tbsp. sugar B. Envy
B. 1 tbsp. salt and 8 tsp. sugar C. Reward
C. 1 tsp. salt and 8 tsp. sugar D. Self-esteem
D. 8 tsp. salt and 8 tsp. sugar 29. “To be the leading hospital in the Philippines” is
20. Gentian Violet is used for: best illustrate in:

A. Wound A. Mission
B. Umbilical infections B. Philosophy
C. Ear infections C. Vision
D. Burn D. Objective
21. Which of the following is a live attenuated 30. It is the professionally desired norms against
bacterial vaccine? which a staff performance will be compared?

A. BCG A. Job descriptions


B. Survey 39. Objective data is also called:
C. Flow chart
D. Standards A. Covert
31. Reprimanding a staff nurse for work that is done B. Overt
incorrectly is an example of what type of C. Inference
reinforcement? D. Evaluation
40. An example of subjective data is:
A. Feedback
B. Positive reinforcement A. Size of wounds
C. Performance appraisal B. VS
D. Negative reinforcement C. Lethargy
32. Questions that are answerable only by choosing D. The statement of patient “My hand is painful”
an option from a set of given alternatives are known 41. What is the best position in palpating the breast?
as?
A. Trendelenburg
A. Survey B. Side lying
B. Close ended C. Supine
C. Questionnaire D. Lithotomy
D. Demographic 42. When is the best time in performing breast self
33. A researcher that makes a generalization based examination?
on observations of an individuals behavior is said to
be which type of reasoning: A. 7 days after menstrual period
B. 7 days before menstrual period
A. Inductive C. 5 days after menstrual period
B. Logical D. 5 days before menstrual period
C. Illogical 43. Which of the following should be given the
D. Deductive highest priority before performing physical
34. The balance of a research’s benefit vs. its risks examination to a patient?
to the subject is:
A. Preparation of the room
A. Analysis B. Preparation of the patient
B. Risk-benefit ratio C. Preparation of the nurse
C. Percentile D. Preparation of environment
D. Maximum risk 44. It is a flip over card usually kept in portable file
35. An individual/object that belongs to a general at nursing station.
population is a/an:
A. Nursing care plan
A. Element B. Medicine and treatment record
B. Subject C. Kardex
C. Respondent D. TPR sheet
D. Author 45. Jose has undergone thoracentesis. The nurse in
36. An illustration that shows how the members of charge is aware that the best position for Jose is:
an organization are connected:
A. Semi fowlers
A. Flowchart B. Low fowlers
B. Bar graph C. Side lying, unaffected side
C. Organizational chart D. Side lying, affected side
D. Line graph 46. The degree of patients abdominal distension
37. The first college of nursing that was established may be determined by:
in the Philippines is:
A. Auscultation
A. Fatima University B. Palpation
B. Far Eastern University C. Inspection
C. University of the East D. Percussion
D. University of Sto. Tomas 47. A male client is addicted with hallucinogen.
38. Florence nightingale is born on: Which physiologic effect should the nurse expect?

A. France A. Bradyprea
B. Britain B. Bradycardia
C. U.S C. Constricted pupils
D. Italy D. Dilated pupils
48. Tristan a 4 year old boy has suffered from full D. Semi-fowlers
thickness burns of the face, chest and neck. What 56. Nurse Hazel knows that Myrna understands her
will be the priority nursing diagnosis? condition well when she remarks that urinary
frequency is caused by:
A. Ineffective airway clearance related to edema
B. Impaired mobility related to pain A. Pressure caused by the ascending uterus
C. Impaired urinary elimination related to fluid B. Water intake of 3L a day
loss C. Effect of cold weather
D. Risk for infection related to epidermal D. Increase intake of fruits and vegetables
disruption 57. How many ml of blood is loss during the first 24
49. In assessing a client’s incision 1 day after the hours post delivery of Myrna?
surgery, Nurse Betty expect to see which of the
following as signs of a local inflammatory response? 100
500
A. Greenish discharge 200
B. Brown exudates at incision edges 400
C. Pallor around sutures 58. Which of the following hormones stimulates the
D. Redness and warmth secretion of milk?
50. Nurse Ronald is aware that the amiotic fluid in
the third trimester weighs approximately: Progesterone
Prolactin
A. 2 kilograms Oxytocin
B. 1 kilograms Estrogen
C. 100 grams 59. Nurse Carla is aware that Myla’s second stage
D. 1.5 kilograms of labor is beginning when the following
51. After delivery of a baby girl. Nurse Gina assessment is noted:
examines the umbilical cord and expects to find a
cord to: Bay of water is broken
Contractions are regular
A. Two arteries and two veins Cervix is completely dilated
B. One artery and one vein Presence of bloody show
C. Two arteries and one vein 60. The leaking fluid is tested with nitrazine paper.
D. One artery and two veins Nurse Kelly confirms that the client’s membrane
52. Myrna a pregnant client reports that her last have ruptures when the paper turns into a:
menstrual cycle is July 11, her expected date of
birth is Pink
Violet
A. November 4 Green
B. November 11 Blue
C. April 4 61. After amniotomy, the priority nursing action is:
D. April 18
53. Which of the following is not a good source of Document the color and consistency of
iron? amniotic fluid
A. Butter Listen the fetal heart tone
B. Pechay Position the mother in her left side
C. Grains Let the mother rest
D. Beef 62. Which is the most frequent reason for
54. Maureen is admitted with a diagnosis of ectopic postpartum hemorrhage?
pregnancy. Which of the following would you
anticipate? Perineal lacerations
Frequent internal examination (IE)
A. NPO CS
B. Bed rest Uterine atomy
C. Immediate surgery 63. On 2nd postpartum day, which height would you
D. Enema expect to find the fundus in a woman who has had a
55. Gina a postpartum client is diagnosed with caesarian birth?
endometritis. Which position would you expect to 1 finger above umbilicus
place her based on this diagnosis? 2 fingers above umbilicus
2 fingers below umbilicus
A. Supine 1 finger below umbilicus
B. Left side lying 64. Which of the following criteria allows Nurse
C. Trendelinburg Kris to perform home deliveries?
Normal findings during assessment Identify the illness
Previous CS Assess the child
Diabetes history Treat the child
Hypertensive history 73. If a young child has pneumonia when should the
65. Nurse Carla is aware that one of the following mother bring him back for follow up?
vaccines is done by intramuscular (IM) injection?
After 2 days
Measles In the afternoon
OPV After 4 days
BCG After 5 days
Tetanus toxoid 74. It is the certification recognition program that
66. Asin law is on which legal basis: develop and promotes standard for health facilities:

RA 8860 Formula
RA 2777 Tutok gamutan
RI 8172 Sentrong program movement
RR 6610 Sentrong sigla movement
67. Nurse John is aware that the herbal medicine 75. Baby Marie was born May 23, 1984. Nurse John
appropriate for urolithiasis is: will expect finger thumb opposition on:

Akapulco April 1985


Sambong February 1985
Tsaang gubat March 1985
Bayabas June 1985
68. Community/Public health bag is defined as: 76. Baby Reese is a 12 month old child. Nurse
Oliver would anticipate how many teeth?
An essential and indispensable equipment of
the community health nurse during home visit 9
It contains drugs and equipment used by the 7
community health nurse 8
Is a requirement in the health center and for 6
home visit 77. Which of the following is the primary antidote
It is a tool used by the community health nurse for Tylenol poisoning?
in rendering effective procedures during home
visit Narcan
69. TT4 provides how many percentage of Digoxin
protection against tetanus? Acetylcysteine
Flumazenil
70 78. A male child has an intelligence quotient of
80 approximately 40. Which kind of environment and
90 interdisciplinary program most likely to benefit this
99 child would be best described as:
70. Third postpartum visit must be done by public
health nurse: Habit training
Sheltered workshop
Within 24 hours after delivery Custodial
After 2-4 weeks Educational
Within 1 week 79. Nurse Judy is aware that following condition
After 2 months would reflect presence of congenital G.I anomaly?
71. Nurse Candy is aware that the family planning
method that may give 98% protection to another Cord prolapse
pregnancy to women Polyhydramios
Placenta previa
Pills Oligohydramios
Tubal ligation 80. Nurse Christine provides health teaching for the
Lactational Amenorrhea method (LAM) parents of a child diagnosed with celiac disease.
IUD Nurse Christine teaches the parents to include
72. Which of the following is not a part of IMCI which of the following food items in the child’s diet:
case management process
Rye toast
Counsel the mother Oatmeal
White bread R.A. 7600
Rice R.A. 6700
81. Nurse Randy is planning to administer oral P.D. 996
medication to a 3 year old child. Nurse Randy is 89. Self governance, ability to choose or carry out
aware that the best way to proceed is by: decision without undue pressure or coercion from
anyone:
“Would you like to drink your medicine?”
“If you take your medicine now, I’ll give you Veracity
lollipop” Autonomy
“See the other boy took his medicine? Now it’s Fidelity
your turn.” Beneficence
“Here’s your medicine. Would you like a 90. A male patient complained because his
mango or orange juice?” scheduled surgery was cancelled because of
82. At what age a child can brush her teeth without earthquake. The hospital personnel may be excused
help? because of:

6 years Governance
7 years Respondent superior
5 years Force majeure
8 years Res ipsa loquitor
83. Ribivarin (Virazole) is prescribed for a female 91. Being on time, meeting deadlines and
hospitalized child with RSV. Nurse Judy prepare completing all scheduled duties is what virtue?
this medication via which route?
Fidelity
Intra venous Autonomy
Oral Veracity
Oxygen tent Confidentiality
Subcutaneous 92. This quality is being demonstrated by Nurse
84. The present chairman of the Board of Nursing in Ron who raises the side rails of a confused and
the Philippines is: disoriented patient?

Maria Joanna Cervantes Responsibility


Carmencita Abaquin Resourcefulness
Leonor Rosero Autonomy
Primitiva Paquic Prudence
85. The obligation to maintain efficient ethical 93. Which of the following is formal continuing
standards in the practice of nursing belong to this education?
body:
Conference
BON Enrollment in graduate school
ANSAP Refresher course
PNA Seminar
RN 94. The BSN curriculum prepares the graduates to
86. A male nurse was found guilty of negligence. become?
His license was revoked. Re-issuance of revoked
certificates is after how many years? Nurse generalist
Nurse specialist
1 year Primary health nurse
2 years Clinical instructor
3 years 95. Disposal of medical records in government
4 years hospital/institutions must be done in close
87. Which of the following information cannot be coordination with what agency?
seen in the PRC identification card?
Department of Health
Registration Date Records Management Archives Office
License Number Metro Manila Development Authority
Date of Application Bureau of Internal Revenue
Signature of PRC chairperson 96. Nurse Jolina must see to it that the written
88. Breastfeeding is being enforced by milk code or: consent of mentally ill patients must be taken from:

EO 51 Nurse
Priest Hallucinogen
Family lawyer Psycho stimulant
Parents/legal guardians Anxiolytic
97. When Nurse Clarence respects the client’s self- Narcotic
disclosure, this is a gauge for the nurses’ 105. In community health nursing, it is the most
important risk factor in the development of mental
Respectfulness illness?
Loyalty
Trustworthiness Separation of parents
Professionalism Political problems
98. The Nurse is aware that the following tasks can Poverty
be safely delegated by the nurse to a non-nurse Sexual abuse
health worker except: 106. All of the following are characteristics of crisis
except
Taking vital signs
Change IV infusions The client may become resistive and active in
Transferring the client from bed to chair stopping the crisis
Irrigation of NGT It is self-limiting for 4-6 weeks
99. During the evening round Nurse Tina saw Mr. It is unique in every individual
Toralba meditating and afterwards started singing It may also affect the family of the client
prayerful hymns. What would be the best response 107. Freud states that temper tantrums is observed
of Nurse Tina? in which of the following:

Call the attention of the client and encourage to Oral


sleep Anal
Report the incidence to head nurse Phallic
Respect the client’s action Latency
Document the situation 108. The nurse is aware that ego development
100. In caring for a dying client, you should begins during:
perform which of the following activities
Toddler period
Do not resuscitate Preschool age
Assist client to perform ADL School age
Encourage to exercise Infancy
Assist client towards a peaceful death 109. Situation: A 19 year old nursing student has
101. The Nurse is aware that the ability to enter into lost 36 lbs for 4 weeks. Her parents brought her to
the life of another person and perceive his current the hospital for medical evaluation. The diagnosis
feelings and their meaning is known: was ANOREXIA NERVOSA. The Primary gain of
a client with anorexia nervosa is:
Belongingness
Genuineness Weight loss
Empathy Weight gain
Respect Reduce anxiety
102. The termination phase of the NPR is best Attractive appearance
described one of the following: 110. The nurse is aware that the primary nursing
diagnosis for the client is:
Review progress of therapy and attainment of
goals Altered nutrition : less than body requirement
Exploring the client’s thoughts, feelings and Altered nutrition : more than body requirement
concerns Impaired tissue integrity
Identifying and solving patients problem Risk for malnutrition
Establishing rapport 111. After 14 days in the hospital, which finding
103. During the process of cocaine withdrawal, the indicates that her condition in improving?
physician orders which of the following:
She tells the nurse that she had no idea that she
Haloperidol (Haldol) is thin
Imipramine (Tofranil) She arrives earlier than scheduled time of
Benztropine (Cogentin) group therapy
Diazepam (Valium) She tells the nurse that she eat 3 times or more
104. The nurse is aware that cocaine is classified as: in a day
She gained 4 lbs in two weeks
112. The nurse is aware that ataractics or psychic 121. Which of the following activity would be best
energizers are also known as: for a depressed client?

Anti manic Chess


Anti depressants Basketball
Antipsychotics Swimming
Anti anxiety Finger painting
113. Known as mood elevators: 122. The nurse is aware that clients with severe
depression, possess which defense mechanism:
Anti depressants
Antipsychotics Introjection
Anti manic Suppression
Anti anxiety Repression
114. The priority of care for a client with Projection
Alzheimer’s disease is 123. Nurse John is aware that self mutilation among
Bipolar disorder patients is a means of:
Help client develop coping mechanism
Encourage to learn new hobbies and interest Overcoming fear of failure
Provide him stimulating environment Overcoming feeling of insecurity
Simplify the environment to eliminate the need Relieving depression
to make chores Relieving anxiety
115. Autism is diagnosed at: 124. Which of the following may cause an increase
in the cystitis symptoms?
Infancy
3 years old Water
5 years old Orange juice
School age Coffee
116. The common characteristic of autism child is: Mango juice
125. In caring for clients with renal calculi, which is
Impulsitivity the priority nursing intervention?
Self destructiveness
Hostility Record vital signs
Withdrawal Strain urine
117. The nurse is aware that the most common Limit fluids
indication in using ECT is: Administer analgesics as prescribed
126. In patient with renal failure, the diet should be:
Schizophrenia
Bipolar Low protein, low sodium, low potassium
Anorexia Nervosa Low protein, high potassium
Depression High carbohydrate, low protein
118. A therapy that focuses on here and now High calcium, high protein
principle to promote self-acceptance? 127. Which of the following cannot be corrected by
dialysis?
Gestalt therapy
Cognitive therapy Hypernatremia
Behavior therapy Hyperkalemia
Personality therapy Elevated creatinine
119. A client has many irrational thoughts. The goal Decreased hemoglobin
of therapy is to change her: 128. Tony with infection is receiving antibiotic
therapy. Later the client complaints of ringing in the
Personality ears. This ototoxicity is damage to:
Communication
Behavior 4th CN
Cognition 8th CN
120. The appropriate nutrition for Bipolar I disorder, 7th CN
in manic phase is: 9th CN
129. Nurse Emma provides teaching to a patient
Low fat, low sodium with recurrent urinary tract infection includes the
Low calorie, high fat following:
Finger foods, high in calorie
Small frequent feedings Increase intake of tea, coffee and colas
Void every 6 hours per day Call the family
Void immediately after intercourse 138. The ANA recognized nursing informatics
Take tub bath everyday heralding its establishment as a new field in nursing
130. Which assessment finding indicates circulatory during what year?
constriction in a male client with a newly applied
long leg cast? 1994
1992
Blanching or cyanosis of legs 2000
Complaints of pressure or tightness 2001
Inability to move toes 139. When is the first certification of nursing
Numbness of toes informatics given?
131. During acute gout attack, the nurse administer
which of the following drug: 1990-1993
2001-2002
Prednisone (Deltasone) 1994-1996
Colchicines 2005-2008
Aspirin 140. The nurse is assessing a female client with
Allopurinol (Zyloprim) possible diagnosis of osteoarthritis. The most
132. Information in the patients chart is significant risk factor for osteoarthritis is:
inadmissible in court as evidence when:
Obesity
The client objects to its use Race
Handwriting is not legible Job
It has too many unofficial abbreviations Age
The clients parents refuses to use it 141. A male client complains of vertigo. Nurse Bea
133. Nurse Karen is revising a client plan of care. anticipates that the client may have a problem with
During which step of the nursing process does such which portion of the ear?
revision take place?
Tymphanic membranes
Planning Inner ear
Implementation Auricle
Diagnosing External ear
Evaluation 142. When performing Weber’s test, Nurse Rosean
134. When examining a client with abdominal pain, expects that this client will hear
Nurse Hazel should assess:
On unaffected side
Symptomatic quadrant either second or first Longer through bone than air conduction
The symptomatic quadrant last On affected side by bone conduction
The symptomatic quadrant first By neither bone or air conduction
Any quadrant 143. Toy with a tentative diagnosis of myasthenia
135. How long will nurse John obtain an accurate gravis is admitted for diagnostic make up.
reading of temperature via oral route? Myasthenia gravis can confirmed by:

3 minutes Kernigs sign


1 minute Brudzinski’s sign
8 minutes A positive sweat chloride test
15 minutes A positive edrophonium (Tensilon) test
136. The one filing the criminal care against an 144. A male client is hospitalized with Guillain-
accused party is said to be the? Barre Syndrome. Which assessment finding is the
most significant?
Guilty
Accused Even, unlabored respirations
Plaintiff Soft, non distended abdomen
Witness Urine output of 50 ml/hr
137. A male client has a standing DNR order. He Warm skin
then suddenly stopped breathing and you are at his 145. For a female client with suspected intracranial
bedside. You would: pressure (ICP), a most appropriate respiratory goal
is:
Call the physician
Stay with the client and do nothing Maintain partial pressure of arterial oxygen (Pa
Call another nurse O2) above 80mmHg
Promote elimination of carbon dioxide C. Positive
Lower the PH B. Avoid vitamin supplements while o therapy
Prevent respiratory alkalosis D. Chancre
146. Which nursing assessment would identify the A. 60 breaths per minute
earliest sign of ICP? D. A and B
D. Cotrimoxazole
Change in level of consciousness C. 1 tsp. salt and 8 tsp. sugar
Temperature of over 103°F B. Umbilical infections
Widening pulse pressure A. BCG
Unequal pupils C. Epidemiological situation
147. The greatest danger of an uncorrected atrial D. 90
fibrillation for a male patient will be which of the B. -15c to -25c
following: A. Bacterial toxin
D. Planning
Pulmonary embolism B. Schedule
Cardiac arrest A. Motivation
Thrombus formation C. Vision
Myocardial infarction D. Standards
148. Linda, A 30 year old post hysterectomy client D. Negative reinforcement
has visited the health center. She inquired about B. Close ended
BSE and asked the nurse when BSE should be A. Inductive
performed. You answered that the BSE is best B. Risk-benefit ratio
performed: A. Element
C. Organizational chart
7 days after menstruation D. University of Sto. Tomas
At the same day each month D. Italy
During menstruation B. Overt
Before menstruation D. The statement of patient “My hand is
149. An infant is ordered to recive 500 ml of painful”
D5NSS for 24 hours. The Intravenous drip is C. Supine
running at 60 gtts/min. How many drops per minute A. 7 days after menstrual period
should the flow rate be? B. Preparation of the patient
C. Kardex
60 gtts/min. C. Side lying, unaffected side
21 gtts/min D. Percussion
30 gtts/min D. Dilated pupils
15 gtts/min A. Ineffective airway clearance related to
150. Mr. Gutierrez is to receive 1 liter of D5RL to edema
run for 12 hours. The drop factor of the IV infusion D. Redness and warmth
set is 10 drops per minute. Approximately how B. 1 kilograms
many drops per minutes should the IV be regulated? C. Two arteries and one vein
D. April 18
A. Butter
13-14 drops
C. Immediate surgery
17-18 drops
D. Semi-fowlers
10-12 drops
A. Pressure caused by the ascending uterus
15-16 drops
B. 500
Answers D. Estrogen
C. Don’t get pregnant at least 3 months C. Cervix is completely dilated
A. Increased urinary output D. Blue
D. Incubation period varies depending on the B. Listen the fetal heart tone
site of the bite D. Uterine atomy
B. Cover IV bottle with brown paper bag C. 2 fingers below umbilicus
C. Rumpel-leede test A. Normal findings during assessment
D. Peripheral blood smear D. Tetanus toxoid
A. Nurse who never had chicken pox C. RI 8172
D. After the delivery of the baby B. Sambong
B. 18 year old sister who recently got married A. An essential and indispensable equipment of
C. BCG vaccination the community health nurse during home visit
C. Klebs-loeffler bacillus D. 99
D. I will come back on Wednesday, same time, B. After 2-4 weeks
to read the result C. Lactational Amenorrhea method (LAM)
B. Identify the illness C. Void immediately after intercourse
A. After 2 days A. Blanching or cyanosis of legs
D. Sentrong sigla movement B. Colchicines
B. February 1985 A. The client objects to its use
D. 6 D. Evaluation
C. Acetylcysteine B. The symptomatic quadrant last
A. Habit training A. 3 minutes
B. Polyhydramios C. Plaintiff
D. Rice B. Stay with the client and do nothing
D. “Here’s your medicine. Would you like a A. 1994
mango or orange juice?” B. 2001-2002
A. 6 years D. Age
C. Oxygen tent B. Inner ear
B. Carmencita Abaquin C. On affected side by bone conduction
A. BON D. A positive edrophonium (Tensilon) test
D. 4 years A. Even, unlabored respirations
C. Date of Application B. Promote elimination of carbon dioxide
A. EO 51 A. Change in level of consciousness
B. Autonomy C. Thrombus formation
C. Force majeure B. At the same day each month
A. Fidelity B. 21 gtts/min
D. Prudence A. 13-14 drops
B. Enrollment in graduate school
C. Primary health nurse
A. Department of Health
D. Parents/legal guardians
C. Trustworthiness
B. Change IV infusions
C. Respect the client’s action
D. Assist client towards a peaceful death
C. Empathy
A. Review progress of therapy and attainment
of goals
D. Diazepam (Valium)
B. Psycho stimulant
C. Poverty
A. The client may become resistive and active
in stopping the crisis
B. Anal
D. Infancy
C. Reduce anxiety
A. Altered nutrition : less than body
requirement
D. She gained 4 lbs in two weeks
C. Antipsychotics
A. Anti depressants
D. Simplify the environment to eliminate the
need to make chores
B. 3 years old
D. Withdrawal
D. Depression
A. Gestalt therapy
D. Cognition
C. Finger foods, high in calorie
D. Finger painting
A. Introjection
B. Overcoming feeling of insecurity
C. Coffee
D. Administer analgesics as prescribed
A. Low protein, low sodium, low potassium
D. Decreased hemoglobin
B. 8th CN

Potrebbero piacerti anche